You are on page 1of 353

MỤC LỤC

Ban biên tập


Lời giới thiệu . . . . . . . . . . . . . . . . . . . . . . . . . . . . . . . . . . . . . . . . 3

Trần Nam Dũng


Học một bài toán như thế nào? . . . . . . . . . . . . . . . . . . . . . . . . . . . . . . . 5

Võ Quốc Bá Cẩn
Một tính chất thú vị của tam thức bậc hai và nhị thứ bậc nhất . . . . . . . . . . . . . . 25

Nguyễn Văn Huyện, Tống Hữu Nhân


Một số vấn đề bất đẳng thức bậc bốn ba biến . . . . . . . . . . . . . . . . . . . . . . 33

Nguyễn Mạnh Linh


Một số vấn đề xung quanh tổng lũy thừa . . . . . . . . . . . . . . . . . . . . . . . . . 55

Nguyễn Hoàng Vinh


Hàm lồi một biến và bất đẳng thức . . . . . . . . . . . . . . . . . . . . . . . . . . . . 69

Kiều Đình Minh


Một số kỹ thuật sử dụng định lý giới hạn kẹp . . . . . . . . . . . . . . . . . . . . . . 83

Trần Minh Hiền


Định lý Ruf và hàm sinh . . . . . . . . . . . . . . . . . . . . . . . . . . . . . . . . . . 107

Nguyễn Đình Thành Công, Nguyễn Văn Hưởng


Một số bài toán về trò chơi . . . . . . . . . . . . . . . . . . . . . . . . . . . . . . . . . 125

Đậu Hoàng Hưng


Phương pháp đếm bằng hai cách thông qua bảng ô vuông . . . . . . . . . . . . . . . . 145

Trần Ngọc Thắng


Phương pháp quỹ đạo . . . . . . . . . . . . . . . . . . . . . . . . . . . . . . . . . . . 157

1
2 Các phương pháp giải toán qua các kỳ thi Olympic

Trần Quang Hùng


Tổng quát và ứng dụng một bài toán chia đôi đoạn thẳng . . . . . . . . . . . . . . . . 169

Huỳnh Chí Hào


Hàng điểm điều hòa . . . . . . . . . . . . . . . . . . . . . . . . . . . . . . . . . . . . 185

Nguyễn Văn Linh


Các tam giác có dạng AB C AC D kBC . . . . . . . . . . . . . . . . . . . . . . . . . 209

Nguyễn Ngọc Giang


Vẻ đẹp bài toán thi Olympic Hungary . . . . . . . . . . . . . . . . . . . . . . . . . . . 229

Nguyễn Duy Liên


Định lý thặng dư Trung Hoa và một số ứng dụng . . . . . . . . . . . . . . . . . . . . . 241

Lê Phúc Lữ
S.n/ – hàm tổng các chữ số . . . . . . . . . . . . . . . . . . . . . . . . . . . . . . . . 267

Ban biên tập


Lời giải và bình luận đề thi VMO 2015 . . . . . . . . . . . . . . . . . . . . . . . . . . 299

Ban biên tập


Đề thi Olympic Toán quốc tế 2015 . . . . . . . . . . . . . . . . . . . . . . . . . . . . . 349
LỜI GIỚI THIỆU
Ban biên tập

Các bạn đang cầm trên tay cuốn kỷ yếu của chương trình Gặp gỡ Toán học 2015 do Viện nghiên
cứu cao cấp về Toán (VIASM) phối hợp cùng với công ty cổ phần Giáo dục Ti tan (Titan
Education) và trường THPT chuyên Lê Quý Đôn Vũng Tàu tổ chức tại Vũng Tàu tháng 7, 2015.
Gặp gỡ toán học 2015 được thực hiện dưới sự bảo trợ của Chương trình trọng điểm quốc gia phát
triển toán học giai đoạn 2010-2020.
Tuy nhiên, có thể coi đây là một ấn phẩm độc lập, một tài liệu tham khảo bổ sung dành cho các
bạn học sinh và các thầy cô giáo.
Chủ đề chính của kỷ yếu này là Các phương pháp giải toán với 16 bài viết về phương pháp giải
toán Đại số, Giải tích, Hình học, Số học, Tổ hợp của các tác giả là các thầy cô giáo chuyên toán,
nghiên cứu sinh, học viên cao học, các bạn sinh viên và học sinh đến từ mọi miền đất nước. Sự
ủng hộ nhiệt tình và có chất lượng của các tác giả dành cho Kỷ yếu đã tạo ra một cuốn Kỷ yếu
dày dặn, có nội dung phong phú, cập nhật. Vì một số lý do khác nhau, một số bài viết chưa được
đưa vào Kỷ yếu lần này, chúng tôi sẽ trao đổi với các tác giả để có thể sử dụng trong các ấn phẩm
sau. Ban biên tập xin chân thành cảm ơn sự ủng hộ của các tác giả cho kỷ yếu Gặp gỡ toán học
2015 cũng như trong suốt những năm qua.
Bên cạnh đó, kỷ yếu giới thiệu Lời giải và Bình luận của các kỳ thi chính trong năm học vừa
qua: VMO 2015, Vietnam TST 2015 và IMO 2015. Qua các lời giải chi tiết đặc biệt là phần bình
luận, độc giả có thể có một cái nhìn rộng hơn về mặt phương pháp giải toán.
Ban biên tập xin cảm ơn lãnh đạo các đơn vị tổ chức đã luôn đồng hành cùng Gặp gỡ toán học,
đặc biệt là sự quan tâm, động viên của GS Ngô Bảo Châu, GS Nguyễn Hữu Dư, TS Nguyễn Thị
Lê Hương, TS Nguyễn Văn Lượng và thầy Lê Quốc Hùng dành cho các hoạt động của chương
trình nói chung và Kỷ yếu nói riêng.
Ban biên tập gửi lời cảm ơn đặc biệt đến họa sĩ thiết kế Phạm Minh Thư (Trento, Italy) đã thiết
kế một bộ áo mới thật đẹp và hiện đại cho Kỷ yếu lần này.
Hy vọng cuốn kỷ yếu này sẽ là một tài liệu bổ ích cho các bạn học sinh và các thầy cô giáo.
Tp Hồ Chí Minh, 7/2015
Ban biên tập

3
4 Các phương pháp giải toán qua các kỳ thi Olympic
HỌC MỘT BÀI TOÁN NHƯ THẾ NÀO?
Trần Nam Dũng
(Đại học KHTN, thành phố Hồ Chí Minh)

Quý hồ tinh bất quý hồ đa


Tôn Vũ
Để cải thiện trí óc, ta cần học ít, thấu hiểu nhiều
Rene Descartes
Đi chậm, tiến xa
Ngạn ngữ Nga
Mỗi một bài toán tôi giải đều trở thành quy tắc mà sau này được dùng để giải những bài toán
khác Hãy chia những khó khăn thành những thành phần đủ nhỏ để có thể giải quyết chúng
Rene Descartes
Làm thế nào để học tốt môn toán luôn là câu hỏi mà nhiều bạn học sinh trăn trở. Tại sao có nhiều
bạn bỏ rất nhiều thời gian cho việc học toán, nhưng tiến bộ thì rất chậm, nhiều lúc còn đứng yên
tại chỗ và thụt lùi? Có nhiều bạn nhìn học rất thoải mái, nhưng học đâu hiểu đó, áp dụng được
những điều mình đã học vào các tính huống khác nhau.
Vấn đề là ở cách học. Khi học một bài toán, có bạn chỉ học qua loa, nắm được lời giải là thôi,
chuyển ngay qua bài toán khác. Không đào sâu khai thác, không so sánh với những bài toán khác
để tìm những cái chung, không tóm tắt lại để biết đâu là điểm chính yếu, không mở rộng để xem
phương pháp giải sẽ còn áp dụng được đến đâu. Vì thế, một lời giải chỉ đơn thuần là một lời giải.
Không phải là phương pháp, không có sự kết nối đa chiều với những bài toán khác.
Chúng ta cứ tưởng tượng, nếu cần thuộc đường ở thành phố Hồ Chí Minh. Với hàng trăm con
đường như thế, chúng ta có nhớ nổi không? Và nếu bỏ công sức ra học thì bao giờ mới nhớ hết?
Nhưng nếu ta học một cách có hệ thống, đầu tiên là các trục ngang, trục dọc lớn, sau đó là các
cụm theo quận, theo phường. Ta không cần nhớ hết, chỉ cần biết ở quận nào, phường nào, gần
đường lớn nào. Khi cần chi tiết ta sẽ nghiên cứu sau, vì khi đã khoanh vùng được thì đó không
còn là việc khó.
Học giải toán cũng vậy. Nếu chúng ta học các bài toán một cách riêng rẽ thì biết bao nhiêu là đủ.
Và ta có đủ thời gian để nhớ được bao nhiêu bài toán. Và nếu ta biết rằng khó hy vọng cho sự
trúng tủ, mà chỉ có thể là những bài toán gần, tương tự, có mối liên hệ với những bài toán mình
biết. Vậy thì khi cần vận dụng mối liên hệ với những bài toán đó, trong kho tàng vài nghìn bài
toán được sắp xếp lộn xộn, không có lớp lang thì ta có đủ thời gian mà đem ra mà dùng không?
Nói một cách hình ảnh, giả sử ta có 100 bài toán, thay vì học cả 100 bài một cách riêng lẻ, ta có
thể chọn ra 10 15 bài chốt.

5
6 Các phương pháp giải toán qua các kỳ thi Olympic

1 3
4
2 13
5 15
14 6
7
8
9
10
11 12

Lúc đó, mỗi một bài toán chốt (ta biết rõ cách giải) hay thường (là những bài toán ta chưa biết
nhưng có thể gặp) đều có những mối liên hệ hàng ngang, cột dọc hay đường chéo với những bài
toán khác. Từ đó, khi cần ta sẽ có thể phục hồi lại, y như ta giải Sudoku vậy.
Dưới đây ta xem xét một số ví dụ về khai thác phương pháp giải toán từ những chứng minh và lời
giải kinh điển.

1. Chứng minh của G.Polya cho bất đẳng thức AM-GM


và bài học về sử dụng hằng số trong bất đẳng thức
không thuần nhất
Chúng ta đều biết bất đẳng thức AM-GM và rất nhiều các cách chứng minh của nó: Dùng quy
nạp lùi, dùng hàm lồi, dùng khai triển, dùng dồng biến ... Tuy nhiên cách chứng minh dưới đây
của G.Polya rất đẹp đẽ và đem đến cho ta nhiều bài học bổ ích.

Định lý 1 (Bất đẳng thức AM-GM). Nếu a1 ; a2 ; : : : ; an là n số thực dương thì ta có


a1 C a2 C    C an p
 n a1 a2    an :
n
Lời giải. Trước hết ta chứng minh bổ đề:
Bổ đề 1. Nếu x1 ; x2 ; : : : ; xn là các số thực dương có tích bằng 1 thì

x1 C x2 C    C xn  n:

Ta chứng minh bổ đề bằng quy nạp theo n:


Với n D 1 mệnh đề hiển nhiên đúng. Giả sử mệnh đề đã đúng với n số. Ta chứng minh mệnh đề
đúng với n C 1 số. Xét n C 1 số x1 ; x2 ; : : : ; xn ; xnC1 có tích bằng 1:
Nếu tất cả các số bằng nhau (và bằng 1/ thì bất đẳng thức trở thành đẳng thức. Trong trường hợp
ngược lại, phải có một số > 1 và một số < 1: Không mất tính tổng quát của bài toán, ta có thể
giả sử xn < 1; và xnC1 > 1:
Khi đó ta có .xn 1/.xnC1 1/ < 0; suy ra xn C xnC1 > xn xnC1 C 1: Từ đó ta có

x1 C    C xn 1 C xn C xnC1 > x1 C    C xn 1 C xn xnC1 C 1: .1/


Học một bài toán như thế nào? 7

Áp dụng giải thiết quy nạp cho n số x1 ; : : : ; xn 1 ; xn xnC1 có tích bằng 1; ta có


x1 C    C xn 1 C xn xnC1  n: .2/
Từ .1/ và .2/ ta suy ra
x1 C    C xn 1 C xn C xnC1 > n C 1:
Như vậy bổ đề đúng đến n C 1: Theo nguyên lý quy nạp toán học, bổ đề đúng với mọi n:
Quay trở lại bài toán, ta đặt
a1 an
x1 D p ; : : : ; x n D p ;
n
a1    an n
a1    an
thì
a1 an a1    an
x1    xn D p  p D D 1:
n
a1    an n
a1    an a1    an
Áp dụng bổ đề ta có
x1 C    C xn  n;
suy ra
a1 an
p C  C p  n;
n
a1    an n
a1    an
tương đương với
a1 C a2 C    C an p
 n a1    an :
n
Bất đẳng thức AM-GM được chứng minh.
Trong lời giải này, có hai ý tưởng chính, thứ nhất là bước đưa bài toán về bài toán với điều kiện
chuẩn hóa x1 x2    xn D 1 và thứ hai là bước sử dụng hằng số 1 để "dồn" xn ; xnC1 thành xn xnC1
(cụ thể là thay xn C xnC1 bằng xn xnC1 C 1/: Nhờ có bước thay thế này mà ta có thể áp dụng
được giả thiết quy nạp.
Ta sẽ tiếp tục phân tích ý tưởng chuẩn hóa ở bước 1 trong mục sau. Ở đây ta khai thác ý tưởng sử
dụng hằng số để so sánh các đại lượng không cùng bậc.
Bài toán 1. Cho x; y; z là các số thực dương. Chứng minh rằng ta có bất đẳng thức
x 2 C y 2 C z 2 C 2xyz C 1  2.xy C yz C zx/: .1:1/
Phân tích. Ở đây x 2 C y 2 C z 2 và xy C yz C zx là cùng bậc, có thể so sánh được, nhưng xyz
là đại lượng khác bậc. Ta muốn dùng hằng số 1 để so sánh xyz với các đại lượng vế trái, làm đơn
giản bớt thành phần này.
Lời giải. Trong 3 số x; y; z luôn có hai số cùng lớn hơn hay bằng 1 hoặc cùng nhỏ hơn hay
bằng 1: Giả sử đó là x; y thì ta có .x 1/.y 1/  0; suy ra xy C 1  x C y: Từ đó
2xyz C 2z  2xz C 2yz: .1:2/
Như vậy, để chứng minh .1:1/; ta chỉ cần chứng minh
x 2 C y 2 C z 2 C 1  2xy C 2z:
Nhưng bất đẳng thức cuối này là hiển nhiên vì nó tương đương với
.x y/2 C .z 1/2  0:
Bài toán được chứng minh.
8 Các phương pháp giải toán qua các kỳ thi Olympic

Bài toán 1 là một bổ đề có nhiều ứng dụng hiệu quả, đặc biệt là trong các bất đẳng thức không
thuần nhất chứa xyz: Bạn đọc có thể sử dụng bài toán 1 để giải 2 bài toán song sinh sau.

Bài tập 1 (Hello IMO, 2007). Cho x; y; z là các số thực dương. Chứng minh rằng

xyz C 2.x 2 C y 2 C z 2 / C 8  5.x C y C z/:

Bài tập 2 (Ninh Thuận, 2014). Cho x; y; z là các số thực dương. Chứng minh rằng ta có bất
đẳng thức
xyz C x 2 C y 2 C z 2 C 5  3.x C y C z/:

Hằng số 1 trong áp dụng nói trên, về nguyên tắc, có thể thay bằng hằng số bất kỳ. Điều quan
trọng trong việc chọn hằng số nào là nó phải liên quan đến các đại lượng ở hai vế và phải giúp ta
giải quyết được bài toán.

Bài toán 2. Cho a; b; c là ba số dương và đặt


1 1 1
x DaC ; y DbC ; z DcC :
b c a
Chứng minh rằng ta có bất đẳng thức

xy C yz C zx  2.x C y C z/: .2:1/

Phân tích. Trong ví dụ này, ta cũng dùng kỹ thuật chuồng và thỏ, nhưng vách ngăn là số 2:
Điều này có thể thấy rõ từ hai vế của bất đẳng thức cần chứng minh.

Lời giải. Trong 3 số x; y; z luôn có hai số cùng lớn hơn hay bằng 2 hoặc cùng nhỏ hơn hay
bằng 2: Giả sử đó là x; y thì ta có .x 2/.y 2/  0; suy ra

xy C 4  2.x C y/: .2:2/

Như vậy, để chứng minh .2:1/; ta chỉ cần chứng minh

yz C zx  2z C 4;

hay là
z.y C x 2/  4: .2:3/
Thật vậy, ta có
      r r
1 1 1 1 1 c a
z .x C y 2/ D c C aC CbC 2  cC aC 2 2 D 4:
a b c a c a c

Ở đây ta đã sử dụng đánh giá b C b1  2 và 2 lần dùng AM-GM ở cuối). Tức là .2:3/ đúng. Cộng
với .2:2/ vế theo vế, ta có điều phải chứng minh.

Bài toán 3 (Việt Nam MO, 1996). Cho x; y; z là các số thực dương thỏa mãn điều kiện
xy C yz C zx C xyz D 4: Chứng minh rằng ta luôn có

x C y C z  xy C yz C zx: .3:1/
Học một bài toán như thế nào? 9

Phân tích. Các đại lượng ở đây ngược chiều so với ví dụ 1; 2: Do đó ta không đi theo cách
chuồng và thỏ mà đi theo cách của Polya.

Lời giải. Giả sử x  y  z: Vì xy C yz C zx C xyz D 4 nên không thể tất cả các số đều > 1;
cũng không thể tất cả các số đều < 1: Từ đây suy ra x  1 và z  1:
Đặt s D x C z và p D xz thì ta có y.s C p/ D 4 p và ta cần chứng minh

s C y  sy C p:

Ta có

.s C p/.s C y sy p/ D s2 C sp C 4 p 4s C ps sp p2 D .s 2/2 C p.s p 1/:

Mặt khác thì


s p 1DxCz xy 1 D .x 1/.1 z/  0;
nên từ đây suy ra
.s C p/.s C y sy p/  0;
có nghĩa là s C y sy p  0: Ta có điều phải chứng minh.
Bài tập 3 (PTNK, 2014). Cho a; b; c là các số thực dương thỏa mãn điều kiện

.a C 1/.b C 1/.c C 1/ D 1 C 4abc:

Chứng minh rằng ta có bất đẳng thức

a C b C c  1 C abc:

2. Bài học từ Polya về chuẩn hóa trong chứng minh bất


đẳng thức
Trong các bài toán ở trên, ta thấy rằng sử dụng các hằng số một cách thích hợp, ta đã rút gọn
được các bất đẳng thức cần chứng minh để đưa về các bất đẳng thức đơn giản hơn. Với bất đẳng
thức thuần nhất thì không thể có những hằng số như thế (vì khi nhân tất cả các biến với cùng một
đại lượng thì bất đẳng thức không thay đổi, trong khi hằng số bị thay đổi). Những hằng số đó sẽ
xuất hiện nếu ta đưa ra các điều kiện chuẩn hóa.
Ngoài ra, chuẩn hóa còn có thể giúp chúng ta đơn giản các biểu thức cồng kềnh, đặc biệt là các
biểu thức chứa căn. Ta bắt đầu bằng một ví dụ kinh điển.
Cho a D .a1 ; a2 ; : : : ; an / là bộ gồm n số thực dương. Với số thực r; ta đặt
 1r
a1r C a2r C    C anr

Mr .a/ D ; (trung bình bậc r của bộ a):
n

Ta có bất đẳng thức trung bình lũy thừa nổi tiếng sau: Nếu r > s thì Mr .a/  Ms .a/: Rất thú vị
là bất đẳng thức cực mạnh này (từ đây suy ra AM-GM, Cauchy-Schwarz, bất đẳng thức trung
bình điều hòa) lại được chứng minh chỉ dùng BDT Bernoulli.
Dưới đây ta xét trường hợp chính của bất đẳng thức này.
10 Các phương pháp giải toán qua các kỳ thi Olympic

Bài toán 4. Nếu r > s > 0 thì


Mr .a/  Ms .a/:
Lời giải. Biểu thức của Mr .a/ và Ms .a/ là khá cồng kềnh và ta chưa biết sẽ xử lý thế nào. Sử
dụng ý tưởng từ chứng minh của G.Polya, ta giải bài toán "đã được chuẩn hóa sau".

Bổ đề 2. Cho r > s: Nếu a1s C a2s C    C ans D n thì


a1r C a2r C    C anr  n:
r
Thật vậy do r > s > 0 nên s
> 1: Áp dụng bất đẳng thức Bernoulli ta có
r r
air D 1 C ais 1 s  1 C ais 1 :

s
Cho i chạy từ 1 đến n rồi cộng lại vế theo vế, ta có điều phải chứng minh.
Bài toán 5. Hãy từ bổ đề suy ra kết quả bài toán 4:
Bài toán 6 (Saudi Arabia TST 2015). Cho x; y; z là các số thực dương thỏa mãn điều kiện
 
1 1 1
.x C y C z/ C C D 10:
x y z
Tìm giá trị lớn nhất, giá trị nhỏ nhất của biểu thức
 
2 2 2 1 1 1
P D .x C y C z / C 2C 2 :
x2 y z
Lời giải. Lời giải bài toán này có thể chia nhỏ thành các bước sau.
1: Dùng tư tưởng chuẩn hóa, đưa bài toán về bài toán tương đương (vì sao?) như sau: Cho
x; y; z là ba số dương thỏa mãn x C y C z D 10; và x1 C y1 C z1 D 1: Tìm giá trị nhỏ nhất
và giá trị lớn nhất của
 
2 2 2 1 1 1
P D .x C y C z / C 2C 2 :
x2 y z

2: Đặt t D xyz D xy C yz C zx: Tính P theo t:


3: Đánh giá x; y; z bằng phương pháp điều kiện có nghiệm của hệ phương trình.
4: Đánh giá t bằng khảo sát hàm số hoặc phương pháp sử dụng hằng số.
5: Đánh giá P:
Bạn đọc hãy dựa vào sơ đồ trên để tìm ra lời giải cụ thể.
Bài toán 7 (Việt Nam MO, 2004). Cho x; y; z > 0 thoả mãn .x C y C z/3 D 32xyz: Tìm
giá trị lớn nhất và giá trị nhỏ nhất của biểu thức
x4 C y4 C z4
P D :
.x C y C z/4
Bài toán 8 (British MO, 1986). Cho x; y; z là các số thực thoả mãn điều kiện x C y C z D 0
và x 2 C y 2 C z 2 D 6: Tìm giá trị lớn nhất và giá trị nhỏ nhất của
P D x 2 y C y 2 z C z 2 x:
Học một bài toán như thế nào? 11

3. Loạt bài toán liên quan đến bài Phổ Thông Năng Khiếu
2014 và Sóc Trăng 2014
Trong kỳ thi chọn đội tuyển của các trường và các tỉnh vừa qua, có hai bài toán sau đây:

Bài toán 9 (PTNK, 2014). Cho X D f1; 2; : : : ; 19g: Chứng minh rằng tồn tại tập hợp F
gồm ít nhất 2600 tập con 7 phần tử của X sao cho với mọi A khác B thuộc F thì jA \ Bj  5:

Bài toán 10 (Sóc Trăng, 2014). Một nhóm học sinh có n người. Họ tổ chức 10 cuộc gặp mặt,
mỗi cuộc gặp mặt có 20 học sinh tham dự. Hai học sinh bất kỳ tham gia chung không quá một
cuộc gặp. Tìm giá trị nhỏ nhất của n:

Bài toán 9 thuộc về dạng xây dựng ví dụ. Những bài này luôn rất khó và không thể làm thủ công
được. Lời giải bài này dựa vào ý tưởng sau.

Lời giải. Với mỗi k D 0; 1; 2; : : : ; 18; nếu đặt P .k/ D A  X j jAj D 7; S.A/  k mod 19;
(ở đây ta ký hiệu S.A/ là tổng các phần tử của A/ thì P .k/ thỏa mãn điều kiện nếu A khác
B thuộc P .k) thì jA \ Bj  5 (vì nếu jA \ Bj D 6 thì ta có A D fa1 ; a2 ; : : : ; a6 ; ag;
B D fa1 ; a2 ; : : : ; a6 ; bg và S.A/ S.B/ D a b ¤ 0 mod 19/: Mặt khác ta có
7
jP .0/j C jP .1/j C    C jP .18/j D C19 :

Từ đó suy ra tồn tại k sao cho


7
C19
jP .k/j  D 2652:
19
Bài toán được chứng minh.

Bài toán 10 thuộc dạng cực trị rời rạc, bao gồm cả đánh giá và xây dựng ví dụ. Và hóa ra, bước
đánh giá là khá đơn giản.

Lời giải. Ta lập danh sách của 10 chuyến đi. Cho chuyến đi đầu tiên, ta cần 20 học sinh. Cho
chuyến đi thứ hai, ta cần ít nhất 19 học sinh mới (tức là học sinh chưa từng đi chuyến trước, do
chỉ có nhiều nhất 1 học sinh đi chuyến trước). Cho chuyến đi thứ ba, ta cần ít nhất 18 học sinh
mới (do chỉ có nhiều nhất 2 học sinh đi các chuyến trước). Cứ như vậy đến chuyến thứ 10; ta cần
ít nhất 11 học sinh mới. Như vậy số học sinh không ít hơn

20 C 19 C 18 C    C 11 D 155:

Lời giải hoàn tất.

Cách 2. Ta mô hình hóa bài toán như sau: Cho X D f1; 2; : : : ; ng: Giả sử tồn tại 10 tập con
A1 ; A2 ; : : : ; A10 của X thỏa mãn các điều kiện:

i) jAi j D 20; với mọi i D 1; 2; : : : ; 10:


ˇ ˇ
ii) ˇAi \ Aj ˇ  1 với mọi 1  i < j  10:
12 Các phương pháp giải toán qua các kỳ thi Olympic

Tìm giá trị nhỏ nhất của n:


Áp dụng bất đẳng thức bao hàm và loại trừ (hay bất đẳng thức Bonferroni), ta có
ˇ 10 ˇ 10
ˇ[ ˇ X X
n  ˇ Ai ˇ  jAi j jAi \ Aj j  10  10 45  1 D 155:
ˇ ˇ
ˇ ˇ
i D1 i D1 1i <j 10

Từ đó cũng có kết luận như trên.


Cách 3. Cách 3 này phức tạp hơn cả, nhưng, như chúng ta sẽ thấy về sau, là cách có khả năng
mở rộng xa nhất và sâu nhất.
Trước tiên, ta mô hình hóa bài toán như ở cách 2: Ta lập bảng thành viên gồm 10 dòng và n cột,
dòng đại diện cho 10 chuyến đi (các tập Ai / và cột đại diện cho n học sinh (phần tử của X/:
1 2 3  n 1 n
A1
A2

A10

Ở dòng i cột j ta sẽ viết số 1 nếu j 2 Ai (học sinh j dự cuộc gặp mặt thứ i/: Trong trường hợp
ngược lại, ta viết số 0: Khi đó, tổng số các số 1 trên mỗi dòng, theo giả thiết, bằng 20: Do đó
tổng số các số 1 trong toàn bảng bằng 200:
Nhưng điều kiện jAi \ Aj j  1 được thể hiện như thế nào trên ngôn ngữ của bảng thành viên?
Đó là không có 4 số 1 tạo thành đỉnh của một hình chữ nhật. Nói cách khác, mỗi cặp số 1 trên
dòng sẽ xuất hiện không quá 1 lần trên 2 cột tương ứng và mỗi cặp số 1 trên cột sẽ xuất hiện trên
2 dòng tương ứng không quá 1 lần.
2
Ta thử dùng ý này để đánh giá n: Nếu đếm số cặp số 1 trên dòng thì trên mỗi dòng có C20 D 190
cặp số 1: Mặt khác, mỗi cặp cột chứa nhiều nhất 1 cặp số 1 trên dòng, do đó số cặp số 1 trên
dòng không lớn hơn Cn2 : Từ đây ta suy ra n.n 1/  3800: Từ đó n  63: Đánh giá này quá
không chặt. Ta cũng có thể nhận ra ngay điều này khi bắt tay xây dựng ví dụ dấu bằng xảy ra.
Ta đổi lại tính số cặp số 1 trên cột. Để làm điều này, ta gọi cj là số số 1 trên dòng j: Ta dễ thấy
c1 C c2 C    C cn bằng tổng số số 1 trong toàn bảng, do đó bằng 200: Ta đếm số cặp số 1 trên
c .c 1/
cột bằng 2 cách. Cách 1 là đếm theo cột. Cột thứ j có cj số 1 do đó có Cc2j D j 2j cặp số 1:
n
P cj .cj 1/
Như vậy tổng số cặp số 1 trên cột của toàn bảng bằng 2
:
j D1
2
Mặt khác, ta có C10 D 45 cặp dòng, mỗi cặp dòng chứa không quá 1 cặp số 1 trên cột. Do đó
toàn bảng có không quá 45 cặp số 1 trên cột. Từ đây ta suy ra bất đẳng thức
n
X cj .cj 1/
 45;
j D1
2

tương đương với


n
X n
X
cj2  cj C 90 D 290:
j D1 j D1
Học một bài toán như thế nào? 13

n
P
Làm thế nào để từ đây đánh giá n‹ Ở đây, do ta biết tổng cj nên ta có thể dùng bất đẳng thức
j D1
Cauchy-Schwarz để đánh giá
!2
n
P
n
cj
X j D1
290  cj2  ;
j D1
n

suy ra
40000
n D 137  9:
290
Suy ra tiếp n  138:
Theo như những lời giải ở trên thì ta đã biết n D 138 không phải là đáp số. Nhưng nếu ta chưa
biết đáp số 155 thì sao? Suy nghĩ một chút, ta thấy nếu n D 138 thì gần như dấu bằng đã xảy ra,
tức là các cj gần như bằng nhau. Nhưng không thể có 138 số nguyên không âm gần như bằng
nhau có tổng bằng 200 được.
Như vậy đánh giá bằng Cauchy-Schwarz là không chặt. Ta nhận thấy rằng do tổng các cj bằng
200 và có hơn 138 số như vậy nên cj sẽ bằng 1; hoặc 2: Như vậy nếu ta dùng bất đẳng thức (bất
đẳng thức này đúng với mọi cj nguyên)

.cj 1/.cj 2/  0;

thì có khả năng dấu bằng xảy ra với mọi j; tức là ta có một bất đẳng thức chặt.
Áp dụng bất đẳng thức này, ta có đánh giá
n
X n
X n
X
290  cj2  .3cj 2/ D3 3c 2n D 600 2n:
j D1 j D1 j D1

Suy ra 2n  310; tức là n  155:


Ta cũng đi đến kết quả như hai cách 1; 2 ở trên. Tuy cồng kềnh hơn nhưng là một mô hình có thể
mở rộng tốt hơn. Điều này sẽ được nhấn mạnh trong phần sau. Nhưng trước hết, thông tin cj D 1
hoặc 2 (cụ thể có 45 số bằng 2 và 110 số bằng 1/ giúp ta dễ dàng hơn trong việc xây dựng ví dụ
cho n D 155:

A1 D f1; 2; : : : ; 20g;
A2 D f1; 21; 22; : : : ; 39g;
A3 D f2; 21; 40; 41; : : : ; 57g;
A4 D f3; 22; 40; 58; : : : ; 74g;
A5 D f4; 23; 41; 58; 75; : : : ; 90g;
A6 D f5; 24; 42; 59; 75; 91; : : : ; 105g;
A7 D f6; 25; 43; 60; 76; 91; 106; : : : ; 119g;
A8 D f7; 26; 44; 61; 77; 92; 106; 120; : : : ; 133g;
A9 D f8; 27; 45; 62; 78; 93; 107; 120; 134; : : : ; 144g;
A10 D f9; 28; 46; 63; 79; 94; 108; 121; 134; 145; : : : ; 155g:
14 Các phương pháp giải toán qua các kỳ thi Olympic

Các bài toán 9; 10 đã được giải quyết trọn vẹn. Tuy nhiên, câu chuyện chỉ mới bắt đầu. Sau đây
là những bình luận tiếp theo về hai bài toán này.
7
Bình luận 1. Trong lời giải bài 9; ta sử dụng lý luận tổng các jP .k/j D C19 ; nên tồn tại k
C7
sao cho jP .k/j  1919 D 2652: Nhưng liệu ta có thể làm mạnh kết quả này bằng cách tính các
jP .k/j và chọn số lớn nhất?
Hóa ra là ta có thể chứng minh tất cả các jP .k/j đều bằng nhau và bằng 2652: Ta chứng minh
bằng phương pháp song ánh như sau.
Ta chứng minh mẫu là jP .1/j D jP .2/j: Bài toán không thay đổi khi ta thay X D f0; 1; 2; : : : ; 18g:
Giả sử A D fa1 ; a2 ; : : : ; a7 g là tập con 7 phần tử của X sao cho

a1 C a2 C    C a7  1 mod 19:

Ta đặt bi D 2ai mod 19 thì b1 ; b2 ; : : : ; b7 cũng thuộc X; đôi một phân biệt và

b1 C b2 C    C b7  1 mod 19:

Ánh xạ rõ ràng là song ánh, từ đây suy ra jP .1/j D jP .2/j: Một song ánh khác cũng có thể sử
dụng là bi D ai C k mod 19 với k được chọn sao cho 7k  1 mod 19:
Trong lời giải này, ta sử dụng tính chất 0; 1; 2; : : : ; 18 vừa đủ một hệ thặng dư đầy đủ mod19:
Nhưng nếu X D f0; 1; 2; : : : ; 18; 19g thì sao? Phương pháp song ánh như trên không còn khả
thi. Thế nhưng ta vẫn có cách.
Ta tiếp tục gọi P .k/ là họ các tập con 7 phần tử của X D f0; 1; : : : ; 18g sao cho tổng các
phần tử đồng dư k mod 19; còn Q.k/ là họ các tập con 6 phần tử của X D f0; 1; : : : ; 18g
sao cho tổng các phần tử đồng dư k mod 19: Lại gọi P  .k/ là họ các tập con 7 phần tử của
X D f0; 1; : : : ; 18; 19g sao cho tổng các phần tử đồng dư k mod 19: Thế thì vẫn bằng cách
chứng minh tương tự như ở trên, ta có tất cả các jP .k/j bằng nhau và tất cả các jQ.k/j bằng
nhau. Nhưng rõ ràng
jP  .k/j D jP .k/j C jQ.k/j;
nên ta tiếp tục vẫn có tất cả các jP  .k/j bằng nhau.
Bằng cách này, ta có thể tiếp tục chứng minh các jP .k/j vẫn bằng nhau với

X D f0; 1; 2; : : : ; 19; 20g:

Nhưng quá trình này có thể kéo dài tiếp được không? Thoạt nhìn ta nghĩ là có thể, nhưng suy nghĩ
một chút, ta thấy điều kiện cần để các jP .k/j bằng nhau là số tập con 7 phần tử của X phải chia
hết cho 19: Mà với jXj D 26 thì rõ ràng điều này không đúng. Vậy với X D f0; 1; 2; : : : ; 25g
thì các jP .k/j bằng bao nhiêu. Nếu làm thủ công tỷ mỉ từ dưới lên thì ta cũng có thể tính được
các jP .k/j nhưng cách thủ công cũng không thể đi xa hơn nữa. Ta đề xuất một phương pháp
tổng quát hơn để giải các bài toán tương tự, sử dụng số phức, cụ thể là căn của đơn vị.
Ta xét trường hợp X D f0; 1; : : : ; 25g và tập con 7 phần tử. Như thường lệ, gọi a là căn nguyên
thủy bậc 19 của 1: Khi đó
1 C a C a2 C    C a18 D 0;

x 19 1 D .x 1/.x a/    .x a18 /:
Học một bài toán như thế nào? 15

Xét đa thức
P .x/ D .x 1/.x a/.x a2 /    .x a25 /:
Ta tính hệ số của x19 trong P .x/ bằng hai cách. Một mặt, nếu khai triển P .x/ ra thì để được
ta cần lấy x từ 19 dấu ngoặc, còn 7 dấu ngoặc khác sẽ lấy các số có dạng ak với k thuộc
f0; 1; : : : ; 25g: Như thế, ta sẽ có tổng các số có dạng aS.A/ với A chạy qua tất cả các tập con 7
phần tử của X: Chú ý rằng aS.A/ chỉ phụ thuộc vào số dư khi chia s.A/ cho 19 (đó là lý do tại
sao ta lấy căn bậc 19 của đơn vị) nên từ đây dễ dàng suy ra tổng nói trên bằng

.jP .0/j C P .1/a C    C P .18/a18 /:

Mặt khác
P .x/ D .x 19 1/.x 1/.x a/    .x a6 /:
Suy ra hệ số của x 1 9 bằng 1  a  a2    a6 D a2 : Từ đây suy ra

jP .0/j C jP .1/ja C .jP .2/j 1/a2 C    C jP .18/ja18 D 0:

Điều này đúng với mọi a là nghiệm của phương trình 1 C x C x 2 C    C x 18 D 0: Suy ra đa
thức
jP .0/j C jP .1/jx C .jP .2/j 1/x 2 C    C jP .18/jx 18 ;
tỷ lệ với đa thức 1 C x C    C x 18 và vì thế

jP .0/j D jP .1/j D jP .2/j 1 D    D jP .18/j:


7
C19 1
Suy ra tất cả các jP .i/j; i ¤ 2; bằng nhau và bằng 19
: Riêng jP .2/j lớn hơn đúng 1 đơn vị!
Lời giải cho bài tính jP .k/j với X D f0; 1; 2; : : : ; 25g mà không dùng số phức như sau. Đặt

X1 D f0; 1; 2; : : : ; 18g; X2 D f19; 20; : : : ; 25g:

Đặt
P1 .k; s/ D fA  X1 j jAj D s; S.A/  k mod 19g:
Với B  X2 đặt

P .k; B/ D fA  Xj jAj D 7; A \ X2 D B; S.A/  k mod 19g:


S
Khi đó rõ ràng P .k/ D P .k; B/: Do đó, để so sánh các P .k/; ta so sánh các P .k; B/:
BX 2

Bổ đề 3. Với mọi 1  s  7 thì các jP1 .k; s/j bằng nhau với k D 0; 1; 2; : : : ; 18:
Chứng minh bổ đề này bằng phương pháp song ánh như ở trên.
Bổ đề 4. Với mọi B  X2 ; B ¤ X2 thì các jP .k; B/j bằng nhau với k D 0; 1; 2; : : : ; 18:
Thật vậy nếu A thuộc P .k; B/ thì A \ X2 D B; jAj D 7; do đó

jA \ X1 j D 7 jBj D s  1  S.A/ D k;

suy ra S.A \ X1 / D k S.B/ mod 19: Vậy

jP .k; B/j D jP1.k S.B/ mod 19; s/j:


16 Các phương pháp giải toán qua các kỳ thi Olympic

Theo bổ đầu tiên các số cuối bằng nhau với k D 0; 1; 2; : : : ; 18 nên các số ở trước cũng thế. Ta
có điều phải chứng minh.
Còn riêng trường hợp B D X2 thì jP .k; B/j D 0 với k ¤ 2 và bằng 1 với k D 2: Tổng hợp tất
cả lại ta có các jP .k/j sẽ bằng nhau với mọi k ¤ 2 và jP .2/j lớn hơn các jP .k/j khác đúng 1
đơn vị. (Một lời giải rất gọn cho thấy nếu dùng ký hiệu một cách bài bản có thể diễn đạt các lý
luận chính xác và ngắn gọn).
Bình luận 2. Ta sẽ thử lửa các phương pháp đã sử dụng trong lời giải bài toán 7 bằng cách thay
đổi điều kiện một chút.

Bài toán 11. Một nhóm học sinh có n người. Họ tổ chức 10 cuộc gặp mặt, mỗi cuộc gặp mặt
có 7 học sinh tham dự. Hai học sinh bất kỳ tham gia chung không quá một cuộc gặp. Tìm giá trị
nhỏ nhất của n:
Làm theo cách 1; ta sẽ có đánh giá: n  7 C 6 C    C 2 C 1 D 28:
Làm theo cách 2; ta sẽ có đánh giá: n  70 45 D 25:
n n
cj2  160; với Œ
P P
Làm theo cách 3; ta có đánh giá: cj D 70:
j D1 j D1

Bây giờ áp dụng bất đẳng thức Cauchy Schwarz thì ta được
!2
n
P
n
cj
X j D1 4900
160  cj2  D ;
j D1
n n

4900
suy ra n  160
D 30  625:
Ta có thể làm mạnh hơn bằng cách dùng bất đẳng thức cj2  cj 2‹
n
X n
X
160  cj2  3 cj 2n D 210 2n;
j D1 j D1

50
suy ra tiếp n  2
D 25:
Đánh giá này, trái với mong muốn, còn yếu hơn đánh giá trước đó! Tại sao vậy? Bởi vì ở đây ta
có khoảng 30 số có tổng bằng 70; như vậy các số này nằm giữa 2 và 3 chứ không phải 1 và 2: Vì
thế ta phải sử dụng bất đẳng thức .cj 2/.cj 3/  0; tức là cj2  5cj 6 thì sẽ chặt hơn. Áp
dụng đánh giá này, ta có
n
X n
X
160  cj2 5 cj 6n D 350 6n:
j D1 j D1

Nên n  190
6
D 31  667: Suy ra n  32: Đây là đánh giá tốt nhất. Việc còn lại của chúng ta là
xây dựng ví dụ dấu bằng xảy ra.
Hai bài toán 9 và 10 có thể phát biểu thành một dạng toán tổng quát như sau.

Bài toán 12. Cho X D f1; 2; : : : ; ng: Các tập con A1 ; A2; : : : ; Am của X có tính chất
Học một bài toán như thế nào? 17

i) jAi j D k với mọi i D 1; 2; : : : ; m:

ii) jAi \ Aj j  p với mọi i ¤ j:

Trong bài toán này có 4 tham số, là n; m; k; p: Nếu biết 3 tham số kia, có thể đánh giá các
tham số còn lại.

1) Cho biết m; k; p; tìm nmin : Bài Sóc Trăng cho m D 10; k D 20; p D 1:

2) Cho biết n; k; p; tìm mmax :

3) Cho biết n; k; p; chứng minh có thể tìm ít nhất m tập con thỏa mãn (bài Phổ Thông Năng
Khiếu là n D 19; k D 7; p D 1; m D 2600/:

4) Cho biết n; k; m: Tìm pmin :

Sau đây là một số bài tập tương tự chúng tôi dành cho bạn đọc ở dưới dạng bài tập.

Bài toán 13. Chúng ta có ba ý nhỏ:

a) Có 8 cái hộp, mỗi cái hộp có 6 viên bi thuộc một trong n màu. Biết rằng không có hai viên
bi cùng màu trong 1 hộp và không có hai màu xuất hiện trong quá 1 hộp. Tìm giá trị nhỏ
nhất của n:

b) Trong quốc hội có n thành viên. Người ta tổ chức 8 cuộc họp (tiếp nối nhau), mỗi cuộc
họp có 6 người tham dự. Biết rằng 2 thành viên bất kỳ không họp chung với nhau quá 1
lần, tìm giá trị nhỏ nhất của n:

c) Trong Duma quốc gia có 1600 đại biểu, lập thành 16000 ủy ban, mỗi ủy ban có 80 đại
biểu. Chứng minh rằng có ít nhất hai ủy ban có không dưới 4 thành viên chung.

Bài toán 14. Sau khi khai trương được đúng 10 ngày, một nhân viên thư viện cho biết:

1) Mỗi ngày có đúng 8 người đến đọc sách.

2) Không có người nào đến thư viện 1 ngày quá 1 lần.

3) Trong 2 ngày bất kỳ của 10 ngày đó thì có ít nhất là 15 người khác nhau cùng đến thư viện.

Căn cứ đồng thời cả 3 điều kiện mà nhân viên thư viện cung cấp hãy cho biết số người tối thiểu
đã đến thư viện trong 10 ngày nói trên là bao nhiêu?

Bài toán 15 (Mông Cổ, 2014). Một lớp học có n học sinh. Thầy giáo muốn tổ chức các tổ trực
gồm 3 học sinh sao cho hai học sinh bất kỳ cùng trực với nhau không quá 1 buổi. Gọi k là số
ngày nhiều nhất mà thầy giáo có thể phân công trực. Chứng minh rằng

n.n 3/ n.n 1/
k :
6 6
18 Các phương pháp giải toán qua các kỳ thi Olympic

4. Từ một bài toán Olympic của Nga


Cách đây 20 năm, khi bắt đầu đi dạy, để chuẩn bị bài vở, tôi tìm được bài toán thú vị sau trong
một tuyển tập đề Olympic của Nga:

Bài toán 16. Cho a < b < c là 3 nghiệm của phương trình x 3 3x C 1 D 0: Chứng minh
rằng ta có hệ thức
a2 c D b 2 a D c 2 b D 2:

Lời giải. Lời giải rất ngắn gọn (tôi đọc được trong sách của thầy Phan Huy Khải):
Đặt x D 2 cos t thay vào phương trình thì được

8 cos 3t 6 cos t D 1:

Áp dụng công thức quen thuộc cos 3t D 4 cos 3t 3 cos t; ta được cos 3t D 21 : Từ đây ta được
3 nghiệm là a D 2 cos 160ı ; b D 2 cos 80ı ; c D 2 cos 40ı : Từ đó các đẳng thức

a2 c D b2 a D c2 b D 2;

chẳng qua là công thức 2 cos 2x D .2 cos x/2 2 mà thôi.

Khi tôi và các học sinh thảo luận bài này, có hàng loạt các câu hỏi nảy sinh:

1) Tại sao ta lại nghĩ đến phép đặt x D 2 cos t‹

2) Tại sao ta có quyền đặt x D 2 cos t‹

3) Liệu có cách giải nào khác ngoài phép thế lượng giác độc đáo nói trên?

Trả lời cho câu .1/; lúc đó chúng tôi tạm chấp nhận hệ số 3x làm chúng ta liên tưởng đến công
thức cos 3t D 4 cos 3t 3 cos t: Ở phần sau chúng ta sẽ thấy rằng đây là một phép đặt mang tính
tổng quát, còn hệ số 2 được chọn cũng theo một quy tắc tổng quát để tạo ra biểu thức cos 3t:
Trả lời cho câu .2/; có thể chứng minh được rằng 2 < a; b; c < 2 bằng cách sử dụng tính chất
hàm liên tục: Đặt f .x/ D x 3 3x C 1 thì f . 2/ D 1; f .0/ D 1; f .1/ D 1; f .2/ D 3 do
đó ta có 2 < a < 0 < b < 1 < c < 2: Từ đó ta có thể đặt x D 2 cos t:
Về mặt logic, để tìm một nghiệm riêng nào đó, ta có thể đặt x D q cos t với q bất kỳ. Vấn đề là
.i / chọn q phù hợp để rút gọn được phương trình .i i/ với q như vậy, có bảo đảm là ta vét hết
nghiệm hay không. Ví dụ nếu trong phương trình x 3 3x C 1 D 0 mà ta đặt x D cos t thì nếu
có giải được, ta cũng chỉ có thể tìm được 1 nghiệm. Ngược lại, nếu ta đặt x D 2 cos t và thay vào
tìm ra 3 nghiệm thì không cần đặt ngược lại vấn đề về “tính pháp lý” của phép đặt này.
Trả lời cho câu hỏi .3/; chúng tôi đã tìm ra một cách tiếp cận khác như sau: Điều cần chứng
minh có thể viết lại thành

a D b2 2; b D c 2 2; c D a2 2: ./

Từ đây ta thấy rằng b 2 2; c 2 2; a2 2 là nghiệm của phương trình x 3 3x C 1 D 0: Bây


giờ, nếu ngược lại ta chứng minh được rằng b 2 2; c 2 2; a2 2 là nghiệm của phương trình
x 3 3x C 1 D 0 thì ta cũng sẽ dễ dàng suy ra được ./: Từ đó có sơ đồ chứng minh sau:
Học một bài toán như thế nào? 19

i) Chứng minh rằng a2 2; b 2 2; c 2 2 là 3 nghiệm của phương trình x 3 3x C 1 D 0:

ii) Sắp được thứ tự b 2 2 < c2 2 < a2 2:

iii) Suy ra ./:

Ta thầy .i i / là khá hiển nhiên từ đánh giá 2 < a < 0 < b < 1 < c < 2 và a C b C c D 0:
Vấn đề .i / cũng khá thú vị. Cách tiếp cận truyền thống và tự nhiên là dùng định lý Viet thuật và
đảo. Ta có theo định lý Viet thì

a C b C c D 0; ab C bc C ca D 3; abc D 1:

Từ đó suy ra
a2 C b 2 C c 2 D .a C b C c/2 2.ab C bc C ca/ D 6;
a2 b 2 C b 2 c 2 C c 2 a2 D .ab C bc C ca/2 2abc.a C b C c/ D 9:
Do đó a2 2 C b2 2 C c2 2 D 0; và

.a2 2/.b 2 2/ C .b 2 2/.c 2 2/ C .c 2 2/.a2 2/


2 2 2 2 2 2
Da b Cb c Cc a 4.a C b C c 2 / C 12
2 2

D 9 24 C 12 D 3;

.a2 2/.b 2 2/.c 2 2/ D a2 b 2 c 2 2.a2 b 2 C b 2 c 2 C c 2 a2 / C 4.a2 C b 2 C c 2 / 8


D 1 18 C 24 8 D 1:
Theo định lý Viet đảo thì a2 2; b 2 2; c 2 2 là ba nghiệm của phương trình x 3 3x C 1 D 0:
Cách tiếp cận thứ hai khá độc đáo là cách lập phương trình bậc 3 có nghiệm là a2 2; b 2 2;
c 2 2 mà không dùng đến định lý Viet.
Ta có a là nghiệm của phương trình x 3 3x C 1 D 0; do đó a3 3a D 1: Bình phương hai vế
của đẳng thức, ta được a6 6a4 C 9a2 D 1; hay là

.a2 /3 6.a2 /2 C 9a2 1 D 0:

Điều này cũng đúng cho b 2 ; c 2 : Suy ra a2 ; b 2 ; c 2 là nghiệm của phương trình

x3 6x 2 C 9x 1 D 0:

Từ đây ta tiếp tục suy ra a2 2; b 2 2; c 2 2 là 3 nghiệm của phương trình

.x C 2/3 6.x C 2/2 C 9.x C 2/ 1 D 0:

Khai triển phương trình cuối và rút gọn, ta được đúng phương trình x 3 3x C 1 D 0Š
Ý tưởng biến đổi phương trình độc đáo này tôi học được từ bài giảng của thầy Nguyễn Vũ Lương
trong đợt bồi dưỡng giáo viên chuyên toán trung học phổ thông vào tháng 12 năm 1996: Cũng
vào năm 1996; trong đợt dẫn hai bạn Lê Long Triều và Lê Quang Nẫm ra Hà Nội học và đợt
bồi dưỡng tháng 12; tôi đã học được rất nhiều điều từ các thầy Phan Đức Chính (ví dụ phép dồn
biến), Nguyễn Văn Mậu (điểm bất động trong phương trình hàm), Nguyễn Khắc Minh (phương
pháp song ánh), Nguyễn Minh Đức (phương trình hàm đa thức), Đặng Hùng Thắng (số phức
20 Các phương pháp giải toán qua các kỳ thi Olympic

trong các bài toán đếm). Những bài học vỡ lòng thường rất ấn tượng và sâu sắc. Và nếu tiếp tục
được nuôi dưỡng, phát triển sẽ trở thành những bửu bối cho mỗi chúng ta.
Quay trở lại với phép đặt x D 2 cos t; hóa ra đây là một cách tiếp cận tổng quát để giải phương
trình bậc 3 trong trường hợp phương trình có 3 nghiệm phân biệt.
Ta biết rằng phương trình bậc 3 dạng tổng quát ax 3 C bx 2 C cx C d D 0 bằng phép đặt
b
y D x C 3a có thể đưa về dạng y 3 C py C q D 0: Lại đặt y D u C v thì ta có

.u C v/3 C p.u C v/ C q D 0;

hay là
u3 C v 3 C q C .3uv C p/.u C v/ D 0:
p3
Nếu ta chọn u; v sao cho 3uv C p D 0 thì u3 C v 3 D q; u3 v 3 D 27
: Như vậy, nếu

0 q2 p3
 D C  0;
4 27
thì ta sẽ có
q p 0 p
r r
q
uD 3
C ;vD 3
0 :
2 2
Từ đây ta có công thức Cardano nổi tiếng
s r s r
3 q q2 p3 3 q q2 p3
yD C C C C :
2 4 27 2 4 27
0
Nhưng nếu  < 0 thì sao? Ta không thể kết luận là phương trình vô nghiệm được. Chẳng hạn
0
với phương trình x 3 3x C 1 D 0 ở trên thì  D 34 < 0 nhưng phương trình thì rõ ràng có 3
nghiệm. Học cách đặt ở trên, ta thử đặt x D m cos t và thay vào phương trình để được

m3  cos 3t C mp  cost C q D 0:

Ta muốn rằng m3  cos 3t C mp  cos t tỷ lệ với 4 cos 3t 3 cos t D cos 3t: Để có điều này, ta
cần chọn m sao cho
m3 4
D :
mp 3
q
4p 0
Tức là phải chọn m D 3
: Do  < 0 nên p < 0 nên điều này thực hiện được. Với giá trị m
như thế, ta viết lại phương trình dưới dạng

m3
cos 3t C q D 0:
4
4q ˇ
ˇ ˇ
Phương trình cuối cùng này có nghiệm khi và chỉ khi ˇ m3
 1: Điều này tương đương với
64p 3 0 0
16q 2  m6 ; tương đương với 16q 2 C 27  0 hay   0: Như vậy trong trường hợp   0
thì phép thế lượng giác giải quyết được phương trình bậc 3 rút gọn tổng quát x 3 C px C q D 0:
Tuy công thức Cardano và phép thế lượng giác nói trên cho phép giải quyết trọn vẹn phương trình
bậc 3 tổng quát nhưng trong thực tiễn những bài toán liên quan đến nghiệm, việc tìm nghiệm và
tính toán trực tiếp trên nghiệm là rất phức tạp và không cần thiết. Ta tránh làm như vậy mà thông
Học một bài toán như thế nào? 21

qua các phương pháp gián tiếp. Bài toán sau đây một đề thi của nước ta năm 2002 là một ví dụ
như vậy. Năm đó, rất nhiều thí sinh, do được trang bị “kỹ” về phương trình bậc 3 đã kỳ công biến
đổi và giải các phương trình bậc 3 tương ứng và cố gắng chứng minh đẳng thức đề bài nhưng
không thành công.

Bài toán 17 (Việt Nam MO, 2002). Cho hai đa thức:

P .x/ D 4x 3 2x 2 15x C 9; Q.x/ D 12x 3 C 6x 2 7x C 1:

a) Chứng minh rằng mỗi một đa thức đều có 3 nghiệm thực phân biệt

b) Gọi ˛ và ˇ là các nghiệm thực lớn nhất của P và Q tương ứng. Chứng minh rằng

˛ 2 C 3ˇ 2 D 4:

Bài toán 18 (PTNK). Tìm tất cả các cặp số thực p; q sao cho phương trình x 3 C px C q D 0
có 3 nghiệm phân biệt a < b < c thỏa mãn điều kiện

a2 c D b2 a D c2 b D 2:

Bài toán 19 (Canadian MO). Cho a; b; c là ba nghiệm của phương trình x 3 3x C 1 D 0:


Hãy tính giá trị của biểu thức sau đây
 3  3  3
1 a 1 b 1 c
P D C C :
1Ca 1Cb 1Cc

5. Tiếp nối câu chuyện về phương trình x3 3x C 1 D 0


Câu chuyện về phương trình x 3 3x C 1 D 0 hóa ra chưa dừng lại. Cũng vào khoảng những
năm 1996; 1997; tôi mua được một cuốn bài tập đại số của Pháp, trong đó có bài toán sau:

Bài toán 20. Cho a < b < c là 3 nghiệm của phương trình x 3 3x C 1 D 0: Hãy tính giá trị
của biểu thức
a b c
T D C C :
b c a
Cái thú vị của bài toán này là, hoàn toàn khác với bài toán 19 hay những bài toán tương tự liên
quan đến biểu thức đối xứng của ba nghiệm, biểu thức T không đối xứng nên không thể trực tiếp
tính T mà chỉ sử dụng định lý Viet.

Lời giải. Lúc đó tôi đã biết bài toán 16 nên tôi có thể giải được khá dễ dàng

a b c b 2 2 c 2 2 a2 2
T D C C D C C
b c a b c a
2.ab C bc C ca/ 2. 3/
D .a C b C c/ D0 D 6:
abc . 1/
22 Các phương pháp giải toán qua các kỳ thi Olympic

Tuy nhiên đây hoàn toàn là một lời giải thiếu tự nhiên, nhờ kiến thức biết trước chứ không phải
do suy luận mà có. Vậy lời giải tự nhiên của bài toán này như thế nào. Tôi đọc được trong sách ý
tưởng tuyệt vời sau:
Đặt S D ab C bc C ac ; thì S C T và S  T là các biểu thức đối xứng đối với a; b; c nên ta có
thể dễ dàng tính được. Cụ thể (bài tập đơn giản này xin dành cho bạn đọc) S C T D 3 và
S  T D 18: Từ đây, áp dụng định lý Viet đảo, ta suy ra fS; T g D f 6; 3g: Cuối cùng dùng
đánh giá 2 < a < 0 < b < 1 < c < 2 và a D .b C c/ ta dễ dàng suy ra T < 0: Từ đó
T D 6:
Ý tưởng “đối xứng hóa” quá đẹp đẽ! Học được cách tiếp cận này, tôi lập tức áp dụng thành công
cho bài toán sau, mà trước đó tôi đã phải làm khá vất vả bằng phương pháp tham số hóa lượng
giác (cũng chính là bài tập 8 từ mục 2/:
Bài toán 21 (British MO, 1986). Cho x; y; z là các số thực thỏa mãn điều kiện x Cy Cz D 0;
x 2 C y 2 C z 2 D 6: Tìm giá trị lớn nhất và nhỏ nhất của biểu thức
P D x 2 y C y 2 z C z 2 x:
Lời giải. Theo cách tiếp cận ở mục 2; ta có
.x C y C z/2 .x 2 C y 2 C z 2 /
xy C yz C zx D D 3:
2
Do đó nếu đặt t D xyz thì mọi biểu thức đối xứng của x; y; z đều có thể biểu diễn được qua t:
Từ đó ta đánh giá t rồi đánh giá biểu thức đó như hàm số của t là xong.
Tuy nhiên, P không là biểu thức đối xứng đối với x; y; z nên sơ đồ này chưa áp dụng được. Áp
dụng ý tưởng đối xứng hóa, ta xét Q D xy 2 C yz 2 C zx 2 thì được
P C Q D xy.x C y/ C yz.y C z/ C zx.z C x/ D 3xyz D 3t;

P  Q D .x 2 y C y 2 z C z 2 x/.xy 2 C yz 2 C zx 2 /
D x 3 y 3 C y 3 z 3 C z 3 x 3 C 3x 2 y 2 z 2 C xyz.x 3 C y 3 C z 3 /
D .xy C yz C zx/3 3.xy C yz/.yz C zx/.zx C xy/ C 6t 2
D 9t 2 27:
Từ đây ta được p
27.4 t 2 /
3t ˙
P D :
2
Cũng từ đây ta suy ra điều kiện của t là 2  t  2: Để tìm giá trị lớn nhất, dĩ nhiên ta sẽ xét
trường hợp p
3t C 27.4 t 2 /
P D :
2
Áp dụng bất đẳng thức Cauchy-Schwarz, ta có
4P 2  .9 C 27/.t 2 C 4 t 2 / D 144:
Từ đó suy ra P  6: Dấu bằng xảy ra, chẳng hạn khi t D 1: Vậy giá trị lớn nhất của P là 6;
đạt được, chẳng hạn khi t D 1 (và khi đó có thể tính ra được x; y; z là hoán vị nào đó của 3
nghiệm a; b; c của phương trình x 3 3x C 1 D 0/: Tương tự, giá trị nhỏ nhất của P là 6: Và
phương trình x 3 3x C 1 D 0 lần nữa lại xuất hiện!
Học một bài toán như thế nào? 23

Bài toán 22 (Junior Turkish MO, 2013). Tìm giá trị lớn nhất của j.x y/.y z/.z x/j với
x; y; z là các số thực thỏa mãn điều kiện x C y C z D 0 và x 2 C y 2 C z 2 D 6:

Bài toán 23 (GGTH lần 1, 2010).


x2 1
a) Chứng minh rằng đồ thị hàm số y D 2
3x x
; có 3 điểm cực trị.

b) Tính bán kính đường tròn ngoại tiếp và diện tích tam giác có đỉnh tại ba điểm cực trị trên.

Tài liệu tham khảo


[1] G.Polya, Giải Một Bài Toán Như Thế Nào, Nhà xuất bản Giáo dục.

[2] G. H. Hardy, J. E. Littlewood, G. Polya, Bất Đẳng Thức, Nhà xuất bản Đại học và Trung học
chuyên nghiệp, 1981.

[3] Arthur Engel, Problem-Solving Strategies, Springer, 1998.

[4] Loren C. Larson, Problem-Solving Through Problems, Springer, 1983.

[5] A.Kanel-Belov, A.Kovaldzi, Giải Bài Toán Không Mẫu Mực Như Thế Nào?, MCCME, 2001.
24 Các phương pháp giải toán qua các kỳ thi Olympic
MỘT TÍNH CHẤT THÚ VỊ CỦA TAM THỨC
BẬC HAI VÀ NHỊ THỨC BẬC NHẤT
Võ Quốc Bá Cẩn
(Archimedes Academy, Hà Nội)

Tam thức bậc hai và nhị thức bậc nhất là hai phần kiến thức quan trọng và gần gũi với học sinh
phổ thông khi học môn Toán. Những tính chất đẹp và đặc trưng của chúng đã được sử dụng rất
hiệu quả trong nhiều lĩnh vực khác nhau của Toán sơ cấp.
Phần lớn các tính chất đó đã được giới thiệu trong nhiều bài viết khác nhau trên các tạp chí như
Toán Học Tuổi Trẻ, Toán Tuổi Thơ, : : :
Ở đây, trong bài viết này, chúng tôi xin được giới thiệu cùng các bạn một tính chất khá đặc biệt
của tam thức bậc hai và nhị thức bậc nhất mà từ tính chất này với một cách sử dụng khéo léo, ta
có thể giải được các bài bất đẳng thức thi Olympic Toán Quốc tế khá dễ dàng.
Tính chất đó chính là:

Định lý 1. Xét hàm số f .x/ D ax 2 C bx C c trên miền D D Œx1 ; x2 : Đặt:

m D min ff .x1 /; f .x2 /g


M D max ff .x1 /; f .x2 /g :
Khi đó, ta có các kết quả sau:

a) Nếu a D 0 thì m  f .x/  M:

b) Nếu a > 0 thì f .x/  M:

c) Nếu a < 0 thì f .x/  m:

Chứng minh kết quả này khá dễ dàng, xin được dành cho bạn đọc. Ở đây, ta chỉ bàn về ý nghĩa
và ứng dụng của nó. Có thể thấy đây là một kết quả khá quan trọng có thể được ứng dụng rộng
rãi trong lĩnh vực bất đẳng thức và cực trị. Nó chỉ ra rằng:

 Với các nhị thức bậc nhất: Hai biên x1 ; x2 của x cũng chính là hai vị trí mà hàm số sẽ
đạt được giá trị lớn nhất .max/ và giá trị nhỏ nhất .min/ của nó.

 Với các tam thức bậc hai có hệ số cao nhất dương .a > 0/: Giá trị lớn nhất của hàm số
sẽ đạt được tại x1 hoặc x2 :

 Ngược lại, với các tam thức bậc hai có hệ số cao nhất âm .a < 0/: Giá trị nhỏ nhất của
hàm số sẽ đạt được tại x1 hoặc x2 :

25
26 Các phương pháp giải toán qua các kỳ thi Olympic

Cho nên, tùy theo trường hợp cụ thể, ta chỉ cần xét biên của biến là có thể tìm được cực trị (ứng
với từng trường hợp) của hàm f .x/:
Và ta cũng có thể tận dụng điều này trong các bài toán chứng minh bất đẳng thức. Chẳng hạn, ta
đang có hàm f .x/ với a > 0 và đề bài yêu cầu chứng minh f .x/  K với mọi x 2 Œx1 ; x2 :
Theo kết quả trên, ta chỉ cần xét bất đẳng thức này tại x D x1 và x D x2 là đủ. Nếu nó đúng tại
hai trường hợp này thì cũng có nghĩa là bài toán được chứng minh xong.
Tính chất này cũng được ứng dụng khá nhiều trong phương pháp dồn biến để chứng minh bất
đẳng thức, một phương pháp mà hiện nay đã không còn xa lạ với nhiều bạn đọc yêu Toán. Bây
giờ, chúng ta sẽ cùng đến với một số ví dụ để minh họa cho tính chất đẹp này.
Ví dụ 1. Cho a; b; c 2 Œ1; 2: Chứng minh rằng
 
1 1 1
.a C b C c/ C C  10:
a b c
Lời giải. Quy đồng và khử mẫu, ta thấy bất đẳng thức cần chứng minh tương đương với
f .a/ D .a C b C c/.ab C bc C ca/ 10abc  0:
Cố định b; c: Ta thấy f .a/ là tam thức bậc hai của a với hệ số cao nhất dương. Do đó, ta chỉ cần
xét a D 1 và a D 2 là đủ. Nếu bất đẳng thức đúng trong hai trường hợp này thì cũng sẽ đúng
trong mọi trường hợp. Tương tự, ta cũng chỉ cần xét b; c 2 f1; 2g: Từ đó, do tính đối xứng, ta
đưa được bài toán về xét tại bốn trường hợp sau:
.a; b; c/ D .1; 1; 1/; .1; 1; 2/; .1; 2; 2/; .2; 2; 2/:
Bằng tính toán trực tiếp, ta thấy cả bốn trường hợp đều cho kết quả đúng. Từ đó, ta có điều phải
chứng minh. Một điều thú vị là hai trong bốn trường hợp trên cho ta dấu bằng của bài toán.
Ví dụ 2. Cho x1 ; x2 ; : : : ; xn 2 Œ0; 1: Chứng minh bất đẳng thức sau
.1 C x1 C x2 C    C xn /2  4.x12 C x22 C    C xn2 /:
Lời giải. Cố định x2 ; : : : ; xn : Đặt:
f .x1 / D 4.x12 C    C xn2 / .x1 C    C xn C 1/2 :
Dễ thấy f .x1 / là tam thức bậc hai của x1 với hệ số cao nhất dương. Và như vậy, f .x1 / đạt giá trị
lớn nhất tại x1 D 0 hoặc x1 D 1; suy ra ta chỉ cần kiểm tra tính đúng đắn của hai bất đẳng thức,
thu được từ bất đẳng thức ban đầu với x1 D 0 và x1 D 1: Lặp lại các lý luận này với mỗi một
trong hai bất đẳng thức thu được, ta nhận được kết quả tương tự: chỉ cần kiểm tra tính đúng đắn
của mỗi một trong hai bất đẳng thức tại x2 D 0 và x2 D 1:
Tiếp tục lý luận tương tự, ta thu được là chỉ cần kiểm tra tính đúng đắn của tất cả 2n bất đẳng
thức, thu được từ bất đẳng thức ban đầu với một phần (có thể là rỗng) các biến số bằng 0 và phần
còn lại bằng 1: Do tính đối xứng của bất đẳng thức ban đầu, có thể xét:
(
x1 D    D xk D 1
.0  k  n/:
xkC1 D    D xn D 0

Với các giá trị như vậy của biến số, bất đẳng thức ban đầu có dạng
.k C 1/2  4k:
Đây là một kết quả hiển nhiên, bởi vì nó tương đương với .k 1/2  0:
Một tính chất thú vị của tam thức bậc hai và nhị thứ bậc nhất 27

Ví dụ 3 (THTT). Cho a; b; c 2 Œ1; 2: Chứng minh rằng

a3 C b 3 C c 3  5abc:

Lời giải. Do tính đối xứng nên không mất tính tổng quát, ta có thể giả sử a  b  c; thế thì từ
giả thiết, ta có 2c  a  b  c: Suy ra

a3 C b 3 C c 3 5abc  2ca2 C b 3 C c 3 5abc:

Từ đây, ta đưa được bài toán về chứng minh

f .a/ D 2ca2 C b 3 C c 3 5abc  0:

Cố định b; c: Ta thấy f .a/ là một tam thức bậc hai với hệ số cao nhất dương và a 2 Œb; 2c:
Theo kết quả trên, ta chỉ cần chứng minh f .b/  0 và f .2c/  0 là đủ. Thật vậy, ta có

f .b/ D b 2 .b 2c/ C c.c 2 b2/  0

f .2c/ D b 3 C 9c 3 10bc 2 D .b c/.b 2 C bc 9c 2 /


 .b c/.4c 2 C 2c 2 9c 2 / D 3c 2 .b c/  0:

Ví dụ 4 (IMO Shortlist, 1993). Cho a; b; c; d là các số thực không âm thỏa mãn điều kiện
a C b C c C d D 1: Chứng minh rằng
1 176
abc C bcd C cda C dab  C abcd:
27 27
Lời giải. Cố định c; d: Khi đó, từ giả thiết suy ra hai tổng c C d và a C b cũng được cố định.
2
Đặt x D ab; ta có 0  x  .aCb/
4
và bất đẳng thức cần chứng minh trở thành
 
176 1
f .x/ D c C d cd x C cd.a C b/  0:
27 27

Dễ thấy f .x/ là một nhị thức bậc nhất nên để chứng minh bất đẳng thức trên, ta chỉ cần xét
2
x D 0 hoặc x D .aCb/
4
: Một cách tương đương, ở bất đẳng thức ban đầu, ta chỉ cần xét ab D 0
.aCb/2
(ứng với x D 0) hoặc a D b (ứng với x D 4
).
Lý luận tương tự, ta cũng chỉ cần xét cd D 0 hoặc c D d: Từ đó, với để ý ở tính đối xứng của
bất đẳng thức đã cho, ta đưa được bài toán về xét một trong hai trường hợp sau:
 Trường hợp 1: Có một số bằng 0: Không mất tính tổng quát, ta có thể giả sử d D 0: Khi
đó, theo yêu cầu của đề bài, ta cần chứng minh
1
abc  :
27
Kết quả này hiển nhiên đúng theo AM-GM:

aCbCc 3
 
1
abc  D :
3 27
28 Các phương pháp giải toán qua các kỳ thi Olympic

1
 Trường hợp 2: a D b và c D d: Lúc này ta có a C c D 2
và bất đẳng thức cần chứng
minh có thể được viết lại dưới dạng:
1 176 2 2
2ac.a C c/  C a c ;
27 27
hay
.1 11ac/  0:
16ac/.1
2
Kết quả này hiển nhiên đúng do ta có ac  aCc
2
1
D 16 :

Ví dụ 5 (IMO, 1983). Cho a; b; c là độ dài ba cạnh của một tam giác. Chứng minh rằng

a2 b.a b/ C b 2 c.b c/ C c 2 a.c a/  0:

Lời giải. Ta viết lại bất đẳng thức dưới dạng

a3 b C b 3 c C c 3 a  a2 b 2 C b 2 c 2 C c 2 a2 :

Không mất tính tổng quát, ta có thể giả sử a  c  b: (Về nguyên tắc, đây là bất đẳng thức hoán
vị nên ta phải xét cả trường hợp a  b  c nữa mới đầy đủ. Tuy nhiên, ở đây ta lại có thể lượt
bớt trường hợp này. Bạn đọc hãy tự giải thích vì sao nhé!)
Sử dụng bất đẳng thức AM-GM, ta có a2  2ac c 2 : Do đó, ta chỉ cần chứng minh

ab.2ac c 2 / C b 3 c C c 3 a  a2 b 2 C b 2 c 2 C c 2 a2 ;

hay
f .a/ D .b c/2 a2 C .c b/ac 2 C b 2 c.b c/  0:
Do hệ số cao nhất của f .a/ không dương và a 2 Œc; b C c nên ta chỉ cần chứng minh f .c/  0
và f .b C c/  0 là đủ. Thật vậy, ta có

f .c/ D bc.b c/2  0


f .b C c/ D b 3 .c b/  0:

Ví dụ 6 (Moldova TST, 2006). Cho a; b; c là ba cạnh của một tam giác. Chứng minh rằng
 
2 b
c  a 
a 1 C b2 1 C c2 1  0:
c a b

Lời giải. Cũng giống như ví dụ trên, ở bài toán này ta chỉ cần xét a  c  b là đủ (và bạn đọc
hãy tự giải thích vì sao). Lúc này, sử dụng bất đẳng thức AM-GM, ta có
c a
2 :
a c
Do đó, ta chỉ cần chứng minh được
 
2 b a 2 a
  
2
a 1 Cb 1 Cc 1  0;
c c b
Một tính chất thú vị của tam thức bậc hai và nhị thứ bậc nhất 29

hay
c2 b2
   
b 2
f .a/ D 1 a C a C b2 c 2  0:
c b c
Do f .a/ có hệ số cao nhất không dương và a 2 Œc; b C c nên ta chỉ cần chứng minh f .c/  0
và f .b C c/  0: Thật vậy, ta có

c.b c/2
f .c/ D 0
b

.b C c/.b c/2
f .b C c/ D  0:
b
Ví dụ 7 (VMO, 2012). Cho các cấp số cộng .an /; .bn / và số nguyên m > 2: Xét m tam thức
bậc hai P1 .x/; P2 .x/; : : : ; Pm .x/ với:

Pk .x/ D x 2 C ak x C bk ; k D 1; 2; : : : ; m:

Chứng minh rằng nếu hai tam thức P1 .x/ và Pm .x/ đều không có nghiệm thực thì tất cả các đa
thức còn lại cũng không có nghiệm thực.

Lời giải. Gọi d1 ; d2 lần lượt là các công sai của hai cấp số cộng .an / và .bn /: Khi đó, theo tính
chất của cấp số cộng, ta có
ak D a1 C .k 1/d1

bk D b1 C .k 1/d2
với mọi k D 1; 2; : : : ; m: Theo yêu cầu của đề bài, ta cần chứng minh

ak2 4bk < 0

với mọi k D 1; 2; : : : ; m; hay

f .k/ D Œa1 C .k 1/d1 2 4 Œb1 C .k 1/d2  < 0:

Do f .k/ là tam thức bậc hai của k với hệ số cao nhất  0 và 1  k  m nên để chứng minh bất
đẳng thức trên, ta chỉ cần chứng minh f .1/ < 0 và f .m/ < 0: Thật vậy, ta có

f .1/ D a12 4b1 < 0


f .m/ D Œa1 C .m 1/d1 2 4 Œb1 C .m 1/d2 
2
D am 4bm < 0;
do giả thiết P1 .x/ và Pm .x/ cùng vô nghiệm.

Ví dụ 8 (VMO, 1996). Cho a; b; c là các số không âm thỏa mãn ab C bc C ca C abc D 4:


Chứng minh rằng
a C b C c  ab C bc C ca:
30 Các phương pháp giải toán qua các kỳ thi Olympic

4 ab
Lời giải. Từ giả thiết, ta suy ra ab C bc C ca D 4 abc và c D aCbCab
: Do đó, bất đẳng thức
cần chứng minh có thể được viết lại thành

.ab C 1/.4 ab/


aCbC  4:
a C b C ab
n o
S2 S2
Đặt S D a C b và P D ab thì ta có 0  P  min 4; 4 (P  4
là kết quả quen thuộc, còn
4 ab
P  4 được suy ra từ c D aCbCab
và c  0). Bất đẳng thức trên trở thành

.P C 1/.4 P /
SC  4;
S CP
hay tương đương
f .P / D .P C 1/.4 P / C .S 4/.S C P /  0:
n h oi
S2
Cố định S: Ta có f .P / là một tam thức bậc hai với hệ số cao nhất âm và P 2 0; min 4; 4
:
 n 2
o
Do đó, ta chỉ cần chứng minh f .0/  0 và f min 4; S4  0: Dễ dàng tính được

f .0/ D .S 2/2  0:
 n o
S2
Còn với f min 4; 4
; ta xét như sau:

 Nếu S  4 thì ta có

S2
  
f min 4; D f .4/ D S.S 4/  0:
4

 Nếu S  4 thì ta có

S2 2/2
 2
S 2 /.S
  
S .16
f min 4; Df D  0:
4 4 4

Lời kết. Các bạn thấy đấy, có những cái thật sự đơn giản nhưng nếu ta biết áp dụng chúng một
cách sáng tạo và khéo léo thì có thể biến những cái đơn giản đó thành một phương pháp, một kỹ
thuật mới giúp ta có thể giải được các bài toán khó mà trước mắt là phục vụ cho các kỳ thi học
sinh giỏi. Bài viết trên vẫn còn nhiều thiếu sót và cần được hoàn thiện thêm, chúng tôi rất mong
nhận được sự trao đổi góp ý cùng các bạn đọc gần xa! Cuối cùng, để kết thúc bài viết này, xin
được nêu lên một số bài tập mà ta có thể áp dụng tính chất đẹp này để giải:

Bài tập 4. Cho a; b; c; d 2 Œ0; 1: Chứng minh rằng


8
a2 .b d / C b 2 .c a/ C c 2 .d b/ C d 2 .a c/  :
27
Bài tập 5. Cho a; b; c là độ dài ba cạnh của một tam giác. Chứng minh rằng
   
a b c b c a
3 C C 1 2 C C :
b c a a b c
Một tính chất thú vị của tam thức bậc hai và nhị thứ bậc nhất 31

Bài tập 6 (AMM). Cho a; b; c là độ dài ba cạnh của một tam giác. Chứng minh rằng
 2
b2 c2
  
a 2 2 2 1 1 1
3 2 C 2 C 2  .a C b C c / 2 C 2 C 2 :
b c a a b c

Bài tập 7. Cho a; b; c; d là các số không âm có tổng bằng 4: Chứng minh rằng

.1 C 3a/.1 C 3b/.1 C 3c/.1 C 3d /  125 C 131abcd:

Bài tập 8. Cho a; b; c; d là các số không âm có tổng bằng 4: Chứng minh rằng

3.a2 C b 2 C c 2 C d 2 / C 4abcd  16:

Bài tập 9. Chứng minh rằng với mọi a; b; c; d  0; ta đều có bất đẳng thức sau

a4 C b 4 C c 4 C d 4 C 2abcd 
 a2 b 2 C a2 c 2 C a2 d 2 C b 2 c 2 C c 2 d 2 :

Bài tập 10 (Mongolia TST, 2008). Tìm hằng số k lớn nhất sao cho bất đẳng thức sau đúng
với mọi số thực không âm x; y; z:

x 3 C y 3 C z 3 C k.xy 2 C yz 2 C zx 2 /  .k C 1/.x 2 y C y 2 z C z 2 x/:

Bài tập 11 (Vietnam TST, 2009). Tìm tập hợp tất cả các số thực k sao cho bất đẳng thức sau
đúng với mọi số thực dương a; b; c:
     3
a b c 1
Ck Ck Ck  Ck :
bCc cCa aCb 2
32 Các phương pháp giải toán qua các kỳ thi Olympic
MỘT SỐ VẤN ĐỀ VỀ BẤT ĐẲNG THỨC
BẬC BỐN BA BIẾN
Nguyễn Văn Huyện
(Trường ĐH Giao Thông Vận Tải, thành phố Hồ Chí Minh)
Tống Hữu Nhân
(Trường ĐH Y Dược, thành phố Hồ Chí Minh)

1. Mở đầu
Trên tạp chí IJPAM 1 nhóm tác giả Vasile Cirtoaje 2 và Võ Quốc Bá Cẩn 3 có đề xuất định lý:

Định lý 1. Với a; b; c là ba số thực, xét đa thức


X X X X X
F .a; b; c/ D a4 C A b 2 c 2 C Babc aCC b3c C D bc 3 :

Nếu 1 C A C B C C C D D 0; bất đẳng thức F .a; b; c/  0 sẽ đúng khi và chỉ khi

3.1 C A/  C 2 C CD C D 2 :

Với định lý này lớp các bài toán bậc 4 ba biến số đã được giải quyết triệt để. Tuy nhiên việc
chứng minh và áp dụng nó trong các kỳ thi không phải là điều dễ dàng. Theo quan điểm cá nhân
định lý này giống như một tiêu chuẩn trong việc sáng tạo ra các bất đẳng thức mới hơn là một
phương pháp để giải quyết các bài toán. Điều mà chúng ta quan tâm không chỉ là những định lý
tổng quát mà còn là những công cụ không quá quá phức tạp để sử dụng trong phòng thi. Vì thế,
trong bài viết nhỏ này, chúng tôi xin chia sẻ một vài kỹ thuật nhỏ khá hiệu quả trong việc xử lý
các bất đẳng thức bậc 4 ba biến.

2. Các bài toán đối xứng


Năm 2011 trên diễn đàn toán học artofproblemsolving.com thành viên mudok có đề
xuất bài toán sau đây:

Bài toán 1. Cho các số thực a; b; c thỏa mãn a C b C c D 0: Chứng minh rằng

.ab C bc C ca/2 C 9abc  3.ab C bc C ca/: .2:1:1/


1
International Journal of Pure and Applied Mathematics
2
Trường đại học Department of Automatic Control and Computers, Ploiesti, Romania
3
Hà Nội

33
34 Các phương pháp giải toán qua các kỳ thi Olympic

Lời giải. Cách tự nhiên nhất là ta sẽ rút c D a b rồi thay vào (2.1.1), và được
2
ab .a C b/2 9ab.a C b/  3 ab .a C b/2 ;
  

.a2 C ab C b 2 /2 C 3.a2 C ab C b 2 /  9ab.a C b/:


Quan sát một chút ta thấy bất đẳng thức lúc này chứa ba đại lượng a2 C ab C b 2 ; ab; a C b và
chúng có mối liên hệ với nhau thông qua đánh giá
3
a2 C ab C b 2  .a C b/2  3ab;
4
suy ra
9
.a2 C ab C b 2 /2 C 3.a2 C ab C b 2 /  .a C b/4 C 9ab:
16
Như vậy ta chỉ cần chứng minh
.a C b/4
C ab  9ab.a C b/:
16
Rất tiếc bất đẳng thức này không đúng (có thể kiểm tra với a D 1; b D 1). Do đó ta sẽ đổi lại
đánh giá theo kiểu
3
.a2 C ab C b 2 /2 C 3.a2 C ab C b 2 /  .3ab/2 C 3  .a C b/2
4 .2:1:2/
9
D 9a2 b 2 C .a C b/2 ;
4
và quy bài toán về chứng minh
.a C b/2
a2 b 2 C  ab.a C b/:
4
Bất đẳng thức này đúng theo bất đẳng thức AM-GM, nhưng có một lỗi trong đánh giá (2.1.2) đó
là .a2 C ab C b 2 /2  .3ab/2 chỉ đúng khi ab  0; nhưng ta có thể khắc phục lỗi này bằng kỹ
thuật sau.
Do ab  bc  ca D a2 b 2 c 2  0 nên trong ba số ab; bc; ca sẽ có ít nhất một số không âm, giả sử
ab  0: Với những lập luận trên, áp dụng bất đẳng thức AM-GM, ta có
3
a2 C ab C b 2  .a C b/2  3ab  0: .2:1:3/
4
Suy ra
9.a C b/2
.a2 C ab C b 2 /2 C 3.a2 C ab C b 2 /  9a2 b 2 C :
4
Ta sẽ chứng minh
.a C b/2
a2 b 2 C  ab.a C b/:
4
Cũng theo bất đẳng thức AM-GM thì
r
2
.a C b/ .a C b/2
a2 b 2 C  2 a2 b 2  D ab ja C bj  ab.a C b/: .2:1:4/
4 4
Một số vấn đề bất đẳng thức bậc bốn ba biến 35

Đẳng thức xảy ra khi (2.1.3) và (2.1.4) trở thành đẳng thức, tức
8
<a D b
2
: a2 b 2 D .a C b/
4
Giải hệ này ta được a D b D c D 0 và a D b D 1; c D 2: Bài được chứng minh.

Bài toán 2. Cho ba số thực a; b; c thỏa mãn a C b C c D 3: Chứng minh rằng

3.a4 C b 4 C c 4 / C a2 C b 2 C c 2 C 6  6.a3 C b 3 C c 3 /: .2:2:1/

Lời giải. Đây cũng là một bài toán đối xứng, nên ta sẽ tìm cách đưa điều kiện về dạng tổng bằng
0; rồi dùng phép thế như trên. Rất tự nhiên đặt a D x C 1; b D y C 1; c D z C 1; khi đó

x C y C z D .a 1/ C .b 1/ C .c 1/ D 0:

Với phép đặt này thì

a2 C b 2 C c 2 D .x C 1/2 C .y C 1/2 C .z C 1/2


D x 2 C y 2 C z 2 C 2.x C y C z/ C 3
D x 2 C y 2 C z 2 C 3;

tương tự

a3 C b 3 C c 3 D x 3 C y 3 C z 3 C 3.x 2 C y 2 C z 2 / C 3;
a4 C b 4 C c 4 D x 4 C y 4 C z 4 C 4.x 3 C y 3 C z 3 / C 6.x 2 C y 2 C z 2 / C 3:

Bất đẳng thức (2.2.1) trở thành

3.x 4 C y 4 C z 4 / C 6.x 3 C y 3 C z 3 / C x 2 C y 2 C z 2  0: .2:2:2/

Giả sử xy  0; rồi thay z D x y vào (2.2.2), ta được

3Œx 4 C y 4 C .x C y/4  C 6Œx 3 C y 3 .x C y/3  C x 2 C y 2 C .x C y/2  0;

3.x 4 C 2x 3 y C 3x 2 y 2 C 2xy 3 C y 4 / C x 2 C xy C y 2  9xy.x C y/


3.x 2 C xy C y 2 /2 C x 2 C xy C y 2  9xy.x C y/:
Áp dụng bất đẳng thức AM-GM, ta có
3
x 2 C xy C y 2  .x C y/2  3xy  0; .2:2:3/
4
suy ra
3
3.x 2 C xy C y 2 /2 C x 2 C xy C y 2  27x 2 y 2 C .x C y/2 :
4
Do đó ta chỉ cần chứng minh

.x C y/2
9x 2 y 2 C  3xy.x C y/:
4
36 Các phương pháp giải toán qua các kỳ thi Olympic

Cũng theo bất đẳng thức AM-GM, thì


r
2 2 .x C y/2 .x C y/2
9x y C  2 9x 2 y 2  D 3xy jx C yj  3xy.x C y/: .2:2:4/
4 4
Đẳng thức xảy ra khi (2.2.3) và (2.2.4) trở thành đẳng thức, tức x; y là nghiệm của hệ
8
<x D y
: 27x 2 y 2 D 3 .x C y/2
4
Giải hệ này ta được x D y D 0 hoặc x D y D 31 ; suy ra a D b D c D 1; hoặc a D b D 43 ; c D
1
3
cùng các hoán vị. Bài toán được chứng minh.

Sau 2 ví dụ đầu ta thấy ý tưởng chính để giải quyết các bất đẳng thức đối xứng bậc 4 được “mô
hình” hóa như sau:

(1) Nếu bất đẳng thức có điều kiện a C b C c D 3 thì ta sẽ tiến hành đổi biến sang x D
a 1; y D b 1; z D c 1; còn nếu bất đẳng thức có các điều kiện khác thì ta cố gắng
thuần nhất bài toán hoặc thông qua một số đánh giá để đưa bài toán về đồng bậc sau đó
chuẩn hóa a C b C c D 3; rồi sử dụng các biến đổi

a4 C b 4 C c 4 D x 4 C y 4 C z 4 C 4.x 3 C y 3 C z 3 / C 6.x 2 C y 2 C z 2 / C 3;
a3 C b 3 C c 3 D x 3 C y 3 C z 3 C 3.x 2 C y 2 C z 2 / C 3;
a2 C b 2 C c 2 D x 2 C y 2 C z 2 C 3;
ab C bc C ca D xy C yz C zx C 3;
abc D xyz C xy C yz C zx C 1


(2) Thay z D x y rồi biến đổi về hai đại lượng x 2 C xy C y 2 ; xy .x C y/ thông qua các
đẳng thức
x 4 C y 4 C z 4 D 2.x 2 C xy C y 2 /2 ;
x3 C y3 C z3 D 3xy.x C y/;
x C y C z D 2.x 2 C xy C y 2 /;
2 2 2

xy C yz C zx D .x 2 C xy C y 2 /;
xyz D xy.x C y/;

Cuối cùng ta sẽ thu được bất đẳng thức có dạng
2
A.x 2 C xy C y 2 / C B.x 2 C xy C y 2 / C C xy .x C y/ C D  0:

Từ đó tùy vào các hệ số các A; B; C; D mà ta có những đánh giá thích hợp để giải quyết
bài toán.

Rất vui mừng vì phát hiện thú vị này chúng tôi đã thử áp dụng vào các bài toán khác và thu được
những kết quả sau:
Một số vấn đề bất đẳng thức bậc bốn ba biến 37

Bài toán 3. Cho ba số thực a; b; c thỏa mãn a C b C c D 3: Chứng minh rằng

.ab C bc C ca 3/2  27.abc 1/: .2:3:1/

Lời giải. Đặt a D x C 1; b D y C 1; c D z C 1; thì x C y C z D 0: Khi đó

ab C bc C ca D xy C yz C zx C 3:
abc D xyz C xy C yz C xz C 1:

Bất đẳng thức (2.3.1) trở thành

.xy C yz C zx/2  27.xyz C xy C yz C xz/: .2:3:2/

Giả sử xy  0; thay z D x y vào (2.3.2) và thu gọn lại, ta được

Œxy .x C y/2 2  27Œ xy.x C y/ C xy .x C y/2 ;

hay là
.x 2 C xy C y 2 /2 C 27.x 2 C xy C y 2 / C 27xy.x C y/  0:
Áp dụng bất đẳng thức AM-GM, ta có
3
x 2 C xy C y 2  .x C y/2  3xy  0;
4
suy ra
81
.x 2 C xy C y 2 /2 C 27.x 2 C xy C y 2 /  9x 2 y 2 C .x C y/2 :
4
Ta sẽ chứng minh
9
x 2 y 2 C .x C y/2 C 3xy.x C y/  0;
4
bất đẳng thức này đúng vì

9 .3x C 3y C 2xy/2
x 2 y 2 C .x C y/2 C 3xy.x C y/ D  0:
4 4
Đẳng thức xảy ra khi a D b D c D 1; hoặc a D b D 2; c D 7 cùng các hoán vị. Chứng minh
của ta được hoàn tất.
Bài toán 4. Cho ba số thực a; b; c thỏa mãn a C b C c D 3: Chứng minh rằng

3.a4 C b 4 C c 4 / C 33  14.a2 C b 2 C c 2 /: .2:4:1/

Lời giải. Đặt a D x C 1; b D y C 1; c D z C 1; thì x C y C z D 0: Bất đẳng thức (2.4.1)


được viết lại như sau

3.x 4 C y 4 C z 4 / C 12.x 3 C y 3 C z 3 / C 4.x 2 C y 2 C z 2 /  0: .2:4:2/

Giả sử xy  0; thay z D x y vào (2.4.2) và thu gọn lại, ta được

3.x 4 C 2x 3 y C 3x 2 y 2 C 2xy 3 C y 4 / C 4.x 2 C xy C y 2 /  18xy.x C y/;

3.x 2 C xy C y 2 /2 C 4.x 2 C xy C y 2 /  18xy.x C y/:


38 Các phương pháp giải toán qua các kỳ thi Olympic

Áp dụng bất đẳng thức AM-GM, ta có

3
x 2 C xy C y 2  .x C y/2  3xy  0;
4
suy ra
3.x 2 C xy C y 2 /2 C 4.x 2 C xy C y 2 /  27x 2 y 2 C 3.x C y/2 ;
lại có
27x 2 y 2 C 3.x C y/2 18xy.x C y/ D 3.x C y 3xy/2  0:
Đẳng thức xảy ra khi a D b D c D 1 hoặc a D b D 53 ; c D 1
3
: Bài toán được chứng minh.

Bài toán 5. Cho ba số thực a; b; c thỏa mãn a C b C c D 3: Chứng minh rằng

.ab C bc C ca/2 C 9  18abc: .2:5:1/

Lời giải. Đặt a D x C 1; b D y C 1; c D z C 1; thì x C y C z D 0: Khi đó bất đẳng thức


(2.5.1) trở thành

.xy C yz C zx C 3/2 C 9  18.1 C xy C yz C zx C xyz/;

hay
.xy C yz C zx/2  12.xy C yz C zx/ C 18xyz: .2:5:2/
Giả sử xy  0; thay z D x y vào (2.5.2), ta được

.x 2 C xy C y 2 /2 C 12.x 2 C xy C y 2 / C 18xy.x C y/  0:

Áp dụng bất đẳng thức AM-GM, ta có

3
x 2 C xy C y 2  .x C y/2  3xy  0;
4
suy ra
.x 2 C xy C y 2 /2 C 12.x 2 C xy C y 2 /  9x 2 y 2 C 9.x C y/2 ;
như vậy ta cần chỉ ra
x 2 y 2 C .x C y/2 C 2xy.x C y/  0;
tương đương với
.x C y C xy/2  0:
Đẳng thức xảy ra khi a D b D c D 1 hoặc a D b D 1; c D 5: Bài toán được chứng minh.

Bài toán 6. Chứng minh rằng bất đẳng thức

.a2 C b 2 C c 2 2ab 2bc 2ca/2 C 9.ab C bc C ca/2  30abc.a C b C c/; .2:6:1/

luôn đúng với mọi số thực dương a; b; c:


Một số vấn đề bất đẳng thức bậc bốn ba biến 39

Lời giải. Nếu a C b C c D 0 thì (2.6.1) hiển nhiên đúng. Nếu a C b C c ¤ 0; thay .a; b; c/
bởi . a; b; c/ thì bất đẳng thức vẫn không thay đổi nên ta có thể giả sử a C b C c > 0; và
chuẩn hóa cho a C b C c D 3: Đặt a D x C 1; b D y C 1; c D z C 1; khi đó x C y C z D 0:
Bất đẳng thức (2.6.1) trở thành

Œ4.xy C yz C zx/ C 32 C 9.xy C yz C zx C 3/2  90.xy C yz C zx C xyz/;

tương đương với

25.xy C yz C zx/2  12.xy C yz C zx/ C 90xyz: .2:6:2/

Giả sử xy  0; thay z D x y vào (2.6.2) và thu gọn lại, ta được

25.x 2 C xy C y 2 /2 C 12.x 2 C xy C y 2 / C 90xy.x C y/  0:

Áp dụng bất đẳng thức AM-GM, ta có


3
x 2 C xy C y 2  .x C y/2  3xy  0;
4
suy ra
25.x 2 C xy C y 2 /2 C 12.x 2 C xy C y 2 /  225x 2 y 2 C 9.x C y/2 :
Ta sẽ chứng minh
25x 2 y 2 C .x C y/2 C 10xy.x C y/  0;
điều này đúng vì

25x 2 y 2 C .x C y/2 C 10xy.x C y/ D .x C y C 5xy/2  0:

Đẳng thức xảy ra khi a D b D c hoặc a D 3b D 3c cùng các hoán vị, như vậy chứng minh của
ta được hoàn tất.
Bài toán 7 (Việt Nam MO, 2015). Cho ba số thực a; b; c không âm. Chứng minh rằng
p p p  X
2 2 2
3.a C b C c /  .a C b C c/ ab C bc C ca C .b c/2  .a C b C c/2 :

Lời giải. Thay .a; b; c/ bởi .a2 ; b 2 ; c 2 / bất đẳng thức cần chứng minh trở thành
X
3.a4 C b 4 C c 4 / > .a2 C b 2 C c 2 /.ab C bc C ca/ C .b 2 c 2 /2 > .a2 C b 2 C c 2 /2 :

Ta chứng minh vế trái

3.a4 C b 4 C c 4 / > .a2 C b 2 C c 2 /.ab C bc C ca/ C 2.a4 C b 4 C c 4 a2 b 2 b2c2 c 2 a2 /:

Điều này tương đương với

a4 C b 4 C c 4 C 2.a2 b 2 C b 2 c 2 C c 2 a2 / > .a2 C b 2 C c 2 /.ab C bc C ca/;

.a2 C b 2 C c 2 /2 > .a2 C b 2 C c 2 /.ab C bc C ca/: .2:7:1/


Chuẩn hóa a C b C c D 3 và đặt a D x C 1; b D y C 1; c D x y C 1 vào (2.7.1) và thu
gọn lại, ta được

6.x 4 C 2x 3 y C 3x 2 y 2 C 2xy 3 C y 4 / C 9.x 2 C xy C y 2 /  0;


40 Các phương pháp giải toán qua các kỳ thi Olympic

6.x 2 C xy C y 2 /2 C 9.x 2 C xy C y 2 /  0:
Bất đẳng thức này hiển nhiên đúng. Đẳng thức xảy ra khi a D b D c; vế trái được chứng minh.
Tiếp đến ta chứng minh vế bên phải

.a2 C b 2 C c 2 /.ab C bc C ca/ C 2.a4 C b 4 C c 4 a2 b 2 b2c2 c 2 a2 / > .a2 C b 2 C c 2 /2 ;

bất đẳng thức này tương đương

a4 C b 4 C c 4 C .a2 C b 2 C c 2 /.ab C bc C ca/ > 4.a2 b 2 C b 2 c 2 C c 2 a2 /;

a4 Cb 4 Cc 4 Cabc.aCbCc/Cab.a2 Cb 2 /Cca.c 2 Ca2 /Cca.c 2 Ca2 / > 4.a2 b 2 Cb 2 c 2 Cc 2 a2 /:


Dễ thấy này là hệ quả của hai bất đẳng thức sau đây

ab.a2 C b 2 / C ca.c 2 C a2 / C ca.c 2 C a2 /  2.a2 b 2 C b 2 c 2 C c 2 a2 /; .2:7:2/

a4 C b 4 C c 4 C abc.a C b C c/  ab.a2 C b 2 / C ca.c 2 C a2 / C ca.c 2 C a2 /: .2:7:3/


Bất đẳng thức (2.7.2) tương đương với

ab.a b/2 C bc.b c/2 C ca.c a/2  0:

Ta chứng minh (2.7.3), chuẩn hóa cho aCbCc D 3 và đặt a D xC1; b D y C1; c D x y C1
với xy  0 rồi thế vào bất đẳng thức trên và thu gọn lại, ta được

4.x 2 C xy C y 2 /2 C 3.x 2 C xy C y 2 /  18xy.x C y/:

Theo bất đẳng thức AM-GM ta có


3
4.x 2 C xy C y 2 /2 C 3.x 2 C xy C y 2 /  4  .3xy/2 C 3  .x C y/2
4
9
D 36x 2 y 2 C .x C y/2 ;
4

9 9.x C y 4xy/2
36x 2 y 2 C .x C y/2 18xy.x C y/ D  0:
4 4
Đẳng thức xảy ra khi a D b D c hoặc a D b; c D 0: Như vậy bài toán được chứng minh.
Bài toán 8 (Iran MO, 2005). Cho ba số thực a; b; c thỏa mãn abc D 1; chứng minh rằng

4 4 4 a2 C b 2 b2 C c2 c 2 C a2
a C b C c C 3.a C b C c/  C C :
c a b
Lời giải. Ta viết bất đẳng thức lại dưới dạng thuần nhất như sau

a4 C b 4 C c 4 3abc.a C b C c/  ab.a2 C b 2 / bc.b 2 C b 2 / ca.c 2 C a2 /: .2:8:1/

Vì X
ab.a2 C b 2 / D .ab C bc C ca/.a2 C b 2 C c 2 / abc.a C b C c/;
nên (2.8.1) tương đương với

a4 C b 4 C c 4 C .ab C bc C ca/.a2 C b 2 C c 2 /  4abc.a C b C c/: .2:8:2/


Một số vấn đề bất đẳng thức bậc bốn ba biến 41

Nếu a C b C c D 0; thay c D a b vào (2.8.2) ta được

a4 C b 4 C .a C b/4 .a2 C ab C b 2 / a2 C b 2 C .a C b/2  0;


 

tuy nhiên dễ thấy

a4 C b 4 C .a C b/4 .a2 C ab C b 2 / a2 C b 2 C .a C b/2 D 0;


 

nên bài toán đúng trong trường hợp này.


Nếu a C b C c ¤ 0; thay .a; b; c/ bởi . a; b; c/ thì bất đẳng thức vẫn không thay đổi nên
ta có thể giả sử a C b C c > 0; và chuẩn hóa cho a C b C c D 3:
Đặt a D x C 1; b D y C 1 thì c D x y C 1; rồi thay vào (2.8.2) và thu gọn lại ta được

27.x 2 C xy C y 2 /  0:

Bất đẳng thức này luôn đúng. Đẳng thức xảy ra khi a D b D c hoặc a C b C c D 0: Bài toán
được chứng minh.
Nhận xét. Bài toán là hệ quả của đẳng thức sau
X X X
a4 C bc a2 D .a C b C c/2 .a2 C b 2 C c 2 ab bc ca/ C 4abc.a C b C c/:

Bài toán 9. Chứng minh rằng với mọi số thực a; b; c ta luôn có


4 4
.a C b/4 C .b C c/4 C .c C a/4  a C b 4 C c 4 C .a C b C c/4 :

.2:9:1/
7
Lời giải. Nếu a C b C c D 0; thay c D a b vào (2.9.1) và thu gọn ta được

a4 C 2a3 b C 3a2 b 2 C 2ab 3 C b 4  0;

ta có
a4 C 2a3 b C 3a2 b 2 C 2ab 3 C b 4 D .a2 C ab C b 2 /2  0:
Nếu a C b C c ¤ 0, thay .a; b; c/ bởi . a; b; c/ thì bất đẳng thức vẫn không thay đổi nên
ta có thể giả sử a C b C c > 0; và chuẩn hóa cho a C b C c D 3:
Đặt a D x C 1; b D y C 1; c D z C 1; thì x C y C z D 0 và x 3 C y 3 C z 3 D 3xyz; ta có

.a C b/4 C .b C c/4 C .c C a/4 D x 4 C y 4 C z 4 C 24.x 2 C y 2 C z 2 / 24xyz C 48;

tương tự thì

a4 C b 4 C c 4 C .a C b C c/4 D x 4 C y 4 C z 4 C 6.x 2 C y 2 C z 2 / C 12xyz C 84:

Như vậy bất đẳng thức trở thành

x 4 C y 4 C z 4 C 48.x 2 C y 2 C z 2 /  72xyz: .2:9:2/

Giả sử xy  0 rồi thay z D x y vào (2.9.2), bất đẳng thức được viết lại dưới dạng

2.x 2 C xy C y 2 /2 C 96.x 2 C xy C y 2 / C 72xy.x C y/  0:


42 Các phương pháp giải toán qua các kỳ thi Olympic

Theo bất đẳng thức AM-GM, ta có


2.x 2 C xy C y 2 /2 C 96.x 2 C xy C y 2 /  18x 2 y 2 C 72.x C y/2 ;
lại có
18x 2 y 2 C 72.x C y/2 C 72xy.x C y/ D 18.2x C 2y C xy/2  0:
Đẳng thức xảy ra khi và chỉ khi a D b D c hoặc 3a D 3b D c: Chứng minh của chúng ta vì
thế hoàn tất.
Nhận xét. Bài toán là một kết quả mạnh hơn của bất đẳng thức sau – Việt Nam TST 1996:
4
.a C b/4 C .b C c/4 C .c C a/4  .a4 C b 4 C c 4 /:
7
Bài toán 10. Với ba số thực a; b; c: Chứng minh rằng với mọi số thực k ta luôn có
X
.a b/.a c/.a kb/.a kc/  0: .2:10:1/

Lời giải. Lập luận như trên giả sử a C b C c > 0; và chuẩn hóa cho a C b C c D 3: Đặt
a D x C 1; b D y C 1; c D x y C 1 với xy  0; khi đó bất đẳng thức (2.10.1) được viết
lại dưới dạng
.k C 2/2 .x 2 C xy C y 2 /2 C 3.k 1/2 .x 2 C xy C y 2 /  9.k C 2/.1 k/xy.x C y/: .2:10:2/
Theo bất đẳng thức AM-GM, ta có
3
x 2 C xy C y 2  .x C y/2  3xy  0:
4
Suy ra
9
.k C 2/2 .x 2 C xy C y 2 /2 C 3.k 1/2 .x 2 C xy C y 2 /  9.k C 2/2 x 2 y 2 C .k 1/2 .x C y/2 ;
4
Ta sẽ chứng minh
2 2 2 .k 1/2 .x C y/2
.k C 2/ x y C  .k C 2/.1 k/xy.x C y/;
4
tương đương với
Œ.k 1/.x C y/ C 2.k C 2/xy2  0:
Đẳng thức xảy a D b D c hoặc a D kb D kc: Chứng minh hoàn tất.
Nhận xét. Bài toán là hệ quả của đẳng thức
X X
2 .a b/.a c/.a kb/.a kc/ D .a b/2 Œa C b .k C 1/c2 :

Đây là một kết quả thú vị, chúng ta có một số kết quả khá đẹp mắt như sau
Nếu k D 0; bất đẳng thức trở thành
a2 .a b/.a c/ C b 2 .b c/.b a/ C c 2 .c a/.c b/  0;
đây chính là bất đẳng thức Schur bậc 4 và nếu khai triển ta sẽ được bất đẳng thức (2.7.3).
Nếu k D 2; bất đẳng thức trở thành
9.a4 C b 4 C c 4 / C 126.a2 b 2 C b 2 c 2 C c 2 a2 /  5.a C b C c/4 :
a
Đẳng thức xảy ra khi 2
D b D c:
Một số vấn đề bất đẳng thức bậc bốn ba biến 43

3. Đưa bài toán về dạng bậc bốn


Có rất nhiều bài toán dạng phân thức, căn thức, ... thoạt nhìn sẽ không có dạng đa thức bậc 4
nhưng nếu thông qua một số đánh giá và biến đổi, ta có thể chuyển chúng về dạng đa thức bậc 4
và có thể áp dụng phương pháp trên. Để hiểu rõ hơn, mời ban đọc xét qua các bài toán sau.

Bài toán 11. Cho a; b; c là các số thực không âm thỏa mãn ab C bc C ca > 0: Chứng minh
rằng bất đẳng thức sau luôn đúng

.a C b/2 .b C c/2 .c C a/2


C 2 C 2  6:
c 2 C ab a C bc b C ca

Lời giải. Áp dụng bất đẳng thức Cauchy-Schwarz ta có


X .a C b/2 Œ.a C b/2 C .b C c/2 C .c C a/2 2

c 2 C ab .ab C c 2 /.a C b/2 C .bc C a2 /.b C c/2 C .ca C b 2 /.c C a C b/2
4.a2 C b 2 C c 2 C ab C bc C ca/2
D :
.ab C c 2 /.a C b/2 C .bc C a2 /.b C c/2 C .ca C b 2 /.c C a C b/2

Vì thế để chứng minh bài toán ta cần chứng minh được

.a2 C b 2 C c 2 C ab C bc C ca/2 3
 :
.ab C c 2 /.a C b/2 C .bc C a2 /.b C c/2 C .ca C b 2 /.c C a/2 2

Bằng cách khai triển trực tiếp ta thấy điều này tương đương với
X X
2Œa4 C b 4 C c 4 C abc.a C b C c/ ab.a2 C b 2 / C 3 ab.a b/2  0:

Bất đẳng thức cuối cùng đúng theo bất đẳng thức (2.7.3), đẳng thức xảy ra khi và chỉ khi
a D b D c hoặc a D b; c D 0 và các hoán vị. Bài toán được chứng minh.

Bài toán 12. Cho a; b; c là các số thực không âm thỏa mãn a2 C b 2 C c 2 C ab C bc C ca D 6:


Chứng minh rằng
1 1 1
C C  1: .3:2:1/
4 ab 4 bc 4 ca
Lời giải. Bài toán này được p
làm chặt từ đề thi Moldova TST 2005 và có đến hai dấu bằng là
a D b D c D 1 và a D b D 2; c D 0: Muốn sử dụng bất đẳng thức Cauchy-Schwarz ta phải
dùng đến kỹ thuật thêm bớt để làm đảo chiều bài toán. Ta xét
1
P .a; b/ D k :
4 ab
Ta có
1
P .1; 1/ D k ;
3
p  1
P 2; 0 D k ;
4
p p  1
P 2; 2 D k :
2
44 Các phương pháp giải toán qua các kỳ thi Olympic

Ta cần tìm k sao cho P .a; b/  0 và đánh giá này càng chặt càng tốt từ các tính toán trên ta thấy
k  12 là giá trị tốt nhất cần tìm, từ đó dẫn đến đẳng thức

2 2 ab
1 D ;
4 ab 4 ab
bất đẳng thức (3.2.1) được viết lại như sau
2 ab 2 bc 2 bc
C C  1:
4 ab 4 bc 4 bc
Do a2 C b 2 C c 2 C ab C bc C ca D 6; nên

3.2 ab/ D .a b/2 C c 2 C bc C ca  0:

Theo bất đẳng thức Cauchy-Schwarz, ta có

Œ .2 ab/2 .6 ab bc ca/2 .a2 C b 2 C c 2 /2


P
X 2 ab
P D P DP :
4 ab .2 ab/.4 ab/ .2 ab/.4 ab/ .2 ab/.4 ab/

Như vậy, ta cần chứng minh


X
.a2 C b 2 C c 2 /2  .2 ab/.4 ab/;

tương đương với

a4 C b 4 C c 4 C a2 b 2 C b 2 c 2 C c 2 a2 C 6.ab C bc C ca/  24;

hay dưới dạng thuần nhất


X
4
X
2 2
X  X
2
X  2 X 2 X 2
a C a b C ab a C bc  a C bc :
3
Khi triển và thu gọn lại ta được

a4 C b 4 C c 4 C abc.a C b C c/  ab.a2 C b 2 / C bc.b 2 C c 2 / C ca.c 2 C a2 /:

Đây chính là bất đẳng thức (2.7.3). Bài toán được chứng minh.

Bài toán 13. Cho ba số thực x; y; z thuộc Œ 1; 1 thỏa mãn x C y C z D 0: Chứng minh rằng
p p p
1 C x C y 2 C 1 C y C z 2 C 1 C z C x 2  3:

Lời giải. Bình phương hai vế, ta được


Xp
x2 C y2 C z2 C 2 .1 C x C y 2 /.1 C y C z 2 /  6:

Áp dụng bất đẳng thức Cauchy-Schwarz ta có


Xp Xp X
.1 C x C y 2 /.1 C y C z 2 /  .1 C x/.1 C y/ C jyzj
Xp X
 .1 C x/.1 C y/ yz:
Một số vấn đề bất đẳng thức bậc bốn ba biến 45

Ta đưa bài toán về chứng minh


X X Xp
x2 2 yz C 2 .1 C x/.1 C y/  6:

Đặt 1Cx D a2 ; 1Cy D b 2 ; 1Cz D c 2 với a; b; c là các số không âm, khi đó a2 Cb 2 Cc 2 D 3;


bất đẳng thức trên trở thành
X X X X
a4 2 b2c2 C 2 a2 C 2 bc 9  0;

tương đương với X X X X


a4 2 b2c2 C 2 bc a2  0;
X X  X X  X 
3 a4 6 b2c2 C 2 bc a2 a2  0;

khai triển và thu gọn lại, ta được


X X
a4 C b 4 C c 4 C abc.a C b C c/ ab.a2 C b 2 / C 2 bc.b c/2  0:

Bất đẳng thức này đúng theo bất đẳng thức (2.7.3), đẳng thức xảy ra khi x D y D z D 0: Bài
toán được chứng minh.
Bài toán 14. Cho ba số thực không âm a; b; c thỏa mãn a2 C b 2 C c 2 > 0: Chứng minh rằng
p
2 2 2
p 3
abc.ab C bc C ca/
a Cb Cc C 3 p  2.ab C bc C ca/:
a2 C b 2 C c 2
Lời giải. Áp dụng bất đẳng thức AM-GM ta có
p p
p 3
abc.ab C bc C ca/ 3 3abc
3 p p
a2 C b 2 C c 2 a2 C b 2 C c 2
p p
3 3abc a2 C b 2 C c 2
D
a2 C b 2 C c 2
3abc.a C b C c/
 :
a2 C b 2 C c 2
Ta cần chứng minh
3abc.a C b C c/
a2 C b 2 C c 2 C  2.ab C bc C ca/;
a2 C b 2 C c 2
quy đồng và thu gọn lại ta được

a4 C b 4 C c 4 C abc.a C b C c/  ab.a2 C b 2 / C bc.b 2 C c 2 / C ca.c 2 C a2 /:

Đây chính là bất đẳng thức (2.7.3), đẳng thức xảy ra khi a D b D c hoặc a D b; c D 0 cùng các
hoán vị. Chứng minh hoàn tất.
Bài toán 15. Cho ba số thực không âm a; b; c thỏa mãn a3 C b 3 C c 3 C abc D 12: Chứng
minh rằng bất đẳng thức sau luôn được thỏa mãn
19.a2 C b 2 C c 2 / C 6.ab C bc C ca/
 36:
aCbCc
46 Các phương pháp giải toán qua các kỳ thi Olympic

Lời giải. Áp dụng bất đẳng thức AM-GM, ta có

19.a2 C b 2 C c 2 / C 6.ab C bc C ca/ D 2  8.a2 C b 2 C c 2 / C 3.a C b C c/2


p
 12 3 3.a2 C b 2 C c 2 /2 .a C b C c/2 :

Suy ra s
19.a2 C b 2 C c 2 / C 6.ab C bc C ca/ 2 2 2 2
3 3.a C b C c /
 12 :
aCbCc aCbCc
Ta quy bài toán về chứng minh

.a2 C b 2 C c 2 /2  9.a C b C c/;

hay
4.a2 C b 2 C c 2 /2  .a3 C b 3 C c 3 C abc/.a C b C c/:
Chuẩn hóa a C b C c D 3 và đặt a D x C 1; b D y C 1; c D x y C 1; với xy  0 bất đẳng
thức cần chứng minh trở thành

16.x 2 C xy C y 2 /2 C 3.x 2 C xy C y 2 / C 36xy.x C y/  0:

Theo bất đẳng thức AM-GM thì


9
16.x 2 C xy C y 2 /2 C 3.x 2 C xy C y 2 /  144x 2 y 2 C .x C y/2 ;
4
lại có
9
2 2 2 9.x C y C 8xy/2
144x y C .x C y/ C 36xy.x C y/ D  0:
4 4
Đẳng thức xảy ra khi a D 2b D 2c cùng các hoán vị. Bài toán được chứng minh.

4. Xung quanh bài toán trong kỳ thi British MO 1986


Trong kỳ thi vô địch toán British MO 1986 có một bài toán bất đẳng thức thú vị sau

Bài toán 16. Với a; b; c là ba số thực thỏa mãn đồng thời các điều kiện

a C b C c D 0; a2 C b 2 C c 2 D 6:

Chứng minh rằng


a2 b C b 2 c C c 2 a  6:

Đây là một bất đẳng thức khó vì đẳng thức của nó không tại tâm, cũng không tại biên mà tại
a D 2 cos 2
9
; b D 2 cos 4
9
; c D 2 cos 8
9
cùng các hoán vị, chính vì dấu bằng “kỳ lạ” này mà
bài toán đã gây khó khăn cho các phương pháp mà ta đã biết thậm thậm chí là các phương pháp
rất mạnh như S.O.S, dồn biến, ... Trong quyển sách “Sử Dụng Phương Pháp Cauchy Schwarz Để
Chứng Minh Bất Đẳng Thức” tác giả Võ Quốc Bá Cẩn 4 đã đưa ra một chứng minh rất độc đáo
bằng bất đẳng thức Cauchy-Schwarz như sau
4
Hà Nội
Một số vấn đề bất đẳng thức bậc bốn ba biến 47

Lời giải. Ta có một số tính toán đơn giản

.a C b C c/2 .a2 C b 2 C c 2 /
ab C bc C ca D D 3;
2
a2 b 2 C b 2 c 2 C c 2 a2 D .ab C bc C ca/2 2abc.a C b C c/ D 9:
Sử dụng đẳng thức

3.a2 b C b 2 c C c 2 a/ D a.2ab C c 2 / C b.2bc C a2 / C c.2ca C b 2 /; .4:1:1/

và bất đẳng thức Cauchy-Schwarz, ta có


2 2 2 2 2 2 2 2 2
 
.a C b C c / .2ab C c / C .2bc C a / C .2ca C b /
.a2 b C b 2 c C c 2 a/2 
9
2 .2ab C c / C .2bc C a / C .2ca C b 2 /2
2 2 2 2
 
D :
3
Bằng cách khai triển trực tiếp, ta thấy
2
X
.2ab C c 2 / D .a2 C b 2 C c 2 /2 C 2.a2 b 2 C b 2 c 2 C b 2 c 2 / C 4abc.a C b C c/ D 54;

nên ˇ 2
ˇa b C b 2 c C c 2 aˇ  6;
ˇ

suy ra
a2 b C b 2 c C c 2 a  6:
Bài toán được chứng minh.

Nhận xét. Có thể nói mấu chốt của lời giải này chính là việc sử dụng đẳng thức (4.1.1) để sau
đó sử dụng thành công bất đẳng thức Cauchy-Schwarz. Trong sách các tác giả cũng lý giải việc
tìm ra đẳng thúc (4.1.1) bằng cách sử dụng phương pháp nhân tử Langrange (sẽ được học trong
chương trình toán cao cấp của bậc đại học) như sau. Bằng cách đặt

F .a; b; c/ D a2 b C b 2 c C c 2 a C 1 .a C b C c/ C 2 .a2 C b 2 C c 2 6/:

Ta thấy điểm cực trị của hàm F .a; b; c/ là nghiệm của hệ phương trình sau đây
8
@F @F @F
D D D0
ˆ
ˆ
ˆ
< @a @b @c
ˆ aCbCc D0
ˆ
: a2 C b 2 C c 2 D 6
ˆ

tương đương với


2ab C c 2 C 1 C 22 a D 0
8
ˆ
ˆ
2
ˆ
< 2bc C a C 1 C 22 b D 0
ˆ
ˆ
ˆ
2ca C b 2 C 1 C 22 c D 0 .4:1:2/
ˆ
aCbCc D0
ˆ
ˆ
ˆ
ˆ
ˆ
: 2
a C b2 C c2 D 6
48 Các phương pháp giải toán qua các kỳ thi Olympic

Cộng tương ứng theo vế các phương trình thứ nhất, thứ hai, thứ ba lại với nhau ta được
.a C b C c/2 C 31 C 22 .a C b C c/ D 0;
Từ đó suy ra 1 D 0; vì thế hệ (4.1.2) lúc này trở thành
2ab C c 2 C 22 a D 0
8
ˆ
ˆ
2
ˆ
< 2bc C a C 22 b D 0
ˆ
ˆ
ˆ
2ca C b 2 C 22 c D 0
ˆ
aCbCc D0
ˆ
ˆ
ˆ
ˆ
ˆ
: 2
a C b2 C c2 D 6
Từ ba phương trình đầu, ta rút ra được
2ab C c 2 2bc C a2 2ca C b 2
D D D 22 :
a b c
Nếu để ý ta sẽ thấy đẳng thức trên chính là điều kiện xảy ra dấu bằng trong bất đẳng thức
Cauchy-Schwarz và điều này lý giải cho việc tách đẳng thức (4.1.1).
Không dừng lại ở đây, bằng kỹ thuật tương tự chúng ta ta có thể giải quyết trọn vẹn được bài toán
tổng quát của bất đẳng thức trên, một bài toán rất hay.
Bài toán 17 (Võ Quốc Bá Cẩn). Với a; b; c là ba số thực và t  0 là một số cho trước thỏa
mãn đồng thời a C b C c D 0; a2 C b 2 C c 2 D 6t 2 : Chứng minh rằng với mọi số thực k ta luôn
có bất đẳng thức sau đây
ˇ 2 p
ˇa b C b 2 c C c 2 a C kabc ˇ  2t 3 k 2 3k C 9:
ˇ
.4:2:1/
Lời giải. Từ giả thiết ta tính được
ab C bc C ca D 3t 2 ;
a2 b 2 C b 2 c 2 C c 2 a2 D 9t 4 :
Lại có
a3 b C b 3 c C c 3 a ab 3 bc 3 ca3 D .a C b C c/.a b/.b c/.c a/ D 0;
nên a3 b C b 3 c C c 3 a D ab 3 C bc 3 C ca3 ; suy ra
ab.a2 C b 2 / C bc.b 2 C c 2 / C ca.c 2 C a2 /
a3 b C b 3 c C c 3 a D
2
ab.a C b / C bc.b C c 2 / C ca.c 2 C a2 / C abc.a C b C c/
2 2 2
D
2
.ab C bc C ca/.a2 C b 2 C c 2 /
D D 9t 4 :
2
Từ đó áp dụng bất đẳng thức Cauchy-Schwarz, ta có
 2 2  2
3.a b C b 2 c C c 2 a C kabc/ D 3.a2 b C b 2 c C c 2 a/ C 3kabc C kt 2 .a C b C c/
hX i2
2 2
D a.2ab C c C kbc C kt /
hX i
2
 .a2 C b 2 C c 2 / .2ab C c 2 C kbc C kt 2 /
hX i
2
D 6t 2 .2ab C c 2 C kbc C kt 2 / :
Một số vấn đề bất đẳng thức bậc bốn ba biến 49

Bằng khai triển trực tiếp ta thấy


2
X
.2ab C c 2 C kbc C kt 2 / D 6t 4 .k 2 3k C 9/:

Suy ra p
ˇ 2
ˇa b C b 2 c C c 2 a C kabc ˇ  2t 3 k 2
ˇ
3k C 9:
Bài toán được chứng minh.
Nhận xét.
.1/ Ngoài ra để tính a3 b C b 3 c C c 3 a D 9t 4 ta có thể sử dụng hằng đẳng thức

.a C b C c/.a2 b C b 2 c C c 2 a/ D 0:

3
.2/ Chọn k D 2
thì bất đẳng thức .4:2:1/ trở thành
ˇ 2 p
ˇ2.a b C b 2 c C c 2 a/ C 3abc ˇ  6 3t 3 :
ˇ
.4:2:2/

Do a C b C c D 0 nên ta dễ dàng kiểm tra được


ˇ 2
ˇ2.a b C b 2 c C c 2 a/ C 3abc ˇ D j.a
ˇ
b/.b c/.c a/j ;

khi đó bất đẳng thức .4:2:1/ tương đương với


p
j.a b/.b c/.c a/j  6 3t 3 : .4:2:3/

Lúc này chọn tiếp t D 1; ta sẽ được bài toán rất đẹp mắt sau đây – JBMO Turkey 2014:
Với a; b; c là ba số thực cho trước thỏa mãn đồng thời a C b C c D 0; a2 C b 2 C c 2 D 6:
Chứng minh bất đẳng thức
p
j.a b/.b c/.c a/j  6 3:

Những bài toán bất đẳng thức khó, hình thức đơn giản, đẹp mắt và có dấu đẳng thức xảy ra khi
các biến lệch nhau (lệch tâm và lệch biên) thường không có nhiều, vì để xây dựng được những
bài toán như vậy đòi hỏi người ra đề phải có một trình độ lão luyện. Chắc hẳn trong mỗi chúng ta
ai cũng đã từng đôi lần chạm trán với những bài toán như vậy. Có thể nêu ra ở đây một bài toán
đại diện cho những tiêu chuẩn nói trên, một bất đẳng thức rất nổi tiếng của Vasile Cirtoaje.
Bài toán 18. Cho ba số thực a; b; c thay đổi bất kỳ. Chứng minh rằng

.a2 C b 2 C c 2 /2  3.a3 b C b 3 c C c 3 a/:

Ẩn bên trong vẻ bề ngoài đơn giản này là một bất đẳng thức vô cùng khó vì ngoài trường hợp
tầm thường a D b D c để đẳng thức  xảy ra thì vẫn còn một trường hợp nữa đặc biệt nữa là
.a; b; c/  sin2 7 ; sin2 2
7
; sin 2 4
7
; chính vì thế mà một lời giải bình thường cho bất đẳng
thức có dấu bằng “bất thường” này dường như là không có ! Những chứng minh ban đầu đa phần
đều đưa bài toán về dạng tổng các bình phương (ngay cả bằng phương pháp tam thức bậc hai
cũng buộc ta phải phân tích biệt thức  thành một tổng bình phương) điều này đòi hỏi người làm
toán phải có một nhãn quan nhạy bén để nhận biết được sự tồn tại của các đại lượng bình phương
đó. Bằng bài toán tổng quát trên chúng ta có một lời giải khá thú vị sau đây.
50 Các phương pháp giải toán qua các kỳ thi Olympic

Lời giải. Tương tự bài (2.9) ta chuẩn hóa a C b C c D 3; khi đó tồn tại số thực t sao cho

a2 C b 2 C c 2 D 3 C 6t 2 :

Tiếp đến đặt a D 1 C x; b D 1 C y; c D 1 C z thì x C y C z D 0 và tính được

x 2 C y 2 C z 2 D 6t 2 :

Bất đẳng thức cần chứng minh trở thành


2
3.1 C 2t 2 /  a3 b C b 3 c C c 3 a:

Trong (4.2.1) chọn k D 1; với chú ý jAj  A; ta được


p
a2 b C b 2 c C c 2 a C abc  2 7  t 3 :

Biến đổi vế trái


a3 b C b 3 c C c 3 a D .1 C x/3 .1 C y/ C .1 C y/3 .1 C z/ C .1 C z/3 .1 C x/
X
D .x 3 C 3x 2 y C 3xy C x 3 y C 3x 2 C 1/
X X X X  X
D x3 C 3 x2y C 3 x2 C yz C x3y C 3
D 3xyz C 3.x 2 y C y 2 z C z 2 x/ 9t 4 C 9t 2 C 3
D 3.x 2 y C y 2 z C z 2 x C xyz/ 9t 4 C 9t 2 C 3
p
 3  2 7  t 3 9t 4 C 9t 2 C 3:

Như vậy ta chỉ cần chứng minh


2 p
.1 C 2t 2 /  2t 3 7 3t 4 C 3t 2 C 1;

hay là p
7t 4 C t 2  2 7t 3 :
Áp dụng bất đẳng thức AM-GM ta có
p p p
7t 4 C t 2  2 7t 4  t 2 D 2 7 jt j3  2 7t 3 :

Bài toán được chứng minh.


Nhận xét. Với aCbCc D 3 thì 6 là hằng số tốt nhất (về mặt tính toán) để a2 Cb 2 Cc 2 D 3C6t 2 :
Thật vậy, theo bất đẳng thức Cauchy-Schwarz ta có

.a C b C c/2
a2 C b 2 C c 2  D 3;
3
nên tồn tại số thực k không âm và 0  t < 1 sao cho a2 C b 2 C c 2 D 3 C kt 2 : Ta có thể chọn k
tùy ý k D 1; k D 10; : : : nhưng ta phải chọn k sao cho những đánh giá trở phải nên đơn giản
nhất. Cũng theo bất đẳng thức Cauchy-Schwarz thì

.b C c/2 .3 a/2
a2 C b 2 C c 2  a2 C D a2 C ;
2 2
Một số vấn đề bất đẳng thức bậc bốn ba biến 51

suy ra
.3 a/2
3 C kt 2  a2 C :
2
Giải bất phương trình này, ta được
p p
3 t 6k 3 C t 6k
a ;
3 3
cho k D 6; thì
1 2t  a  1 C 2t:
Như vậy k D 6 là hằng số tốt nhất.
Bài toán 19 (IMO, 2006). Tìm hằng số M nhỏ nhất sao cho bất đẳng thức
ˇab.a2 b 2 / C bc.b 2 c 2 / C ca.c 2 a2 /ˇ  M.a2 C b 2 C c 2 /2 ;
ˇ ˇ

luôn đúng với mọi số thực a; b; c bất kỳ.


p p p p
Lời giải. Cho a D 3 22C2 ; b D 2 32 2 ; c D 1 thì M  9 2
32
; ta sẽ chứng minh M D 9 2
32

giá trị nhỏ nhất cần tìm, tức chứng minh
p
ˇab.a2 9 2 2
b 2 / C bc.b 2 c 2 / C ca.c 2 a2 /ˇ  .a C b 2 C c 2 /2 :
ˇ ˇ
32
Thật vậy bất đẳng thức tương đương với
p
9 2 2
j.a C b C c/.a b/.b c/.c a/j  .a C b 2 C c 2 /2 : .4:4:1/
32
Chuẩn hóa a C b C c D 3; khi đó tồn tại số thực t  0 để a2 C b 2 C c 2 D 3 C 6t 2 ; lúc này
(4.4.1) được viết lại dưới dạng
p
27 2
j.a b/.b c/.c a/j  .1 C 2t 2 /2 : .4:4:2/
32
Đặt a D x C 1; b D y C 1; c D z C 1 rồi thay vào (4.4.2) ta được
p
27 2
j.x y/.y z/.z x/j  .1 C 2t 2 /2 :
32
Áp dụng bất đẳng thức .4:2:3/ ta đưa bài toán về chứng minh bất đẳng thức mạnh hơn là
p
p 27 2
6 3  t3  .1 C 2t 2 /2 ;
32
hay p
32 6 3
2 2
.1 C 2t /  t :
9
Áp dụng bất đẳng thức AM-GM, ta có
s
 3 r
2 2 2 4 2 4 8
1 C 2t 2 D 1 C  t 2 C  t 2 C  t 2  4 1   t2 D 4  t 6;
3 3 3 3 27
52 Các phương pháp giải toán qua các kỳ thi Olympic

bình phương hai vế ta được


p
32 6 3
.1 C 2t 2 /2  t :
9
p
9 2
Điều này cho phép ta kết luận M D 32
; là giá trị nhỏ nhất cần tìm.

Không chỉ áp dụng được với lớp các bài toán bậc 4, bổ đề tổng quát này còn áp dụng tốt với các
bài toán khác, chúng ta cùng xem.

Bài toán 20. Với a; b; c là ba số thực thỏa mãn đồng thời các điều kiện

a C b C c D 0; a2 C b 2 C c 2 D 3:

Chứng minh rằng


a5 b C b 5 c C c 5 a  3:

Lời giải. Do a2 C b 2 C c 2 D 3 nên từ bài (4.2) ta được t D p1 : Ta có có một số tính toán


2

3
ab C bc C ca D ;
2
9
a3 b C b 3 c C c 3 a D :
4
a2 b C b 2 c C c 2 a C 3abc D .ab 2 C bc 2 C ca2 /


27
a3 b 3 C b 3 c 3 C c 3 a3 D .ab C bc C ca/3 C 3a2 b 2 c 2 D C 3a2 b 2 c 2 :
8
Ta có biến đổi

a5 b D a3 b  a2 D a3 b.3 b2 c 2 / D 3a3 b a3 b 3 a3 bc 2 ;

suy ra

a5 b C b 5 c C c 5 a D 3.a3 b C b 3 c C c 3 a/ .a3 b 3 C b 3 c 3 C c 3 a3 / .ab 2 C bc 2 C ca2 /abc


   
9 27
D3 C 3a b c C .a2 b C b 2 c C c 2 a C 3abc/abc
2 2 2
4 8
27
D C abc.a2 b C b 2 c C c 2 a/:
8

Bài toán quy về chứng minh

3
abc.a2 b C b 2 c C c 2 a/  :
8

Trong (4.2.1) chọn k D 3; t D p1 ; ta được


2

ˇa b C b 2 c C c 2 a C 3abc ˇ  p3 :
ˇ 2 ˇ
.4:6:1/
2
Một số vấn đề bất đẳng thức bậc bốn ba biến 53

Áp dụng bất đẳng thức AM-GM, ta có


1
abc.a2 b C b 2 c C c 2 a/ D  3abc  .a2 b C b 2 c C c 2 a/
3
2
1 3abc C a2 b C b 2 c C c 2 a


3 2
3
 :
8
p p p 
Ta thấy đẳng thức xảy ra khi và chỉ khi .a; b; c/ D 2 cos 29
; 2 cos 13
9
; 2 cos 7
9
; hoặc
 p p p 
.a; b; c/ D 2 cos 2
9
; 2 cos 13
9
; 2 cos 7
9
cùng các hoán vị.
Bài toán được chứng minh.
Nhận xét. Chắc hẳn bạn đọc sẽ thắc mắc tại sao ta lại chọn k D 3 để có đánh giá (4.6.1). Ta có
thể lý giải điều này như sau:
Do t D p1 ; nên từ (4.2.1) ta được
2
r
ˇ 2
ˇa b C b 2 c C c 2 a C kabc ˇ 
ˇ k2 3k C 9
:
2
Áp dụng bất đẳng thức AM-GM ta có
ˇ ˇ
ˇ1ˇ ˇ
abc.a b C b c C c a/  ˇˇ ˇˇ  ˇkabc  .a2 b C b 2 c C c 2 a/ˇ
2 2 2
ˇ
k
ˇ 1 ˇ kabc C a2 b C b 2 c C c 2 a 2
ˇ ˇ 
ˇ ˇ
ˇ ˇ
k 2
k2 3k C 9
 :
8 jkj

Như vậy ta cần tìm k sao cho


k2 3k C 9 D 3 jkj :
Giải phương trình này ta được k D 3:

5. Bài tập rèn luyện


Bài tập 12. Cho a; b; c là ba số thực không âm sao cho ab C bc C ca > 0: Chứng minh rằng

a2 b2 c2 2
16 C C  :
2a2 C .b C c/2 2b 2 C .c C a/2 2c 2 C .a C b/2 3

Bài tập 13. Chứng minh rằng bất đẳng thức


2
7 a2 C b 2 C c 2 C .a C b C c/2  12 a4 C b 4 C c 4 C .a C b C c/4 ;
  

luôn đúng với mọi số thực a; b; c thay đổi bất kỳ.


54 Các phương pháp giải toán qua các kỳ thi Olympic

Bài tập 14. Cho a; b; c là các số thực không âm.Chứng minh rằng
p ˇ
a4 C b 4 C c 4 abc.a C b C c/ > 2 2 ˇa3 b C b 3 c C c 3 a ab 3 bc 3 ca3 ˇ :
ˇ

Bài tập 15. Cho ba số thực x; y; z có tổng bằng 3: Chứng minh rằng
4.x 4 C y 4 C z 4 / C 45  19.x 2 C y 2 C z 2 /:
Bài tập 16. Cho a; b; c là độ dài ba cạnh của tam giác. Chứng minh rằng
a4 C b 4 C c 4 C 9abc.a C b C c/  10.a2 b 2 C b 2 c 2 C c 2 a2 /:
Bài tập 17. Cho a; b; c là ba số thực thỏa mãn a2 C b 2 C c 2 D 3: Tìm giá trị lớn nhất của
P D a4 C b 4 C c 4 C 3.ab C bc C ca/:
Bài tập 18. Cho a; b; c là ba số thực thỏa mãn ab C bc C ca D 3: Tìm giá trị nhỏ nhất của
F D 4.a4 C b 4 C c 4 / C 11abc.a C b C c/:
Bài tập 19. Tìm hằng số k lớn nhất sao cho bất đẳng
a4 C b 4 C c 4  3 C k.a3 C b 3 C c 3 C 3abc 6/;
luôn đúng với mọi số thực dương a; b; c thỏa mãn a C b C c D 3:
Bài tập 20. Chứng minh rằng với mọi số thực không âm a; b; c thì
a4 C b 4 C c 4 a2 b 2 b 2 c 2 c 2 a2 > 2 ˇa3 b C b 3 c C c 3 a ab 3 bc 3 ca3 ˇ :
ˇ ˇ

Bài tập 21. Cho ba số thực ˛; ˇ; thỏa mãn 1 C ˛ C ˇ D 2 : Chứng minh rằng bất đẳng thức
X X X X
a4 C ˛ b 2 c 2 C ˇabc a bc.b 2 C c 2 /;
2
luôn đúng với mọi số thực a; b; c khi và chỉ khi 1 C ˛  :

Tài liệu tham khảo


[1] Võ Quốc Bá Cẩn, Chuyên Đề Bất Đẳng Thức Hiện Đại, 2008.
[2] Võ Quốc Bá Cẩn, Trần Quốc Anh, Sử Dụng Phương Pháp Cauchy Schwarz Để Chứng Minh
Bất Đẳng Thức, Nhà Xuất Bản Đại Học Sư Phạm, 2010.
[3] Lê Việt Hải, Phương Pháp Nhân Tử Langrange & Bất Đẳng Thức Cauchy-Schwarz, Câu
Lạc Bộ Toán Trường Phổ Thông Năng Khiếu Thành Phố Hồ Chí Minh, 2011.
[4] Vasile Cirtoaje, Algebraic Inequalities Polynomial Rational Symmetric Inequalities, GIL
Publishing House, 2011.
[5] Vasile Cirtoaje, Võ Quốc Bá Cẩn , On Some Cyclic Homogeneous Polynomial Inequality
Of Degree Fourth In Real Variables Under Constraints, International Journal of Pure and
Applied Mathematics, 2012.
[6] Art of Problem Solving: http://artofproblemsolving.com
[7] JIPAM: http://www.emis.de/journals/JIPAM/
MỘT SỐ VẤN ĐỀ XUNG QUANH
TỔNG LŨY THỪA
Nguyễn Mạnh Linh
(Toán 2012-2015, THPT Chuyên Hạ Long)

1. Hoán vị của một tập hợp


Ký hiệu Œn D f1; 2; : : : ; ng: Một hoán vị của Œn là một song ánh:

f W Œn ! Œn:

Dễ thấy có đúng nŠ hoán vị của Œn:

1. Với mỗi hoán vị f của Œn; ta có thể biểu diễn nó bởi một đồ thị có hướng G .V ; A/ như sau:

 Tập đỉnh V D Œn:


 Nếu i là điểm bất động của f thì có một khuyên ở đỉnh i :
 Nếu f .i / D j thì có một cung đi từ i vào j:

Xét i 2 Œn và n C 1 số: i ; f .i /; f 2 .i /; : : : ; f n .i /; theo nguyên lý Dirichlet thì


tồn tại hai số bằng nhau, giả sử là:

f k .i / D f l .i / .n  k > l  0/:

Do f là song ánh nên f k l .i / D i : Do đó có thể gọi t là số dương nhỏ nhất sao cho
f t . i / D i ; khi đó i ; f .i /; f 2 .i /; : : : ; f t 1 .i / đôi một phân biệt.

Thiết lập một quan hệ tương đương trong Œn W i  j khi và chỉ khi tồn tại số t sao cho
f t . j / D i : Ta sử dụng quan hệ này để phân hoạch Œn thành một số lớp. Gọi G là đồ
thị biểu diễn hoán vị f; dễ thấy:

 i  j khi và chỉ khi i liên thông với j:


 Các đỉnh biểu diễn các phần tử cùng lớp tạo thành một chu trình (khuyên được coi là
chu trình độ dài 1).

Mỗi lớp trên được gọi là một chu trình của hoán vị f: Nói cách khác, mỗi hoán vị đều là
tích các chu trình rời nhau. Vậy ta có thể biểu diễn mỗi hoán vị f ở dạng chu trình. Các
chu trình được đặt giữa hay dấu . / và các phần tử trong mỗi chu trình được viết theo thứ
tự. Chẳng hạn, với n D 3 thì:

.1; 2 ; 3/ D .1/.2/.3/; .2 ; 1; 3/ D .3/.12/; .3; 1; 2/ D .132/;


.1; 3; 2/ D .1/.23/; .2 ; 3; 1/ D .123/; .3; 2 ; 1/ D .2/.13/:

55
56 Các phương pháp giải toán qua các kỳ thi Olympic
   
n n
2. Ký hiệu là số hoán vị của Œn chứa đúng k chu trình. Số này được gọi là số
k k
 
n
Stirling loại I (unsigned). Còn số . 1/ n k được gọi là số Stirling loại I (signed).
k
Ta có      
0 n n
D 1; D 0 .n > 0/; D 0 .k > n/;
0 0 k
     
n n n.n 1/ n
D 1; D ; D nŠ:
n n 1 2 1
 
n
3. Ta có thể tính trực tiếp giá trị theo đúng định nghĩa:
k
 
n nŠ X 1
D  ;
k kŠ m1 m2    mk
1  m1 ; m2 ; : : : ; mk
m1 C m2 C    C mk D n

với tổng chạy trên tất cả kn 11 nghiệm nguyên dương .m 1 ; m 2 ; : : : ; m k / của phương


trình m 1 C m 2 C    C m k D n :
 
n
4. Một vài giá trị của số :
k

k 0 1 2 3 4 5 6 7 8 9
n
0 1
1 0 1
2 0 1 1
3 0 2 3 1
4 0 6 11 6 1
5 0 24 50 35 10 1
6 0 120 274 225 85 15 1
7 0 720 1764 1624 735 175 21 1
8 0 5040 13068 13132 6769 1960 322 28 1
9 0 40320 109584 118124 67284 22449 4536 546 36 1
 
n
5. Một đẳng thức truy hồi quan trọng cho số :
k
     
n n 1 n 1
D .n 1/ C :
k k k 1

Thật vậy, xét một hoán vị f của Œn chứa đúng k chu trình. Gọi g là hoán vị của Œn 1
thu được bằng cách bỏ đỉnh n (vẫn giữ nguyên các chu trình). Có hai trường hợp:
 Nếu n là điểm bất động của f thì sau khi bỏ n ; g chính là một hoán vị của Œn 1
chứa đúng k 1 chu trình.
Một số vấn đề xung quanh tổng lũy thừa 57

 Nếu n nằm trong một chu trình với độ dài không nhỏ hơn 2 thì sau khi bỏ n ; g chính
là một hoán vị của Œn 1 chứa đúng k chu trình.

Bây giờ, nhận xét rằng mỗi hoán vị g như thế ứng với đúng n 1 hoán vị f khả dĩ, vì
trên mỗi chu trình độ dài l ta có thể thêm phần tử n vào đúng một vị trí khác nhau, nên có
đúng n 1 vị trí khác nhau để có thể thêm phần tử n, từ đó ta suy ra công thức trên.

6. Xét các đa thức đối xứng cơ sở của các số 1; 2 ; : : : ; n 1:

e 0 .n 1/ D 1;
e 1 .n 1/ D 1 C 2 C    C .n 1/;
X
e 2 .n 1/ D ij ;
1 i < j  n 1

:::;
e n 1 .n 1/ D 1  2    .n 1/;
e k .n 1/ D 0 .k > n 1/:

Khi đó ta có  
n
D en k .n 1/:
k
Chứng minh kết quả này bằng đại số. Ta quy nạp theo n :

Dễ dàng kiểm tra kết quả cho các trường hợp nhỏ n D 1; 2 : Xét các đa thức đối xứng cơ
sở của các số 1; 2 ; : : : ; n : Chú ý rằng ta có đẳng thức

e nC 1 k .n/ D ne n k .n 1/ C e nC 1 k .n 1/:

Do đó, sử dụng giả thiết quy nạp và đẳng thức ở mục 5, ta được
     
nC1 n n
Dn C
k k k 1
D ne n k .n 1/ C e n .k 1 / .n 1/
D e nC 1 k .n/:

7. Ký hiệu: (
1 với n D 0
.x / n D
x .x 1/    .x n C 1/ với n > 0
Đa thức này được gọi là “falling factorial”. Theo định lý Vietta và kết quả ở mục 6, ta có
khai triển Taylor của .x / n như sau:
n n  
X n i i
X n i n
.x / n D . 1/ en i .n 1/x D . 1/ xi :
i
i D0 i D0

Hay với một đa thức khác là “rising factorial”:


(
n 1 với n D 0
.x /
x .x C 1/    .x C n 1/ với n > 0
58 Các phương pháp giải toán qua các kỳ thi Olympic

thì ta có
n  
n
X n
.x / D xi :
i
i D0

Như vậy, có thể thấy rằng các số Stirling loại I cho chúng ta biểu diễn “falling factorial”
thành tổ hợp tuyến tính của các đơn thức (khai triển Taylor). Ở mục sau, chúng ta sẽ nghiên
cứu điều ngược lại, tức là biểu diễn đơn thức thành tổ hợp tuyến tính của các “falling
factorial” (khai triển Hilbert).

2. Xếp bóng vào hộp


   
n n
1. Ký hiệu là số cách phân hoạch Œn thành k khối. Nói cách khác, chính là số
k k
  xếp n quả bóng khác nhau vào k hộp giống nhau sao cho không có hộp nào rỗng.
cách
n
được gọi là số Stirling loại II, ta có
k
     
0 n n
D 1; D 0 .n > 0/; D 0 .k > n/;
0 0 k
       
n n n n.n 1/ n
D D 1; D ; D 2 n 1 1:
n 1 n 1 2 2
   
n n
2. Ta có thể tính toán số bằng nguyên lý bù trừ như sau: Để ý rằng k Š chính là số
k k
cách xếp n quả bóng khác nhau vào k hộp khác nhau. Gọi A i là tập các cách xếp sao cho
hộp i là hộp rỗng. Ta có nhận xét:
 Với mỗi tập con fi 1 ; i 2 ; : : : ; i j g của Œk  thì A i 1 \ A i 2 \    \ A i j chính là tập
các cách xếp sao cho các hộp i 1 ; i 2 ; : : : ; i j đều rỗng, do đó:
ˇ A i \ A i \    \ A i ˇ D .k j / n :
ˇ ˇ
1 2 j

 A 1 [ A 2 [    [ A k chính là tập các cách xếp sao cho ít nhất một trong các hộp là
rỗng. Ta có
k
X X
. 1/ j 1
ˇ ˇ
jA 1 [    [ A k j D ˇAi \    \ Ai ˇ
1 j
j D1 fi1 ; i2 ; :::; ij gŒk 
k  
X j 1 k
D . 1/ .k j /n :
j
j D1

Do có đúng k n cách xếp tùy ý n bóng vào k hộp nên


  k   k  
n X j 1 k n
X j k
kŠ Dk n
. 1/ .k j / D . 1/ .k j /n
k j j
j D1 j D0
k   k  
X k j k n
X k j k
D . 1/ j D . 1/ j n:
k j j
j D0 j D0
Một số vấn đề xung quanh tổng lũy thừa 59
 
n
Tóm lại, ta có công thức tổng quát của là:
k

  k  
n 1 X k
D . 1/ k j
j n:
k k Š j D0 j

 
n
3. Một vài giá trị của số :
k

k 0 1 2 3 4 5 6 7 8 9
n
0 1
1 0 1
2 0 1 1
3 0 1 3 1
4 0 1 7 6 1
5 0 1 15 25 10 1
6 0 1 31 90 65 15 1
7 0 1 63 301 350 140 21 1
8 0 1 127 966 1701 1050 266 28 1
9 0 1 255 3025 7770 6951 2646 462 36 1

 
n
4. Một đẳng thức truy hồi cho số :
k

     
n n 1 n 1
Dk C :
k k k 1

Thật vậy, xét một cách xếp n bóng khác nhau vào k hộp giống nhau. Có hai trường hợp:

 Nếu hộp chứa quả bóng thứ n chỉ chứa quả bóng thứ n thì n 1 quả bóng còn lại
được xếp vào k 1 hộp.

 Nếu hộp chứa quả bóng thứ n chứa không ít hơn hai quả bóng, ta bỏ quả bóng n ra
thì n 1 quả bóng còn lại được xếp trong k hộp.

Bây giờ, nhận xét rằng mỗi cách xếp n 1 bóng vào k hộp như thế ứng với đúng k cách
xếp n bóng vào k hộp, vì với mỗi cách xếp ta có thể thêm n vào một trong k hộp (chú ý
rằng cách đếm như vậy không bị lặp lại), từ đó ta suy ra công thức trên.
60 Các phương pháp giải toán qua các kỳ thi Olympic

5. Khai triển Hilbert của đơn thức:


 
0
1 D .x / 0 D .x / 0 ;
0
   
1 1
x D .x / 1 D .x / 1 C .x / 0 ;
1 0
     
2 2 2 2
x D .x / 2 C .x / 1 D .x / 2 C .x / 1 C .x / 0 ;
2 1 0
       
3 3 3 3 3
x D .x / 3 C 3.x / 2 C .x / 1 D .x / 3 C .x / 2 C .x / 1 C .x / 0 ;
3 2 1 0
:::

Một cách tổng quát, ta có


n  
n
X n
x D .x / i :
i
i D0

Công thức này có thể được chứng minh bằng quy nạp theo n : Dễ thấy kết quả đúng với
n D 0; 1: Sử dụng đẳng thức ở mục 4, ta có
nC
X1   nC1
X n C 1
nC1
.x / i D .x / i
i i
i D0 i D1
nC
X1  n  nC
X1  n 
D i .x / i C .x / i
i i 1
i D1 i D1
nC
X1  n  n  
X n
D i .x / i C .x / i C 1
i i
i D0 i D0
n   n  
X n X n
D i .x / i C .x / i C 1
i i
i D0 i D0
n  
X n 
D i .x / i C .x / i C 1
i
i D0
n  
X n 
D i .x / i C .x i /.x / i
i
i D0
n   n  
X n X n
D x .x / i D x .x / i
i i
i D0 i D0
n nC 1
Dxx Dx :

3. Sai phân của đa thức


Đầu tiên, ta dễ thấy khai triển Hilbert của một đa thức là duy nhất. Nếu như biểu diễn một đa
thức ở dạng khai triển Taylor:

P .x / D a n x n C a n 1x
n 1
C    C a0
Một số vấn đề xung quanh tổng lũy thừa 61

giúp ta dễ dàng tính đạo hàm:


d
P D na n x n 1
C .n 1/a n 1x
n 2
C    C a1 ;
dx
thì với khai triển Hilbert:

P .x / D ˛ n .x / n C ˛ n 1 .x / n 1 C : : : C ˛0 ;

ta lại có thể tính sai phân rất tiện lợi:

P .x / D n˛ n .x / n 1 C .n 1/˛ n 1 .x / n 2 C    C ˛1 :

Từ các kết quả của các phần 1 và 2, ta có thể chuyển đổi giữa khai triển Taylor và Hilbert:
n  
j k j
X
ak D . 1/ ˛j ;
k
j Dk
n  
X j
˛k D aj :
k
j Dk

4. Tổng lũy thừa


Ta có
n.n C 1/ 1 1
1C2CCn D D n2 C n ;
2 2 2
n.n C 1/.2 n C 1/ 1 1 1
12 C 22 C    C n2 D D n3 C n2 C n ;
6 3 2 6
2 2
n .n C 1/ 1 1 1
13 C 23 C    C n3 D D n4 C n3 C n2 :
4 4 2 4
Ở mục này ta tìm hiểu một vài cách tiếp cận các đa thức S k .x / biểu diễn tổng các lũy thừa bậc
k của n số nguyên dương đầu tiên:

S k .n/ D 1 k C 2 k C    C n k :

4.1. Sai phân


Quy ước S k .0/ D 0: Để tìm S k .n/ thực chất ta cần tìm đa thức S k .x / bậc k C 1 thỏa mãn:
(
S k .x / D .x C 1/ k
S k .0/ D 0

Giả sử S k . x 1/ có khai triển Hilbert:

S k .x 1/ D ˛ k .x / k C 1 C ˛ k 1 .x / k C    C ˛ 0 .x / 1 C ˛ :

Do S k . 0 / D 0 nên 0 D ˛ 0 C ˛ ; tức ˛ D ˛ 0 : Từ đây suy ra

S k .x / D ˛ k .x C 1/ k C 1 C ˛ k 1 .x C 1/ k C    C ˛ 0 .x C 1/ 1 ˛0 :
62 Các phương pháp giải toán qua các kỳ thi Olympic

Khi đó, ta có

S k . x / D .k C 1/˛ k .x C 1/ k C k ˛ k 1 .x C 1/ k 1 C    C ˛ 0 .x C 1/ 0 :

Do tính duy nhất của khai triển Hilbert và đẳng thức:


     
k k k k
. x C 1/ D .x C 1/ k C .x C 1/ k 1 CC .x C 1/ 0 ;
k k 1 0
 
1 k
ta suy ra ˛ i D i C1
và ˛ 0 D 0: Vậy:
i
   
1 k 1 k
Sk .x / D .x C 1/ k C 1 C ˛ k 1 .x C 1/ k
kC1 k k k 1
  k  
1 k X 1 k
CC .x C 1/ 2 D .x C 1/ i C 1
2 1 i C1 i
i D1
0 1
k   k k j i    
X 1 k X X . 1/ k j A i
D .x C 1/ .x / i D .x C 1/ @ x
i C1 i j C1 j i
i D1 i D1 j Di
k
. 1/ j .j
 
1 k C1
X 1/Š k
D x x
kC1 j C 1 j
j D1
0 1
k k
X X . 1/ j i     k
X . 1/ j i    
k j k j A xi :
C @
j C1 j i j C1 j i 1
i D2 j Di j Di 1

4.2. Giải tích


Ta có
kX
C1  
k C2 k C2 k j C1 kC2
i .i 1/ D . 1/ ij:
j
j D0

Thay i D 1; 2 ; : : : ; n và cộng lại, ta thu được


kX
C1  
k C2 k j C1 kC2
n D . 1/ S j .n/:
j
j D0

Đẳng thức này đúng với vô hạn n nên ta có


kX
C1  
k C2 k j C1 kC2
x D . 1/ S j .x /;
j
j D0

từ đó suy ra
Pk C1
x k C2 C . 1/ k j k C2

j D0 j
S j .x /
S k C 1 .x / D :
kC2
Một số vấn đề xung quanh tổng lũy thừa 63

Đây chính là một hệ thức truy hồi cho dãy đa thức S k .x /: Từ hệ thức này, ta có thể rút ra một
kết quả rất thú vị sau đây:
d
S k .x / D k S k 1 .x / C . 1/ k B k
dx
với B k C 1 là hằng số, phụ thuộc vào k : Thật vậy, ta chứng minh kết quả này bằng quy nạp. Dễ
dàng kiểm tra kết quả với k D 1: Giả sử kết quả đã đúng tới k nguyên dương nào đó, ta có

x k C 2 C kj D
P C1 k j k C2

0 . 1/ j
S j .x /
S k C 1 .x / D :
kC2
d
Đạo hàm hai vế, với chú ý S 0 .x / D x ; ta tính được dx
S k C 1 .x / bằng
Pk
. k C 2 /x k C 1 C . 1/ k j k C2
C . 1/ j B j C . 1/ k k C2
  
j D1 j
j Sj 1 .x / 0
kC2
Pk
k
. 1/ k k C2

Bj
 
X k j kC2 j j D0 j
D x k C1 C . 1/ Sj 1 .x / C
j kC2 kC2
j D1
k  
X k j kC1
Dx k C1
C . 1/ Sj 1 .x / C . 1/ k C 1 B k C 1
j 1
j D1
k
X1  
k 1 j kC1
Dx k C1
C . 1/ S j .x / C . 1/ k C 1 B k C 1
j
j D0

D . k C 1/S k .x / C . 1/ k C 1 B k C 1 ;

trong đó: €
B0 D 1
Pk k C2

j D0 j
Bj
Bk C1 D
kC2
Vậy kết quả cũng đúng với k C 1, theo nguyên lý quy nạp nó đúng với mọi k :
Hằng số B n xác định như trên được gọi là các số Bernoulli. Một vài giá trị đầu là:
1 1
B 0 D 1; B1 D ; B2 D ;
2 6
1
B 3 D 0; B4 D ; B 5 D 0;
30
1 1
B6 D ; B 7 D 0; B8 D ;
42 30
5
B 9 D 0; B10 D ; B 1 1 D 0:
66
Một công thức tính trực tiếp số B n :
n i  
X 1 X j i
Bn D . 1/ j n:
i C 1 j D0 j
i D0
64 Các phương pháp giải toán qua các kỳ thi Olympic

Ta có thể viết lại hai đẳng thức xác định số B n ở dạng:


˚B 0 D1
Pn nC 1

i D0 i
Bi
Bn D
nC1
hay
€B
0 D 1
n
X Bi 1
 D0
i D0
i Š .n C 1 i /Š

Nhận thấy vế trái của hai đẳng thức trên là các hệ số của tích hai khai triển hàm sinh:

x x2 xn
f .x / D 1 C C CC C ;
2Š 3Š .n C 1/Š
B1 B2 2 Bn n
g .x / D B 0 C xC x CC x C :
1Š 2Š nŠ

Do đó, f . x /  g .x / D 1: Dễ thấy:

x2 xn
x  f .x / C 1 D 1 C x C CC C    D ex
2Š nŠ
ex 1
nên ta có f .x / D x
: Từ đó suy ra

B1 B2 2 Bn n x
B0 C xC x CC x C    D g .x / D x :
1Š 2Š nŠ e 1

Và do đó, một định nghĩa giải tích cho số B n là:

dn
ˇ
x ˇˇ
B n D lim :
a ! 0 dx n e x 1 ˇx Da
d
Các đa thức B k .x / D dx S k .x 1/ được gọi là các đa thức Bernoulli, có những ứng dụng to
lớn trong Toán học. Trở lại với các đa thức S k .x /; từ đẳng thức:

d
S k .x / D k S k 1 .x / C . 1/ k B k ;
dx

lấy nguyên hàm hai vế, ta được


Z
S k .x / D k Sk 1 .x /dx C . 1/ k B k x :

Từ công thức này, ta xây dựng được công thức tường minh cho S k .x / (công thức Faulhaber):

k  
1 X i kC1
S k .x / D . 1/ B i x k C1 i
:
k C 1 i D0 i
Một số vấn đề xung quanh tổng lũy thừa 65

Thật vậy, ta chứng minh bằng quy nạp. Dễ thấy công thức đúng với k D 0: Giả sử kết quả đã
đúng tới k tự nhiên nào đó, do:
˚ Z
S k C1 .x / D .k C 1/ S k .x /dx C . 1/ k C 1 B k C1 x
S k C 1 .0/ D 0

nên ta có
k  Z
X i kC1
Sk C1 . x / D . 1/ B i x k C1 i
dx C . 1/ k C 1 B k C1 x
i
i D0
k
x k C2 i
 
X i kC1
D . 1/ B i C . 1/ k C 1 B k C 1 x
i kC2 i
i D0
k   !
1 X i kC1 kC2 k C 1
D . 1/ B i x k C 2 i C . 1/ .k C 2/B k C 1 x
kC2 i k C 2 i
i D0
k     !
1 X k C 2 k C 2
D . 1/ i B i x k C 2 i C . 1/ k C 1 B k C1 x
kC2 i kC1
i D0
kX
C1  
1 i kC2
D . 1/ B i x k C2 i
:
kC2 i
i D0

Vậy công thức cũng đúng với k C 1; theo nguyên lý quy nạp nó đúng với mọi số tự nhiên k :
Tóm lại, ta có
k  
1 X i kC1
S k .x / D . 1/ B i x k C1 i :
kC1 i
i D0

Kết hợp công thức này với công thức:

k
. 1/ j .j
 
1 k C1
X 1/Š k
Sk .x / D x x
kC1 j C 1 j
j D1
0 1
k
X X . 1/ k j i     k
X . 1/ j i    
k j k j A xi :
C @
j C1 j i j C1 j i 1
i D2 j Di j Di 1

Bằng cách so sánh hệ số bậc nhất ở hai công thức trên, ta được

k
. 1/ j .j
 
k
X 1/Š k
. 1/ B k D :
j C 1 j
j D1

Do đó,
k
. 1/ k j C1  
X .j 1/Š k
Bk D :
j C1 j
j D1
66 Các phương pháp giải toán qua các kỳ thi Olympic

Kết hợp với công thức:


  k  
n 1 X k
D . 1/ k j
j n;
k k Š j D0 j

ta tìm được một công thức khác để tính trực tiếp số Bernoulli:
n n  
X 1 X i
Bn D . 1/ n j C1
j n:
i .i C 1/ j D 0 j
i D1

Một vài giá trị đầu của đa thức S k .x /:

S0 .x / D x ;
1 1 1
S 1 . x / D x 2 C x D x .x C 1/;
2 2 2
1 3 1 2 1 1
S 2 . x / D x C x C x D x .x C 1/.2x C 1/;
3 2 6 6
1 4 1 3 1 2 1
S 3 . x / D x C x C x D x 2 .x C 1/ 2 ;
4 2 4 4
1 5 1 4 1 3 1 2 1
S4 .x / D x C x C x x D x .x C 1/.2x C 1/.3x 2 C 3x 1/
5 2 3 30 30
1 1 5 4 1 2 1 2
S5 .x / D x 6 C x 5 C x x D x .x C 1/ 2 .2x 2 C 2x 1/;
6 2 12 12 12
1 7 1 6 1 5 1 3 1
S6 .x / D x C x C x x C x
7 2 2 6 42
1
D x .x C 1/.2x C 1/.3x 4 C 6x 3 3x C 1/;
42
1 1 7 6 7 4 1 2
S7 .x / D x 8 C x 7 C x x C x
8 2 12 24 12
1 2
D x .x C 1/ 2 .3x 4 C 6x 3 x 2 4 x C 2/;
24
1 1 2 7 5 2 1
S8 .x / D x 9 C x 8 C x 7 x C x3 x
9 2 3 15 9 30
1
D x .x C 1/.2x C 1/.5x 6 C 15x 5 C 5x 4 15x 3 x 2 C 9x 3/;
90
1 10 1 3 7 6 1 3 2
S9 .x / D x C x9 C x8 x C x4 x
10 2 4 110 2 20
1 2
D x .x C 1/ 2 .x 2 C x 1/.2x 4 C 4x 3 x 2 3x C 3/:
20
Một số vấn đề xung quanh tổng lũy thừa 67

4.3. Một kết quả bên lề


Sử dụng công thức khai triển Abel, ta có

Sk C1 . n / D 1k C1 C 2k C1 C    C nk C1
D 1 k  .1 2/ C .1 k C 2 k /  .2 3/ C    C .1 k C 2 k C    C n k /  n
D nS k .n/ S k .1/ S k .2/    S k .n 1/

D .n C 1/S k . n/ S k .1/ C S k .2/ C    C S k .n/
n k  
X 1 X j kC1
D .n C 1/S k .n/ . 1/ B j i k C1 j
kC1 j
i D1 j D0
k  
1 X j C1 kC1
D .n C 1/S k .n/ C . 1/ Bj Sk C1 j .n/
kC1 j
j D0
1 1
D .n C 1/S k .n/ S k C1 .n/ S k .n/
kC1 2
k  
1 X j C1 kC1
C . 1/ Bj S k C 1 j .n/:
kC1 j
j D2

Từ đó suy ra
1
 Pk j C1 k C1

.k C 1/ n C 2
S k .n/ C j D 2 . 1/ Bj j
S k C1 j .n/
S k C 1 .n/ D ;
kC2
nghĩa là
Pk
1
. 1/ j C 1 B j k C1
 
.k C 1/ x C 2
S k .x / C j D2 j
Sk C1 j .x /
S k C 1 .x / D :
kC2

5. Thay lời kết


5.1. Từ một đẳng thức đẹp
Chúng ta đã biết kết quả đẹp nổi tiếng dưới đây, được gọi là định lý Nicomachus:

1 3 C 2 3 C    C n 3 D .1 C 2 C    C n/ 2 :

Chứng minh đẹp sau đây thuộc về Charles Wheatstone (1854):

13 C 23 C 33 C    C n3
D 1 C .3 C 5/ C    C .n 2 n C 1/ C    C .n 2 C n

1/
D 1 C 3 C 5 C    C n2 C n 1

2
n.n C 1/ 2
 2  
n Cn 1C1
D D
2 2
2
D .1 C 2 C    C n/ :
68 Các phương pháp giải toán qua các kỳ thi Olympic

Một cách viết khác là:


2
S 3 .x / D S 1 .x / :
Liệu có thể có một kết quả tổng quát hơn? Câu trả lời là có.

 Với n lẻ thì S n .x / là một đa thức của S 1 .x /:


Sn .x /
 Với n chẵn và dương thì 2x C1
là một đa thức của S 1 .x /:

Bài toán này xin dành lại cho bạn đọc.

5.2. Một bài toán mở


Tìm tất cả cặp số nguyên dương .m ; n/ để S m .x / là một đa thức của S n .x /:

Tài liệu tham khảo


[1] en.wikipedia.org

[2] diendantoanhoc.net

[3] Trần Nam Dũng (chủ biên), Các phương pháp giải toán qua các kỳ thi Olympic, Kỷ yếu
Gặp gỡ Toán học 2013.
HÀM LỒI MỘT BIẾN VÀ BẤT ĐẲNG THỨC
Nguyễn Hoàng Vinh
(THPT Chuyên Lương Thế Vinh, Đồng Nai)

1. Lý thuyết
Định nghĩa 1. Cho x là số thực thuộc .a; b/ và f .x/ là hàm số khả vi cấp 2: Khi đó nếu:

 f 00 .x/  0 thì f .x/ được gọi là lồi trên .a; b/:

 f 00 .x/  0 thì f .x/ được gọi là lõm trên .a; b/:

Chẳng hạn, các hàm số

f .x/ D e x ; f .x/ D x 2 ; f .x/ D x 4 ; : : :

là hàm lồi trên R: Còn các hàm số như


1
f .x/ D sin x; f .x/ D ln.x/; f .x/ D ; :::
x

là hàm lồi hoặc hàm lõm trên từng khoảng nhất định.
Lưu ý. Nếu không nói gì thêm thì các hàm f .x/ trong toàn bộ bài viết này được quy ước là khả
vi vô hạn lần.

2. Một vài bất đẳng thức liên quan đến hàm lồi – lõm
Định nghĩa 2. Khai triển Taylor của hàm số f .x/ bậc n
n
X f .i / .x0 / f .nC1/ .c/
f .x/ D .x x0 /i C .x x0 /nC1 ;
i D0
iŠ .n C 1/Š

trong đó f .x/ có đạo hàm cấp n C 1 tại x0 và c 2 .x; x0 /; hoặc c 2 .x0 ; x/:

2.1. Bất đẳng thức tiếp tuyến


Định lý 1. Cho hàm số f .x/ lồi trên .a; b/; khi đó với mọi x; x0 thuộc .a; b/ ta luôn có
0
f .x/  f .x0 / C f .x0 /.x x0 /:

69
70 Các phương pháp giải toán qua các kỳ thi Olympic

Cách chứng minh khá đơn giản, xét khai triển Taylor bậc 1 của f .x/ tại x0 2 .a; b/; ta được

1 00
f .x/ D f .x0 / C f 0 .x0 /.x x0 / C f .c/.x x0 /2 :
2

Theo giả thiết, ta có điều phải chứng minh. Tương tự, ta có bất đẳng thức cho hàm lõm

f .x/  f .x0 / C f 0 .x0 /.x x0 /:

Hệ quả 1. Cho f .x/ là hàm lõm trên .a; b/; chứng minh rằng với mọi x1 ; x2 ; x3 2 .a; b/;

f 0 .x1 /.x2 x1 / C f 0 .x2 /.x3 x2 / C f 0 .x3 /.x1 x3 /  0:

Kết quả tương tự cho hàm lồi

f 0 .x1 /.x2 x1 / C f 0 .x2 /.x3 x2 / C f 0 .x3 /.x1 x3 /  0:

Mở rộng 1. Cho f .x/ lõm trên .a; b/ và 2 bộ n số .x1 ; x2 ; : : : ; xn /; .y1 ; y2 ; : : : ; yn / thuộc


.a; b/ thỏa mãn điều kiện
Xn n
X
f .xi /  f .yi /;
i D1 i D1

thì ta luôn có
n
X
f 0 .yi /.xi yi /  0:
i D1

Áp dụng bất đẳng thức tiếp tuyến cho hàm lõm lần lượt tại

.x; x0 / D f.x1 ; y1 /; .x2 ; y2 /; .x3 ; y3 /g ;

rồi cộng theo vế ta có điều cần chứng minh.

Ví dụ 1. Một số bài tập để bạn đọc áp dụng:

1) Chứng minh rằng với mọi số thực dương x; thì

3x 1
ex 1
 :
xC1


 
2) Chứng minh rằng với mọi a; b; c thuộc 0; 2
; ta luôn có

cos a  .b a/ C cos b  .c b/ C cos c  .a c/  0:

3) Cho n số thực dương a1 ; a2 ; : : : ; an ; chứng minh với mọi bộ b1 ; b2 ; : : : ; bn là một hoán


vị của các số đã cho thỏa
Xn
bin .ai bi /  0:
i D1
Hàm lồi một biến và bất đẳng thức 71

4: Cho hai tam giác nhọn ABC và EIF; thỏa mãn


cos A C cos B C cos C  cos E C cos I C cos F:
Chứng minh rằng
sin E  .A E/ C sin I  .B I / C sin F  .C F /  0:
ACBCC
Và nếu đặt S D 3
; thì
E  sin E C I  sin I C F  sin F
S :
sin E C sin I C sin F

Hệ quả 2. Cho hàm số f .x/ lồi trên .a; b/ khi đó với mọi x1 ; x2 ; c thuộc .a; b/ ta luôn có
f .x1 / C f .x2 /  2f .c/ C f 0 .c/  .x1 C x2 2c/:
Áp dụng khai triển Taylor cấp 1 tại x0 D c; ta có
f .x1 / C f .x2 /  2f .c/ C f 0 .c/  .x1 C x2 2c/:
Ví dụ 2. Cho các số dương a; b; c chứng minh rằng
  
ab
a C b  c 2 C ln :
c2
Mở rộng 2. Cho hàm số f .x/ lồi trên .a; b/ khi đó với mọi x1 ; x2 ; : : : ; xn ; c thuộc .a; b/ và
các số dương ˛1 ; ˛2 ; : : : ; ˛n sao cho ˛1 C ˛2 C    C ˛n D 1; thì ta có
n n
!
X X
˛i f .xi /  f .c/ C f 0 .c/ ˛i xi c :
i D1 i D1

Nhận xét. Nếu f .x/ đồng biến trên .a; b/ thì với mọi c  ˛1 x1 C ˛2 x2 C    C ˛n xn ; ta có
n
X
˛i f .xi /  f .c/:
i D1
Pn
Nếu c thỏa f 0 .c/ c  0; thì niD1 ˛i f .xi /  f .c/:
 P
i D1 ˛i xi

2.2. Bất đẳng thức Jesen


Định lý 2. Cho hàm số f .x/ lồi trên .a; b/ khi đó với mọi x1 ; x2 2 .a; b/; và ˛1 ; ˛2 là hai số
dương thỏa mãn ˛1 C ˛2 D 1: Ta luôn có
˛1 f .x1 / C ˛2 f .x2 /  f .˛1 x1 C ˛2 x2 / :
Khai triển Taylor cấp 1 cho hàm số f .x/ tại x0 D ˛1 x1 C ˛2 x2 và cho x D x1 ; x2 ta có
˛1 f .x1 /  ˛1 f .˛1 x1 C ˛2 x2 / C ˛1 ˛2 f 0 .˛1 x1 C ˛2 x2 /.x1 x2 /;
˛2 f .x2 /  ˛2 f .˛1 x1 C ˛2 x2 / ˛1 ˛2 f 0 .˛1 x1 C ˛2 x2 /.x1 x2 /:
Cộng theo vế cho ta điều phải chứng minh.
72 Các phương pháp giải toán qua các kỳ thi Olympic

Mở rộng 3. Cho hàm số f .x/ lồi trên .a; b/ khi đó với mọi x1 ; x2 ; : : : ; xn thuộc .a; b/ và các
số thực dương ˛1 ; ˛2 ; : : : ; ˛n thỏa mãn ˛1 C ˛2 C    C ˛n D 1 thì ta có
n n
!
X X
˛i f .xi /  f ˛i xi :
i D1 i D1

Hệ quả 3. Nếu hàm số f .e x / lồi thì bằng cách đặt ẩn mới ta đưa được về
n n
!
X Y ˛i
˛i f .xi /  f xi :
i D1 i D1

Mở rộng 4 (Bất đẳng thức AM-GM suy rộng). Chứng minh rằng với mọi số thực dương
x1 ; x2 ; : : : ; xn và các số dương ˛1 ; ˛2 ; : : : ; ˛n thỏa mãn ˛1 C ˛2 C    C ˛n D 1; thì ta có
n
X n
Y
˛i xi  xi˛i :
i D1 i D1

Gợi ý. Xét hàm số f .x/ D e x :


Ví dụ 3. Chứng minh rằng với mọi x1 ; x2 ; : : : ; xn thuộc Œ1; C1/ và các số dương ˛1 ; ˛2 ; : : : ; ˛n
thỏa mãn ˛1 C ˛2 C    C ˛n D 1; thì
n
X ˛i 1
 n
:
1 C xi
xi˛i
Q
i D1 1C
iD1

1
Gợi ý. Xét hàm số f .x/ D 1Ce x
:

Hệ quả 4. Cho hàm số f .x/ lồi trên .a; b/ khi đó với mọi x1 < x2 thuộc .a; b/ thì
f .x1 / f .x2 /
.x x1 / C f .x1 /  f .x/; 8x 2 Œx1 ; x2 :
x1 x2
Tương ứng với hàm lõm là có
f .x1 / f .x2 /
.x x1 / C f .x1 /  f .x/; 8x 2 Œx1 ; x2 :
x1 x2

Mở rộng 5. Cho hàm số f .x/ lồi trên .a; b/ khi đó với mọi số thực

x1 < x2 < x3 <    < xnC1 2 .a; b/;

thì
n 1
X xi C1 xi
.f .xi / f .xi C2 // > f .xn / f .x1 /:
i D1
xi xi C2

Ví dụ 4. Chứng minh rằng nếu a  b  c  d thuộc 0; 2 ; ta luôn có


 

.sin a sin c/.b a/.b d / C .sin b sin d /.c b/.a c/  .sin c sin a/.a c/.b d /:
Hàm lồi một biến và bất đẳng thức 73

Ví dụ 5. Chứng minh rằng nếu x1 < x2 < x3 <    < xnC1 là các số thực, thì
n 1
X  xi C1 xi
ximC1 xim > xnm x1m :
iD1
xi xi C2

Hệ quả 5. Cho hàm số f .x/ lõm trên .a; b/ khi đó với mọi số thực dương P x1 ; x2 ; x3 ; : : : ; xn
thuộc .a; b/ và ˛1 ; ˛2 ; : : : ; ˛n thỏa mãn ˛1 C ˛2 C    C ˛n D 1: Đặt S D niD1 ˛i xi thì ta có
n
X
˛i f 0 .xi /.S xi /  0:
i D1

Gợi ý. Theo bất đẳng thức tiếp tuyến

˛i f .S /  ˛i f .xi / C f 0 .xi /  .S
 
xi / ; 8i D 1; 2; : : : ; n:

Cộng theo vế và dùng bất đẳng thức Jensen, ta có điều phải chứng minh.

Ví dụ 6. Chứng minh rằng với mọi a; b; c thuộc 0; 2 ta luôn có


 

cos a  . 2a C b C c/ C cos b  .a 2b C c/ C cos c  .a C b 2c/  0:

Ví dụ 7. Chứng minh rằng với mọi a; b; c dương ta đều có

an  . 2a C b C c/ C b n  .a 2b C c/ C c n  .a C b 2c/  0:

2.3. Bất đẳng thức Kamarata


Khai triển Abel. Cho các số thực a1 ; a2 ; : : : ; an và b1 ; b2 ; : : : ; bn thì ta có đẳng thức
n
X
ai bi D a1 .b1 b2 / C .a1 C a2 /.b2 b3 / C    C .a1 C a2 C    C an / .bn 1 bn /:
i D1

Bộ trội. Cho hai bộ số .a1 ; a2 ; : : : ; an / ; .b1 ; b2 ; : : : ; bn / được sắp thứ tự giảm dần và thỏa
j j
X X
1) ai  bi với mọi j D 1; 2; : : : ; n 1:
i D1 i D1

n
X n
X
2) ai D bi :
i D1 i D1

Khi đó, ta nói bộ .a1 ; a2 ; : : : ; an / trội hơn bộ .b1 ; b2 ; : : : ; bn / và kí hiệu

.a1 ; a2 ; : : : ; an /  .b1 ; b2 ; : : : ; bn / :

Bộ trội không hoàn toàn. 2 bộ số thỏa điều kiện như trên nhưng .a1 ; a2 ; : : : ; an / không  cần
sắp thứ tự, khi đó ta có thể sắp lại thứ tự cho .a1 ; a2 ; : : : ; an / trở thành a1 ; a2 ; : : : ; an thì

a1 ; a2 ; : : : ; an  .b1 ; b2 ; : : : ; bn / :



74 Các phương pháp giải toán qua các kỳ thi Olympic

Khi đó, ta gọi .a1 ; a2 ; : : : ; an / là trội không hoàn toàn so với .b1 ; b2 ; : : : ; bn / và kí hiệu là

.a1 ; a2 ; : : : ; an /  .b1 ; b2 ; : : : ; bn / :

Bất đẳng thức Karamata. Cho hàm số f .x/ lồi trên .a; b/ khi đó, với mọi bộ .a1 ; a2 ; : : : ; an /
và .b1 ; b2 ; : : : ; bn / 2 .a; b/n thỏa .a1 ; a2 ; : : : ; an /  .b1 ; b2 ; : : : ; bn / thì ta luôn có

f .a1 / C f .a2 / C    C f .an /  f .b1 / C f .b2 / C    C f .bn /:

Gợi ý. Ta có
n
X n
X
.f .ai / f .bi //  f 0 .bi / .ai bi / :
i D1 i D1
Pn
Đặt A D i D1 f 0 .bi /.ai bi /; theo khai triển Abel ta có

A D f 0 .b1 / f 0 .b2 / .a1 b1 / C f 0 .b2 / f 0 .b3 / .a1 C a2


   
b1 b2 / C     0:

Kết quả tương tự với hàm lõm

f .a1 / C f .a2 / C    C f .an /  f .b1 / C f .b2 / C    C f .bn /:

Ví dụ 8. Cho các số thực dương a; b; c thì khi đó

8.2a b C 1/.2b c C 1/.2c a C 1/  .a C b C 2/.b C c C 2/.c C a C 2/:

c; 2c a/  aCb cCa bCc



Lấy ln hai vế và lưu ý .2a b; 2b 2
; 2
; 2
:

Hệ quả 6. Bộ .a; b; c/ gọi là gần đều hơn bộ .x; y; z/ nếu chúng thỏa
1) maxfa; b; cg  maxfx; y; zg:

2) minfa; b; cg  minfx; y; zg:

3) a C b C c D x C y C z:
Khi đó, ta có .x; y; z/  .a; b; c/ (trong đó .a; b; c/ là bộ hoán vị của .a; b; c/ đã sắp thứ
tự giảm dần) và nếu f .x/ là hàm lồi trên khoảng chứa a; b; c; x; y; z thì ta có

f .x/ C f .y/ C f .z/  f .a/ C f .b/ C f .c/:

Ví dụ 9. Cho tam giác ABC gần đều hơn tam giác A0 B 0 C 0 (các góc gần đều). Chứng minh
1) sin A C sin B C sin C  sin A0 C sin B 0 C sin C 0 :

2) Với 0 < a < 1; thì


 a  b  c 0
!a 0
!b 0
!c
A B C A B C
sin C sin C sin  sin C sin C sin :
2 2 2 2 2 2
Hàm lồi một biến và bất đẳng thức 75

Hệ quả 7. Cho hàm số f .x/ lồi trên .a; b/ và bộ n số a1 ; a2 ; : : : ; an I b1 ; b2 ; : : : ; bn thỏa


mãn đồng thời các điều kiện
1) ai ; bi thuộc .a; b/; với mọi i D 1; 2; : : : ; n:

2) bi  ai I bi  bi C1 ; với mọi i D 1; 2; : : : ; n 1:

3) a1 C a2 C    C an D b1 C b2 C    C bn :
Thì ta có .a1 ; a2 ; : : : ; an /  .b1 ; b2 ; : : : ; bn / nên

f .a1 / C f .a2 / C    C f .an /  f .b1 / C f .b2 / C    C f .bn /:

Đặt biệt với giả thiết như trên


n   n
X ˛i ai ˇi ai C1 X
f  f .ai /;
i D1
˛i ˇi i D1

Với ˛i ; ˇi 2 RC W ˛i > ˇi ; với mọi i D 1; 2; 3; : : : ; n:


Ví dụ 10. Cho các số dương a1 ; a2 ; : : : ; an chứng minh rằng

.a1 C a2 /.a2 C a3 /    .an C a1 /  .2a1 a2 /.2a2 a3 /    .2an a1 /:

Ví dụ 11. Cho các số dương a1 ; a2 ; : : : ; an chứng minh rằng


a12 a22 an2
    
.1 C a1 /.1 C a2 /    .1 C an /  1 C 1C  1 C :
a2 a3 a1

Gợi ý. Xét hàm f .x/ D ln.1 C e x / với hai bộ

.2 ln a1 ln a2 ; 2 ln a2 ln a3 ; : : : ; 2 ln an ln a1 / ; .ln a1 ; ln a2 ; : : : ; ln an / :

Hệ quả 8. Cho 2n số a1  a2     a2n 2 .a; b/ khi đó gọi b1 ; b2 ; : : : ; b2n là một hoán vị


của các số đã cho. Dễ thấy

.a1 C a2 ; a3 C a4 ; : : : ; a2n 1 C a2n /  .b1 C b2 ; b3 C b4 ; : : : ; b2n 1 C b2n /

nên với mọi hàm số f .x/ lồi trên .a; b/ ta luôn có

f .a1 Ca2 /Cf .a3 Ca4 /C  Cf .a2n 1 Ca2n /  f .b1 Cb2 /Cf .b3 Cb4 /C  Cf .b2n 1 Cb2n /:

Ví dụ 12. Cho các số dương a1  a2      a2n và gọi b1 ; b2 ; : : : ; b2n là một hoán vị của
chúng. Chứng minh rằng

.1 C a1 a2 / .1 C a3 a4 / :::.1 C a2n 1 a2n /  .1 C b1 b2 / .1 C b3 b4 /    .1 C b2n 1 b2n /

Hệ quả 9 (Bất đẳng thức Popoviciu). Cho f .x/ lồi trên .a; b/ và x; y; z 2 .a; b/ ta có
        
xCyCz 4 xCy yCz xCz
f .x/ C f .y/ C f .z/ C f  f Cf Cf :
3 3 2 2 2
76 Các phương pháp giải toán qua các kỳ thi Olympic

Gợi ý. Không mất tính tổng quát, giả sử x  y  z: Đặt


xCyCz xCy xCz yCx
uD ;vD ;mD ;nD ;
3 2 2 2
Thì

.x; x; x; y; u; u; u; y; y; z; z; z/  .v; v; v; v; m; m; m; m; n; n; n; n/:

Áp dụng bất đẳng thức Kamarata ta có điều cần chứng minh.

Ví dụ 13 (Bất đẳng thức Popoviciu mở rộng). Cho hàm f .x/ lồi trên .a; b/ và x; y; z 2 .a; b/
và m; n 2 RC : Ta có
 
xCyCz
f .x/ C f .y/ C f .z/ C f
3
      
4 mx C ny my C mz mx C nz
 f Cf Cf :
3 mCn mCn mCn

Ví dụ 14. Cho x; y; z là các số thực dương, chứng minh rằng

81.x 3 C y 3 C z 3 / C 3.x C y C z/3  4 .2x C y/3 C .2y C z/3 C .2z C x/3 :


 

2.4. Bất đẳng thức xấp xỉ


Định lý 3. Cho hàm số f .x/ lồi trên .a; b/ và f .x/  0; với mọi x 2 .a; b/: Khi đó, với bất
kì bộ n số xỉ thỏa a  x1  x2      xn  b; thì ta luôn có

n 1
X Zb
Œf .a/ C f .b/.b a/  Œf .xi / C f .xi C1 /.xi C1 xi /  2 f .x/dx:
i D1 a

Với hàm lõm ta cũng có tương tự

n 1
X Zb
Œf .a/ C f .b/.b a/  Œf .xi / C f .xi C1 /.xi C1 xi /  2 f .x/dx:
i D1 a

Bất đẳng thức được suy ra từ hình ảnh trực quan và các bất đẳng thức cơ bản của tích phân.

Ví dụ 15. Cho 0  a  b  c: Chứng minh rằng

2
.an C c n / .c a/  .an C b n / .b a/ C .b n C c n / .c c nC1 anC1 :

b/ 
nC1

Ví dụ 16. Cho 0  a  b  c  : Chứng minh rằng

.sin a C sin c/ .c a/  .sin a C sin b/ .b a/ C .sin b C sin c/ .c b/  2 .cos a cos c/ :


Hàm lồi một biến và bất đẳng thức 77

Cho hàm số f .x/ lõm trên .a; b/ và f .x/  0; với mọi x 2 .a; b/: Gọi A; B là hai điểm có tọa
độ A.a; f .a//; B.b; f .b// và A0 ; B 0 là hình chiếu của chúng lên trục Ox; lấy C.c; f .c// bất kì
trên đồ thị của f .x/ bị giới hạn bởi hai đường x D a và x D b: Khi đó, ta có bất đẳng thức sau

Zb
SABB 0 A0 C SABC  f .x/dx:
a

Rõ ràng, bất đẳng thức này chặt hơn bất đẳng thức xấp xỉ dạng I:
Lưu ý. Nếu f .x/ lồi trên .a; b/ thì ta có

Zb
SABB 0 A0 SABC  f .x/dx:
a

Có thể tính được

1
SABB 0 A0 D Œf .a/ C f .b/.b a/;
2
1
SABC D j.a c/Œf .b/ f .c/ .b c/Œf .a/ f .c/j :
2
Vì c thuộc .a; b/ suy ra c D ˛a C ˇb; với ˛; ˇ là hai số không âm thỏa mãn ˛ C ˇ D 1; nên

1
SABC D .b a/ jˇ .f .b/ f .˛a C ˇb// C ˛ .f .a/ f .˛a C ˇb//j
2
1
D .b a/ jˇf .b/ C ˛f .a/ f .˛a C ˇb/j :
2

2.5. Bất đẳng thức xếp xỉ dạng 2


Định lý 4. Với f .x/ lõm thì

1
SABC D .b a/ .f .˛a C ˇb/ ˇf .b/ ˛f .a// :
2
Cho hàm số f .x/ lõm trên .a; b/ và f .x/  0; với mọi x 2 .a; b/: Khi đó, với mọi ˛; ˇ là hai
số không âm thỏa mãn ˛ C ˇ D 1; ta luôn có

Zb
.b a/ Œf .˛a C ˇb/ C ˛f .b/ C ˇf .a/  2 f .x/dx:
a

Với hàm lồi ta có điều tương tự

Zb
.b a/ Œf .˛a C ˇb/ C ˛f .b/ C ˇf .a/  2 f .x/dx:
a
78 Các phương pháp giải toán qua các kỳ thi Olympic

Ví dụ 17. Chứng minh rằng với mọi số a; b; ˛; ˇ  0 thỏa ˛ C ˇ D 1; ˛; ˇ  0 và mọi số tự


nhiên n ta luôn có
2
.b a/ .˛a C ˇb/n C ˛b n C ˇan  b nC1 anC1 :
  
nC1
Ví dụ 18. Chứng minh rằng với mọi số b  a  1; ta có
   
2 1 1 ab C b
.b a/ p C C  4 ln :
1 C ab 1Ca 1Cb ab C a

2.6. Bất đẳng thức đường cong


Định nghĩa 3. Cho hàm số f .x/ lồi trên .a; b/ và f .4/ .x/  0; với mọi x 2 .a; b/: Khi đó,
với mọi x; c 2 .a; b/ ta có
1
f .x/  f .c/ C f 0 .c/.x c/ C f .3/ .c/  .x c/3 :
6
Bất đẳng thức trên gọi là bất đẳng thức đường cong bậc 3:
Kết quả được suy ra dễ dàng từ khai triển Taylor.
Nhận xét. Bất đẳng thức tiếp tuyến là một bất đẳng thức đường cong bậc nhất, nếu
f .3/ .c/  0; 8c 2 .a; b/;
thì bất đẳng thức đường cong bậc 3 chặt hơn bất đẳng thức tiếp tuyến
1
f .x/  f .c/ C f 0 .c/.x c/ C f .3/ .c/  .x c/3  f .c/ C f 0 .c/.x c/:
6
Ví dụ 19. Chứng minh rằng với mọi a không âm, ta luôn có
a3
ea  1 C a C :
6
Mở rộng 6. Nếu hàm số f .x/ có đạo hàm cấp 2; 4; 6; : : : ; 2n không âm trên .a; b/ thì với
mọi x; c 2 .a; b/ ta luôn có
n
X f .2i 1/
.c/
f .x/  f .c/ C .x c/2i 1
:
i D1
.2i 1/Š

Đây gọi là bất đẳng thức đường cong bậc 2n 1:


Với đạo hàm cấp chẵn âm, ta cũng có những bất đẳng thức tương tự.
Lưu ý. Hệ quả ở bất đẳng thức tiếp tuyến đều mở rộng được sang bất đẳng thức đường cong.

Hệ quả 10. Cho hàm số f .x/ có đạo hàm cấp 2; 4 đều không âm trên .a; b/ thì với mọi
xi 2 .a; b/; i D 1; 2; : : : ; n ta luôn có
n
X n
X
6 0
f .xi /.xi C1 xi /  f .3/ .xi /.xi xi C1 /3 :
i D1 i D1

Quy ước xnC1 D x1 :


Hàm lồi một biến và bất đẳng thức 79

Ví dụ 20. Cho 3 số dương a; b; c: Chứng minh rằng

Cn3  n 3
an 1 .b a/Cb n 1 .c b/Cc n 1 .a c/  b/3 C b n 3 .b c/3 C c n 3 .a c/3 :

a .a
n
Ví dụ 21. Cho ba số thực a; b; c: Chứng minh rằng
X
e a .b a/2
 
6 .b a/  0:

Mở rộng 7. Cho f .x/ lõm trên .a; b/ và có đạo hàm cấp 4 âm, lấy hai bộ n số .x1 ; x2 ; : : : ; xn / ;
và .y1 ; y2 ; : : : ; yn / thuộc .a; b/ thỏa mãn
n
X n
X
f .xi /  f .yi /;
i D1 i D1

thì ta luôn có
n  
X 0 1 3
f .yi /.xi yi / C f .3/ .yi /.xi yi /  0:
i D1
6

Hệ quả 11. Cho hàm số f .x/ có đạo hàm cấp 2; 4 không âm trên .a; b/ thì với mọi số thực
xi 2 .a; b/; i D 1; 2; : : : ; n; với mọi c 2 .a; b/; ta luôn có có bất đẳng thức
n n n
X X 1 .3/ X
0
f .xi /  nf .c/ C f .c/  .xi c/ C f .c/  .xi c/3 :
i D1 i D1
6 iD1

Khai triển Taylor tại x0 D c và cho x D xi I i D 1; 2; 3; : : : ; n rồi cộng theo vế.

Ví dụ 22. Cho x1 ; x2 ; : : : ; xn > 1 và thỏa niD1 xi D niD1 xi3 ; chứng minh rằng
P P

n
X xi .2xi C 1/
 0:
i D1
1 C xi

aCbCc
Ví dụ 23. Cho các số thực dương a; b; c: Đặt S D 3
; chứng minh rằng

n.n 1/.n 2/
an C b n C c n  3S n C Sn 3
 .S a/ .S b/ .S c/ :
6

3. Mở rộng bất đẳng thức Jensen


Trường hợp hai biến. Cho hàm số f .x/ có đạo hàm cấp 4 không âm trên .a; b/ : Khi đó, với
mọi x; y 2 .a; b/ ; ta luôn có
   
xCy 1 .2/ x C y
f .x/ C f .y/  2f C f .x y/2 :
2 4 2

Bất đẳng thức được chứng minh dễ dàng bằng khai triển Taylor.
80 Các phương pháp giải toán qua các kỳ thi Olympic

Ví dụ 24. Cho các số thực dương a; b: Chứng minh rằng


1 n.n 1/
an C b n  .a C b/n C .a C b/n 2 .a b/2 :
2n 1 2 n

Ví dụ 25. Cho a; b 2 Œ0;  ; chứng minh rằng


 " #
b/2
  
aCb aCb .a
2 sin  sin a C sin b  sin  2 :
2 2 4

Ví dụ 26. Cho 2 số dương x; y; chứng minh


p 1p
x C y  2 xy C xy  .ln x ln y/2 :
4
Ví dụ 27. Cho các số thực dương a; b; c thỏa a C b C c D 2: Chứng minh rằng
a2 b2 c2 2a 2b 2c
C C  1 C a  ln2 C b  ln2 C c  ln2 :
bCc aCc aCb bCc aCc aCb
Mở rộng 8. Cho hàm số f .x/ có đạo hàm cấp 2n+2 không âm trên .a; b/ : Khi đó, với mọi
x; y 2 .a; b/ ; ta luôn có
  X n  
xCy 2 .2i / x C y
f .x/ C f .y/  2f C 2i
f .x y/2i :
2 i D1
2iŠ2 2

Nhận xét. Bằng phương pháp này, ta làm cho bất đẳng thức càng ngày càng chặt.
Ví dụ 28. Cho a; b 2 Œ0;  ; chứng minh rằng
" #
.a b/2 .a b/4 .a b/6

aCb
sin a C sin b  sin 2 :
2 4 4Š24 6Š26

Ví dụ 29. Cho hai số dương x; y: Chứng minh rằng


 
p 1 1
x C y  xy 2 C  .ln x ln y/2 C .ln x ln y/ 4
:
4 384
Trường hợp ba biến. Cho hàm số f .x/ có đạo hàm cấp 4 không âm trên .a; b/ : Khi đó, với
mọi x; y; z 2 .a; b/ ; ta luôn có
1 X 1
f .x/ C f .y/ C f .z/  3f .S/ C f .2/ .S/ .a S/2 C f .3/ .S/  .a S/.b S/.c S/:
2 2
aCbCc
Trong đó S D 3
:
.3/
Nhận xét. Nếu f .S/ D 0; ta có một bất đẳng thức đẹp hơn
1
f .x/ C f .y/ C f .z/  3f .S/ C f .2/ .S/ .S a/2 C .S b/2 C .S c/2 :
 
2
Ví dụ 30. Cho các số thực x; y; z có tổng là 3: Chứng minh rằng
x 4 C y 4 C z 4  3 C 6 .x 1/2 C .y 1/2 C .z 1/2 C 12.x
 
1/.y 1/.z 1/:
Ví dụ 31. Cho x; y; z 2 Œ0;  thỏa mãn x C y C z D 3
2
: Chứng minh rằng
 2 
1  2   2  
sin x C sin y C sin z  3 x C y C z :
2 2 2 2
Hàm lồi một biến và bất đẳng thức 81

Tài liệu tham khảo


[1] Nguyễn Đình Trí, Tạ Văn Đĩnh, Nguyễn Hồ Quỳnh, Toán Học Cao Cấp - Tập 2: Phép Tính
Giải Tích Một Biến Số, Nhà Xuất Bản Giáo Dục.

[2] Phạm Kim Hùng, Sáng Tạo Bất Đẳng Thức, Nhà Xuất Bản Hà Nội.

[3] Đoàn Quỳnh, Trần Nam Dũng, Hà Huy Khoái, Đặng Hùng Thắng, Nguyễn Trọng Tuấn, Tài
Liệu Giáo Khoa Chuyên Toán Lớp 12, Nhà Xuất Bản Giáo Dục.
82 Các phương pháp giải toán qua các kỳ thi Olympic
MỘT SỐ KỸ THUẬT SỬ DỤNG ĐỊNH LÝ
GIỚI HẠN KẸP CỦA DÃY SỐ
Kiều Đình Minh
(THPT Chuyên Hùng Vương, Phú Thọ)

Định lý giới hạn kẹp (SLT: Sandwich Limit Theorem) là một trong những định lý cơ bản, quan
trọng và hay của Giải tích. SLT cho phép chúng ta tìm giới hạn của dãy số một cách gián tiếp.
Trong bài giảng này, chúng tôi sẽ nêu một số kỹ thuật và tình huống sử dụng SLT thường gặp.
Hy vọng bài giảng sẽ giúp đỡ các bạn áp dụng thành thạo SLT khi giải toán để chuẩn bị tốt cho
các kỳ thi Olympic sắp tới.

Định lý 1 (Định lý giới hạn kẹp – SLT). Cho ba dãy số .an /; .bn /; .cn /: Nếu an  bn  cn ; 8n
và lim an D lim cn D L thì lim bn D L:

Hệ quả 12. Nếu jan j  bn ; 8n và lim bn D 0 thì lim an D 0:

Hệ quả này thường được sử dụng khi dùng định nghĩa giới hạn dãy hoặc dùng định lý Lagrange
với các hàm co.

Định lý 2 (Định lý tương đương). Cho ba dãy số .an /; .bn /; .cn / thoả mãn an  bn  cn ; 8n:
Khi đó các điều kiện sau là tương đương:

i) lim an D lim cn D L:

ii) lim.cn an / D 0 và lim bn D L:

iii) lim an D lim bn D lim cn D L:

Một số kết quả hay dùng:


p
1. Nếu n 2 N ; n  2 thì lim n n D 1:
p
2. Cho a > 0, khi đó lim n a D 1:

3. Nếu jqj < 1 thì lim q n D 0:

Tiếp theo, chúng ta sẽ đi tìm hiểu một số kỹ thuật sử dụng SLT hay gặp. Nói chúng cách phân
chia dạng cũng chỉ là tương đối mà thôi.

1. Kỹ thuật làm trội


p
n
Ví dụ 1 (KSTN TST, ĐHBK HN, 2012). Tính giới hạn: I D lim n2012 C 2012n:

83
84 Các phương pháp giải toán qua các kỳ thi Olympic

Lời giải. Ta có
p
n
p
n
p
n p 2013
1< n2012 C 2012n < n2012 C n2012 < n2013 D n n :
p
n
Mà lim n D 1 nên I D 1.

Ví dụ 2 ((Olympic sinh viên, 2003). Cho dãy .xk / được xác định bởi:

1 2 3 k
xk D C C C  C :
2Š 3Š 4Š .k C 1/Š

x1n C x2n C    C x2003


n
p
Tìm giới hạn J D lim n
:
k 1 1
Lời giải. Dễ thấy .xk / đơn điệu tăng. Từ đẳng thức .kC1/Š
D kŠ .kC1/Š
; ta suy ra

1
xk D 1 :
.k C 1/Š

Do đó
n
x2003 < x1n C x2n C    C x2003
n n
< 2003x2003 :
Từ kết quả này, ta có
p p
n
x2003 < n
x1n C x2n C    C x2003
n
< 2003  x2003 :
1
x1n C x2n C    C x2003
n
p
Suy ra J D lim n
D x2003 D 1 2004Š
.

Ví dụ 3. Cho ˛ > 2. Xétdãy số dương .an / W a1 > 0; an˛ D a1 C a2 C    C an 1 ; 8n  2.


Chứng minh rằng dãy ann có giới hạn hữu hạn và tìm giới hạn đó.

Lời giải. Nhận xét rằng dãy .an / đơn điệu tăng. Do đó:
˛
D an˛ C an D an an˛ 1 C 1 < anC1 an˛ 1
 
anC1 C1
˛ 1
Suy ra anC1 < an˛ 1
C 1; 8n  2: Cộng các bất đẳng thức tương tự lại, ta thu được
˛ 1
anC1 < a2˛ 1
Cn 1:

Từ đó suy ra
 ˛ 1  ˛ 1
anC1 a2 n 1
0< < C :
nC1 nC1 .n C 1/˛ 1

a2 ˛ 1 n 1 anC1 ˛ 1

Vì lim D 0 và lim D 0 nên theo SLT, ta tính được lim D 0 hay
nC1 .nC1/˛ 1 nC1

an
lim D 0:
n
Ví dụ 4. Cho a 2 R. Chứng minh rằng

an
lim D 0:

Một số kỹ thuật sử dụng định lý giới hạn kẹp 85

Lời giải. Chọn m 2 N sao cho m C 1 > jaj. Ta có


n m n m
jajm jm C 1jm
ˇ nˇ  
ˇ a ˇ jaj jaj jaj jaj jaj jaj jaj
0ˇ ˇD
ˇ ˇ        :
nŠ 1 2 m mC1 n mŠ mC1 mŠ mC1
 n m
jaj
Mà lim mC1 D 0 nên theo SLT thì
ˇ nˇ
ˇa ˇ an
lim ˇˇ ˇˇ D 0 ) lim D 0:
nŠ nŠ
Nhận xét. Ta nói nŠ trội hơn an .
1 5 9
Ví dụ 5. Tính giới hạn của dãy số an D 2
 6
 10
   4nC1
4nC2
:
Lời giải. Ta có an D nkD0 4kC2
4kC1
Q
. Đặt:
n n n
Y 4k C 2 Y 4k C 3 Y 4k C 4
bn D ; cn D dn D :
4k C 3 4k C 4 4k C 5
kD0 kD0 kD0
4kC1 4kC2 4kC3 4kC4
Dễ thấy 4kC2
< 4kC3
< 4kC4
< 8k  0: Suy ra an < bn < cn < dn : Do đó:
4kC5
;
r
4 1 4 1
an < an bn cn dn D ) 0 < an < :
4n C 5 4n C 5
Áp dụng SLT, ta được lim an D 0.
Ví dụ 6 (ĐHSPHN, 2011). Cho dãy số .an / được xác định bởi a1 D 1 và:
2n 3
an D
an 1 ; 8n  2:
2n
Đặt bn D niD1 ai ; n  1: Chứng minh rằng .bn / có giới hạn hữu hạn và tìm giới hạn đó.
P

Lời giải. Ta có 2nan D .2n 3/ an 1 nên


an 1 D 2 Œ.n 1/ an 1 nan  ; 8n  2:
Do đó:
n
X n
X
bn D ai D 2 Œiai .i C 1/ ai C1  D 2 Œ1 .n C 1/ anC1  :
i D1 i D1
2n 3 .2n 3/.2n 5/53
Lại có an D 2n
an 1 D 2n.2n 2/42
nên

12 32 52    .2n 3/2 1


an2 < D ;
2/2 1/ 4n2

.22 1/ .42 1/    .2n 1 4n2 .2n
từ đó suy ra
1
an < p :
2n 2n 1
Với kết quả này, ta thu được
 
1
2 1 p < 2 Œ1 .n C 1/ anC1  D bn < 2:
2 2n C 1
Theo SLT, ta có lim bn D 2.
Nhận xét. Có thể chứng minh bằng quy nạp rằng nan  p1 ; 8n  1.
n
86 Các phương pháp giải toán qua các kỳ thi Olympic

2. Sử dụng tổng các bình phương không âm


Ví dụ 7. Giả sử a; b; c 2 R sao cho b 2 4ac < 0 và .un / ; .vn / là hai dãy thoả mãn
au2n C bun vn C cvn2 ! 0 khi n ! C1. Chứng minh rằng un ! 0; vn ! 0, khi n ! C1.
Lời giải. Ta có
2
b2
  
bvn 4ac
au2n cvn2 vn2 :

a C bun vn C D aun C C
2 4
Vì b 2 4ac < 0 nên từ biến đổi trên, ta suy ra
4ac b 2
 
vn2  a au2n C bun vn C cvn2 :

0
4
Theo SLT ta có vn ! 0 khi n ! C1. Chứng minh tương tự (bằng cách nhân c vào và tách bình
phương), ta cũng có un ! 0 khi n ! C1:
Ví dụ 8 (Đề nghị Olympic 30/4/2012). Cho 2 dãy .un / ; .vn / với u1 D 2011; v1 D 2012 và:
   
1 un vn 1 un vn
unC1 D C vn sin un ; vnC1 D C un cos vn :
2 vn2 C 1 2 u2n C 1
Tính giới hạn của hai dãy đã cho.
Lời giải. Áp dụng bất đẳng thức Cauchy-Schwarz, ta có
2 " #
2

1 u n vn 1 v n 2
u2nC1 D u2 C vn2

C vn sin un  2 C sin un
4 vn2 C 1 4 n 2
vn C 1
 
1 2  1 5
un C vn2 u2n C vn2

 C1 D
4 4 16
và 2 " #
2

2 1 u n vn 1 u n
v 2 C u2n 2

vnC1 D C un cos vn  2 C cos vn
4 u2n C 1 4 n 2
un C 1
 
1 2  1 5
un C vn2 u2n C vn2 ;

 C1 D
4 4 16
2 2
trong đó các bất đẳng thức cuối đúng do ta có 2a 2  41 , a2 1  0: Từ đó suy ra
.a C1/
5 2
u2nC1 C vnC1
2
un C vn2 ) lim u2n C vn2 D 0 ) lim un D lim vn D 0:
 

8

3. Sử dụng hệ quả SLT và định nghĩa


Ví dụ 9 (Canada MO, 1985). Cho dãy số .xn / được xácđịnh bởi x1 2 .1; 2/ và:
xn2
xnC1 D 1 C xn ; 8n  1:
2
Chứng minh rằng .xn / có giới hạn hữu hạn và tìm giới hạn đó.
Một số kỹ thuật sử dụng định lý giới hạn kẹp 87

a2
Lời giải. Dễ thấy xn > 0; 8n. Giả sử .xn / có giới hạn là a .a > 0/ thì a D 1 C a ; tức
p p 2
a D 2: Ta chứng minh lim xn D 2. Thật vậy, ta có
ˇ ˇ p2 C x
ˇ ˇ
p ˇˇ ˇ 2 p p
ˇ ˇ ˇ
ˇ x n
ˇ n 2 ˇ
ˇxnC1 2ˇ D ˇ 1 C xn 2ˇ D ˇxn 2ˇ ˇ ˇ:
ˇ ˇ ˇ ˇ ˇ ˇ ˇ
2 ˇ 2 ˇ

Bằng quy nạp, ta chỉ ra được 1 < xn < 23 ; 8n  2. Suy ra


p ˇp ˇ p
2 1 ˇˇ 2 C xn 2 ˇˇ 2 12
<ˇ ˇ< D q < 1:
2 ˇ 2 ˇ 2
ˇ p ˇˇ ˇ p ˇˇ ˇ p ˇˇ
Như vậy, ta có ˇxnC1 2ˇ < q ˇxn 2ˇ <    < q n ˇx1 2ˇ ! 0 khi n ! C1: Từ đó,
ˇ ˇ ˇ
p
suy ra lim xn D 2:
Ví dụ 10 (T11/422 THTT). Cho dãy số .un / được xác định bởi:
u2n 1 1
u0 D a 2 Œ0; 2/ ; un D ; 8n D 1; 2; : : :
n
p 
Tìm giới hạn lim un n .
Lời giải. Ta chỉ ra rằng luôn tìm được số hạng thứ k nào đó của dãy mà uk 2 Œ 1; 1. Thật vậy,
nếu a 2 .0; 1 thì u0 D a 2 .0; 1  Œ 1; 1.
Xét trường hợp a 2 .1; 2/, tức u0 2 .1; 2/. Nếu xảy ra trường hợp mọi un … Œ 1I 1 thì từ hệ
u2 1
thức un D n n1 , ta suy ra un > 1; 8n  1. Lúc này, ta có để ý rằng
n C 2 un n C 1 C un 1 nC1
D > ;
n C 1 un 1 n n
tức là
k C 2 uk kC1
> ; 8k D 1; 2; : : : ; n:
k C 1 uk 1 k
Nhân theo vế của n bất đẳng thức trên với k D 1; 2; : : : ; n; ta thu được
n C 2 un .n C 1/ .n C 2/
> ;
2 u0 2
hay
 
.n C 1/ .n C 2/ 1 2 u0
un < n C 2 .2 u0 / D .n C 1/ .n C 2/ :
2 nC1 2
 
Điều này là không thể, vì khi n đủ lớn n > 2 u0u0 thì vế trái là số dương, còn vế phải là số âm.
u2n 1
Vậy luôn tồn tại k 2 N sao cho uk 2 Œ 1; 1. Khi đó, theo hệ thức un D 1
n
, ta suy ra
1
jun j  ; 8n > k:
n
Do đó
ˇ p ˇ
ˇun nˇ  p1 ; 8n > k:
n
p 
Từ bất đẳng thức này, ta dễ dàng thu được lim un n D 0.
88 Các phương pháp giải toán qua các kỳ thi Olympic

4. Sử dụng các bất đẳng thức cổ điển


c n

Ví dụ 11. Chứng minh rằng lim 1 C n2 D 1; 8c 2 R:

Lời giải. Ở đây, ta sẽ sử dụng bất đẳng thức Bernulli:

.1 C a/n  1 C na; 8a 2 Œ 1; 1 ; n 2 N:
 p 
Khi n rất lớn n  jcj thì njcj2 < 1, ta có
 n
 c n jcj jcj jcj
1C 2  1 1 nD1 :
n n2 n2 n
Mặt khác, ta cũng có
jcj n
 
 c n 1 1 1
1C 2  1C 2 D n D  n  n:jcj
:
n n n2
1 jcj 1
n2 Cjcj n2 Cjcj n2 Cjcj

   
jcj njcj c n

Do lim 1 n
D lim 1 n2 Cjcj
D 1 nên theo SLT, ta có lim 1 C n2 D 1; 8c 2 R.
bn
Nhận xét. Kết quả vẫn đúng nếu ta thay c bằng một đại lượng bị chặn. Chẳng hạn, với c D aCn
thì
 n
b
lim 1 C D 1:
n . a C n/

Ví dụ 12. Tính giới hạn:


n
r !
X k
S D lim 1C 2 1 :
n
k D1

Lời giải. Áp dụng bất đẳng thức AM-GM, ta có


p 1CxC1 x
1Cx 1 1D ; 8x > 0:
2 2
p x
p x
Do 1Cx 1D p
1C x C 1
và 1CxC1<2C 2
nên

p x x 2x
1Cx 1D p > x D :
1CxC1 2C 2
4Cx

Kết hợp với bất đẳng thức ở trên, ta thu được


2x p x
< 1Cx 1< ; 8 x > 0: . /
4Cx 2
k
Áp dụng . / thay x bởi n2
; ta được
n n n
!
2 nk2
r
X X k X k
k
< 1C 2 1 < ;
4C n2
n 2n 2
k D1 k D1 k D1
Một số kỹ thuật sử dụng định lý giới hạn kẹp 89

hay
n n
r !
X 2k X k n . n C 1/
< 1C 2 1 < :
4n 2 C k n 4n 2
k D1 k D1

Mặt khác, ta lại có


n n n n
X k X 2k X k2 X k2
D <
2n 2 4n 2 C k 2n 2 . 4n 2 C k / 8n 4
k D1 k D1 k D1 k D1
n . n C 1/ .2 n C 1/
D ! 0:
48n 4
Suy ra
n n
X 2k X k n .n C 1/ 1
lim 2
D lim 2
D lim 2
D :
4n C k 2n 4n 4
k D1 k D1
1
Do đó, theo SLT thì S D 4
:
Nhận xét.
1. Có thể dùng bất đẳng thức giữa trung bình điều hòa, trung bình nhân và trung bình cộng
để đánh giá:
x 2 p 1CxC1 x
1C D 1
 .1 C x /  1  D1C : . /
2Cx 1Cx
C1 2 2

Từ đó cũng thu được lời giải bài toán.


2x x x2
2. Ta có 4Cx 2Cx
D . 4C x / . 2C x /
> 0; 8 x > 0 nên . / chặt hơn .  /.

Ví dụ 13. Cho a > 0. Xác định dãy số . x n / với:


1
x0 D a ; x nC 1 D x n C ; 8 n  0:
x n2
x
Tìm giới hạn lim pn .
3
n

Lời giải. Ta có
 3
1 3 1
x k3 C 1 D xk C 2 D x k3 C 3 C 3
C 6
> x k3 C 3;
xk xk xk
từ đó suy ra
x n3 > x 03 C 3n ; 8 n  1: .1/
Từ . 1/ suy ra
3 1 1 1
x k3 C1 < x k3 C 3 C C < x k3 C 3 C C :
x 03 C 3k x 03 C 3k
2
k 9k 2


Do vậy, ta có
n
X1 n 1
1 1 X 1
x n3 < x 13 C 3 .n 1/ C C ;
k 9 k2
k D1 k D1
90 Các phương pháp giải toán qua các kỳ thi Olympic

suy ra
n n
X 1 1 X 1
x n3 < x 13 C 3n C C : . 2/
k 9 k2
k D1 k D1

Để ý rằng:
n
X 1 1 1 1 1
2
<1C C CC D2 < 2: . 3/
k 12 23 . n 1/ n n
k D1

Áp dụng bất đẳng thức Cauchy-Schwarz, ta được


n
!2 n n
X 1 X 1 X 1 p
n < 2 n ) < 2 n: .4/
k k2 k
k D1 k D1 k D1

Từ các bất đẳng thức . 1/ ; . 2/ ; . 3/ và . 4/ ; suy ra


r
x 03 x3 x3 2 2
C3< n < 1 C3C C :
n n n n 9n
Chuyển qua giới hạn khi n ! C1, theo SLT ta được

x n3 xn p
3
lim D 3 ) lim p D 3:
n 3
n

5. Sử dụng quy nạp


1
Ví dụ 14 (IMC, 2011). Cho dãy số . a n / W 2
< a n < 1; 8n  0. Xác định dãy . x n / bởi:
a n C1 C x n
x0 D a0 ; x nC 1 D ; 8 n  0:
1 C a nC 1 x n

Chứng minh rằng . x n / hội tụ và tìm giới hạn của dãy đó.
1 1
Lời giải. Ta có 2
< x0 D a0 < 1 ) 0 < 1 x0 < 2
: Ta chứng minh quy nạp:

1
0<1 xn < :
2 n C1
1
Thật vậy, giả sử 0 < 1 xn < 2 nC 1
. Ta có

a nC 1 C x n 1 C a nC 1 x n a nC 1 xn 1 a nC 1
1 x nC 1 D 1 D D .1 xn / :
1 C a nC 1 x n 1 C a nC 1 x n 1 C a nC 1 x n
1 a nC1 1 12 1
Vì 0 < 1 C a nC 1 x n
< 1C 0
D 2
nên

1 1 1 1
0<1 x nC 1 < .1 xn / < : nC 1 D nC 2 :
2 2 2 2
Theo SLT, ta có lim .x n 1/ D 0; từ đó suy ra lim x n D 1.
Một số kỹ thuật sử dụng định lý giới hạn kẹp 91

Ví dụ 15 (Olympic sinh viên, 2005). Cho dãy .x n / xác định bởi x 1 D 5 và x nC 1 D x n2 2


với mọi n D 1; 2 ; : : : Tìm giới hạn:
x nC 1
lim :
x1 x2    xn

Lời giải. Theo giả thiết, ta có


2
2
4 D x n2 4 D x n2 x n2 4 D x n2 x n2 1 x n2 1 4
 
x nC 1 2
D    D x n2 x n2    x 12 x 12 4 D 21. x 1 x 2    x n / 2 :

1

Từ đó suy ra
 2
x nC 1 4
D 21 C :
x1 x2    xn .x1 x2    xn /2
Bằng quy nạp, ta chứng minh được x n > 2 ; 8 n  1: Suy ra
1 1
x1 x2    xn > 2n ; 8n  1 ) 0 < 2
< :
.x 1 x 2    x n / 4n

Theo SLT, ta có
2
p

x nC 1 x nC 1
lim D 21 ) lim D 21:
x1 x2    xn x1 x2    xn

6. Một số kỹ thuật khác


Ví dụ 16 (T9/405 THTT). Cho k 2 N  ; ˛ 2 R. Xét dãy số . a n / với:

b1k ˛ c C b2k ˛ c C    C bnk ˛ c


an D ; 8n D 1; 2 ; : : : ;
nk C1
trong đó bx c là ký hiệu phần nguyên của x. Tìm lim a n .
Lời giải. Ta có x 1 < b x c  x nên

˛ 1k C 2k C    C nk ˛ 1k C 2k C    C nk
 
1
< an  :
nk C1 nk nk C1
Lại có
n  k n
1k C 2k C    C nk
 
1X i 1X i
D D f
n k C1 n i D1 n n i D1 n

với f . x / D x k . Theo định nghĩa tích phân xác định thì

n Z1 Z1
1k C 2k C    C nk
 
1X i 1
lim k C 1
D lim f D f . x / dx D x k dx D :
n n i D1 n kC1
0 0

˛
Do đó, theo SLT thì lim a n D k C1
.
92 Các phương pháp giải toán qua các kỳ thi Olympic

1
Ví dụ 17. Chứng minh rằng lim . 1 C x / x D e :
x !0

Lời giải. Ta sẽ dùng SLT của dãy số để tìm giới hạn này. Trước hết ta chứng minh cho
1
lim .1 C x / x D e :
x !0C

Giới hạn này tương đương với


 y
1
lim 1C D e:
y !C1 y

Ta có bất đẳng thức kẹp


 Œy   y  Œy C1
1 1 1
1C  1C  1C
Œy  C 1 y Œy 

Chuyển qua giới hạn khi y ! C1 và theo định nghĩa


n
e D lim 1 C n1 ta có
1 y
 
lim 1C D e:
y !C1 y
Để chứng minh
 y
1 1
lim .1 C x / x De , lim 1C D e;
x !0 y! 1 y
ta đặt t D y thì có điều cần chứng minh.
˛
Ví dụ 18. Cho a > 1; ˛ > 0. Tìm giới hạn lim na n .

Lời giải. Chọn k 2 N  sao cho k > ˛ với n  k .


Do a > 1 nên đặt a D 1 C q ; q > 0, từ khai triển nhị thức Newton ta có

n .n 1/ ::: . n k C 1/
.1 C q /n  q k

n˛ n˛ k Šn ˛
) D  k
an .1 C q /n q n .n 1/ ::: . n k C 1/
˛
Theo SLT, ta có lim na n D 0.

Nhận xét. Ta nói dãy a n trội hơn dãy n ˛ :

Ví dụ 19. Cho dãy số

1
ln 1 C x n2

. x n / W x 1 D a 2 R I x n C1 D 2002 ; n D 1; 2 ; ::::
2

Tìm giới hạn của dãy số .x n /.


Một số kỹ thuật sử dụng định lý giới hạn kẹp 93

1
ln 1 C x 2

Lời giải. Xét hàm số f . x / D 2
2002 liên tục trên R và có
ˇ ˇ
ˇ f . x / ˇ D ˇ x ˇ  1 ; 8 x 2 R:
ˇ 0 ˇ ˇ ˇ
ˇ 1 C x2 ˇ 2
Xét hàm số g . x / D x f . x / liên tục trên R và có
x2 x C 1
g0 .x / D > 0; 8 x 2 R I g . 0/  g . 2002/ < 0:
x2 C 1
Nên suy ra phương trình f . x / D x có nghiệm duy nhất x D l 2 . 2002I 0 /.
Áp dụng định lý lagrange ta có
1
l j D j f . x n / f .l / j D ˇ f 0 .c / ˇ jx n l j  j x n
ˇ ˇ
jx nC 1 lj
 n 2
1
   j x 1 l j ! 0; n ! C1;
2
với c 2 . x n I l / : Vậy theo hệ quả SLT có lim x n D l .
Nhận xét. Nội dung định lýLagrange được phát biểu như sau: Nếu hàm số f liên tục trên đoạn
Œ a I b  và có đạo hàm trên khoảng .a I b / thì tồn tại c 2 . a I b / sao cho
f .b / f .a /
D f 0 .c / :
b a

7. Sử dụng tính chất nghiệm của phương trình, tính đơn


điệu của hàm số
Ví dụ 20. Chứng minh rằng với mọi n 2 N  phương trình x 3 2 nx C n D 0 có ba nghiệm
phân biệt x 1 < x 2 < x 3 . Đặt ˛ n D x 2 . Tìm lim ˛ n .
Lời giải. Xét f n .x / D x 3 2 nx C n ; n 2 N  , ta có

lim f n . x / D C1; f n . 0/ D n > 0; f n . 1/ D 1 n  0; lim f n . x / D 1:


x !C1 x! 1

Mà f n .x / liên tục trên R nên phương trình có 3 nghiệm x 1 < x 2 < x 3 và 0 < x 2 < 1. Lại

f n 21 D 13 > 0I f n 12 C n1 D n13 C 2 1n 2 C 43n 15
! 185 , khi n ! C1.
 
8

Vậy khi n đủ lớn thì f n 21 C n1 < 0. Do đó




1 1 1 1 1 1 1
< x 2 < C ) < ˛ n < C ) lim ˛ n D :
2 2 n 2 2 n 2
Ví dụ 21 (VMO, 2002-A). Cho n 2 N  . Chứng minh phương trình
1 1 1 1
C C ::: C D
x 1 4x 1 n2 x 1 2
có nghiệm duy nhất x n > 1 và khi n ! C1 thì x n ! 4.
94 Các phương pháp giải toán qua các kỳ thi Olympic

Lời giải. Với mỗi n 2 N  , xét hàm


1 1 1 1
fn .x / D C C ::: C
x 1 4x 1 n2 x 1 2
liên tục và nghịch biến trên . 1I C1/. Hơn nữa
1
lim f n . x / D C1I lim f n .x / D :
x !1C x !C1 2
Do đó với mỗi n 2 N  thì phương trình f n .x / D 0 có nghiệm duy nhất x n > 1. Ta có
1 1 1 1 1
f n . 4/ D C C ::: C D < 0 D fn .xn /
1:3 3:5 .2 n 1/ .2 n C 1/ 2 4n
) x n < 4 ; 8 n  1:
Theo định lý Lagrange thì
1
f n . 4/ j D ˇ f n 0 .c / ˇ jx n
ˇ ˇ
D jf n .x n / 4j
4n
với c 2 . x n I 4/ : Mặt khác
ˇ 0 ˇ
ˇfn .c /ˇ D 1 4 n2 1
2
C 2
C ::: C 2
> :
.c 1/ .4 c 1/ .n2 c 1/ 9
Suy ra
1 9
jxn 4j D ˇ 0 ˇ < ! 0; n ! C1:
4 n ˇfn .c /ˇ 4n
Vậy lim x n D 4.
Nhận xét. Nếu bài toán yêu cầu tìm giới hạn của dãy . x n / thì sẽ khó hơn. Nếu không dùng
định lý Lagrange thì ta xây dựng dãy số u n ! 4 mà f n . 4/ < f n . x n / < f n . u n / ; rồi từ đó
suy ra u n < x n < 4. Có thể xét dãy u n D 4 kn không? k là hằng số sẽ chọn.

Ví dụ 22 (Vĩnh Long, 2011). Xét phương trình x n D x 2 C x C 1; n 2 N ; n > 2 :


a) Chứng minh rằng với mỗi số tự nhiên n > 2 thì phương trình trên có đúng một nghiệm
dương duy nhất.

b) Chứng minh rằng lim x n D 1.


Lời giải. a) Ta thấy rằng nếu x là nghiệm của phương trình x n D x 2 C x C 1 thì x > 1. Với
n  3 cố định, xét hàm

fn .x / D x n x 2 C x C 1 ; x > 1;


ta có
0
f n . x / D nx n 1
. 2x C 1/  3x .2x C 1/ D x 1>0
nên f n . x / đồng biến trên . 1I C1/, do đó f n .x / D 0 có không quá một nghiệm. Hơn
nữa f n . 1/ D 2 và lim f n . x / D C1 và hàm số f n .x / liên tục trên . 1I C1/ nên
x !C1
phương trình f n . x / D 0 có đúng một nghiệm x n > 1.
Một số kỹ thuật sử dụng định lý giới hạn kẹp 95

b) Ta có
f nC 1 . x n / D x nnC 1 x n2 C x n C 1 D x n x n2 C x n C 1 x n2 C x n C 1
  

D x n3 1 > 0 D f nC 1 . x nC 1 / ) x n > x n C1 ; 8 n :
Do . x n / bị chặn dưới  . x n / hội tụ.
 bởi 1 nên
1
Ta chứng minh f n 1 C p n > 0; 8 n  4. Thật vậy:
n
C n1 C n2 p 1 2
    
1 n 1 1
1C p > 1C p C D 1C nC > 1C p C 1C p C1
n n n 2 n n
 
1 1
) fn 1 C p > 0 D f n .x n / ) 1 < x n < 1 C p
n n
Theo SLT thì lim x n D 1.
Nhận xét. Sau khi chỉ ra được . x n / giảm và bị chặn dưới bởi 1 nên ta dự đoán giới hạn của
dãy này bằng 1 và rồi đi xây dựng bất đẳng thức kẹp 1 < x n < 1 C p1n .

8. Xây dựng dãy phụ để kẹp dãy con


Ví dụ 23. Cho dãy số . x n / W x 0 D x 1 D 1; 3x nC 2 D x nC 1 C x n ; 8 n  0. Chứng minh
rằng dãy . x n / có giới hạn hữu hạn và tìm giới hạn ấy.
Lời giải. Với bài toán này chúng ta có thể tìm CTTQ của x n thông qua việc giải phương trình
đặc trưng của dãy. Tuy nhiên ở đấy chúng ta sẽ xây dựng dãy số phụ để kẹp dãy đã cho và dùng
SLT để suy ra giới hạn.
Dễ thấy x n > 0; 8 n  0 và dự đoán giới hạn của dãy bằng không. Vì thế, ý tưởng là xây dựng
dãy . a n / sao cho:
1. Dãy . a n / hội tụ đến 0:
2. max fx 2 n ; x 2 nC 1 g  a n ; 8 n :
Một cách tự nhiên ta chọn dãy . a n / xác định bởi
2a n
a 0 D 1I a nC 1 D ; 8n  0:
3
Ta có a nC 1 a n D 3a n < 0 ) a nC 1 < a n . Vậy . a n / có giới hạn hữu hạn và lim a n D 0
 nC 1
(hoặc thấy ngay a nC 1 D 23 a n D ::: D 23 ! 0; n ! C1).
Ta chứng minh max f x 2 n ; x 2 nC 1 g  a n ; 8n bằng quy nạp. Với n D 0 thì x 0 D 1; x 1 D 1;
a 0 D 1 ) max f x 0 ; x 1 g D a 0 ; khẳng định đúng. Với n D 1 thì x 2 D 23 ; x 3 D 95 ; a 1 D 32 ;
suy ra max fx 2 ; x 3 g D a 1 ; khẳng định đúng.
Giả sử ta đã có max fx 2 n ; x 2 nC 1 g  a n ; 8 n. Khi ấy
3 x 2 nC 2 D x 2 n C 1 C x 2 n  a n C a n D 2a n D 3a nC 1 ) x 2 n C2  a nC 1 . < a n /
3 x 2 nC 3 D x 2 nC 2 C x 2 n C 1  a n C a n D 2a n D 3a nC 1 ) x 2 n C 3  a nC 1 :
Vậy max fx 2 nC 2 ; x 2 nC 3 g  a nC 1 . Ta có 0 < x 2 n ; x 2 n C 1  a n mà lim a n D 0 nên SLT
suy ra lim x 2 n D lim x 2 nC 1 D 0. Do đó lim x n D 0.
96 Các phương pháp giải toán qua các kỳ thi Olympic

Nhận xét. Trong bài toán trên chúng ta đã dự đoán được giới hạn của dãy . x n / bằng 0 và
x n > 0 ; 8 n  0 nên ta lợi dụng điều này và chỉ cần xây dựng một dãy phụ. Có những bài toán
cần phải xây dựng hai dãy phụ để kẹp hai đầu.

Ví dụ 24. Cho a ; b 2 . 0I 1 / và dãy

1 4 2009 p
. x n / W x 0 D a ; x 1 D b I x nC 2 D x nC 1 C
n
x n ; 8 n  0:
2010 2010

Chứng minh rằng dãy .x n / có giới hạn hữu hạn và tìm giới hạn đó.

Lời giải. Tương tự thí dụ trên, ta dự đoán được giới hạn của dãy . x n / bằng 1 và 0 < x n < 1
với mọi n. Bây giờ ta xây dựng dãy . a n / sao cho:

1. . a n / tăng dần tới 1:

2. a n  min fx 2 n ; x 2 nC 1 g ; 8 n  0:

Ta chọn dãy .a n / xác định bởi:

1 2009 p
a 0 D min f a ; b g I a nC 1 D a n4 C n
a n ; 8 n  0:
2010 2010

Xét 2 0 1 0 . a nC 1 a n / > 0 ) . a n / tăng. Bằng quy nạp thì 0 < a n < 1; 8 n  0. Vậy tồn
tại giới hạn hữu hạn lim a n . Từ đó suy ra lim a n D 1.
Tiếp theo ta chứng minh a n  min fx 2 n ; x 2 nC 1 g ; 8 n  0 bằng quy nạp. Với n D 0 thì
x 0 D a ; x 1 D b ) min f x 0 ; x 1 g D min f a ; b g D a 0 ; khẳng định đúng.
Giả sử a n  min f x 2 n ; x 2 n C 1 g ; 8 n  0, ta phải chứng minh a nC 1  min fx 2 nC 2 ; x 2 nC 3 g :
Ta có
1 2009 p 1 2009 p
x 2 nC 2 D x 24 nC 1 C 4
x2n  a n4 C 4
a n D a nC1
2010 2010 2010 2010

1 2009 p 1 2009 p
x2 nC3 D x 24 n C 2 C 4
x 2 nC 1  4
a nC 1 C
4
an
2010 2010 2010 2010
1 2009 p
 a n4 C 4
a n D a nC 1 :
2010 2010
Vậy a n  min fx 2 n ; x 2 nC 1 g < 1; 8 n  0. Áp dụng SLT, ta suy ra

lim x 2 n D lim x 2 nC 1 D 1 ) lim x n D 1:

Ví dụ 25. Cho dãy số . x n / W x 1 ; x 2 > 0 cho trước,


p p
x nC 2 D x n C1 C x n ; 8 n  1:

Tìm lim x n .
Một số kỹ thuật sử dụng định lý giới hạn kẹp 97

Lời giải. Khác với hai thí dụ trên, trong bài toán này ta chưa chặn được dãy . x n / vì thế ta cần
phải xây dựng hai dãy chẵn hai đầu của . x n /. Cụ thể xây dựng hai dãy .a n / ; . b n / như sau:
(
a 1 D max fx 1 ; x 2 ; 4g
p
a nC 1 D 2 a n ; 8n  1

và (
b 1 D min f x 1 ; x 2 ; 4g
p
b nC 1 D 2 b n ; 8 n  1
Khi đó . a n / giảm dần về 4 còn .b n / tăng dần về 4. Thật vậy

p .4 an / an
a 1  4 ; a 2  4 ; ::: ) a n  4I a nC 1 an D 2 an an D p  0:
2 an C an

Từ đó lim a n D 4. Tương tự lim b n D 4. Bằng quy nạp ta có

b n  min fx 2 n ; x 2 nC 1 g  max f x 2 n ; x 2 n C1 g  a n :

Trước hết chứng minh


max f x 2 n ; x 2 nC 1 g  a n :
Với n D 1 thì max fx 2 n ; x 2 nC 1 g D max fx 2 ; x 3 g  a 1 vì
p p p p
x 2  a 1 D max f x 1 ; x 2 ; 4g ; x 3 D x 2 C x 1  a 1 C a 1 D a 2  a 1 :

Giả sử max fx 2 n ; x 2 nC 1 g  a n ; 8n  1. Ta phải chứng minh

max f x 2 n C 2 ; x 2 nC 3 g  a nC 1 :

Thật vậy, ta có
p p p
x2 nC2 D x2 nC1 C x2n  2 a n D a nC 1 .  a n /

và p p
p p p
x 2 nC 3 D x 2 nC 2 C x 2 nC 1  a nC 1 C an  2 a n D a nC 1 :
Tương tự thì b n  min fx 2 n ; x 2 nC 1 g. Do đó theo SLT thì

lim x 2 n D lim x 2 nC 1 D 4 ) lim x n D 4 :

Ví dụ 26. Cho dãy số không âm . u n / thoả mãn điều kiện:


   
1 1 1 1 1 1
4 u nC 2  C C ::: C u nC 1 C C C ::: C un
nC1 nC2 2n 2n 2n C 1 4n

với mọi n  1: Tìm giới hạn lim u n .

Lời giải. Ta có
1 1 1 n
C C ::: C < < 1; 8 n  1I
nC1 nC2 2n nC1
98 Các phương pháp giải toán qua các kỳ thi Olympic

1 1 1 2n C 1 3
C C ::: C <  ; 8 n  1:
2n 2n C 1 4n 2n 2
Như vậy 4 u n C2  u nC 1 C 23 u n ; 8 n  1. Xét dãy

3
.v n / W v 1 D u 1 ; v 2 D u 2 I 4v nC 2 D v nC 1 C v n ; 8n  1
2
có phương trình đặc trưng

3 3 1
4x 2 x D 0 , x1 D ; x2 D
2 4 2
suy ra  n  n
3 1
vn D C1 C C2 ) lim v n D 0
4 2
Tiếp theo ta chứng minh bằng quy nạp v n  u n  0; 8n  1.
Với n D 1 ; 2 thì khẳng định đúng theo định nghĩa của dãy .v n /.
Giả sử khẳng định đã đúng với mọi n  k C 1. Khi đó ta có

3 3
4 vk C2 D vk C1 C v k  u k C 1 C u k  4u k C 2 ) v k C 2  u k C2  0:
2 2
Cuối cùng, áp dụng SLT, ta được lim u n D 0:

Tiếp theo ta nghiên cứu một tình huống hoàn toàn khác và rất thú vị.

Ví dụ 27 (Vĩnh Phúc, 2012). Cho dãy . u n / W u 1 ; u 2 > 0 và


u nC 1
u nC 2 D 1 C ; 8 n  1:
un

Chứng minh rằng .u n / có giới hạn hữu hạn. Tìm lim u n .

Lời giải. Nhận xét rằng:

 u n > 1; 8 n  3
1 1
 u nC 3 D 1 C u nC 1
C un
) u n < 3; 8 n  4 :

Suy ra u n D 1 C uu nn 12 > 43 ; 8 n  6. Do đó muốn chứng minh dãy . u n / có giới hạn thì có


thể xét n  6 bằng cách chứng minh dãy sau hội tụ:
x nC 1
x 0 D u 6 ; x 1 D u 7 I x n C2 D 1 C ; 8 n  0:
xn

Dễ thấy 43 < x n < 3; 8 n  0


1
Từ x nC 3 D 1 C x nC 1
C x1n ta có
ˇ ˇ
ˇ 2 x nC 1 2 x n ˇ
j x nC 3 2j D ˇˇ C ˇ:
2x n C1 2x n ˇ
Một số kỹ thuật sử dụng định lý giới hạn kẹp 99

Đặt y n D j x n 2 j thì 0  y n < 1 .  /


Sử dụng bất đẳng thức
1
ja C b j  jaj C jb j  max . ja j I j b j /
2
4
và điều kiện x n > 3
ta quy về chứng minh dãy sau có giới hạn:

y 0 D a ; y 1 D b ; y 2 D c .0  a ; b ; c < 1/


3
y nC 3  max . y nC 1 ; y n /
4
Xét  2
3 3
y 3 n  max . y 3 n 2 ; y 3 n 3 /  max . y 3 n 4 ; y 3 n 5 ; y 3 n 6/
4 4
 n  n
3 3
 :::  max . y n ; y n 1 ; :::; y 0 / <
4 4
n n
Tương tự có y 3 n C1 < 43 ; y 3 n C2 < 34 .
 
b n c
Vậy 0  y n < 34 3 ) lim y n D 0 ) lim x n D 2 ) lim u n D 2.

Nhận xét. Ta có bài toán tổng quát như sau: Giả sử f W R kC ! R là hàm tăng theo từng
biến và tồn tại số thực dương a > 0 sao cho f . x ; x ; :::; x / > x với 0 < x < a và
f . x ; x ; : : :; x / < x với x > a. Cho x 1 ; x 2 ; :::; x k là các số dương. Định nghĩa dãy truy
hồi . x n / như sau: x n D f . x n 1 ; x n 2 ; :::; x n k / với n > k. Khi đó lim x n D a.

Lời giải. Ta có thể chỉ ra hàm số f liên tục tại điểm . a ; a ; ::::; a / và f . a ; a ; :::; a / D a.
Xét các dãy .a n / ; . b n / được xác định như sau:

a 1 D a 2 D ::: D a k D min fx 1 ; x 2 ; :::; x k g
a n D f . a n 1 ; a n 2 ; ::::; a n k / ; n > k

và 
b 1 D b 2 D ::: D b k D max f x 1 ; x 2 ; :::; x k g
b n D f . b n 1 ; b n 2 ; ::::; b n k / ; n > k
Nếu min f x 1 ; x 2 ; :::; x k g > a thì . a n / là dãy giảm bị chặn dưới bởi a.
Nếu min f x 1 ; x 2 ; :::; x k g < a thì . a n / là dãy tăng bị chặn trên bởi a. Như vậy ta luôn có
. a n / hội tụ và lim a n D a.
Hơn nữa , do hàm f tăng theo từng biến nên x n  a n ; 8 n  1. Tương tự với dãy . b n / thì
lim b n D a và b n  x n ; 8 n  1. Theo SLT suy ra lim x n D a.

9. Sử dụng SLT trong phương trình, bất phương trình hàm


Ví dụ 28 (Turkey TST, 2005). ] Tìm tất cả các hàm số f W Œ 0I C1/ ! Œ 0I C1/ thoả
mãn đồng thời hai điều kiện sau:

a) 4f . x /  3x ; 8 x  0:
100 Các phương pháp giải toán qua các kỳ thi Olympic

b) f . 4f . x / 3x / D x ; 8 x  0:
3 13
Lời giải. Xét hai dãy số . a n / ; . b n / xác định bởi a 1 D ; b1 D và
€
4 12

3b n 1 C 1
an D
4b n 1 8 n > 1:
3a n C 1
bn D
4a n

Ta có thể chứng minh bằng quy nạp bất đẳng thức kẹp

a n x  f . x /  b n x ; 8 x  0; 8 n  1

Lại bằng quy nạp ta có . a n / tăng, . b n / giảm và a n < 1 < b n ; 8 n.


Ngoài ra với n > 1 ta có
 
3 1 1 an 1
0 < 1 an D 1 C D
4 4b n 1 4 1 C 3a n 1
1 an 1 1 an 1 1 an 1
D  D :
4 .1 C 3a n 1 / 4 . 1 C 3a 1 / 13
1
Do đó 0 < 1 an  4 :13n 1 ; 8 n  1. Theo SLT thì lim a n D 1, suy ra lim b n D 1. Do đó

f .x / D x ; 8 x  0:

Câu hỏi. Tại sao lại chọn được hai dãy . a n / ; . b n / như vậy?

Ví dụ 29 (China MO). Tìm tất cả các hàm số f W Œ 1I C1/ ! Œ 1I C1/ thoả mãn đồng
thời các điều kiện sau:
x C1
a) 2
 f . x /  2 . x C 1/ ; 8 x  1

b) xf . x C 1/ D .f .x // 2 1; 8 x  1:

Lời giải. Thay x bởi x C 1 trong a) ta có


xC2
 f . x C 1/  2 . x C 2/ : .c/
2
Từ b) suy ra
xf . x C 1/ C 1 D . f . x // 2 ; 8x  1
1
) C xf . x C 1/ < . f . x // 2 < 2 C xf . x C 1/ ; 8 x  1: .d/
2
Từ c),d) ta có
1 C x . x C 2/
< . f . x // 2 < 2 . 1 C x .x C 2//
2
. x C 1/ 2
, < .f .x // 2 < 2 . x C 1/
2
1 p
, p . x C 1/ < f . x / < 2 .x C 1/ ; 8 x  1: .e/
2
Một số kỹ thuật sử dụng định lý giới hạn kẹp 101

Bằng quy nạp, áp dụng e) và cách lập luận trên k lần ta được

1 k p k
 
p .x C 1/ < f . x / < 2 . x C 1/ :
2
Với mỗi x  1, chuyển qua giới hạn bất đẳng thức trên khi k ! C1 và sử dụng SLT ta có
x C 1  f . x /  x C 1 ) f . x / D x C 1:
Thử lại thoả mãn. Vậy f . x / D x C 1; 8 x  1.
Nhận xét. Liên quan đến bài toán này ta có bài toán sau: Tìm giới hạn của dãy số . x n /, biết
v s
u r
p
u q
t
xn D 1 C 2 1 C 3 1 C ::: C 1 C . n 1/ 1 C n; n  1:

Xét hàm số
p
r q xC1 p
f .x / D 1Cx 1 C .1 C x /
p ::: )
< f . x / < 2 . x C 1/
2
 n p n
1
) p . x C 1/ < f .x / < 2 . x C 1/
2
 n p n
Cho x D 2 ta được 3 p12 < xn < 3 2 . Áp dụng SLT thì lim x n D 3.

Ví dụ 30. Tìm tất cả hàm f W N  ! N  thoả mãn các điều kiện sau:
a) f . 2/ D 4 :
b) . n C L / f . n/  nf . n C L / ; 8 n ; L 2 N  :
c) f . m n/ D f . m/ f . n/ ; 8 m ; n 2 N  :
C 1
Lời giải. Ta có N  D 2 k I 2 k C 1 . Quy nạp ta được f 2k D 22k .
S   
k D0
k k C1
 
Xét n 2 2 I 2 , từ b) suy ra

f 2k f 2k C1
 
f .n/ f .n C L/ f . n/
 )  
n nCL 2k n 2k C1
22k f . n/ 2 2 . k C 1/ 1 f . n/
)   )   2:
2 k :2 k C 1 n2 2 k :2 k C 1 2 n2
Do đó
1 f .n /  1 f .n m /
  2 ; 8 n 2 N )  2
 2 ; 8n 2 N
2 n2 2 m
.n /
m r
p

1 f . n/  m 1 f .n / m
)   2 ; 8 n ; m 2 N )   2:
2 n2 2 n2
f . n/
Với mỗi n 2 N  , Chuyển qua giới hạn khi m ! C1, theo SLT ta được n2
D 1; từ đó suy
ra f . n/ D n 2 ; 8 n 2 N  .
102 Các phương pháp giải toán qua các kỳ thi Olympic

Ví dụ 31 (T6/216 THTT). Tìm tất cả các hàm số liên tục f W Œ 0I 1 ! R thoả mãn

f .x /  2xf x 2 ; 8 x 2 Œ 0I 1 :

. 1/

Lời giải. Cho x D 0 thì f . 0/  0: Cho x D 1 thì

f . 1/  2f . 1/ , f . 1/  0: . 2/

Với 0 < x < 21 , sử dụng . 1/ n lần ta được


n n
f .x /  2xf x 2  2 2 x 3 f x 4  :::  . 2x / n x 2 n 1
x2 ; 8 n 2 N  : . 3/
  
f

Vì x 2 0I 21 và f liên tục nên




n n
. 2x / n :x 2 n 1
x2
 
lim :f D 0:f . 0/ D 0: . 4/
n! C 1

Từ . 3/ và . 4/ cho ta  
1
f . x /  0; 8 x 2 0I : . 5/
2
Mặt khác với x 2 . 0I 1/ thì từ . 1/, ta có

p 
 1 
p  p f x f x 2n
f x  2 x f .x / ) f .x /  p  :::  1
: . 6/
2 x 2 n x 1 2n
Mà  
1
f x2
n
lim 1
D0
n! C 1 2n x 1 2n

nên từ . 6/, ta có
f .x /  0; 8 x 2 . 0I 1/ : . 7/
 1
Từ . 5/ và . 7/ cho ta f . x / D 0; 8 x 2 0I 2 .
2n
1
Với mỗi x 2 2 I 1 tồn tại n 2 N để x < 12 thì



n n
f .x /  2n x 2 1
x2

f D 0:

Do đó  
1
f . x /  0; 8 x 2 I1 : . 8/
2
Từ . 7/ ; . 8/ suy ra  
1
f .x / D 0; 8 x 2 I1 :
2
Tóm lại f . x / D 0; 8 x 2 Œ 0I 1/.
Vì hàm f liên tục trên Œ 0I 1 nên với dãy . x n / 2 Œ 0I 1/ sao cho lim x n D 1 thì

f . 1/ D f .lim x n / D lim f . x n / D lim f . 0/ D 0:

Vậy f . x / D 0; 8 x 2 Œ 0I 1, thử lại thoả mãn.


Một số kỹ thuật sử dụng định lý giới hạn kẹp 103

Ví dụ 32. Tìm f W Œ 0I 1 ! Œ 0I 1 thoả mãn các điều kiện sau:

a) 2 x f . x / 2 Œ0I 1 ; 8 x 2 Œ 0I 1

b) f . 2 x f . x // D x ; 8 x 2 Œ 0I 1 :

Lời giải. Xét dãy số . x n / W x n D nx . n 1/ f . x / ; 8 n  1.


Bằng quy nạp được
(
x n 2 Œ 0I 1
E
f . x n / D x n 1 ; 8 n D 2 ; 3; :::

Do 0  x n  1; 8 n  1 nên 0  nx .n 1 / f . x /  1; 8 n suy ra

nx 1 nx
nx 1  .n 1/ f . x /  nx )  f .x /  ; 8 n > 1:
n 1 n 1

Với mỗi x 2 Œ 0I 1 cho n ! C1, ta được f . x / D x.


Thử lại f . x / D x ; 8 x 2 Œ 0I 1 là nghiệm của phương trình đã cho.

Ví dụ 33. Cho dãy các hàm số . f n . x // xác định bởi:

x f n2 . x /
f 0 .x / D 0I f nC 1 . x / D f n .x / C ; 8 n  0; x 2 Œ 0I 1 :
2
Tìm giới hạn lim f n . x /.
n! C 1

Lời giải. Trước hết ta chứng minh bằng quy nạp bất đẳng thức
p
0  fn .x /  x ; 8 n  0: . 1/
p p
Thật vậy, với x 2 Œ 0I 1  ) x 2 x  0 ) 0  x2 D f 1 . x /   x ; suy ra .1/ đúng
khi n D 1. Giả sử . 1/ đúng đến n. Xét hàm số g . t / D t C 12 x t 2 ; t 2 Œ 0I 1 có

g0 .t / D 1 t  0; 8 t 2 Œ 0I 1

nên g . t / đồng biến trên Œ 0I 1. Theo giả thiết quy nạp thì
p
0  f n .x /  x  1; 8 x 2 Œ 0I 1 . 2/
p  p
nên f nC 1 . x / D g . f n . x //  g x D x ; 8 x 2 Œ0 I 1 : Mặt khác, từ . 2/ có

x f n2 .x /  0 ) f nC 1 . x /  f n . x /  0:
p
Do vậy 0  f n C1 . x /  x, tức .1 / đúng đến n C 1. Theo nguyên lý quy nạp thì có . 1/.
Tiếp theo ta chứng minh

p 2
x fn .x / < ; 8 x 2 Œ 0I 1 ; 8 n  0:
nC1
104 Các phương pháp giải toán qua các kỳ thi Olympic

Thật vậy, ta có
p
p p
 
 x C f n 1 .x /
x fn .x / D x fn 1 .x / 1
2
p  p n
p
 
p  x  x
 x fn 1 .x / 1  :::  x f0 .x / 1
2 2
2 npx  p  3 nC 1
p  p n x
2 n x x 24 2 C n 1 2
D : 1  5
n 2 2 n nC1
 n
2 n 2
D < :
nC1 nC1 nC1
p 2
Vậy 0  x f n .x / < nC 1
; 8 x 2 Œ0 I 1 ; 8 n  0. Theo SLT, ta có
p
lim f n . x / D x ; 8 x 2 Œ0 I 1 :
n! C 1

Ví dụ 34 (ĐHKHTN HN, 2010). Tìm tất cả các hàm số f W R C ! R C sao cho


f .xy /
x 3 C y D . f . x // 3 C ; 8x ; y 2 RC :

f
f .x /
Lời giải. Cho y D 1 ta được
f .x C 1 / D . f . x // 3 C 1: . 1/
Cho x D 1 ta được
f .y /
f . y C 1/ D .f .1 // 3 C : . 2/
f . 1/
Đặt f . 1/ D a và sử dụng . 1/ ta lần lượt tính được
3
f .2/ D a 3 C 1; f . 9/ D f 23 C 1 D a3 C 1 C 1

. 3/
Mặt khác sử dụng . 2/ta có
f . 2/ 1 f . 3/ 1
f . 3/ D a 3 C D a 3 C a 2 C ; f . 4/ D a 3 C D a3 C a2 C a C 2 ;
a a a a
1
f . 5/ D a 3 C a 2 C a C 1 C 3 ; :::;
a
1 1 1 1 1
f . 9/ D a 3 C a 2 C a C 1 C C 2 C 3 C 4 C 7 : . 4/
a a a a a
Từ . 3/ và . 4/ suy ra
3 1 1 1 1 1
a3 C 1 D a3 C a2 C a C 1 C C 2 C 3 C 4 C 7
a a a a a
1/ a 3

, .a a C 1 g . a / D 0;
trong đó
g . a / D a 1 2 C a 1 1 C 2a 1 0 C 4a 9 C 5a 8 C 6a 7 C 7a 6 C 6a 5 C 5a 4 C 4a 3 C 3a 2 C 2 a C 1:
Giải ra ta tìm được a D 1. Vậy ta có
Một số kỹ thuật sử dụng định lý giới hạn kẹp 105

f . x C 1/ D f . x / C 1 và f . x / D . f . x // 3 .
 
 p C 2 3

Từ đây suy ra f . x C n / D f . x / Cn ; 8 n 2 N . Với r D q
2 Q , ta tính f r Cq
bằng hai cách như sau: Ta có
  
2 3
 3 3
r C q2 D f .r / C q 2 :

f r Cq D f

Mặt khác:

p2 2
 

2 3
 
3 p 4 6
f r Cq Df r C 3: 2 :q C 3: :q C q
q q
r C 3p C 3p q C q 6
3 2 3

Df
r 3 C 3p 2 C 3p q 3 C q 6 D . f .r // 3 C 3p 2 C 3p q 3 C q 6 :

Df

Từ đây ta được phương trình

q 2 . f . r // 2 C q 4 f . r / D p 2 C p q 3 :
p
Giải phương trình này, với chú ý f . r / > 0, ta được f . r / D q
D r.
Vậy f .r / D r; 8 r 2 Q C . Bây giờ để ý rằng với mọi x ; y > 0; ta có
 p 3 p 
3
 p
3
 3 f 3 x :y
f .x C y / D f x Cy D f x C p 
f 3 x
p 
f 3 x :y
D f .x / C p  > f .x / :
f 3 x

Suy ra f . x / là hàm tăng trên R C . Cuối cùng, với mỗi x 2 R C , xét các dãy số hữu tỷ
. u n / ; . v n / sao cho . u n / tăng và dần về x (suy ra u n  x ; 8 n), .v n / giảm và dần về x (suy
ra v n  x ; 8 n). Khi đó ta có bất đẳng thức kẹp:

un D f .un /  f .x /  f .vn / D vn :

Chuyển qua giới hạn khi n ! C1, theo SLT ta được

f .x / D x ; 8 x 2 R C :

Thử lại thoả mãn bài toán.

Tài liệu tham khảo


[1] Đoàn Quỳnh (chủ biên), Tài Liệu Chuyên Toán – Lớp 11, NXB GD, 2014.

[2] Nguyễn Văn Mậu, Chuyên đề chọn lọc Dãy số và áp dụng, NXB GD, 2008.

[3] Các tài liệu trên mạng.


106 Các phương pháp giải toán qua các kỳ thi Olympic
ĐỊNH LÝ RUF VÀ HÀM SINH
Trần Minh Hiền
(THPT Chuyên Quang Trung, Bình Phước)

1. Một số kiến thức bổ trợ


Định nghĩa 1. Hàm sinh thường của dãy số vô hạn ha0 ; a1 ; : : :i là chuỗi luỹ thừa hình thức
P .x/ D a0 C a1 x C a2 x 2 C a3 x 3 C   
Ta gọi hàm sinh là chuỗi hình thức bởi vì thông thường ta sẽ chỉ coi x là một ký hiệu thay thế
thay vì một số. Chỉ trong một vài trường hợp ta sẽ cho x nhận các giá trị thực, vì thế ta gần như
cũng không để ý đến sự hội tụ của các chuỗi.
Định nghĩa 2. Với u là một số thực và k là số nguyên không âm. Lúc đó hệ số nhị thức mở
u

˚
rộng k được định nghĩa như sau
! u.u 1/ : : : .u k C 1/
u ; k > 0;
D kŠ
k 1; k D 0:

Định lý 1. Cho x là một số thực với jxj < 1, u là một số thực. Khi đó:
1
!
X u
.1 C x/u D xk :
k
kD0

n n
Ví dụ 1. Tìm khai triển luỹ thừa của các hàm sinh .1 C x/ và .1 x/
Lời giải. Theo định lý nhị thức mở rộng thì:
1
!
n
X n k
.1 C x/ D x :
k
kD0

Theo định nghĩa


!
n . n/. n 1/ : : : . n k C 1/ n.n C 1/ : : : .n C k 1/
D D . 1/k  D . 1/k CnCk
k
1
k kŠ kŠ

Từ đó
1
X
n
.1 C x/ D . 1/k CnCk
k
1x
k

kD0

Thay x bằng x, ta được


1
X
n k k
.1 x/ D CnCk 1x :
kD0

107
108 Các phương pháp giải toán qua các kỳ thi Olympic

Thông thường ta có thể dịch các bài toán đếm thẳng sang ngôn ngữ hàm sinh để giải. Ý tưởng
được thể hiện thông qua định lý đơn giản dưới đây.

Định lý 2. Hàm sinh của số các cách chọn phần tử phân biệt từ tập hợp n phần tử là .1Cx/n .

Chứng minh.

1. Đầu tiên, ta hãy xét tập hợp có một phần tử fa1 g. Hàm sinh cho số cách chọn các phần tử
từ tập hợp này đơn giản là 1 C x. Thật vậy, ta có 1 cách chọn không phần tử nào, 1 cách
chọn 1 phần tử và 0 cách chọn hai phần tử trở lên. Tương tự, số cách chọn n phần tử từ tập
hợp fa2 g cũng cho bởi hàm sinh 1 C x. Sự khác biệt của các phần tử trong hai trường hợp
trên là không quan trọng.

2. Và bây giờ là ý tưởng chính: hàm sinh cho số cách chọn các phần tử từ hợp của hai tập
hợp bằng tích các hàm sinh cho số cách chọn các phần tử từ mỗi tập hợp. Chúng ta sẽ giải
thích chặt chẽ điều này, nhưng trước hết, hãy xem xét một ví dụ. Theo nguyên lý này, hàm
sinh cho số cách chọn các phần tử từ tập hợp fa1 ; a2 g là:

.1 C x/.1 C x/ D 1 C 2x C x 2

Có thể kiểm chứng rằng đối với tập hợp fa1 ; a2 g ta có 1 cách chọn 0 phần tử, 2 cách chọn
1 phần tử, 1 cách chọn 2 phần tử và 0 cách chọn 3 phần tử trở lên.
Tiếp tục áp dụng quy tắc này, ta sẽ được hàm sinh cho số cách chọn các phần tử từ tập hợp
n phần tử
.1 C x/.1 C x/ : : : .1 C x/ D .1 C x/n :
Đây chính là công thức hàm sinh mà ta đã nhận được bằng cách sử dụng định lý nhị thức.
k
 k phần tử phân biệt, từ tập hợp có n phần tử chính là hệ số x
Từ đây suy ra số cách chọn
n
của hàm sinh, tức là k .

Trong chứng minh trên, chúng ta đã sử sụng quy tắc sau đây.

Định lý 3 (Quy tắc xoắn). Gọi A.x/ là hàm sinh cho cách chọn các phần tử từ tập hợp A và
B.x/ là hàm sinh cho cách chọn các phần tử từ tập hợp B. Nếu A và B là rời nhau thì hàm sinh
cho cách chọn các phần tử từ A [ B là A.x/:B.x/.

Chứng minh. Định nghĩa:


1
X 1
X 1
X
n n
A.x/ D an x ; B.x/ D bn x ; C.x/ D A.x/:B.x/ D cn x n :
nD0 nD0 nD0

Đầu tiên ta hãy tính tích A.x/:B.x/ và biểu diễn hệ số cn thông qua các hệ số an và bn . Ta có
thể sắp xếp các số hạng này thành dạng bảng
b0 b1 x b2 x 2 b3 x 3 
a0 a0 b0 a0 b1 x a0 b 2 x 2 a0 b3 x 3 
a1 x a1 b 0 x a1 b 1 x 2 a1 b 2 x 3 a1 b3 x 4 
a2 x 2 a2 b0 x 2 a2 b 1 x 3 a2 b 2 x 4 a2 b3 x 5 
a3 x 3 a3 b0 x 3 a3 b 1 x 4 a3 b 2 x 5 a3 b3 x 6 
     
Định lý Ruf và hàm sinh 109

Chú ý rằng các số hạng có cùng luỹ thừa của x xếp trên các đường chéo /. Nhóm tất cả các số
hạng này lại, ta thấy rằng hệ số của xn trong tích bằng

c n D a0 b n C a1 b n 1 C    C an b0

Bây giờ ta chứng minh rằng đây cũng chính là số cách chọn n phần tử từ A [ B. Một cách tổng
quát, ta có thể chọn n phần tử từ A [ B bằng cách chọn j phần tử từ A và n j phần tử từ B,
trong đó j là một số từ 0 đến n. Điều này có thể được thực hiện bằng aj bn j cách. Lấy tổng từ 0
đến n, ta có
a0 bn C a1 bn 1 C    C an b0
cách chọn n phần tử từ A [ B. Đó chính xác là giá trị cn đã được tính ở trên.

Định lý 4. Hàm sinh của số các cách chọn một số phần tử, có lặp, từ tập hợp n phần tử là
1
.k  n/:
.1 x/n

Chứng minh. Giả sử ta chọn n phần tử (có lặp) từ tập hợp chỉ có duy nhất một phần tử. Khi đó
có 1 cách chọn 0 phần tử, 1 cách chọn 1 phần tử, 1 cách chọn 2 phần tử . . . Như thế, hàm sinh
của cách chọn có lặp từ tập hợp có 1 phần tử bằng
1
h1; 1; 1; 1; : : :i $ 1 C x C x 2 C x 3 C    D
1 x
Quy tắc xoắn nói rằng hàm sinh của cách chọn các phần tử từ hợp của các tập hợp rời nhau bằng
tích của các hàm sinh của cách chọn các phần tử từ mỗi tập hợp:
1 1 1 1
  D
1 x 1 x 1 x .1 x/n
1
Như thế, hàm sinh của cách chọn các phần tử từ tập hợp n phần tử có lặp là . Bây giờ ta
.1 x/n
cần tính các hệ số của hàm sinh này. Theo kết trong ví dụ 1.4 thì hệ số của x k trong hàm sinh
bằng:
k
CnCk 1;
k
tức số cách chọn k phần tử có lặp từ n phần tử bằng CnCk 1

Định nghĩa 3. Nếu A D .a0 ; a1 ; a2 ; : : :/ là dãy số nguyên. Khi đó

PA .x/ D x a0 C x a1 C x a2 C   

là hàm sinh loại 2 (hàm sinh đa thức) liên kết với A. Nếu tất cả các phần tử của A không âm, thì
PA .x/ là một đa thức. Trong trường hợp ngược lại thì PA .x/ là thương của hai đa thức. Hàm
sinh loại 2 này giải quyết hữu hiệu cho nhưng bài toán tổng của hai dãy số nguyên.

Định lý 5. Đặt A D .a1 ; a2 ; : : : ; am / và B D .b1 ; b2 ; : : : ; bn / là hai dãy số nguyên với hai


hàm sinh liên kết là PA .x/; PB .x/. Khi đó dãy

C D fci;j D ai C bj j1  i  m; 1  j  ng
110 Các phương pháp giải toán qua các kỳ thi Olympic

có hàm sinh liên kết là


PC .x/ D PA .x/:PB .x/ :

Đặc biệt hệ số của x k trong PC .x/ là số các cặp có thự tự số nguyên .ai ; bj /, với ai 2 A và
bj 2 B, sao cho ai C bj D k .

Chứng minh. Ta có
m
X n
X
ai
PA .x/ D x ; PB .x/ D x bj :
i D1 j D1

Khi đó
m
X n
X X
ai
PA .x/:PB .x/ D x  x bj D x ai Cbj D PC .x/:
i D1 j D1 1i m
1j n

Chú ý là lũy thừa x k xuất hiện trong PC .x/ khi và chỉ khi tồn tại cặp ai 2 A; bj 2 B sao cho
k D ai C bj , và mỗi cặp như vậy đóng góp chính xác một lần xuất hiện x k trong PC .x/.

Ví dụ 2. Giả sử với mỗi số tự nhiên n có hai dãy số dương a1 ; a2 ; : : : ; an và b1 ; b2 ; : : : ; bn sao


cho dãy tổng a1 C a2 ; a1 C a3 ; : : : ; an 1 C an là một hoán vị của dãy các tổng b1 C b2 ; b1 C
b3 ; : : : ; bn 1 C bn . Chứng minh rằng n là lũy thừa của 2.

Lời giải. Đặt A D .a1 ; a2 ; : : : ; an / và B D .b1 ; b2 ; : : : ; bn /, xét các hàm sinh liên kết là
n
X n
X
ai
PA .x/ D x ; PB .x/ D x bi :
i D1 j D1

Ta có:
X X X
ŒPA .x/2 D x 2ai C x ai Caj D PA .x 2 / C x ai Caj
ai 2A ai ;aj 2A ai ;aj 2A
X X X
ŒPB .x/2 D x 2bi C x bi Cbj D PB .x 2 / C x bi Cbj
bi 2B bi ;bj 2B bi ;bj 2B
2 2 2 2
) ŒPA .x/ ŒPB .x/ D PA .x / PB .x /:

Vì PA .1/ D PB .1/ D n do đó 1 là nghiệm của PA .x/ PB .x/ suy ra

PA .x/ PB .x/ D .x 1/k :H.x/; k 2 N


PA2 .x/ PB2 .x/ .x 2 1/k H.x 2 / 2
k H.x /
) PA .x/ C PB .x/ D D D .x C 1/ :
PA .x/ PB .x/ .x 1/k H.x/ H.x/
Chọn x D 1 ta nhận được :

H.12 /
2n D PA .1/ C PB .1/ D .1 C 1/k D 2k
H.1/
) n D 2k 1 :
Định lý Ruf và hàm sinh 111

Định nghĩa 4. Số phức z được gọi là căn bậc n của đơn vị nếu nó thỏa mãn đẳng thức z n 1 D 0.
Tập hợp các căn bậc n của đơn vị là tập hợp
 
2
i k2 k2
e n D cos C i sin jk D 0; 1; 2; : : : ; n 1 :
n n
Nếu z ¤ 1 là căn bậc n của đơn vị, suy ra z là nghiệm của phương trình
zn 1
C zn 2
C    C z C 1 D 0:
Khi đó ta có thể kết luận phương trình z n 1
C zn 2
C    C z C 1 D 0, có n 1 nghiệm là
k2 k2
cos C i sin ; k D 1; 2; : : : ; n 1:
n n
Ví dụ 3. Một phân hoạch  của số nguyên dương n  1 là cách biểu diễn n thành tổng của
một hay một số số nguyên dương (không tính đến thứ tự của các số trong tổng). Ví dụ với n D 4
thì các phân hoạch  là
1 C 1 C 1 C 1; 1 C 1 C 2; 1 C 3; 2 C 2; 4:
Với mỗi số nguyên dương n, ký hiệu .n/ là số cách phân hoạch  của n.
Định lý 6. Với mỗi số nguyên dương n, thì hàm sinh của .n/ là
1
Y 1
P .x/ D :
nD1
1 xn

Chứng minh. Một phân hoạch  của n có thể xem như là một dãy .a1 ; a2 ; : : :/, với ai là số lần
xuất hiện số i trong phân hoạch  của n và
n
X
iai D n:
i D1

Rõ ràng, số các dãy .ai / như vậy là hệ số của x n trong


1
Y
.1 C x i C x 2i C    /
i D1

do mỗi số hạng x n được tạo thành từ việc nhân các thừa số x i ai , ở đó x i ai lấy từ hạng tử thứ i
trong chuỗi trên. Từ đó ta có điều phải chứng minh.

2. Định lý Ruf và ý nghĩa


Định lý 7. Giả sử X
P .x/ D ak :x k
k
2 i
là hàm sinh của dãy fak g, với " D e n (" là căn bậc n của đơn vị). Khi đó
X 1
P .1/ C P ."/ C    C P ."n 1 / :

a0 C an C a2n C a3n C    D an:k D ./
n
k0
112 Các phương pháp giải toán qua các kỳ thi Olympic

Ý nghĩa của định lý trên cho phép tính tổng tất cả các hệ số với chỉ số là bội của n trong khai
triển P .x/.
Chứng minh. Thật vậy, vì
P .x/ D a0 C a1 x C a2 x 2 C a3 x 3 C    C an 1 x n 1
C an x n C   
dẫn đến
P .1/ D a0 C a1 C a2 C a3 C    C an 1 C an C   
P ./ D a0 C a1  C a2  C a3  C    C an 1  n
2 3 1
C an  n C   
P . 2 / D a0 C a1  2 C a2  4 C a3  6 C    C an 1  2.n 1/
C an  2n C   
:::::::::::::::
P . n 1 / D a0 C a1  n 1
C a2  2.n 1/
C a3  3.n 1/
C : : : C an 1  .n 1/.n 1/
C an  n.n 1/
C  :
Ta để ý hệ f0; 1; 2; : : : ; n 1g lập thành hệ thặng dư đầy đủ modulo n, thì f0; k; 2k; : : : ; .n 1/kg
cũng lập thành hệ thặng dư đầy đủ modulo n, với .k; n/=1. Để ý
n 1
1 C  C 2 C n 1
D D 0:
 1

 k n D 1; 8k D 0; 1; 2; : : :
Khi đó
a1 C a1  C a1  2 C    C a1  n 1
D 0;
a2 C a2  2 C a2  4 C    C a2  2.n 1/
D 0; : : :
Từ đây ta nhận thấy nếu .k; n/ D 1 thì
ak C ak  k C ak  2k C    C ak  k.n 1/
D 0:
Từ đó ta có
X
P .1/ C P ./ C    C P . n 1 / D n.a0 C an C a2n C    / D n ak n :
k0

Nhận xét. Trong định lý trên, ta đã xác lập được cách tính tổng các hệ số dạng ak n .k  0/, tức
là tổng tất cả các hệ số có chỉ số chia hết cho n. Vấn đề đặt ra là nếu ta muốn tính tổng
a1 C anC1 C a2nC1 C a3nC1 C   
tức là cần tính tổng các hệ số mà có chỉ số chia cho n dư 1 thì làm thế nào? Một cách suy nghĩ là
ta cố gắng làm cho vai trò của a1 giống như a0 , anC1 giống như an , do đó mỗi phương trình trên
ta sẽ chia tương ứng
P .1/ D a0 C a1 C a2 C a3 C    C an 1 C an C   
P ./ a0
D C a1 C a2  C a3  2 C    C an 1  n 2 C an  n 1 C   
 
P . 2 / a0
2
D 2
C a1 C a2  2 C a3  4 C    C an 1  2n 3 C an  2n 2 C   
 
:::::::::::::::
P . n 1 / a0
n 1
D n 1 C a1 C a2  n 1
C a3  2n 2
C : : : C an 1  .n 2/.n 1/
C an  .n 1/.n 1/
C  :
 
Định lý Ruf và hàm sinh 113

Khi đó
P ./ P . 2 / P . n 1 /
 
X 1
a1 C anC1 C a2nC1 C    D ak nC1 D P .1/ C C C  C
n  2 n 1
k0

Tương tự
P ./ P . 2 / P . n 1 /
 
X 1
a2 C anC2 C a2nC2 C    D ak nC2 D P .1/ C 2 C C    C 2.n 1/ :
n  4 
k0

Từ đó ta có công thức cổng quát sau


ar C anCr C a2nCr C a3nCr C   
X
D ak nCr
k0

P ./ P . 2 / P . n 1 /
 
1
D P .1/ C r C 2r C    C .n 1/r
n   
k
1 X P . /
D ; r D 0; 1; 2; : : : ; n 1:
n  kr
k0

Nhận xét. Khi đã xây dựng xong hàm sinh và chuyển ngôn ngữ đếm về hàm sinh. Thì vấn đề
còn lại là rút gọn vế phải trong công thức (*).

3. Một số ứng dụng trong Tổ hợp


Ta bắt đầu với một ví dụ đơn giản
Ví dụ 4 (Rumania, 2003). Cho tập A D f2; 3; 7; 9g. Tìm số các số có n chữ số lập từ A mà số
đó chia hết cho 3.
Lời giải. Ta cần tìm số các bộ .x1 ; x2 ; : : : ; xn /; xi 2 A; 8i D 1; : : : ; n sao cho :
:
.x1 C x2 C    C xn /::3:
Chúng ta xét hàm sinh:
P .x/ D .x 2 C x 3 C x 7 C x 9 /n :
Số các số cần tìm chính là tổng các hệ số của các lũy thừa có số mũ chia hết cho 3. Giả sử P .x/
có khai triển X
P .x/ D ak x k :
k0

Chúng ta cần phải tính X


AD a3k :
k0

Áp dụng định lý RUF ta có:


X 1 2 i
P .1/ C P ."/ C P ."2 / ;

AD a3k D e 3 ;
3
k0
114 Các phương pháp giải toán qua các kỳ thi Olympic

trong đó:

P .1/ D 4n
P ."/ D ."2 C "3 C "7 C "9 /n D ."2 C 1 C " C 1/n D 1
P ."2 / D ."4 C "6 C "14 C "18 /n D ." C 1 C "2 C 1/n D 1:
4n C 2
Vậy A D .
3
Ví dụ 5 (China, 1999). Với mỗi tập A, ký hiệu s.A/ là tổng các phần tử của tập A .nếu A D ;
thì s.A/ D 0/: Đặt
S D f1; 2; : : : ; 1999g:
Với mỗi r D 0; 1; 2; 3; 4; 5; 6, ta xác định

Tr D fT jT  S; s.T /  r.mod7/g:

Với mỗi r, tìm số phần tử của mỗi tập Tr .


Phân tích giải.
1. Mỗi số nguyên i; 1  i  1999, nếu i ở trong T thì i sẽ tham gia trong tổng s.T /, nếu
i 62 T thì ta coi như là 0 trong tổng s.T /. Do đó mỗi giá trị i, ta liên kết với hàm sinh

x0 C xi D 1 C xi :

Khi đó xét hàm sinh


X
P .x/ D .1 C x/.1 C x 2 /.1 C x 3 / : : : .1 C x 1999 / D an x n :
n

2. Ta thấy tồn tại một song ánh giữa mỗi tập T D fa1 ; a2 ; : : : ; am g  S với một phần tử
lũy thừa x a1 x a2 : : : x am D x a1 Ca2 CCam . Như vậy cả tập T và tổng S.T / được đặc trưng
trong phần tử
x a1 Ca2 CCam :
Khi đó với mỗi giá trị r D 0; 1; 2; 3; 4; 5; 6 thì
X
jTr j D a7kCr :
k0

3. Từ nhận xét ở ví dụ 10.2 ta được


P . 6 /
 
1 P ./ P .2/
jTr j D P .1/ C r C 2r C    C 6r ; r D 0; 1; 2; : : : ; 6:
7   

4. Ta có ;  2 ; : : : ;  6 ;  7 D 1 là các nghiệm của đa thức x 7 1. Do đó

G.x/ D x 7 1 D .x /.x  2 / : : : .x  7 /:

Suy ra

G. 1/ D 2D. 1 /. 1 2/ : : : . 1  7 / ) .1 C /.1 C  2 / : : : .1 C  7 / D 2:


Định lý Ruf và hàm sinh 115

5. Ta có P .1/ D .1 C 1/.1 C 1/ : : : .1 C 1/ D 21999 .


„ ƒ‚ …
1999 số 2

6. Tính P ./, ta có

P ./ D Œ.1 C /.1 C  2 / : : : .1 C  7 /Œ.1 C  8 /.1 C  9 / : : : .1 C  14 / : : :


: : : Œ.1 C  1989 / : : : .1 C  1995 /.1 C  1996 /.1 C  1997 /.1 C  1998 /.1 C  1999 /
D Œ.1 C / : : : .1 C  7 / : : : Œ.1 C / : : : .1 C  7 /.1 C /.1 C  2 /.1 C  3 /.1 C  4 /
„ ƒ‚ …
985 lần
D 2:2
„ ƒ‚: : : …2.1 C /.1 C  2 /.1 C  3 /.1 C  4 /
985 số 2
985 2 6
D2  7 /.1 C  3 /
„ C  C ƒ‚C    C … C „ƒ‚…
.1
D0 D1
985 3
D2 .1 C  /:

7. Tính P . 2 /, ta có

P . 2 / D Œ.1 C  2 /.1 C  4 / : : : .1 C  6 /.1 C  8 /.1 C  10 / : : : .1 C  12 /.1 C  14 / : : :


: : : Œ.1 C  16 /.1 C  18 / : : : .1 C  20 /.1 C  22 /.1 C  24 /.1 C  26 /.1 C  28 / : : :
: : : Œ.1 C  3978 / : : : .1 C  3988 /.1 C  3990 /.1 C  28 /
: : : .1 C  3992 /.1 C  3994 /.1 C  3996 / : : : .1 C  3998 /
D Œ.1 C  2 /.1 C  4 /.1 C  6 /.1 C /.1 C  3 /.1 C  5 /.1 C  7 / : : :
: : : Œ.1 C  2 /.1 C  4 /.1 C  6 /.1 C /.1 C  3 /.1 C  5 /.1 C  7 /
 .1 C  2 /.1 C  4 /.1 C  6 /.1 C /
D 2985 .1 2 6
 7 /.1 C  6 /
„ C  C ƒ‚C    C … C „ƒ‚…
D0 D1
985 2:3
D2 .1 C  /:

8. Từ đây ta có P . i / D 2985 .1 C  3i /; i D 1; 2; 3; 4; 5; 6.

9. Từ đó ta được
6
!
1 1999 985
X
ri
jTr j D 2 C2  .1 C  3i /
7 i D1
6 h
!
1 1999 285
X
ri
i
D 2 C2  C  .3 i /r
7 i D1

10. Mặt khác ta có ngay


6
(
X 6 r chia hết cho 7
 kr D
kD1
1 r không chia hết cho 7:
116 Các phương pháp giải toán qua các kỳ thi Olympic

11. Từ đây dẫn tới


6 h
(
X
ri
i 6 1D5 r  0; 3 .mod 7/
 C  .3 i /r
D
i D1
1 1D 2 trong các trường hợp còn lại:

12. Từ đây dẫn đến €2 1999


C 5:2285
r D 0 hoặc r D 3
jTr j D 7
21999 2286
r D 1; 2; 4; 5; 6
7

Ví dụ 6 (VMO, 2015). Cho số nguyên dương K. Tìm số các số tự nhiên n không vượt quá 10K
thỏa mãn đồng thời các điều kiện sau:

a) n chia hết cho 3:

b) Các chữ số trong biểu diễn thập phân của n thuộc tập hợp f2; 0; 1; 5g.

Lời giải.

1. Chuyển hóa ngôn ngữ đếm về hàm sinh: Vì 10K không chia hết cho 3 nên ta chỉ cần xét các
số từ 0 cho đến 99 : : : 9 (K chữ số 9). Bổ sung các chữ số 0 vào trước nếu cần thiết, ta đưa
về xét các số có dạng a1 a2 : : : aK với ai thuộc f2; 0; 1; 5g. Ta cần đếm các số như vậy và
chia hết cho 3. Chú ý là a1 a2 : : : aK chia hết cho 3 khi và chỉ khi a1 C a2 C : : : C aK chia
hết cho 3, ta đưa bài toán về việc đếm số các bộ .a1 ; a2 ; : : : ; aK /, với mỗi ai 2 f0; 1; 2; 5g,
sao cho a1 C a2 C : : : C aK chia hết cho 3. Đến đây hướng giải quyết giống như hai bài
toán trước. Xét đa thức P .x/ D .1 C x C x 2 C x 5 /K . Ta có
X
P .x/ D .x 2 C 1 C x C x 5 /K D x a1 Ca2 C:::CaK :
.a1 ;a2 ;:::;aK /2f2;0;1;5gK

Ta thấy tổng các hệ số của P .x/ bằng số các bộ .a1 ; a2 ; : : : ; aK / 2 f2; 0; 1; 5gK và bằng
:
4K : Hơn nữa số các bộ .a1 ; a2 ; : : : ; aK / 2 f2; 0; 1; 5gK sao cho a1 C a2 C : : : C aK ::3
bằng tổng các hệ số của các số mũ chia hết cho 3 trong P .x/:

2. Sử dụng RUF: Theo định lý RUF thì tổng S cần tính bằng

1
S D ŒP .1/ C P ."/ C P ."2 /:
3

Gọi " là nghiệm của phương trình x 2 C x C 1 D 0 thì ta có "3 D 1: Từ đó dễ dàng suy ra
1 C " C "2k D 0 với mọi k không chia hết cho 3 và bằng 3 với k chia hết cho 3. (*). Ta dễ dàng
tính được

P .1/ D 4k I
P ."/ D "2K
P ."2 / D "4K :
Định lý Ruf và hàm sinh 117

Suy ra
P .1/ C P ."/ C P ."2 / 4K C "2K C "4K
SD D
3 3
Cuối cùng, lại áp dụng tính chất (*) ta suy ra
4K 1
SD nếu K không chia hết cho 3
3

4K C 2
SD nếu K chia hết cho 3:
3
Ví dụ 7. Có bao nhiêu số tự nhiên có 9 chữ số trong đó không chứa số 0 và chia hết cho 11:
Lời giải.
1. Chuyển hóa ngôn ngữ đếm về hàm sinh: Ta biết số tự nhiên a1 a2 : : : a9 chia hết cho 11 khi
và chỉ khi
:
a1 C a3 C a5 C a7 C a9 a2 a4 a6 a8 ::11:
Xét đa thức
9 4
P .x/ D .x C x 2 C    C x 9 /5 x 1 2

Cx C  C x :

Khi khai triển đa thức P .x/ thì các số hạng của P .x/ có dạng

x c1 Cc2 Cc3 Cc4 Cc5 c6 c7 c8 c9


; với ci 2 f1; 2; 3; : : : ; 9g:
:
Khi đó số bộ số .c1 ; : : : ; c9 / mà c1 C c2 C c3 C c4 C c5 c6 c7 c8 c9 ::11 bằng tổng
các tổng các hệ số của các số hạng chia hết 11 trong khai triển của P .x/.
2. Sử dụng RUF: Gọi S là tổng cần tính thì theo định lý RUF
1
ŒP .1/ C P ."/ C    C P ."10 /
11
2 i
trong đó " D e 11 . Ta có

P .x/ D x 5 .1 C x C x 2 C x 8 /5 :x 36
.1 C x C    C x 8 /4 D x 31
.1 C x C    C x 8 /9 :

Ta có
P .1/ D 99
và với mọi k D 1; 2; : : : ; 10 thì
9 k
! !
 9 X 9 X 9 .j C6/k
P ."k / D " 31k
"9k "10k D "50k "j k D " :
j D0
j j D0
k

Chú ý là nếu j C 6 không chia hết cho 11 thì khi k chạy qua hệ thặng dư thu gọn modulo
11, thì .j C 6/k cũng thế và ta có
9
X
".j C6/k D 1
kD1
118 Các phương pháp giải toán qua các kỳ thi Olympic

còn khi j C 6 chia hết cho 11 (tức j D 5) thì tổng đã cho bằng 10. Vậy
9
! !
10 9
X 9 9
P .1/ C P ."/ C    C P ." / D 9 C 11 :
j D0
j 5

Từ đó ta có kết quả đề bài là


!
99 C 29 9
:
11 5

Ví dụ 8 (Vietnam TST, 2008). Cho M là tập hợp 2008 số nguyên dương đầu tiên, mỗi số đó
được tô bởi một trong 3 màu: xanh, đỏ và vàng, và mỗi màu được tô ít nhất một số. Xét 2 tập:

S1 D f.x; y; z/ thuộc M 3 mà x; y; z tô cùng màu và x C y C z  0 .mod2008/g;

S2 D f.x; y; z/ thuộc M 3 mà x; y; z đôi một khác màu và x C y C z  0 .mod 2008/g:


Chứng minh rẳng 2jS1 j > jS2 j.
Lời giải. Gọi A; B; C là ba tập hợp các số tự nhiên được tô màu xanh, đỏ, vàng tương ứng. Xét
các hàm sinh liên kết:
X X X
PA .x/ D x a ; PB .x/ D x b ; PC .x/ D xc :
a2A b2B c2C

Khi đó
I.x/ D PA3 .x/ C PB3 .x/ C PC3 .x/
X X X
D x a1 Ca2 Ca3 C x b1 Cb2 Cb3 C x c1 Cc2 Cc3
ai 2A bi 2B ci 2C
X
D an x n :
n

Trong đó an chính là số bộ .x; y; z/ có cùng một màu và có tổng là n. Gọi " là nghiệm của
phương trình x 2008 1 D 0. Theo định lý RUF ta có
1 X
I."k / D a0 C a2008 C a2:2008 C a3:2008 D jS1 j :
2008
k

Vậy
1 X 1 Xh 3 k i
jS1 j D I."k / D PA ." / C PB3 ."k / C PC3 ."k / :
2008 2008
k k

Lý luận tương tự ta có :
6 X 2 X
jS2 j D PA ."k /PB ."k /PC ."k / D 3:PA ."k /PB ."k /PC ."k /:
2008 2008
k k

Với k ¤ 0 ta có
PA ."k / C PB ."k / C PC ."k / D 0:
Do đó:
PA3 ."k / C PB3 ."k / C PC3 ."k / D 3PA ."k /PB ."k /PC ."k /:
Định lý Ruf và hàm sinh 119

Vậy ta chỉ cần chứng minh

PA3 .1/ C PB3 .1/ C PC3 .1/ > 3PA .1/PB .1/PC .1/:

Thật vậy ta luôn có

PA3 .1/ C PB3 .1/ C PC3 .1/  3PA .1/PB .1/PC .1/:

Dấu bằng xảy ra khi và chỉ khi PA .1/ D PB .1/ D PC .1/, suy ra 3PA .1/ D 2008 điều này vô lý
vì 2008 không chia hết cho 3.

Ví dụ 9. Cho p là một số nguyên tố lẻ và n là một số nguyên dương không chia hết cho p. Tính
số tất cả các bộ số .x1 ; x2 ; : : : ; xp 1 / 2 f0; 1; : : : ; p 1gp 1 sao cho
p 1
:
ixi ::p:
X

i D1

Lời giải. Xét hàm sinh


p 1
Y
P .x/ D .1 C x i C x 2i C    C x .n 1/i
/:
i D1

Ta có
P .1/ D np 1 :
2
Đặt " D e p i . Khi đó, với mọi 1  j  p 1 thì
p 1
Y 1 "nij
j
P ." / D D 1:
i D1
1 e ij

Theo định lý RUF ta có số bộ cần tính là:


p 1
1X np 1
Cp 1
P ."j / D :
p j D0 p

Ví dụ 10 (IMO, 1995). Cho tập hợp A D f1; 2; : : : ; 2pg, p là số nguyên tố. Tìm số các tập
con của A thỏa mãn:

 Mỗi tập có đúng p phần tử.

 Tổng các phần tử của tập con đó chia hết cho p.

Lời giải. Xét tập con B của A thỏa mãn yêu cầu bài toán.

 Mỗi số nguyên i; 1  i  2p, nếu i ở trong B thì i sẽ tham gia trong tổng s.B/, nếu
i 62 B thì ta coi như là 0 trong tổng s.T /. Do đó mỗi giá trị i , ta liên kết với hàm sinh

x0 C xi D 1 C xi :
120 Các phương pháp giải toán qua các kỳ thi Olympic

Khi đó xét hàm sinh


X
P .x/ D .1 C x/.1 C x 2 /.1 C x 3 / : : : .1 C x 2p / D an x n :
n

Tuy nhiên với hàm sinh này, hệ số của x p ; x 2p ; x 3p ; : : : sẽ cho ta tất cả các tập con của
A, mà tổng các phần tử của mỗi tập con đó chia hết cho p. Do đó điều kiện thứ hai chưa
được đảm bảo. Do đó ta cần một hàm sinh khác cho ta chỉ số đếm, chỉ chọn ra các tập con
trong số các tập ở trên mà có p phần tử. Đến đây ta cần dùng định lý 1.5.

 Xét hàm sinh


P .y/ D .1 C y/.1 C y/ : : : .1 C y/ D .1 C y/2p :
Hệ số của y p cho ta số cách chọn p phần tử từ tập A.

Do đó, ta xét hàm sinh như sau:

P .x; y/ D .1 C xy/.1 C x 2 y/ : : : .1 C x 2p y/:

Khai triển dưới dạng tổng của P .x; y/ có dạng:


X
P .x; y/ D an;k :x n :y k :
k;n

Ta sẽ đi tìm ý nghĩa của an;k . Khi khai triển P .x; y/ thành tổng thì các số mũ của x có dạng
x1 C x2 C    C xk với x1 ; x2 ; : : : ; xk là các số đôi một phân biệt của tập A, số mũ tương ứng
của y là k, như vậy an;k chính là số tập con của A có k phần tử mà tổng các phần tử là n. Ví
dụ, với p D 3, và số hạng
xy:1:x 3 y:1:x 5 y:1 D x 9 :y 3
tương ứng với tập con f1; 3; 5g. Do đó bài toán tương đương với việc đi tìm tổng
X X
AD akp;p D an;p :
k0 pjn

Áp dụng định lý RUF cho P .x; y/ trong đó ta coi x là biến số ta được:


p 1
X
k 1X 2 i
an;k :y D P ."j ; y/; với " D e p :
p j D0
k0;pjn

Ta có:

P .1; y/ D .1 C y/2p
P ."j ; y/ D .1 C "j y/.1 C "2j y/ : : : .1 C "2jp y/; 8j  1:

Vì p là số nguyên tố nên fj; 2j; : : : ; pj g; f.p C 1/j; : : : ; 2pj g tạo thành các hệ thặng dư thu
gọn mod p, mặt khác ta có "p D 1 do đó:

P ."j ; y/ D .1 C "j y/.1 C "2j y/ : : : .1 C "2jp y/


D .1 C "y/.1 C "2 y/ : : : .1 C "p y/; 8j  1:
Định lý Ruf và hàm sinh 121

Dễ nhận thấy phương trình y p C 1 D 0 có các nghiệm là " 1 ; " 2 ; : : : ; " p


do đó ta có:

.1 C "y/.1 C "2 y/ : : : .1 C "p y/ D 1 C y p :

Từ đây ta được:
X 1h i
) an;k y k D .1 C y/2p C .p 1/.1 C y p /2
p
:
k0;n::p
2p

X p
C 2.p 1/
) an;p D
p
:
n::p

.2p
p /C2.p 1/
Vậy số tập con tìm được là: p
.

Nhận xét. Nếu bỏ qua điều kiện đầu tiên thì bài toán trở nên đơn giản hơn rất nhiều. Khi đó ta
chỉ cần xét hàm sinh:
P .x/ D .1 C x/.1 C x 2 / : : : .1 C x 2p /:
Khi đó số tập con cần tìm là
X 1  22p C 4.p 1/
P D akp D P .1/ C P ."/ C    C P ."p 1 / D :
p p
k0

Việc thêm biến, mỗi biến phục vụ một yêu cầu đếm khác nhau cũng là một kỹ năng cần có trong
hàm sinh. Chẳng hạn với ví dụ đơn giản sau.
Ví dụ 11. Cho n là số nguyên dương. Đặt
n
!2
X n
P .x/ D .1 C x/2n 2k
.1 x/2k :
k
kD0

Chứng minh mọi hệ số của lũy thừa bậc lẻ trong P .x/ đều bằng 0.
Lời giải. Ta viết lại P .x/ dưới dạng
n
!2
X n
P .x/ D .1 C x/2n 2k
.1 x/2k
k
kD0
n
! !
X n n
D Œ.1 x/2 k Œ.1 C x/2 n k
:
k n k
kD0

Nhận thấy
n
! !
X n n
Œ.1 x/2 k Œ.1 C x/2 n k
k n k
kD0
n
là hệ số của y trong khai triển

P .x; y/ D Œy C .1 C x/2 n Œy C .1 x/2 n :


122 Các phương pháp giải toán qua các kỳ thi Olympic

Mặt khác
P .x; y/ D Œ.y C 1 C x 2 C 2x/.y C 1 C x 2 2x/n
D Œ.y C 1 C x 2 /2 4x 2 n :
Ta thấy P .x; y/ là hàm chẵn của x. Do đó sẽ không có lũy thừa bậc lẻ của x trong khai triển
của P .x; y/. Do đó hệ số của y n trong khai triển của P .x; y/ chỉ có lũy thừa bậc chẵn của x.
Tức trong khai triển của P .x/, mọi hệ số lũy thừa bậc lẻ của x đều bằng 0. Bài toán được chứng
minh.

4. Bài tập vận dụng


Bài tập 22. Có bao nhiêu số có 2014 chữ số lập từ các chữ số f1; 3; 5; 9g và chia hết cho 3.
Bài tập 23. Cho n là số nguyên dương lớn hơn 1. Tìm số tập con A của tập hợp X D
f1; 2; : : : ; ng sao cho tổng các phần tử của A chia hết cho 7.
Bài tập 24. Có bao nhiêu tập con của tập f1; 2; : : : ; 2000g sao cho tổng các phần tử của nó
chia hết cho 5.
Bài tập 25. Có bao nhiêu tập con của tập f1; 2; : : : ; 2007g (kể cả tập rỗng) mà tổng các phần
tử của mỗi tập đó chia hết cho 17.
Bài tập 26. Cho tập hợp A D f1; 2; : : : ; mg; p là số nguyên tố. Tìm số các tập con của A thỏa
mãn:
 Mỗi tập có đúng p phần tử.
 Tổng các phần tử của tập con đó chia hết cho p.
Bài tập 27. Chứng minh rằng số tập con n phần tử của tập f1; 2; : : : ; 2ng và tổng các phần tử
của chúng chia hết cho n bằng
!
. 1/n X  n  2d
 . 1/d ' :
n d d
d jn

Bài tập 28 (IMO Shortlist 1999). Cho p > 3 là số nguyên tố. Nếu X là tập con khác rỗng
của tập f0; 1; 2; : : : ; p 1g, đặt f .X/ là số dãy fa1 ; a2 ; : : : ; ap 1 g sao cho aj 2 X với mọi j và
p 1
X
pj jaj :
j D1

Chứng minh rằng f .f0; 1; 3g/  f .f0; 1; 2g/. Dấu bằng xảy ra khi và chỉ khi p D 5.
Bài tập 29. Với mỗi phân hoạch  của số nguyên dương n, đặt A./ là số các số 1 xuất hiện
trong  và B./ là số các số nguyên phân biệt trong . Ví dụ với n D 13, với phân hoạch  là
1C1C2C2C2C5
thì A./ D 2; B./ D 3: Chứng minh rằng với mọi số nguyên dương n thì tổng tất cả các
A./, với  lấy trên tất cả các phân hoạch nguyên của n, bằng với tổng tất cả các B./, với 
lấy trên tất cả các phân hoạch nguyên của n.
Định lý Ruf và hàm sinh 123

Bài tập 30 (USA TST, 2004).

1. Cho n là số nguyên dương lẻ. Tìm số các dãy .a0 ; a1 ; : : : ; an / sao cho ai 2 f1; 2; : : : ; ng
với mọi i , an D a0 và ai ai 1 ¤ i.modn/ với mọi i D 1; 2; : : : ; n.

2. Cho n là số nguyên tố lẻ. Tìm số các dãy .a0 ; a1 ; : : : ; an / sao cho ai 2 f1; 2; : : : ; ng với
mọi i , an D a0 và ai ai 1 ¤ i; 2i.modn/ với mọi i D 1; 2; : : : ; n.

Tài liệu tham khảo


[1] T. Andreescu, Z. Feng, A Path to Combinatorics for Undergraduated, Birkhauser, 2004.

[2] Zachary R. Abel, Multivariate Generating Functions and Other Tidbit, Mathematical
Reflections 2, 2006.

[3] T. Andreescu, G. Dospinescu, Straight from The Book, XYZ Press. 2012.

[4] Holden Lee, Generating Function, Bài giảng trên mạng, 2001.
124 Các phương pháp giải toán qua các kỳ thi Olympic
MỘT SỐ BÀI TOÁN VỀ TRÒ CHƠI
Nguyễn Đình Thành Công, Nguyễn Văn Hưởng
(Hà Nội)

1. Khái quát chung


Toán trò chơi là một đặc thù riêng trong tổ hợp và và thường xuất hiện trong đề thi với vai trò
là những câu hỏi khó. Một bài toán trò chơi có thể phân loại theo nhiều cách nhưng trong bài
viết này, ta sẽ xem xét cách chia theo số lượng người chơi: một người; hai người chơi hoặc nhiều
người chơi.
Rõ ràng trong một bài toán trò chơi nói riêng và một bài tổ hợp nói chung, điều đầu tiên mà cũng
quan trọng nhất là tìm ra tính chất đặc biệt của bài toán.
Muốn tìm được tính chất đặc biệt này cần phải quan tâm:
1. Mệnh đề giả thiết.
2. Mệnh đề kết luận.
3. Mối liên kết các giả thiết với nhau.
Các nguyên lý hay được sử dụng trong toán trò chơi là: nguyên lý đơn biến, bất biến và cực trị.
Một vấn đề cần sự tư duy cao nhất không chỉ trong toán trò chơi mà trong mọi bài toán tổ hợp,
đó là: xây dựng cấu hình thỏa mãn.
Để hiểu rõ những vấn đề đã nêu, chúng ta bắt đầu làm quen một vài bài tập sau.

2. Các bài toán chọn lọc


2.1. Bài toán một người chơi
2.1.1. Đặc thù bài toán
Dạng bài toán này thường cho giả thiết là một quy tắc điền, xóa, lấy, ... để đưa ra kết luận. Tính
chất hay sử dụng ở đây là: tính đơn biến, bất biến.

2.1.2. Các ví dụ minh họa


Bài toán 1. Trên hệ trục tọa độ Oxy người ta đặt một viên sỏi tại gốc tọa độ rồi thực
hiện một trò chơi sau: Tại mỗi bước, người ta sẽ rút đi một viên sỏi ở tọa độ điểm .xI y/
và đặt thêm hai viên sỏi vào hai vị trí khác nhau; mỗi viên đó tại một trong bốn tọa độ sau
f.x C 1I y C 1/ I .x 1I y 1/ I .x C 1I y 1/ I .x 1I y C 1/g. Chứng minh rằng: Trong suốt
quá trình thực hiện trò chơi, luôn tồn tại viên sỏi cách gốc tọa độ một khoảng không vượt quá 5:

125
126 Các phương pháp giải toán qua các kỳ thi Olympic

Lời giải. Gọi Sk là tập tất cả các tọa độ các điểm mà mỗi điểm có chứa ít nhất một viên sỏi ở
bước thứ k: Khi đó, ta xét tổng:
X 1
Tk D
2jijCjj j
.iIj /2Sk

Tại bước thứ k C 1, ta lấy một viên ở tọa độ .i0 I j0 / và thêm vào hai viên ở tọa độ .i1 I j1 / và
.i2 I j2 / thì khi đó, ta sẽ có:
1 1 1 1 1
D C  C
2ji0 jCjj0 j 2ji0 jCjj0 jC1 2ji0 jCjj0 jC1 2ji1 jCjj1 j 2ji2 jCjj2 j
Do đó, ta có: Tk  TkC1 . Tiếp theo, giả sử không tồn tại viên sỏi nằm ở tọa độ mà khoảng cách
từ điểm đó tới gốc tọa độ không vượt quá 5 tại bước thứ m nào đó thì ta phải có: trong hình
vuông được giới hạn bởi các cạnh x ˙ y D ˙5 sẽ không chứa viên đá nào. (hình minh họa )

Khi đó, ta có:


1 X 1
X 1 X 1 X 1
Tm  ji jCjj j
D 1 C 4 i Cj ji jCjj j
2 i D1 j D0
2 2
ji jCjj j>5 ji jCjj j5
1 1  
X 1 X 1 1 1 1 1 1
! Tm  1 C 4 1 4 1 C2 C3 C4 C5
i D1
2i j D0 2j 2 4 8 16 32
65 7
! Tm  9 D < 1 D T0
8 8
Điều vô lý này cho thấy điều giả sử là sai và bài toán được chứng minh.
Nhận xét. TínhP đơn biến được sử dụng trong bài toán này rất rõ, thế nhưng vấn tại sao lại chọn
1
được hàm Tk D .i Ij /2S 2jijCjj j mà không phải hàm khác. Muốn hiểu rõ thì bạn cần nhìn lại quy

tắc của bài toán. Và cũng với hàm kiểu này ta cũng có một bài toán gần như anh em sinh đôi sau.
Bài toán 2. Cho bốn tam giác vuông bằng nhau. Người ta thực hiện mỗi bước cắt như sau:
Chọn ra một tam giác vuông, sau đó cắt tam giác vuông này thành hai tam giác vuông bởi đường
cắt là đường cao ứng với cạnh huyền. Chứng minh rằng: sau một số hữu hạn bước, dù cắt theo
cách nào cũng có hai tam giác vuông bằng nhau.
Một số bài toán về trò chơi 127

Lời giải. Đầu tiên, ta có thể giả sử: mỗi tam giác vuông ban đầu có cạnh huyền bằng 1 và một
trong hai góc nhọn bằng .
Khi đó, ta có nhận xét: một tam giác vuông có cạnh huyền x và góc nhọn  thì sau bước cắt theo
chiều cao, ta được hai tam giác vuông đều có một góc nhọn  còn cạnh huyền của chúng tương
ứng là: x sin I x cos .
Như vậy, tất cả các tam giác vuông sau mỗi bước đều có cạnh huyền dạng: .sin x/m .cos x/n với
m; n 2 N. Khi đó, ta xét ánh xạ sau:
.sin x/m .cos x/n 7! .mI n/ :
Như vậy mỗi tam giác tương ứng với một điểm có hai thành phần là số nguyên không âm trên tọa
độ Descartes Oxy (dĩ nhiên là sẽ có trường hợp nhiều tam giác có cùng ánh xạ là một điểm).
Gọi tập hợp các tam giác sau bước thứ k là Sk .
Ta có đại lượng bất biến ở đây là:
X 1
P D :
2xCy
8.xIy/2Sk

1 1
P P
Thật vậy, ta xét hiệu: 8.xIy/2Sk 2xCy 8.xIy/2Sk 1 2xCy
ta thấy, do theo bước cắt thì sự thay
đổi từ bước thứ k 1 tới bước thứ k là:
.x0 I y0 / ! .x0 C 1I y0 / I .x0 I y0 C 1/ :
Do đó ta có:
X 1 X 1 1 1 1
D C D0
2xCy 2xCy 2.x0 C1/Cy0 2x0 C.y0 C1/ 2x0 Cy0
8.xIy/2Sk 8.xIy/2Sk 1

Vì thế P là đại lượng bất biến.


Tại thời điểm ban đầu, ta có bốn tam giác vuông cạnh huyền bằng 1 tương ứng với bốn điểm
1
.0I 0/. Do đó, ta có P D 4  20C0 D 4.
Giả sử rằng tồn tại m bước mà sau bước thứ m, ta thu được các tam giác mà không có hai tam
giác nào bằng nhau. Điều đó chứng tỏ: không có hai tam giác nào có ánh xạ cùng là một điểm.
Ta phân hoạch tất cả các điểm này thành các tập hợp: Uk D f.xI y/ jx C y D kg.
Do xI y 2 ZI xI y  0 nên jUk j  k C 1. Khi đó, ta có:
h 1
X jUk j X kC1
4DP D < D4
2k 2k
kD0 kD0

Điều vô lý này chứng tỏ rằng điều giả sử ở trên là sai và bài toán được chứng minh.
Nhận xét. Hàm số này không chỉ áp dụng ở bài toán một người chơi. Chúng tôi sẽ giới thiệu
thêm trong phần bài toán nhiều người chơi.
Bài toán 3. Cho một bộ số S gồm ba số nguyên dương không nhất thiết phân biệt. Thực hiện
một quy tắc: Chọn hai số nguyên x và y trong số đó. Nếu có x  y thì ta thay hai số này bằng
hai số 2xI y x để được bộ mới. Chứng minh rằng có thể thực hiện một chuỗi hữu hạn các quy
tắc như trên để thu được một tập S 0 chứa số 0:
128 Các phương pháp giải toán qua các kỳ thi Olympic

Lời giải. Giả sử bộ ban đầu S D .aI bI c/ với 0 < a  b  c. Ta sẽ chứng minh rằng từ bộ S
này, có thể đưa được về bộ .b 0 I a0 I c 0 / trong đó a0 là số dư của b khi chia cho a: Thật vậy, ta đặt:

b D qa C rI q 2 N I 0  r < a
 
q D q0 C 2  q1 C 22  q2 C ::: C 2k  qk I qi 2 f0I 1g I i D 1I k

Từ quy tắc của đề bài, ta xét các hàm số sau:

f0 .d I eI g/ D .2d I eI g d /
f1 .d I eI g/ D .2d I e d I g/

Xét Sr D fqk :::fq1 fq0 .aI bI c/ : Rõ ràng, với quy tắc này ta sẽ thu được Sr D .b 0 I rI c 0 /.


Như vậy, gọi số nhỏ nhất trong mỗi bộ là số “quyết định”.


Bây giờ ta xét chuỗi các quy tắc sao cho thu được S 0 D .a0 I b 0 I c 0 / sao cho số quyết định của nó
là số nhỏ nhất trong tập các số quyết định. Ta chứng minh số quyết định này phải bằng 0:
Thật vậy; giả sử số quyết định của bộ S 0 dương thì theo nhận xét trên ta thu được bộ có số quyết
định nhỏ hơn. Điều này vô lý. Vậy bài toán được chứng minh.

Nhận xét. Phương pháp tìm ra hàm để chỉ ra tính đơn biến hay bất biến trong toán trò chơi có
ứng dụng khá mạnh và phổ biến. Thế nhưng vẫn có những lớp bài toán mà ta không thể chỉ ra
hàm số mà chỉ có thể chỉ ra tính chất trong bài toán là tính chất đơn biến hay bất biến thôi. Ta bắt
đầu tìm hiểu tiếp thông qua bài toán sau.

Bài toán 4. Từ các số 1I 2I 3I 4I 5 người ta thực hiện quy tắc sau: Thay hai số aI b bằng hai số
a C bI ab. Hỏi sau một số hữu hạn chuỗi quy tắc; có thể thu được các số sau hay không?

a) 22015 C 22011 I 22015 C 2  22011 I 22015 C 3  22011 I 22015 C 4  22011 I 22015 C 5  22011 :

b) 21I 27I 64I 180I 540:

Lời giải. a) Giả sử rằng sau hữu hạn chuỗi quy tắc ta có thể thu được năm số: 22015 C
22011 I 22015 C 2  22011 I 22015 C 3  22011 I 22015 C 4  22011 I 22015 C 5  22011 :
Khi đó, theo giả thiết thì phải tồn tại các số nguyên dương a; b để

a C b D 22015 C i I ab D 22015 C j I i; j 2 1:22011 I 2  22011 I 3  22011 I 4  22011 I 5  22011


˚

! ab .a C b/ D j i
! .a 1/ .b 1/ D j i C 1
! max .a 1I b 1/  jj i C 1j  5  22011
! max .aI b/  6  22011
! a C b  12  22011 < 22015

Điều cuối cùng vô lý nên điều giả sử ban đầu là sai. Câu trả lời là phủ định.
b) Dễ thấy rằng: Số các bội số của 3 trong bài luôn không giảm.
Ta xét các trường hợp sau:

 Khi aI b cùng chia hết cho 3 thì a C bI ab cũng chia hết cho 3:
Một số bài toán về trò chơi 129

 Nếu trong aI b có đúng một số chia hết cho 3 thì ab cũng chia hết cho 3 nên trong a CbI ab
cũng có đúng một số chia hết cho 3:

 Nếu aI b cùng số dư khi chia cho 3 và số dư đó thuộc tập thì a C bI ab cũng cùng không
chia hết cho 3:

 Nếu trong aI b có một số chia cho 3 dư 1I một số chia cho 3 dư 2 thì a C b chia hết cho 3
và ab chia cho 3 dư 2:

Để ý rằng ban đầu có một số chia hết cho 3; kết thúc có 4 số chia hết cho 3: Điều này muốn xảy
ra thì số còn lại phải chia cho 3 dư 2: Tuy nhiên, 64 chia cho 3 dư 1: Đây là điều này vô lý.
Câu trả lời vẫn là phủ định.

Bài toán 5 (Duyên hải, 2011). Điền 29 số tự nhiên dương đầu tiên vào bảng 6  5 như bảng
1: Thay đổi các số trong bảng theo quy tắc: mỗi lần lấy một số nằm ở ô liền kề với ô trống rồi
chuyển vào ô trống. Hỏi sau hữu hạn lần chuyển như vậy có thể chuyển từ bảng 1 thành bảng 2
được hay không?

1 2 3 4 5 29 2 3 4 5
6 7 8 9 10 6 7 8 9 10
11 12 13 14 11 12 13 14
15 16 17 18 19 15 16 17 18 19
20 21 22 23 24 20 21 22 23 24
25 26 27 28 29 25 26 27 28 1

Lời giải. Giả sử nhờ phép chuyển số theo quy tắc đã cho, từ bảng 1 ta có thể nhận được bảng 2:
Ta coi mỗi ô trống trong bảng là ô được điền số 0: Với mỗi bảng số nhận được trong quá trình
chuyển số, ta liệt kê tất cả các số của bảng theo thứ tự từ trái qua phải, từ trên xuống dưới. Khi
đó, ứng với mỗi bảng ta được 1 hoán vị của 30 số tự nhiên đầu tiên.
Do đó, với giả sử trên, từ hoán vị

.1I 2I 3I : : : I 11I 12I 0I 13I 14I : : : I 28I 29/ (gọi là hoán vị I).

Sau khi thực hiện phép chuyển số ta có thể nhận được hoán vị

.29I 2I 3I : : : I 11I 12I 0I 13I 14I : : : I 27I 28I 1/ (gọi là hoán vị II).

Giả sử .a1 I a2 I : : : I a30 / là một hoán vị của 30 số tự nhiên đầu tiên. Ta gọi cặp số .ai I aj / với
1  i; j  30 là cặp số ngược của hoán vị nếu ai > aj và i < j: Như vậy, sau mỗi lần thực hiện
phép chuyển vị trí các số thỏa mãn bài thì số cặp ngược của hoán vị đó sẽ tăng hoặc giảm 1 số lẻ
đơn vị. Ta có số cặp ngược của hoán vị I là 12 và số cặp ngược của hoán vị II là 67: Từ đó, kết
hợp với .2/ ! từ hoán vị I chỉ nhận được hoán vị II sau lẻ lần thực hiện phép đổi chổ các số.
Tô màu tất cả các ô con của bảng bởi 2 màu xanh, đỏ sao cho 2 ô kề nhau được tô hai màu khác
nhau. Như vậy sau mỗi lần chuyển ô số 0 sẽ chuyển từ màu này sang màu kia. Do ô số 0 ở bảng
1 và bảng 2 nằm ở 2 ô cùng màu, nên việc chuyển từ bảng 1 sang bảng 2 chỉ được thực hiện sau
chẵn lần di chuyển. Từ đó ta thấy có sự mâu thuẫn. Vậy không thể chuyển từ bảng 1 sang bảng 2
sau hữu hạn lần chuyển số theo quy tắc của đề bài.
130 Các phương pháp giải toán qua các kỳ thi Olympic

Bài toán 6. Trên bảng 2014  2015 có 2014 ô vuông bị tô đen. Mỗi bước biến đổi, ta tô đen
một ô kề với ít nhất hai ô bị tô đen, trong đó quy ước gọi hai ô kề nhau tức là hai ô có chung một
cạnh. Hỏi có tồn tại cấu hình ban đầu nào để sau một số hữu hạn bước; cả bảng bị tô đen?
Lời giải. Ta có một nhận xét vô cùng quan trọng để giải quyết bài toán là: Sau mỗi bước biến
đổi, tổng chu vi của tất cả các ô vuông đen là một đại lượng không tăng.

Như vậy ta chỉ cần quan tâm:


 Thời điểm ban đầu; chu vi lớn nhất của 2014 ô vuông đen là: 4  2014 D 8056 (đây là
trường hợp 2014 ô vuông này không có hai ô nào kề nhau ).

 Thời điểm kết thúc; chu vi tất cả các ô đen là chu vi của bảng và bằng

2 .2014 C 2015/ D 8058:

Do đó, từ nhận xét ta suy ra không thể đạt được trạng thái tất cả các ô của bảng bị tô đen.
Nhận xét. Hai bài toán trên cho thấy rõ sự tinh ý trong lời giải. Vấn đề bất biến cũng hoàn toàn
tương tự.
Bài toán 7 (Lê Quý Đôn, 2014). Cho số nguyên dương n .n  2/. Ta đặt trên đường tròn n ô
vuông sao cho trong mỗi ô vuông có sẵn một viên bi. Ta thực hiện các phép di chuyển bi theo
quy tắc sau: Trong mỗi lần di chuyển bi, ta chọn 2 viên bi, không nhất thiết trong cùng 1 ô vuông
và di chuyển chúng sang ô vuông kế bên cạnh, sao cho 1 viên đi theo chiều kim đồng hồ, viên
còn lại đi ngược chiều kim đồng hồ.
Hãy xác định tất cả các giá trị của n để sau một số hữu hạn phép di chuyển bi, ta có thể đưa tất
cả các viên bi về cùng một ô vuông.
Lời giải. Ta kí hiệu các ô vuông theo thứ tự ngược chiều kim đồng hồ như là các đỉnh của n-giác
đều A1 A2 : : : An nội tiếp trong đường tròn lượng giác với điểm A1 là gốc, ta đánh số thứ tự các
viên bi (như số thứ tự các đỉnh đa giác).
Gọi W là tập hợp các kết quả trong quá trình di chuyển bi và ! 2 W là trạng thái kết quả sau
mỗi bước chuyển bi. Rõ ràng với mỗi ! 2 W cho ta một tương ứng 1 1 với bộ .'1 ; '2 ; : : : ; 'n /
Một số bài toán về trò chơi 131

trong đó 'i số đo nhỏ nhất cung lượng giác theo chiều dương từ điểm đầu là A1 và điểm cuối là
đỉnh mà viên bi thứ i đang nằm ở đó.
Xét biểu thức tổng sau
n
X
f .!/ D 'i :
i D1

Khi đó, sau mỗi phép di chuyển bi, tương ứng với viên bi di chuyển cùng chiều kim đồng hồ thì
giá trị hàm số f giảm đi 2n
.mod2/, và đối với viên bi còn lại di chuyển ngược chiều kim
đồng hồ làm giá trị hàm tăng 2
n
.mod2/ nên biểu thức này có giá trị không đổi theo mod2:
Do đó, f là một bất biến theo mod2:
Trạng thái lúc đầu  
2 4 2 .n 1/
!0 D 0I ; ;:::;
n n n
tương ứng với:
n 1
X 2k
f .!0 / D D  .n 1/ .mod2/ :
n
kD0

Do đó với Œn D 2k .k 2 N /, gọi !  D .0; 0; :::; 0/ là trạng thái cuối để đưa các bi về 1 ô vuông
thì
f .!  / D 0 .mod2/
nhưng
f .!0 /   .mod2/ :
Điều này cho thấy không thể đưa tất cả các bi về 1 ô vuông sau khi thực hiện một số bước hữu
hạn các bước di chuyển.
Ta sẽ chứng minh rằng n D 2k C 1I k 2 N thỏa mãn, ta xây dựng thuật toán sau để đưa các bi
về cùng một ô A1 trong trường hợp này.

1. Đầu tiên lấy hai bi ở các ô A2 ; An di chuyển về ô A1 , tiếp theo ta lấy các bi ở 2 ô A3 ; An 1
sao cho bi ở ô A3 di chuyển về vị trí A2 ; bi ở ô An 1 di chuyển về ô An , thực hiện tiếp
bước nữa ta di chuyển được các bi ở hai ô A2 ; An (mới được chuyển bi vào) về ô A1 :

2. Tiếp tục các bước trên, ta cứ lấy lần lượt hai bi ở các cặp ô Ai ; An i C2 di chuyển về các vị
trí như sơ đồ:

.Ai C1 ; An i C1 / ! .Ai ; An i C2 / ! : : : ! .A2 ; An / ! .A1 ; A1 /

Vậy n lẻ là tất cả các số nguyên dương cần tìm.

Bài toán 8. Các dấu cộng và trừ được viết vào bảng 2014  2014. Mỗi lần ta cho phép đảo
ngược tất cả các dấu trong một hàng; trong một cột hoặc dọc theo một đường bất kì song song
với một trong hai đường chéo của bảng (dĩ nhiên cũng được đảo dấu các góc của bảng ). Hỏi có
thể thực hiện được các phép đảo mà sau một số hữu hạn lần, ta thu được bảng chỉ có cùng dấu
cộng nếu như thời điểm ban đầu có đúng một dấu cộng không nằm trên ô vuông thuộc đường
chéo của bảng còn các ô còn lại đều là dấu trừ?
132 Các phương pháp giải toán qua các kỳ thi Olympic

Lời giải. Câu trả lời là không. Trước hết ta đưa ra khái niệm: hình vuông trung tâm là hình
vuông tạo ra từ bảng mà có hai đường chéo là đường chéo chính của bảng. (Hai ví dụ các ô bị tô
đen trong hình dưới đây tạo thành các hình vuông trung tâm).

Bây giờ ta thay mỗi dấu cộng bằng số 1 còn mỗi dấu trừ thay bằng số 1.

Khi đó, ta giả sử số 1 duy nhất của bảng nằm ở ô .i I j /.

Lúc này; ta xét tập hợp tất cả các ô vuông nằm trên cạnh của hình vuông trung tâm đi qua ô
.i I j / ngoại trừ đỉnh. (Hình vẽ minh họa dưới đây; giả sử số 1nằm ở ô .2I 3/ của bảng 6  6).

Ta gọi tập hợp tất cả các ô đó là tập S: Rõ ràng sau mỗi bước biến đổi thì tích các số trong tập
hợp S là một hằng số không đổi.

Thời điểm ban đầu tích này bằng 1 nên ở thời điểm kết thúc tích này cũng phải bằng 1.

Do đó; không thể đưa về trường hợp tất cả các ô của bảng cùng dấu được.

Bài toán được giải quyết.

Nhận xét. Câu hỏi đặt ra là nếu dấu trừ duy nhất đó đặt ở ô nằm trên đường chéo của bảng thì
sao? Câu hỏi này xin dành cho bạn đọc tự suy ngẫm.

Tiếp tục xoay quanh vấn đề này thì nếu như có hai dấu trừ thì sao?

Dưới đây là mô hình để bạn đọc thừ suy nghĩ thêm về vấn đề này.
Một số bài toán về trò chơi 133

Ngoài tính chất đơn biến, bất biến, có những bài toán đòi hỏi xây dựng cấu hình và tìm ra tính
chất là bản chất của bài toán. Ta tìm hiểu thông qua hai ví dụ rất khó sau:
Bài toán 9 (IMO, 2010). Cho sáu cái hộp nằm trên một hàng: S1 I S2 I : : : I S6 ; mỗi hộp chứa
đúng một viên bi. Thực hiện một trong hai quy tắc sau:

 Quy tắc 1: Nếu hộp Sk k D 1I 5 chứa ít nhất một viên bi; có thể lấy viên một viên trong
Sk ra và cho hai viên bi vào SkC1 .

 Quy tắc 2: Nếu hộp Sk k D 1I 4 chứa ít nhất một viên bi; có thể lấy một viên trong Sk và
đổi chỗ các viên bi trong hai hộp SkC1 I SkC2 cho nhau.
Chứng minh rằng có thể sử dụng một chuỗi quy tắc sao cho cuối cùng thu được năm hộp đầu
2010
rỗng và hộp S6 chứa đúng 20102010 viên bi.
Lời giải. Ta kí hiệu .a1 I a2 I : : : I an / 7! .b1 I b2 I : : : I bn / tương ứng với ý nghĩa: ban đầu các hộp
S1 I S2 I : : : I Sn lần lượt có số bi là a1 I a2 I : : : I an và sau một số hữu hạn quy tắc ta thu được số bi
tương ứng là b1 I b2 I : : : I bn .
2010
Đặt A D 20102010 . Mục tiêu của bài toán là chứng minh có một bộ các quy tắc để có:

.1I 1I 1I 1I 1I 1/ 7! .0I 0I 0I 0I 0I A/

Ta chứng minh hai nhận xét sau:


Nhận xét 1. .aI 0I 0/ 7! .0I 2a I 0/ :
Thật vậy, ta chứng minh quy nạp theo k khẳng định sau:
 
k
.aI 0I 0/ 7! a kI 2 I 0 I 8k D 1I a

Với k D 1; khẳng định đúng bởi vì đó chính là quy tắc 1:

.aI 0I 0/ 7! .a 1I 2I 0/ :

Giả sử khẳng định đúng với k < a nào đó; tức là ta có:
 
.aI 0I 0/ 7! a kI 2k I 0

Sử dụng quy tắc 1 ta có:


       
a kI 2k I 0 7! a kI 2k 1I 2 7! a kI 2k 2I 2:2 7! : : : 7! a kI 0I 2kC1
134 Các phương pháp giải toán qua các kỳ thi Olympic

Dùng quy tắc 2 ta có:


     
k kC1 kC1
.aI 0I 0/ 7! a kI 2 I 0 7! a kI 0I 2 7! a k 1I 2 I0

Khẳng định đúng với k C 1. Do đó, ta dễ dàng suy ra nhận xét 1 đúng.
Nhận xét 2. Đặt P1 D 2I PnC1 D 2Pn ; 8n  1. Khi đó, ta có:

.aI 0I 0I 0/ 7! .0I Pa I 0I 0/

Thật vậy, ta chứng minh quy nạp theo k khẳng định sau:

.aI 0I 0I 0/ 7! .a kI Pk I 0I 0/ I k D 1I a

Với k D 1; khẳng định đúng bởi vì đó chính là quy tắc 1 W

.aI 0I 0I 0/ 7! .a 1I 2I 0I 0/ D .a 1I P1 I 0I 0/

Giả sử khẳng định đúng với k < a nào đó; tức là ta có:

.aI 0I 0I 0/ 7! .a kI Pk I 0I 0/

Áp dụng nhận xét 1 và quy tắc 1 ta suy ra:

.a kI Pk I 0I 0/ 7! a kI 0I 2Pk I 0 D .a

kI 0I PkC1 I 0/ 7! .a k 1I PkC1 I 0I 0/

Suy ra:
.aI 0I 0I 0/ 7! .a kI Pk I 0I 0/ 7! .a k 1I PkC1 I 0I 0/
Khẳng định đúng với k C 1. Do đó, ta dễ dàng suy ra nhận xét 2 đúng.
Ta áp dụng quy tắc 1 và 2 ta có:

.1I 1I 1I 1I 1I 1/ 7! .1I 1I 1I 1I 0I 3/ 7! .1I 1I 1I 0I 3I 0/ 7! .1I 1I 0I 3I 0I 0/


7! .1I 0I 3I 0I 0I 0/ 7! .0I 3I 0I 0I 0I 0/

Áp dụng bổ đề 2 hai lần có:

.0I 3I 0I 0I 0I 0/ 7! .0I 0I P3 I 0I 0I 0/ D .0I 0I 16I 0I 0I 0/ 7! .0I 0I 0I P16 I 0I 0/

Ta có:
2010 20102010 2010
A D 20102010 < 211 D 211:2010
2011 215
< 2.2 /
2011 11 112011
< 22010 D 22 < 22 D P16
Ta dùng quy tắc 2 suy ra:
 
A
.0I 0I 0I P16 I 0I 0/ 7! .0I 0I 0I P16 1I 0I 0/ 7! .0I 0I 0I P16 2I 0I 0/ 7! : : : 7! 0I 0I 0I I 0I 0
4

Ta dùng quy tắc 1 sẽ có:


       
A A A A
0I 0I 0I I 0I 0 7! 0I 0I 0I 1I 2I 0 !
7 0I 0I 0I 2I 2:2I 0 7! : : : 7! 0I 0I 0I 0I I 0
4 4 4 2
Một số bài toán về trò chơi 135

Tiếp tục sử dụng quy tắc 1 như vậy ta có:


 
A
0I 0I 0I 0I I 0 7! .0I 0I 0I 0I 0I A/
2

Như vậy ta thu được:


 
A
.1I 1I 1I 1I 1I 1/ 7! .0I 3I 0I 0I 0I 0/ 7! .0I 0I 0I P16 I 0I 0/ 7! 0I 0I 0I I 0I 0
4
 
A
7! 0I 0I 0I 0I I 0 7! .0I 0I 0I 0I 0I A/
2

Như vậy bài toán được chứng minh xong.

2.2. Bài toán hai người chơi


Bài toán 10 (Tập huấn IMO Việt Nam, 2010). Cho 2011 viên kẹo trên bàn. Hai người chơi
trò chơi: họ được bốc số kẹo là một trong các số 1; 2; 6: Hai người lần lượt bốc kẹo, ai là người
bốc viên kẹo cuối cùng là người thắng cuộc. Giả sử hai người đều thông minh, hỏi ai là người có
chiến thuật thắng.

Lời giải. Trước khi giải bài toán này, ta tìm kết quả với số kẹo nhỏ để xem tính chất của kết quả.
Giả sử ban đầu bàn có n viên kẹo. Ta xem xét bài toán với các giá trị n cụ thể như sau:

 Với n D 1; 2 ta thấy ngay là người thứ nhất thắng (vì bốc một lần có thể hết).

 Với n D 3 thì người thứ hai thắng vì người thứ nhất chỉ có thể bốc 1 hoặc 2 viên, tương
ứng với người thứ hai bốc 2 hoặc 1 viên để thắng.

 Với n D 4 người thứ nhất bốc 1 viên ở lượt đầu, và sử dụng chiến thuật với n D 3 đẩy
người thứ hai vào thế thua và người thứ nhất sẽ thắng.

 Với n D 5 người thứ nhất bốc 2 viên ở lượt đầu, tương tự, người thứ nhất sẽ thắng.

 Với n D 6 thì hiển nhiên người thứ nhất sẽ thắng.

 Với n D 7 thì người thứ hai thắng vì với ba cách bốc người thứ nhất .1I 2I 6/ thì tương ứng
với số viên trên bàn là .6I 5I 1/ và người thứ hai có chiến thuật thắng.

 Với n D 8 thì người thứ nhất ở lượt đầu bốc 1 viên, và bàn còn 7 viên nên người thứ nhất
có chiến thuật thắng.

Tương tự như vậy, ta thu được bảng kết quả sau (dòng đầu là các giá trị của n, dòng dưới là người
chơi có chiến lược thắng):
1 2 3 4 5 6 7 8 9 10 11 12 13 14 15 16 17
1 1 2 1 1 1 2 1 1 2 1 1 1 2 1 1 2
136 Các phương pháp giải toán qua các kỳ thi Olympic

Ta thấy rằng người thứ nhất sẽ thắng nếu n có số dư khi chia cho 7 là 1; 2; 4; 5; 6 và người thứ
hai sẽ thắng khi n có số dư khi chia cho 7 là 0; 3.
Sau khi dự đoán như vậy, ta chứng minh bài toán theo phương pháp quy nạp. Với n D 7k C rI r 2
f1I 2I 3I 4I 5I 6I 7g. Quy nạp theo k ta có: Với k = 0 thì theo bảng ta thấy nhận xét đúng. Xét
n D 7 .k C 1/ C rI r 2 f1I 2I 3I 4I 5I 6I 7g

1. Nếu r D 1; 2; 6 thì người thứ nhất bốc tương ứng 1; 2; 6 viên và trên bàn còn 7k C 7 viên
nên theo quy nạp thì người thứ nhất sẽ thắng.

2. Nếu r D 3 thì ta thấy:

 Người thứ nhất bốc 1 viên thì người thứ hai bốc 2 viên và trên bàn còn 7k C 7 viên
nên theo quy nạp thì người thứ hai sẽ thắng.
 Người thứ nhất bốc 2 viên thì người thứ hai bốc 1 viên và trên bàn còn 7k C 7 viên
nên theo quy nạp thì người thứ hai sẽ thắng.
 Người thứ nhất bốc 6 viên và trên bàn còn 7k C 4 viên nên theo quy nạp thì người
thứ hai có chiến thuật thắng.

3. Nếu r D 4; 5 thì người thứ nhất bốc tương ứng 1; 2 viên để đưa về trường hợp trên bàn
còn 7 .k C 1/ C 3 và người thứ nhất sẽ thắng.

4. Nếu r D 7 thì ta thấy:

 Người thứ nhất bốc 1 viên thì trên bàn còn 7 .k C 1/ C 6 viên là trường hợp trên nên
người thứ hai thắng.
 Người thứ nhất bốc 2 viên thì trên bàn còn 7 .k C 1/ C 5 viên là trường hợp trên nên
người thứ hai thắng.
 Người thứ nhất bốc 6 viên thì trên bàn còn 7 .k C 1/ C 1 viên là trường hợp trên nên
người thứ hai thắng.

Tóm lại nhận xét là đúng. Do 2011 chia cho 7 dư 2 nên theo ý luận này thì người thứ nhất có
chiến thuật thắng.

Nhận xét. Bài toán này xuất phát khởi điểm từ những trường hợp cụ thể số nhỏ dễ xét để dự
đoán kết quả bài toán và phương pháp được hình thành. Tương tự như vậy ta bắt đầu quay lại bởi
bài toán sau:

Bài toán 11 (Bulgari MO, 1995). Hai người A và B chơi chơi một trò chơi với tảng đá có
n  2 viên. Ở lượt đầu tiên, A lấy đi một số viên đá (ít nhất là 1 viên và nhiều nhất là n 1
viên). Sau đó, hai người luân phiên nhau, mỗi người lấy đi ít nhất 1 viên nhưng không được lấy
quá số viên mà lượt liền trước người kia lấy. Người lấy được viên đá cuối cùng là người thắng
cuộc. Hỏi ai là người có chiến thuật thắng.

Lời giải. Ở mỗi lượt ta xét cặp số .mI l/ trong đó m là số đá đang có; l là số đá nhiều nhất có
thể lấy đi ở lượt đó. Ta cần tìm số nguyên dương n để cặp .nI n 1/ là cặp tốt với A nếu như A
thắng và là tốt với B nếu như B thắng.
Ta thấy rằng cặp .mI l/ là tốt với B thì mọi cặp .mI l0 /với l0 < l cũng là tốt với B:
Một số bài toán về trò chơi 137

Cũng tương tự như bài toán trên, ta cũng thử vài giá trị đầu của n và dự đoán rằng: cặp .nI n 1/
là tốt với B khi và chỉ khi n là lũy thừa của 2:
Bây giờ, ta chứng minh quy nạp theo k với n D 2k . Với k D 1 thì hiển nhiên A chỉ có thể lấy 1
viên ở lượt đầu và sau đó B cũng lấy 1 viên và B là người thắng.
Giả sử đúng tới k; tức là ta có cặp 2k I 2k 1 là tốt đối với B ta xét cặp 2kC1 I 2kC1 1 . Nếu
 

A lấy ít nhất 2k viên ở lượt đầu thì hiển nhiên B có thể lấy tất cả số viên còn lại và là người thắng
cuộc. Nếu A lấy l viên 1  l < 2k . Khi đó, do nhận xét thì cặp 2k I l cũng tốt với B:
Do đó, B có chiến thuật để lượt nào đó lấy được viên thứ 2k (là viên cuối cùng của lượt đó). Khi
đó, lượt tiếp theo ngay sau đó là lượt của A với số viên còn lại là 2k viên. Do đó, lại quay về
trường hợp với cặp 2k I l 0 nào đó.


Và khi đó, do giả thiết quy nạp và nhận xét thì B sẽ thắng. Như vậy với n D 2k thì B thắng.
Còn với n D 2k C r 1  r < 2k thì lượt đầu A lấy r viên, và do đó A sẽ thắng.


Kết luận:
1. n D 2k C r 1  r < 2k thì A thắng.


2. n D 2k thì B thắng.
Nhận xét. Với những bài toán bốc viên sỏi thì ý tưởng xét trường hợp nhỏ là ý tưởng rất tốt
để dự đoán phương pháp giải và thông thường, khi đi theo hướng đi này, người ta thường dùng
phương pháp quy nạp. Vấn đề tiếp theo quay sang một góc nhìn khác.
Bài toán 12 (IMC, 2012). Cho đa thức

f .x/ D x 2012 C a2011 x 2011 C    C a1 x C a0 :

Esten và Homer cùng chơi một trò chơi. Ở mỗi lượt, mỗi người chọn một hệ số a0 ; a1 ; : : : ; a2011
chưa được gán giá trị và thay hệ số đó bằng một số thực mình chọn. Esten là người chơi trước.
Hai người chơi luân phiên cho đến khi các hệ số được gán hết giá trị. Homer chiến thắng trò
:
chơi này khi kết thúc trò chơi thì có f .x/ ::m .x/, ngược lại thì Esten thắng.
a) Ai sẽ là người thắng nếu m .x/ D x 2012?

b) Ai sẽ là người thắng nếu m .x/ D x 2 C 1?


Lời giải. a) Ta có: Homer thắng khi và chỉ khi f .2012/ D 0. Điều này tương đương với:

20122012 C a2011  20122011 C    C ak  2012k C    C a1  2012 C a0 D 0

Nhận thấy rằng có 2012 hệ số nên có 2012 lượt chơi mà Esten là người chơi đầu nên Homer sẽ
chơi lượt cuối, tức là điền hệ số cuối cùng của đa thức.
Giả sử đã biết các hệ số: a0 ; a1 ; : : : ; ak 1 ; akC1 ; akC2 ; : : : ; a2011 và lượt cuối, Homer điền cho
hệ số ak và giá trị đó bằng:

20122012 C a2011  20122011 C    C a1  2012 C a0


ak D
2012k
Như vậy, trong trường hợp này, Homer thắng.
138 Các phương pháp giải toán qua các kỳ thi Olympic

b) Xét hai đa thức:

1005
X 1005
X
k 1006
g .y/ D a2k y C y và h .y/ D a2kC1 y k :
kD0 kD0

Khi đó, ta có:


f .x/ D g x 2 C x  h x 2 :
 

Homer thắng khi và chỉ khi: g . 1/ D h . 1/ D 0.


Nhận thấy rằng: mỗi đa thức g .x/ và h .x/ đều có chẵn hệ số chưa biết. Chiến thuật của Homer
là: Khi Esten gán giá trị cho hệ số của đa thức nào trong hai đa thức thì lượt sau đó, Homer cũng
gán giá trị của hệ số khác trong đa thức đó.
Như vậy, ở cả hai đa thức g .x/ I h .x/ thì hệ số cuối cùng là do Homer quyết định. Như vậy,
tương tự ý (a) thì Homer sẽ là người chiến thắng.

Nhận xét. Bài toán này có thể cho m .x/ D x 3 C 1 và kết quả thu được vẫn là Homer thắng.
Câu hỏi đặt ra là liệu có đa thức nào ( với bậc nhỏ nhỏ ) mà luôn có chiến thuật để Esten thắng?
Vấn đề này dành cho bạn đọc.

Bài toán 13 (Vietnam TST, 2002). Người ta ghi lên bảng số nguyên ˚ dương NN00. ˘Hai người A
và B chơi trò chơi sau: Người A xóa số N0 rồi ghi lên bảng số N1 2 N0 1I 3 . Tiếp theo
người B xóa số N1 rồi ghi lên bảng số N2 2 N1 1I N31 . Đến lượt mình người A lại thực
˚  ˘

hiện phép toán trên đối với N2 : : : trò chơi cứ tiếp tục cho đến khi bảng xuất hiện số 0: Người
ghi số 0 đầu tiên được coi là thắng cuộc, người còn lại bị coi là thua cuộc. Hỏi ai, người A hay
người B là người có cách chơi để chắc chắn thắng nếu:

a) N0 D 120:

32002 1
b) N0 D 2
:

32002 C1
c) N0 D 2
:

Lời giải. Trước hết, ta thấy rằng với mọi giá trị N0 cho trước thì hoặc người A có chiến lược
thắng hoặc người B có chiến lược thắng.
Xét hàm số f W N ! f0I 1g thỏa mãn điều kiện sau:

 f .n/ D 1 nếu người A có chiến lược thắng cuộc nếu N0 D n:

 f .n/ D 0 nếu người B có chiến lược thắng cuộc nếu N0 D n:

Dễ thấy rằng f .1/ D f .2/ D 1; f .3/ D 0 và với mỗi giá trị n  3, ta có:

 Nếu f .n 1/ D 0 hoặc f n3 D 0 thì f .n/ D 1:


 ˘

 Nếu f .n 1/ D 1 hoặc f n3 D 1 thì f .n/ D 0: Suy ra f .n/ D 1


 ˘  n ˘
f .n 1/f 3
với mọi n  3.
Một số bài toán về trò chơi 139

Chú ý rằng f .k/ D f 2 .k/; 8k vì f .k/ 2 f0I 1g nên ta thấy rằng với mọi k  1, ta có

f .3k/ D 1 f .3k 1/f .k/


f .3k C 1/ D 1 f .3k/f .k/ D 1 .1 f .3k 1/f .k// f .k/ D 1 f .k/ C f .3k 1/f .k/

Cộng từng vế của hai đẳng thức trên lại, ta được

f .k/ C f .3k/ C f .3k C 1/ D 2; 8k  1:

Từ đó suy ra f .3k C 2/ D f .3k/; 8k  1.


Do đó, nếu f .k/ D 0 thì f .k C1/ D f .3k/
 k Df .3k C1/ D f .3k C2/ D 1 hay f .3k C3/ D 0.
Từ đây, ta thấy rằng vì f .3/ D 0 nên f 3 2 3 D 0; 8k  2. Ta được các kết quả sau:
 
35 3
 f .120/ D f 2
D 0:
    j k
32002 1 32002 3 32002 1
 f 2
D1 f 2
f 6
D 1.
     
32002 C1 32002 3 32002 3
 f 2
Df 2C 2
Df 2
D 0.

32002 1
Vậy nếu N0 D 120 thì người B có chiến lược thắng, nếu N0 D 2
thì người A có chiến lược
2002
thắng, nếu N0 D 3 2 C1 thì người B có chiến lược thắng.

Bài toán 14 (IMO, 2012). Trò chơi đoán kẻ nói dối là một trò chơi giữa hai người chơi A và
B: Quy tắc của trò chơi phụ thuộc vào hai số nguyên dương k và n mà cả hai người chơi đều đã
biết trước. Bắt đầu trò chơi, A sẽ chọn các số nguyên x và N với 1  x  N: A sẽ giữ bí mật số
x và nói số N cho B một cách chính xác. B sẽ thu nhận thông tin về số x bằng cách hỏi A các
câu hỏi như sau: Với mỗi câu hỏi, B chọn một tập S tùy ý các số nguyên dương (có thể là một
tập đã được nhắc đến trong câu hỏi trước đó) và hỏi A xem x có thuộc S hay không. Sau mỗi
câu hỏi, A phải trả lời có hoặc không, nhưng có thể nói dối bao nhiêu lần tùy thích, chỉ có điều
là phải trả lời đúng ít nhất một trong số k C 1 câu hỏi liên tiếp.
Sau khi B đã hỏi xong, B phải chỉ ra một tập X có tối đa n số nguyên dương. Nếu x 2 X; B
thắng, nếu ngược lại, B thua. Chứng minh rằng

1) Nếu n  2k ; B có thể đảm bảo một chiến thắng.

2) Với mọi k đủ lớn, tồn tại một số nguyên n  .1:99/k sao cho B không thể đảm bảo có một
chiến thắng.

Lời giải. Ta phát biểu lại trò chơi thành dạng tập hợp như sau:
Người chơi A chọn một phần tử x 2 S với jS j D N và người chơi B hỏi một dãy các˚câu hỏi liên
tiếp. Tương ứng với câu hỏi j; B sẽ chọn một tập con Dj  S và A chọn Pj 2 Qj ; QjC :
Người A phải đảm bảo rằng với mọi j  1 thì luôn có x 2 Pj [ Pj C1 [    [ Pj Ck : Người B
thắng nếu sau một số bước hữu hạn thì có thể chọn được một tập hợp X mà jX j  n và x 2 X:
1) Ta chỉ cần chứng minh rằng N  2k C 1 thì người B có thể xác định được một tập S 0  S
mà jS 0 j  N 1 mà x 2 S 0 : Giả sử N  2n C 1: Trong lượt đầu tiên, B chọn một tập hợp con
140 Các phương pháp giải toán qua các kỳ thi Olympic
ˇ ˇ
tùy ý D1  S sao cho jD1 j  2k 1 và ˇD1C ˇ  2k 1 : Sau khi nhận được tập hợp P1 từ A; người
chơi B có thể đi bước tiếp theo. Người chơi B chọn một tập D2  S sao cho
ˇD2 \ P C ˇ  2k 2 ; ˇD C \ P C ˇ  2k 2 :
ˇ ˇ ˇ ˇ
1 2 1

Người chơi B sẽ tiếp tục theo cách này: Trong bước thứ j thì chọn Dj sao cho
ˇDj \ P C ˇ  2k j và ˇD C \ P C ˇ  2k j :
ˇ ˇ ˇ ˇ
j j j

Như thế, người chơi B sẽ nhận được các tập hợp P1 ; P2 ; : : : ; Pk mà

.P1 [ P2 [    [ Pk /C  1:

Sau đó, người chơi B chọn một tập DkC1 chứa bất cứ phần tử nào không thuộc P1 [P2 [  [Pk :

Có 2 trường hợp xảy ra:


C
 Nếu người chơi A chọn PkC1 D DkC1 thì B chọn S 0 D S nDkC1 và bài toán đúng.

 Nếu người chơi A chọn PkC1 D DkC1 thì B lặp lại quy trình trên S1 D S nDkC1 để nhận
được các dãy PkC2 ; PkC3 ; : : : ; P2kC1 . Điều kiện sau sẽ được thỏa mãn

jS1 n .PkC2 [ PkC3 [    [ P2kC1 /j  1; từ jS1 j  2k :


ˇ ˇ
Tuy nhiên, ta có ˇ.PkC1 [ PkC2 [    [ P2kC1 /C ˇ  1 nên ta có thể chọn
ˇ ˇ

S 0 D PkC1 [ PkC2 [    [ P2kC1 :


p
2) Gọi p; q là các số nguyên dương thỏa mãn 1; 99 < q
< 2 và chọn số k0 2 N sao cho
 k0
p  q
2 1 ; p k0 1; 99k0 > 1:
q 2
 
Ta sẽ chứng minh rằng với mọi k  k0 ; nếu jS j 2 1; 99k I p k thì tồn tại một chiến thuật để
người A có thể chọn các tập P1 ; P2 ; : : : tương ứng với tập D1 ; D2 ; : : : mà người B chọn sao cho
với mọi j thì điều kiện sau thỏa mãn Pj [ Pj C1 [    [ Pj Ck D S:
 
Giả sử rằng S D f1; 2; 3; : : : ; N g ; người chơi A sẽ xem xét bộ N số là .x/ D x1j ; : : : ; xNj
:
Ban đầu, ta có x10 D x20 D    D xN
0
D 1: Sau khi tập hợp Pj được chọn thì ta có thể định nghĩa
j C1 j
x dựa trên x với quy tắc sau
(
1; i 2 Pj
xij C1 D
q  xij ; i … Pj

Người chơi A có thể chiến thắng nếu xij  q k với mỗi cặp .i; j /: Với một dãy x; định nghĩa
T .x/ D N j k
P
i D1 xi : Khi đó, người A sẽ cần đảm bảo rằng T .x /  q với mỗi j: Chú ý rằng

T .x 0 / D N  p k < q k :
Một số bài toán về trò chơi 141

Tiếp theo, ta sẽ chứng ˚minh rằng x j thỏa mãn T .x j /  q k và một tập Dj C1 mà người chơi
A có thể chọn Pj C1 2 Dj C1 ; DjCC1 thỏa mãn T .x j C1 /  q k : Gọi y là dãy nhận được nếu
Pj C1 D Dj C1 và z là dãy nhận được nếu Pj C1 D DjCC1 : Khi đó, ta có

qxij C ˇDj C1 ˇ ; T .z/ D qxij C ˇDjCC1 ˇ :


X ˇ ˇ X ˇ ˇ
T .y/ D
i 2DjCC1 i 2Dj C1

Cộng theo vế hai bất đẳng thức trên, ta được

T .y/ C T .z/ D qT .x j / C N  q kC1 C p k :


pk
Suy ra min fT .y/; T .z/g  q2 q k C 2
 q k ; do cách chọn k0 :
Từ đó ta có điều phải chứng minh.
Nhận xét. Bài toán này rất khó, rất phù hợp cho đề thi chọn học sinh giỏi quốc tế. Ý tưởng giải
quyết bài toán khá rõ ràng. Và sau đây, để kết thúc vấn đề Toán trò chơi, chúng tôi xin đưa ra một
số bài toán về trò chơi nhiều người tham gia.

2.3. Bài toán nhiều người chơi


Bài toán 15 (Mathematical Excalibur Vol.7, No.5). Có 20 cô gái ngồi xung quanh một cái
bàn và chơi một trò chơi với n quân bài. Ban đầu, một người giữ tất cả quân bài. Tới lượt mỗi
người, nếu có ít nhất một người giữ ít nhất 2 quân bài, một trong số họ sẽ phải chuyển một quân
bài cho mỗi người ngồi bên cạnh cô. Trò chơi kết thúc khi vào chỉ khi mỗi cô gái giữ nhiều nhất
một quân bài.
a) Chứng minh rằng nếu n  20, thì trò chơi không thể kết thúc.

b) Chứng minh rằng nếu n < 20, thì trò chơi cuối cùng cũng kết thúc.
Lời giải. Nếu n > 20 thì theo nguyên lý Dirichlet, trong mọi thời điểm luôn tồn tại một cô gái
giữ ít nhất hai quân bài. Do đó trò chơi không thể kết thúc.
Nếu n D 20, ta đánh dấu các cô gái bởi G1 ; G2 ; : : : ; G20 theo chiều kim đồng hồ và giả sử G1 là
cô gái giữ tất cả quân bài lúc ban đầu. Ta định nghĩa giá trị của một quân bài là i nếu nó đang
được Gi giữ. Gọi S là tổng giá trị của tất cả các quân bài. Ban đầu, S D 20. Xét lúc trước và sau
khi Gi chuyển một quân bài cho mỗi người ngồi cạnh cô. Ta xét các trường hợp sau:
1. Nếu i D 1, thì S sẽ tăng lên 1 1 C 2 C 20 D 20.

2. Nếu 1 < i < 20, thì S không thay đổi vì i i C .i 1/ C .i C 1/ D 0.

3. Nếu i D 20, thì S sẽ giảm đi 20 vì 20 20 C 1 C 19 D 20.


Do đó, trước và sau các lượt chơi, S luôn luôn là bội của 20: Giả sử rằng trò chơi có thể kết thúc.
Khi đó mỗi cô gái cầm một quân bài và S D 1 C 2 C    C 20 D 210, khi đó S không là bội của
của 20; mâu thuẫn. Vậy trò chơi không thể kết thúc.
b) Để chứng minh rằng trò chơi phải kết thúc nếu n < 20, ta sẽ để cho hai cô gái đánh dấu quân
bài nếu đây là lần đầu tiên một trong hai cô gái chuyển bài cho người kia. Bất cứ khi nào một cô
gái chuyển một quân bài cho người bên cạnh, ta sẽ yêu cầu cô ấy dùng quân bài đã được đánh
142 Các phương pháp giải toán qua các kỳ thi Olympic

dấu giữa hai người nếu nó tồn tại. Do vậy mỗi quân bài đã được đánh dấu sẽ bị kẹt lại giữa cặp
đôi đã đánh dấu nó.
Nếu n < 20 do có 20 khoảng kẹp giữa hai người liên tiếp nên sẽ tồn tại một cặp cạnh nhau mà
không bao giờ đánh dấu quân bài nào, do đó hai người đó không trao đổi bất kì quân bài nào.
Nếu trò chơi có thể tiếp diễn mãi mãi,ta ghi lại số lần mỗi cô gái đã chuyển các quân bài. Do trò
chơi có thể kéo dài mãi mãi nên không phải tất cả các cô gái đều chuyển bài hữu hạn lần.
Do vậy ta sẽ bắt đầu với cặp mà không trao đổi bài lần nào và xét theo chiều kim đồng hồ, cuối
cùng cũng có một cặp Gi và Gi C1 mà Gi đã chuyển bài một số hữu hạn lần và Gi C1 đã chuyển
bài vô hạn lần. Điều này rõ ràng là không thể với việc Gi cuối cùng sẽ đến lúc không chuyển bài
nữa nhưng vẫn tiếp tục nhận bài từ Gi C1 . Vậy trò chơi chắc chắn sẽ kết thúc với n < 20.

Bài toán 16. Cho n người chơi được đánh số thứ tự 1; 2; 3; : : : ; n ngồi quanh một bàn tròn.
Ban đầu mỗi người cầm đúng một quả bóng. Một luật chơi được thực hiện như sau: Người 1 đưa
người 2 một quả bóng. Sau đó người 2 đưa người 3 hai quả bóng. Nhận bóng xong thì người 3
đưa người 4 một quả bóng. Và rồi người 4 đưa người 5 một quả bóng. Trò chơi cứ tiếp tục như
vậy; mỗi người khi nhận một quả bóng thì sẽ đưa hai quả bóng và mỗi người nhận hai quả bóng
thì sẽ đưa một quả bóng. Người nào sau khi đưa bóng mà không còn quả bóng nào sẽ tự động
rời khỏi cuộc chơi. Tìm tất cả các giá trị của n để trò chơi kết thúc là một người cầm n quả bóng.

Lời giải. Trước tiên, ta quy ước rằng số thứ tự đầu tiên của mỗi người sẽ không thay đổi trong
suốt quá trình. Một vòng chơi là tính từ người đầu tiên đưa bóng cho đến lúc người cuối cùng
còn lại mỗi lượt đưa bóng. Ta chứng minh các giá trị của n thỏa mãn là:

n D 2m C 1I n D 2m C 2:

Với n D 1I 2 dễ thấy trò chơi kết thúc thỏa mãn. Giả sử n  3.


Ta sẽ xác định trò chơi sau vòng đầu tiên.

 Người 1 đưa một quả bóng và bị rời khỏi trò chơi.

 Người 2 đưa hai quả bóng và rời khỏi trò chơi.

 Người 3 đưa một quả bóng và giữ hai quả bóng.

 Người 4 đưa hai quả bóng và rời khỏi trò chơi.

 Người 5 đưa một quả bóng và giữ hai quả bóng.

Cứ tiếp tục như vậy ta suy ra còn n 2 1 người chơi; đó là những người nhận hai quả bóng và đưa
 ˘

đi một quả bóng; mỗi người (ngoại trừ người 3) thì mỗi người sẽ có hai quả bóng sau vòng một;
riêng người 3 có ba hoặc bốn quả bóng.
Ta có nhận xét sau: Với k  2 xét cấu hình ban đầu .có n người chơi/:

1. Mỗi người chơi (ngoại trừ người bắt đầu chơi đầu tiên) thì mỗi người có đúng k quả bóng.

2. Người chơi đầu tiên có ít nhất k C 1 quả bóng .tức là lượt đầu tiên của anh ta là cho đi
một hoặc hai quả bóng/:
Một số bài toán về trò chơi 143

Trò chơi vẫn được giữ nguyên luật chơi trên. Khi đó, trò chơi được kết thúc là một người cầm n
quả bóng khi và chỉ khi n là lũy thừa của 2:
Thật vậy,

 Nếu số người là lẻ thì rõ ràng, sau hai vòng thì ta lại thu được cấu hình ban đầu và như vậy
thì trò chơi sẽ không bao giờ dừng lại.

 Nếu số người là chẵn, sau mỗi vòng thì những người ở vị trí lẻ tăng nên một quả bóng; còn
những người ở vị trí chẵn sẽ giảm đi một quả bóng.

Do đó, sau k vòng thì những người vị trí chẵn sẽ không còn bóng và do đó; một nửa số người
chơi phải rời khỏi cuộc chơi.
Khi đó; cấu hình mới thu được là: số người giảm đi một nửa; mỗi người có đúng 2k quả bóng;
riêng người chơi đầu tiên của cấu hình này có ít nhất 2k C 1 quả bóng. Tiếp tục lập luận như trên
ta phải có: n D 2m với m là số tự nhiên nào đó. Nhận xét được chứng minh.
Quay lại bài toán, ta có hai trường hợp xảy ra:

1. Người n nhận một quả bóng và đưa người 3 hai quả bóng trong vòng 1 (sau đó người n rời
khỏi trò chơi).

Khi đó, cấu hình mới là mỗi người có đúng 2 quả bóng; ngoại trừ người chơi đầu tiên của
cấu hình này (đây là người 3) thì có đúng 4 quả bóng.
n 2
Khi đó, n chẵn và số người còn lại là 2
.
n 2
Áp dụng bổ đề trên ta suy ra: 2
D 2m ! n D 2mC1 C 2 với m là số tự nhiên nào đó.

2. Người n nhận hai quả bóng và đưa người 3 một quả bóng.

Khi đó, cấu hình mới là: mỗi người có đúng 2 quả bóng; ngoại trừ người chơi đầu tiên của
cấu hình này (đây là người 3) thì có đúng 3 quả bóng.

Khi đó n lẻ và số người còn lại là n 2 1 . Áp dụng bổ đề trên ta suy ra n 1


2
D 2m k hay
n D 2mC1 C 1 với m là số tự nhiên nào đó.

Vậy tập các giá trị n thỏa mãn đề là n D 2m C 1I n D 2m C 2 với m là số tự nhiên nào đó.

Tài liệu tham khảo


[1] Tạp chí Toán Học Và Tuổi Trẻ số 298, trang 12 , NXBGD.

[2] Titu Andreescu, Gabriel Dospinescu, Problem From the Book.

[3] Diễn đàn Toán học http://artofproblemsolving.com.

[4] Jin Hant, A Course in Combinatorics.


144 Các phương pháp giải toán qua các kỳ thi Olympic
PHƯƠNG PHÁP ĐẾM BẰNG HAI CÁCH
THÔNG QUA BẢNG Ô VUÔNG
Đậu Hoàng Hưng
(Trường THPT Chuyên Phan Bội Châu, Nghệ An)

Trong bài viết này, chúng tôi muốn chia sẻ với bạn đọc về phương pháp đếm bằng hai cách bằng
bảng ô vuông nhằm giải quyết một số bài toán tổ hợp thường xuất hiện trong các kỳ thi học sinh
giỏi môn Toán các cấp liên quan đến việc đếm số phần tử của tập hợp.

Bài toán 1. Cho F .S/ là họ các tập con khác rỗng của tập hữu hạn S. Với mỗi x 2 S, gọi
d.x/ là số các tập con thuộc F .S/chứa x. Chứng minh rằng:
X X
a) d.x/ D jAj: (1.1)
x2S A2F .S /

X X X
b) d 2 .x/ D jA \ Bj: (1.2)
x2S A2F .S / B2F .S /

Lời giải. a) Giả sử S D fx1 ; x2 ; : : : ; xn g và F .S/ D fA1 ; A2 ; : : : ; Ak g .k  2n /: Xét bảng


n  k ô vuông (gồm n hàng, k cột) sao cho tại mỗi ô vuông thứ .i; j / (hàng thứ i , cột thứ
j ), ta đặt một số aij với aij D 1 nếu xi 2 Aj và aij D 0 nếu xi … Aj Khi đó, với mỗi
i 2 f1; 2; : : : ; ng ta có d.xi / D kj D1 aij , suy ra
P

n
X n X
X k
d.xi / D ai j :
i D1 i D1 j D1

Pn
Tương tự, với mỗi j 2 f1; 2; : : : ; kg ta có jAj j D i D1 aij , suy ra

k
X k X
X n
jAj j D ai j :
j D1 j D1 i D1

Pn Pk Pk Pn
Do i D1 j D1 ai j D j D1 i D1 aij nên ta có

n
X k
X
d.xi / D jAj j;
i D1 j D1

hay X X
d.x/ D jAj:
x2S A2F .S /

145
146 Các phương pháp giải toán qua các kỳ thi Olympic

a11 a1 2 ::: a1.k 1/


a1k
a21 a2 2 ::: a2.k 1/
a2k
:: :: :: :: ::
: : : : :
a.n 1/1 a.n 1/2 ::: a.n 1/.k 1/ a.n 1/k
an1 an2 ::: an.k 1/ ank
Pk
b) Với mỗi i 2 f1; 2; : : : ; ng; ta có d.xi / D j D1 aij WD ki , suy ra
0 12
n
X n
X k
X n
X
2
d .xi / D @ ai j A D ki2 : .1:3/
i D1 i D1 j D1 i D1

˚
Xét tập hợpˇ P .S/ˇ D .A; B/ W A; B 2 F .S/; A ¤ B; A \ B ¤ ˇ ; (ởˇ đâyP.A; B/ không kể
n
thứ tự). Do P .S/ là số cặp .1; 1/ trong mỗi hàng của bảng nên P .S/ D j D1 Ck2j , suy ra
ˇ ˇ ˇ ˇ

X X n n
ˇ X X
kj2 :
ˇ
jA \ Bj D 2 P .S/ C
ˇ ˇ kj D .1:4/
A2F .S / B2F .S / j D1 j D1

Từ (1.3) và (1.4), ta có X X X
d 2 .x/ D jA \ Bj:
x2S A2F .S / B2F .S /

Nhận xét. Từ các đẳng thức (1.1) và (1.2), ta có


X X
d.x/ D jX \ Aj; 8X  S .1:5/
x2X A2F

và X X X X
d.x/ D jA \ Bj: .1:6/
A2F .S / x2A A2F .S / B2F .S /

Trong chứng minh bài toán 1, chúng ta đã chuyển bài toán về đếm số số 1 trong bảng n  k ô
vuông theo hai cách khác nhau là đếm theo hàng và đếm theo cột. Tiếp tục sử dụng phương pháp
đếm này, ta xét bài toán sau:

Bài toán 2. Cho tập hợp S gồm 7 phần tử và A1 ; : : : ; A7 là các tập con khác nhau của S thoả
mãn jAi j  3; 8i 2 f1; : : : ; 7g, với hai phần tử bất kỳ a; b 2 S tồn tại duy nhất j 2 f1; : : : ; 7g
sao cho a; b 2 Aj . Chứng minh rằng với mọi i; j 2 f1; : : : ; 7g mà i ¤ j thì jAi \ Aj j D 1:

Lời giải. Giả sử S D fx1 ; x2 ; : : : ; x7 g và F .S/ D fA1 ; A2 ; : : : ; A7 g: Xét bảng 7  7 ô vuông


(gồm 7 hàng, 7 cột) sao cho tại mỗi ô vuông thứ .i; j / (hàng thứ i , cột thứ j ), ta đặt một số
aij với aij D 1 nếu xi 2 Aj , aij D 0 nếu xi … Aj : Khi đó, với mỗi i; j 2 f1; 2; : : : ; 7g; đặt
ri D 7j D1 aij và lj D 7iD1 aij , suy ra ri là số các tập hợp thuộc F .S/ chứa xi , lj là số các
P P

phần tử thuộc Aj . Từ giả thiết jAi j  3; 8i 2 f1; 2; : : : ; 7g; ta có


7
X 7 X
X 7 7 X
X 7 7
X
ri D ai j D ai j D jAj j  21: .2:1/
i D1 i D1 j D1 j D1 i D1 j D1
Phương pháp đếm bằng hai cách thông qua bảng ô vuông 147

Mặt khác, do mỗi bộ gồm 2 phần tử xj1 ; xj2 của A luôn tồn tại duy nhất Ai 2 F .A/ để xj1 ;
xj2 2 Ai nên ta có jAi \ Aj j  1; 8i; j 2 f1; 2; : : : 7g và i ¤ j , suy ra
X
jAi \ Aj j  C72 D 21:
i <j

Kết hợp với đẳng thức:


7 7
X X X r2 rk
jAi \ Aj j D Cr2k D k
;
i <j
2
kD1 kD1

ta được
7
X r2 k rk
 21: .2:2/
2
kD1

Theo bất đẳng thức Cauchy-Schuwarz, ta lại có


2 P 2 3
7
kD1 rk
7 7 7 7
!
X rk2 rk 1 X X 16 X
D rk2 rk  rk 5
7
2 2 2 7
4
kD1 kD1 kD1 kD1

7 7
! !
1 X X
D rk rk 7 :
14
kD1 kD1

Kết hợp với (2.1), ta có


7
X r2 k rk
 21: .2:3/
2
kD1

Từ (2.2) và (2.3), ta được


7
X r2 k rk X
D 21 ) jAi \ Aj j D 21:
2 i <j
kD1

Do vậy, ta phải có jAi \ Aj j D 1 với mọi 1  i < j  7:

Bài toán 3 (Romania MO, 1994). Cho S D fA1 ; A2 ; : : : ; A11 g là họ các tập hợp thoả mãn
Ai \ Aj ¤ ; .1  i < j  11/, jAj j D 5 .1  i  11/ và tập hợp A D A˚1 [ A2 [    [ A11 :
Với mỗi x 2 A, ký hiệu d.x/ là số các tập hợp thuộc S chứa x và d D max d.x/ W x 2 A : Tìm
giá trị nhỏ nhất của d:

Lời giải. Giả sử S D fx1 ; x2 ; : : : ; xm g và F .S/ D fA1 ; A2 ; : : : ; A11 g: Xét bảng m  11 ô


vuông (gồm m hàng, 11 cột) sao cho tại mỗi ô vuông thứ .i; j /(hàng thứ i , cột thứ j ), ta đặt
một số aij với aij D 1 nếu xi 2 Aj và aij D 0 nếu xi … Aj : Khi đó:

11
X m
X
d.xi / D aij ; 8i 2 f1; 2; : : : ; mg; jAj j D aij ; 8j 2 f1; 2; : : : ; 11g
j D1 i D1
148 Các phương pháp giải toán qua các kỳ thi Olympic


m
X m X
X 11 11 X
X m 11
X
d.xi / D aij D ai j D jAj j D 55: .3:1/
i D1 i D1 j D1 j D1 i D1 j D1

Ta có
m m m
Xˇ ˇ X
ˇAi \ Aj ˇ D 2 1X   1 X
Cd.xk / D d.xk / d.xk / 1  .d 1/ d.xk /: .3:2/
i <j
2 2
kD1 kD1 kD1

Từ (3.1) và (3.2), suy ra


X 55.d 1/
jAi \ Aj j  : .3:3/
i <j
2

Mặt khác, từ giả thiết Ai \ Aj ¤ ; nên với mỗi cặp .A Pi ; Aj / luôn tồn tại xk 2 Ai \ Aj hay
cặp .Ai ; Aj / được tính ít nhất một lần trong biểu thức i <j jAi \ Aj j, do đó ta có
X
2
jAi \ Aj j  C11 D 55: .3:4/
i <j

Từ (3.3) và (3.4), ta suy ra


d  3:
Nếu d D 3 thì với mỗi xi 2 S ta có d.xi /  d D 3: Ta chứng minh không tồn tại xj 2 S để
d.xj /  2. Thật vậy, giả sử tồn tại xj 2 S để d.xj /  2, khi đó
m
1X  
55  d.xi / d.xi / 1
2 i D1
1X   1  
D 1 C d.xj / d.xj /
d.xi / d.xi / 1
2 2
i ¤j
 
n 1X d.xj / d.xj / 1
 d.xi / C :
2 2
i ¤j

Mặt khác, ta lại có


  m  
n 1X d.xj / d.xj / 1 n 1X d.xj / d.xj / d
d.xi / C D d.xi / C
2 2 2 i D1
2
i ¤j
.3 1/55 2.2 3/
 C D 54:
2 2
Điều này dẫn đến vô lý hay ta có d.xi / D 3; 8xi 2 S, suy ra 3m D 55 (không thoả mãn). Vậy
d D 4 và ta chỉ ra có 11 tập hợp thoả mãn yêu cầu bài toán là:

A1 D A2 D f1; 2; 3; 4; 5g; A3 D f1; 6; 7; 8; 9g; A4 D f1; 10; 11; 12; 13g;


A5 D f2; 6; 9; 10; 14g; A6 D f3; 7; 11; 14; 15g; A7 D f4; 8; 9; 12; 15g;
A8 D f5; 9; 13; 14; 15g; A9 D f4; 5; 6; 11; 14g; A10 D f2; 7; 11; 12; 13g;
A11 D f3; 6; 8; 10; 13g:
Phương pháp đếm bằng hai cách thông qua bảng ô vuông 149

Bài toán 4. Trong một kỳ thi có 11 thí sinh tham gia giải 9 bài toán. Hai thí sinh bất kỳ giải
được nhiều nhất một bài giống nhau. Tìm giá trị lớn nhất của k để mỗi bài toán đều có ít nhất k
thí sinh giải được.
Lời giải. Trước hết ta chuyển bài toán về ngôn ngữ tập hợp. Giả sử tập hợp các bài toán là
S D fx1 ; x2 ; : : : ; x9 g và Ai .1  i  11/ là tập hợp các bài toán giải bởi thí sinh thứ i: Theo
bài ra, ta có jAi \ Aj j  1 .1  i < j  9/: Gọi d.xi / là số tập Aj chứa xi , ta có
9
X 11
X
d.xi / D jAj j
i D1 j D1


11 X
X 11 X
jAi \ Aj j D d 2 .x/;
i D1 j D1 x2S
suy ra
11
X X X
jAi j C 2 jAi \ Aj j D d 2 .x/:
i D1 1i <j m x2S

Một cách tương đương, ta có

X 11
X X
2
d .x/ jAi j D 2 jAi \ Aj j;
x2S i D1 1i <j 11

hay X X X
d 2 .x/ d.x/ D 2 jAi \ Aj j:
x2S x2S 1i <j 11

Từ đó suy ra
X X
d 2 .x/ 2

d.x/ D 2 jAi \ Aj j  2C11 D 110:
x2S 1i <j 11

Với kết quả này, ta có 9.k 2 k/  110; do đó k  4:


 Với k D 4, giả sử tồn tại xi 2 S để d.xi /  5, ta có
9
X  2 
d .xj / d.xj /  8  12 C 20 D 116 > 110 (vô lý):
j D1

Do đó, ta có d.x1 / D d.x2 / D    D d.x9 / D 4: Suy ra


X
2
jAi \ Aj j D 54 D C11 1;
1i <j 11

do vậy tồn tại duy nhất cặp .i; j / để jAi \ Aj j D 0: Không mất tính tổng quát ta giả sử
cặp đó là .1; 2/, suy ra A1 \ A2 D ; và jAi \ Aj j D 1, 8.i; j / ¤ .1; 2/ và i < j .

Nếu tồn tại i 2 f3; 4; : : : ; 11g để jAi j  3 thì từ giả thiết jAi \ Aj j D ˘1 với mọi
j 2 f1; 2; : : : ; 11g n fig; ta thấy tồn tại một phần tử của Ai thuộc ít nhất 10
3
C1D4
150 Các phương pháp giải toán qua các kỳ thi Olympic

tập Aj .j ¤ i/, suy ra tồn tại một phần tử thuộc ít nhất 5 tập Ak (vô lý). Như vậy, ta có
jAi j  4; 8i 2 f3; 4; : : : ; 11g, suy ra
11
X
jAi j  36 C jA1 j C jA2 j > 36:
i D1

Tuy nhiên, điều này không thể xảy ra. Do đó k D 4 không thoả mãn.

 Với k D 3, ta chỉ ra trường hợp sau thoả mãn bằng cách lập bảng ô vuông 11  9 (11 hàng,
9 cột), trong đó nếu thí sinh Ai giải được bài xj thì ở ô vuông .i; j / ta điền số 1, ngược
lại ta điền số 0:

x1 x2 x3 x4 x5 x6 x7 x8 x9
A1 1 0 0 1 0 0 1 0 0
A2 1 0 0 0 1 0 0 0 1
A3 0 1 1 0 0 1 0 1 0
A4 0 1 0 1 1 0 0 0 0
A5 0 0 0 1 0 0 0 1 1
A6 0 0 1 0 0 0 1 0 1
A7 0 0 0 0 1 1 0 0 0
A8 1 1 0 0 0 0 0 0 0
A9 0 0 1 0 0 0 0 0 0
A10 0 0 0 0 0 1 1 0 0
A11 0 0 0 0 0 0 0 1 0

Vậy k D 3 là giá trị cần tìm.

Bài toán 5. Cho A1 ; A2 ; : : : Ak là các tập con của S D f1; 2; : : : ; 10g sao cho jAi j  5; với
mọi i D 1; 2; : : : k và jAi \ Aj j  2 với mọi 1  i < j  k. Tìm giá trị lớn nhất của k?

Lời giải. Giả sử F .S/ D fA1 ; A2 ; : : : ; Ak g: Xét bảng 10  k ô vuông (gồm 10 hàng, k cột)
sao cho tại mỗi ô vuông thứ .i; j / (hàng thứ i , cột thứ j ) ta đặt một số aij với aij D 1 nếu
xi 2 Aj và aij D 0 nếu xi … Aj : Khi đó:

k
X 10
X
d.i/ D aij ; 8i 2 f1; 2; : : : ; 10g; jAj j D aij ; 8j 2 f1; 2; : : : ; kg
j D1 i D1


10
X 10 X
X k k X
X 10 k
X ˇ ˇ
d.i/ D ai j D ai j D ˇAj ˇ  5k: .5:1/
i D1 i D1 j D1 j D1 i D1 j D1

Ta có
10 m
X X
2 1X  
jAi \ Aj j D Cd.j / D d.j / d.j / 1: .5:2/
i <j j D1
2 j D1
Phương pháp đếm bằng hai cách thông qua bảng ô vuông 151

Mặt khác, từ giả thiết bài toán ta có


X
jAi \ Aj j  2Ck2 D k.k 1/: .5:3/
i <j

Từ (5.2) và (5.3), suy ra


m
1X
d.j / .d.j / 1/  k.k 1/:
2 j D1
Một cách tương đương, ta có
2 3
m m
14 X X
d 2 .j / d.j /5  k.k 1/:
2 j D1 j D1

Tới đây, bằng cách sử dụng bất đẳng thức Cauchy-Schwarz, ta suy ra
2 0 12 3
m m
14 1 @ X X
d.j /A d.j /5  k.k 1/;
2 10 j D1 j D1

hay 2 32 3
m m
1 4 X X
d.j /5 4 d.j / 105  k.k 1/:
20 j D1 j D1

Kết hợp với (5.1), ta có


1
 5k  .5k 10/  k.k 1/ , k  6:
20
Với k D 6, ta chỉ ra được 6 tập con của S thoả mãn điều kiện bài toán là:
A1 D f1; 2; 3; 4; 5g; A2 D f1; 2; 6; 7; 8g; A3 D f1; 3; 6; 9; 10g;
A4 D f2; 4; 7; 9; 10g; A5 D f3; 5; 7; 8; 10g; A6 D f4; 5; 6; 8; 9g:

Bài toán 6 (IMO, 1998). Trong một cuộc thi có m thí sinh và n giám khảo .m; n 2 N ; n  3;
n lẻ/: Giả sử k 2 N là số sao cho hai giám khảo bất kỳ cho không quá k thí sinh đánh giá giống
nhau .giám khảo đánh giá thí sinh đạt hoặc không đạt/: Chứng minh rằng
k n 1
> :
m 2n
Lời giải. Xét bảng ô vuông m  n (m hàng, n cột) trong đó các hàng biểu diễn thí sinh và các
cột biểu diễn cho các giám khảo. Mỗi ô sẽ nhận giá trị 1 nếu giám khảo đánh giá thí sinh đạt và
nhận giá trị 0 nếu giám khảo đánh giá thí sinh trượt.
Gọi S là tập hợp các cặp số 0 hoặc 1 ở cùng một hàng. Do giả thiết hai giám khảo đánh giá giống
nhau ở nhiều nhất k thí sinh nên ở hai cột bất kỳ thì có không quá k cặp số 0 hoặc 1 thuộc S, do
đó ta có jS j  kCn2 D k n.n2 1/ : Mặt khác, xét một hàng nào đó trong bảng ô vuông, giả sử có p
số 0 và q số 1 thì số cặp số 0 hoặc số 1 trong hàng đó là:
p.p 1/ C q.q 1/
Cp2 C Cq2 D :
2
152 Các phương pháp giải toán qua các kỳ thi Olympic

Từ p C q D n và n lẻ nên ta có
p.p 1/ C q.q 1/ .p C q 1/2
 :
2 4
Thật vậy, bằng một số biến đổi, ta thấy bất đẳng thức này tương đương với .p q/2  1; hiển
nhiên đúng do p; q khác tính chẵn lẻ. Như vậy, ta đã chứng minh được
.n 1/2
Cp2 C Cq2  :
4
m.n 1/2
Do có m hàng nên ta có jS j  4
, suy ra

k n.n 1/ m.n 1/2 k n 1


 ,  :
2 4 m 2n
Đây chính là kết quả cần chứng minh.
Bài toán 7. Trong một kỳ thi Toán học có 8 thí sinh tham gia và có n câu hỏi .mỗi câu hỏi có 2
cách chọn là đúng hoặc sai). Tìm giá trị lớn nhất của n biết rằng với mỗi cặp hai câu hỏi bất kỳ
.A; B/ .ở đây cặp .A; B/ là một cặp có thứ tự/ có đúng hai thí sinh trả lời là .đúng, đúng/; hai
thí sinh trả lời là .đúng, sai/, hai thí sinh trả lời là .sai, đúng/ và hai thí sinh trả lời là .sai, sai/:
Lời giải. Giả sử 8 thí sinh tham dự kỳ thi là p1 ; p2 ; : : : ; p8 và n câu hỏi là A1 ; A2 ; : : : ; An :
Xét bảng 8  n ô vuông (gồm 8 hàng, n cột) sao cho tại mỗi ô vuông thứ .i; j / (hàng thứ i , cột
thứ j ), ta đặt một số aij với aij D 1 nếu với câu hỏi Aj , thí sinh pi trả lời đúng và aij D 0 nếu
với câu hỏi Aj , thí sinh pi trả lời sai. Khi đó, ta có
n
X 8
X
d.pi / D aij ; 8i 2 f1; 2; : : : ; 8g; jAj j D aij D 4; 8j 2 f1; 2; : : : ; ng
j D1 i D1


8
X 8 X
X n n X
X 8 n
X ˇ ˇ
d.pi / D ai j D ai j D ˇAj ˇ D 4n:
i D1 i D1 j D1 j D1 i D1 j D1

Từ giả thiết ta thấy, nếu trong một cột tất cả số 1 đổi thành số 0 và tất cả số 0 đổi thành số 1 thì
tính chất trong bảng vẫn không đổi. Do đó, không mất tính tổngPn quát, ta giả sử tất cả các số trong
ô vuông của hàng đầu tiên đều bằng 1, suy ra d.1/ D n và i D2 d.i/ D 3n.
Số các cặp số .1; 1/ trong mỗi hàng của bảng là:
8
X
2 2
Cd.i / D 2Cn D n.n 1/:
iD1

Mặt khác, sử dụng bất đẳng thức Cauchy-Schwarz, ta thấy


8 8
" 8 8
#
X
2 2
X
2 2 1 X 2 X
Cd.i / D Cd.1/ C Cd.i / D Cn C d .i/ d.i/
i D1 i D2
2 i D2 i D2
2
8
!2 8
3
n.n 1/ 1 4 1 X X
 C d.i/ d.i/5 :
2 2 7 i D2 iD2
Phương pháp đếm bằng hai cách thông qua bảng ô vuông 153

Do đó, kết hợp với trên, ta suy ra

8
2 !2 8
3
n.n 1/ 1 4 1 X X
C d.i/ d.i/5  n.n 1/;
2 2 7 i D2 i D2

hay
n.n 1/ 1
.3n/2

C 7  3n  n.n 1/ , n  7:
2 14
Ta chứng minh với n D 7 thoả mãn yêu cầu bài toán thông qua bảng sau:

A1 A2 A3 A4 A5 A6 A7
p1 1 1 1 1 1 1 1
p2 1 0 0 0 0 1 1
p3 1 0 0 1 1 0 0
p4 1 1 1 0 0 0 0
p5 0 1 0 1 0 1 0
p6 0 1 0 0 1 0 1
p7 0 0 1 1 0 0 1
p8 0 0 1 0 1 1 0

Vậy n D 7 là giá trị cần tìm.

Bài toán 8. Cho S là tập hợp gồm n phần tử và A1 ; A2 ; : : : ; Ak là các tập con của S có cỡ
trung bình nhỏ nhất bằng nr . Chứng minh rằng nếu k  2r 2 thì tồn tại i; j 2 f1; 2; : : : ; kg,
j ¤ i sao cho jAi \ Aj j  2rn2 . .Ở đây, cỡ trung bình của các tập A1 ; : : : ; Ak là k1 kiD1 jAi j:/
P

Lời giải. Ta có
k
X X kn
d.x/ D jAi j  :
x2S i D1
r

d 2 .x/ nhỏ nhất khi d.x/ D kr , suy ra


P
Do đó x2S

X k2n
d 2 .x/  : .8:1/
x2S
r2

Giả sử khẳng định bài toán là sai, khi đó:

X k X
X k X X
d 2 .x/ D jAi \ Aj j D jAi j C 2 jAi \ Aj j
x2S i D1 j D1 i 1i <j k
2
2r 2 nk 2
 
nk.k 1/ nk 1
< nk C D 2 1C  :
2r 2 2r k k r2

Điều này mâu thuẫn với (8.1) nên ta có điều phải chứng minh.
154 Các phương pháp giải toán qua các kỳ thi Olympic

Bài toán 9 (Định lý Erdos-Ko-Rado). Cho X là tập hợp gồm n phần tử và F .X/ là họ các tập
con gồm k phần tử của X , k  n2 sao cho với mọi tập hợp A; B 2 F .X/ thì A \ B ¤ ;, khi đó:
ˇF .X/ˇ  C k 1 :
ˇ ˇ
n 1

Lời giải. Không mất tính tổng quát, ta giả sử X D f0; 1; : : : ; n 1g và với mỗi s 2 X, đặt:

Bs D fs; s C 1; : : : ; s C k 1g (trong đó các số mod n):

Trong số các tập Bs , có không quá k tập nằm trong F .X/. Thật vậy, giả sử B0 2 F .X/, khi
đó có đúng 2k 2 tập khác là Bi .k 1/  i  k 1 có giao khác rỗng với B0 (ở đây
B j  Bn j ). Tuy nhiên các cặp Bi và Bi Ck lại rời nhau nên ngoài B0 ra, F .X/ chỉ chứa nhiều
nhất là k 1 tập.
Với mỗi hoán vị ı W f0; 1; : : : ; n 1g ! f0; 1; : : : ; n 1g; ta đặt:
˚
ı.Bs / D ı.s/; ı.s C 1/; : : : ; ı.s C k 1/ .mod n/:

Theo lý luận trên, ta thấy F .X/ chứa nhiều nhất k tập dạng ı.Bs / s 2 f0; 1; : : : ; n 1g : Đặt:
˚
L D .ı; s/ W ı.Bs / 2 F .X/ :

Do có nŠ hoán vị ı nên ta xét bảng .nŠ/  n ô vuông (gồm nŠ hàng, n cột) sao cho tại mỗi ô
vuông thứ .i; j / (hàng thứ i , cột thứ j ), ta đặt một số aij xác định bởi:
(
1 nếu ı.Bs / 2 F .X/
ai;j D
0 nếu ı.Bs / … F .X/

a1 0 a11 ::: a1.n 2/


a1.n 1/

a2 0 a21 ::: a2.n 2/


a2.n 1/
:: :: :: :: ::
: : : : :
a.nŠ 1/0 a.nŠ 1/1 ::: a.nŠ 1/.n 2/ a.nŠ 1/.n 1/
a.nŠ/0 a.nŠ/1 ::: a.nŠ/.n 2/ a.nŠ/.n 1/
Từ bảng, ta đếm số phần tử của L theo hai cách:
 Đếm theo cột: Với mỗi s và phần tử A 2 F .X/ cố ˇ số hoán vị ı mà ı.Bs / D A là
ˇ định,
kŠ  .n k/Š; do đó ta có jLj D n  kŠ  .n k/Š  ˇF .X/ˇ:

 Đếm theo hàng: Từ mỗi hàng của bảng có không quá k số 1 nên jLj  nŠ  k:
Như vậy, ta có
nŠ  k
D Cnk 11 :
ˇ ˇ ˇ ˇ
n  kŠ  .n k/Š  ˇF .X/ˇ  nŠ  k , ˇF .X/ˇ 
n  kŠ  .n k/Š
Sau đây là một số bài tập dành cho bạn đọc tự luyện tập:
Bài tập 31. Cho S là tập hợp gồm n phần tử và F .S/ D fA1 ; A2 ; : : : ; Ak g là họ các tập con
của S có cỡ trung bình nhỏ nhất bằng nr . Chứng minh rằng nếu k  2˛r ˛ thì tồn tại các tập
Ai1 ; Ai2 ; : : : ; Ai˛ 2 F .S/ sao cho jAi1 \ Ai2 \    \ Ai˛ j  2rn˛ .
Phương pháp đếm bằng hai cách thông qua bảng ô vuông 155

Bài tập 32. Cho tập S gồm n phần tử và n C 1 tập con M1 ; M2 ; : : : ; Mn  S: Chứng minh
rằng tồn tại r; s  1 và các tập chỉ số rời nhau fi1 ; i2 ; : : : ; ir g; fj1 ; j2 ; : : : ; js g sao cho:
r
[ s
[
Mik D Mjk :
kD1 kD1

Bài tập 33. Cho F .S/ là họ các tập con khác rỗng của tập S gồm n phần tử thỏa mãn:
ˇ ˇ
i) ˇF .S /ˇ  2 và jAj  2; 8A 2 F .S/:

ii) Với hai phần tử bất kỳ x; y 2 S; tồn tại duy nhất A 2 F .S/ sao cho x; y 2 A:
ˇ ˇ
Chứng minh rằng ˇF .S/ˇ  n:

Bài tập 34. Cho tập S là tập hữu hạn và F .S/ D fA1 ; A2 ; : : : ; An g là họ các tập con của S
sao cho mỗi phần tử của S đều thuộc ít nhất t tập con của F .S/. Chứng minh rằng
n
X
jAi j  tjSj:
i D1

Bài tập 35. Cho n; k là các số nguyên dương thỏa mãn n > k 2 k C 1 và fA1 ; A2 ; : : : ; An g
là họ các tập hợp thỏa mãn đồng thời hai điều kiện:

i) jAi j D k; 8i 2 f1; 2; : : : ; ng:

ii) jAi [ Aj j D 2k 1; 8i; j 2 f1; 2; : : : ; ng; i ¤ j:

Tính số phần tử của tập hợp niD1 Ai .


S

Sn 36 (Định lý Corrádi, 1969). Cho A1 ; A2 ; : : : ; An là các tập hợp gồm r phần tử và


Bài tập
S D i D1 Ai . Chứng minh rằng nếu jAi \ Aj j  k; 8i; j 2 f1; 2; : : : ; ng; i ¤ j thì
nr
jS j  :
r C .n 1/k

Tài liệu tham khảo


[1] A. Engel, Problem-Solving Strategies, Springer, 1997.

[2] S. Jukna, Extremal Combinatorics, Springer, 2011.

[3] P.S. Bravo, Problem-Solving Methods in Combinatorics, Birkhauser, 2013.

[4] T. Andreescu, D. Andrica, Number theory-Structures: Examples and Problems, Birkhauser, 2009.

[5] N. Alon, Problems and Results in Extremal Combinatorics, lecture notes.


156 Các phương pháp giải toán qua các kỳ thi Olympic
PHƯƠNG PHÁP QUỸ ĐẠO
Trần Ngọc Thắng
(THPT Chuyên Vĩnh Phúc)

1. Lý thuyết
1.1. Đường đi ngắn nhất
Định nghĩa 1. Cho hai số nguyên dương m; n: Xét mạng lưới ô vuông kích thước n  m và
điểm A.m; n/: Khi đó mỗi đường gấp khúc nối hai điểm O và A gồm m bước đi ngang và n
bước đi lên được gọi là một đường đi ngắn nhất từ điểm O đến điểm A:

Định lý 1. Cho hai số nguyên dương m; n: Khi đó số đường đi ngắn nhất nối điểm O.0; 0/ với
n
điểm A.m; n/ bằng CmCn :

Chứng minh. Mỗi đường đi ngắn nhất từ điểm O đến điểm A tương ứng với bộ .x1 ; : : : ; xmCn /;
trong đó xi 2 f0; 1g và có đúng n số xi bằng 1; xi D 1 nếu đi ngang ở bước thứ i; xi D 1 nếu
đi lên ở bước thứ i: Khi đó mỗi đường đi ngắn nhất từ điểm O đến điểm A tương ứng với một tổ
n
hợp chập n của m C n suy ra số đường đi ngắn nhất là CmCn :

Định lý 2. Cho bốn số nguyên a; b; x; y và x > a: Khi đó số đường đi ngắn nhất nối điểm
x a
A .a; b/ với điểm B .x; y/ bằng CxCy a b:

!
Chứng minh. Xét phép tịnh tiến hệ tọa độ Oxy thành hệ trục AXY theo vector OA D .a; b/ :
Khi đó trong hệ trục tọa độ AXY thì A .0; 0/ ; M .x a; y b/ : Khi đó theo định lý 1 số
x a
đường đi ngắn nhất nối điểm A với điểm M bằng CxCy a b:

1.2. Quỹ đạo


Định nghĩa 2. Cho a; b; m; n là các số nguyên và m > a: Trong mặt phẳng tọa độ vuông góc
Oxy; ta gọi quỹ đạo nối điểm A.a; b/ với điểm B.m; n/ là đường gấp khúc nối các điểm

A.a; b/; A1 .a C 1; y1 / ; A2 .a C 2; y2 / ; : : : ; Am a .m; ym a / ;

trong đó yi yi 1 2 f 1; 1g ; ym a D n:

Định lý 3. Kí hiệu S.m; n/ (với m; n là các số nguyên dương) là số quỹ đạo nối điểm O.0; 0/
mCn
với điểm A.m; n/: Khi đó S .m; n/ D Cm 2 nếu m; n cùng tính chẵn lẻ và S.m; n/ D 0 nếu
m; n khác tính chẵn lẻ.

Chứng minh 1. Theo định nghĩa của quỹ đạo thì m C n chẵn suy ra S.m; n/ D 0 nếu
m; n khác tính chẵn lẻ. Ta xét trường hợp m; n cùng tính chẵn lẻ. Bây giờ ta chứng minh
mCn
S.m; n/ D Cm 2 quy nạp theo m:

157
158 Các phương pháp giải toán qua các kỳ thi Olympic

Để đến được điểm Am .m; n/ thì Am 1 D .m 1; n 1/ hoặc Am 1 D .m 1; n C 1/; kết hợp


với giả thiết quy nạp ta được
m 1Cn 1 mCn
2
S .m; n/ D S .m 1; n 1/ C S .m 1; n C 1/ D Cm 1 C Cm 21
mCn mCn mCn
2 1 2 2
D Cm 1 C Cm 1 D Cm :
mCn
Do đó S .m; n/ D Cm 2 nếu m; n cùng tính chẵn lẻ và bằng 0 nếu m; n khác tính chẵn lẻ.
Chứng minh 2. Giả sử quỹ đạo gồm p đoạn hướng lên và q đoạn hướng xuống. Khi đó ta có
p D m C n; q D m n suy ra m D pCq2
; n D p 2 q : Từ đó suy ra được đính lý.
Định lý 4. Cho x; y; a; b là các số nguyên và x > a: Khi đó số quỹ đạo nối điểm A.a; b/ với
xCy a b
2
điểm M.x; y/ bằng Cx a :
!
Chứng minh. Xét phép tịnh tiến hệ tọa độ Oxy thành hệ trục AXY theo vector OA D .a; b/ :
Khi đó trong hệ trục tọa độ AXY thì A .0; 0/ ; M .x a; y b/ : Khi đó số quỹ đạo nối điểm
xCy a b
2
A với điểm M bằng Cx a :
0
Định lý 5. Cho A; B là hai điểm bất kì nằm phía trên trục hoành. Lấy điểm A đối xứng với
điểm A qua trục hoành. Khi đó số quỹ đạo nối điểm A với điểm B và có điểm chung với trục
0
hoành bằng số quỹ đạo nối điểm A với điểm B:
Chứng minh. Giả sử quỹ đạo nối điểm A với điểm B có điểm chung lần đầu với trục hoành là
điểm M .u; 0/ ; A .a; b/ : Theo định lý 4 ta được:
u a b
2
Số quỹ đạo nối điểm A .a; b/ với điểm M .u; 0/ bằng Cu a :
0
Số quỹ đạo nối điểm A .a; b/ với điểm M .u; 0/ bằng
u aCb u aCb u a b
2 u a 2 2
Cu a D Cu a D Cu a :
Do đó số quỹ đạo nối điểm A .a; b/ với điểm M .u; 0/ bằng số quỹ đạo nối điểm A0 .a; b/
với điểm M .u; 0/ suy ra số quỹ đạo nối điểm A với điểm B và có điểm chung với trục hoành
bằng số quỹ đạo nối điểm A0 với điểm B:

2. Bài tập áp dụng


Bài toán 1. Cho số nguyên dương n > 1 và số tự nhiên k; với 0  k  n: Chứng minh rằng
a) Cnk D Cnn k
:
b) Cnk D Cnk 1 C Cnk 1
1 với k > 1:
Lời giải. Đây là các tính chất cơ bản của Cnk và có thể chứng minh theo nhiều cách khác nhau,
sau đây chúng ta sẽ sử dụng đường đi ngắn nhất để chứng minh các đẳng thức này.
a) Xét đường đi ngắn nhất nối điểm O.0; 0/ với điểm A.n k; k/ gồm n k bước đi ngang
và k bước đi lên. Khi đó theo định lý 1 ta có số đường đi ngắn nhất nối O.0; 0/ với điểm
A.n k; k/ bằng Cnk kCk D Cnk : Mặt khác mỗi đường đi ngắn nhất nối điểm O.0; 0/ với
điểm A.n k; k/ tương ứng với một cách chọn ra n k bước đi ngang từ n bước đi suy ra
số đường đi nối O.0; 0/ với điểm A.n k; k/ bằng Cnn k : Do đó Cnn k D Cnk :
Phương pháp quỹ đạo 159

b) Xét đường đi ngắn nhất nối hai điểm O.0; 0/ với điểm A.n k; k/: Khi đó theo định lý 1
ta có số đường đi ngắn nhất nối O.0; 0/ với điểm A.n k; k/ bằng Cnk kCk D Cnk :

Xét mỗi đường đi ngắn nhất từ điểm O.0; 0/ với điểm A.n k; k/: Trước khi đến điểm
A.n k; k/ ta phải đến được điểm A1 .n k; k 1/ hoặc A2 .n k 1; k/: Khi đó số
đường đi ngắn nhất nối O.0; 0/ với điểm A.n k; k/ bằng số đường đi ngắn nhất nối
O.0; 0/ với điểm A1 .n k; k 1/ cộng với số đường đi ngắn nhất nối O.0; 0/ với điểm
A2 .n k 1; k/ cộng với số. Khi đó theo định lý 1 ta được

Cnk D Cnk 1
kCk 1 C Cnk k 1Ck D Cnk 1
1 C Cnk 1 :

Bài toán được chứng minh.

Bài toán 2. Cho n là một số nguyên dương. Chứng minh rằng


2 2
.Cn0 / C .Cn1 / C    C .Cnn /2 D C2n
n
:

Lời giải. Xét đường đi ngắn nhất từ điểm O.0; 0/ với điểm A.n; n/: Khi đó theo định lý 1 thì
n n
số đường đi ngắn nhất từ điểm O.0; 0/ với điểm A.n; n/ bằng CnCn D C2n :
Tiếp theo ta xét hình vuông OBAC; trong đó B.0; n/; C.n; 0/: Khi đó mỗi đường đi ngắn nhất
từ điểm O.0; 0/ với điểm A.n; n/ phải đi qua một điểm M.k; n k/ với 0  k  n nằm trên
đường chéo BC: Như vậy mỗi đường đi ngắn nhất từ điểm O.0; 0/ với điểm A.n; n/ gồm đường
đi ngắn nhất từ điểm O.0; 0/ với điểm M.k; n k/ và đường đi ngắn nhất từ điểm M.k; n k/
với điểm A.n; n/: Theo định lý 2 ta được số đường đi ngắn nhất từ điểm O.0; 0/ với điểm
k k
M.k; n k/ bằng CkCn k D Cn và số đường đi ngắn nhất từ điểm M.k; n k/ với điểm
A.n; n/ bằng
n k n k
CnCn .kCn k/ D Cn D Cnk ;
nên số đường đi ngắn nhất từ điểm O.0; 0/ với điểm A.n; n/ và đi qua điểm M.k; n k/ là
2
.Cnk / : Do đó số đường đi ngắn nhất từ điểm O.0; 0/ với điểm A.n; n/ bằng
2 2
.Cn0 / C .Cn1 / C    C .Cnn /2 :

Từ đó ta được đẳng thức sau


2 2
.Cn0 / C .Cn1 / C    C .Cnn /2 D C2n
n
:

Bài toán được chứng minh.

Bài toán 3. Cho n > m > 1 là các số nguyên dương. Chứng minh đẳng thức sau

Cnm D Cnm 11 C Cnm 21 C    C Cmm 11 :

Lời giải. Xét đường đi ngắn nhất từ điểm O.0; 0/ với điểm A.m; n m/: Khi đó theo định lý
m m
1 thì số đường đi ngắn nhất từ điểm O.0; 0/ với điểm A.m; n m/ bằng CmCn m D Cn :

Mỗi đường đi ngắn nhất từ điểm O.0; 0/ với điểm A.m; n m/ và đi qua điểm P . 12 ; k/ với
0  k  n: Khi đó đường đi ngắn nhất này gồm một đường đi ngắn nhất từ điểm O.0; 0/ đến
điểm Q.1; k/ và một đường đi ngắn nhất từ điểm Q.1; k/ đến điểm A.m; n m/: Do phải đi
160 Các phương pháp giải toán qua các kỳ thi Olympic

qua điểm P . 21 ; k/ nên chỉ có duy nhất một đường đi từ điểm O.0; 0/ đến điểm Q.1; k/; và theo
định lý 2 số đường đi ngắn nhất từ điểm Q.1; k/ đến điểm A.m; n m/ bằng
m 1
CmCn m .kC1/ D Cnm k1 1 :

Do đó số đường đi ngắn nhất từ điểm O.0; 0/ với điểm A.m; n m/ và đi qua điểm P . 12 ; k/
bằng Cnm k1 1 : Từ đó suy ra số đường đi từ điểm O.0; 0/ với điểm A.m; n m/ bằng
n
X m
Cnm k1 1 :
kD0

Vậy ta có đẳng thức


Cnm D Cnm 11 C Cnm 21 C    C Cmm 11 :
Chứng minh hoàn tất.
Bài toán 4 (Việt Nam TST, 2003). Cho các số nguyên dương m; n; p; q với p < m; q < n:
Trên mặt phẳng tọa độ lấy bốn điểm A.0; 0/; B.p; 0/; C.m; q/; D.m; n/: Xét các đường đi
ngắn nhất f ngắn nhất từ điểm A đến điểm D và các đường đi ngắn nhất g từ điểm B đến điểm
C: Gọi S là số cặp đường đi .f; g/ mà f và g không có điểm chung. Chứng minh rằng
n q q n
S D CmCn  CmCq p CmCq  CmCn p:

n
Lời giải. Theo định lý 1 thì số đường đi ngắn nhất từ điểm A đến điểm D là CmCn và theo định
q
lí 2 thì số đường đi ngắn nhất từ điểm B đến điểm C là CmCq p :
Suy ra số cặp đường đi .f; g/ bằng
n q
CmCn  CmCq p: .1/

Tiếp theo ta đếm số cặp .f; g/ mà f và g cắt nhau. Giả sử f và g cắt nhau tại điểm đầu tiên là
E (điểm có hoành độ nhỏ nhất trong tất cả các giao điểm của nó). Khi đó ta có một song ánh từ
tập các cặp đường đi .f; g/ với cặp các đường đi ngắn nhất từ điểm A đến điểm C và từ điểm
q
B đến điểm D: Theo định lý 1 thì số đường đi ngắn nhất từ điểm A đến điểm C bằng CmCq và
n
theo định lý 2 thì số đường đi ngắn nhất từ điểm B đến điểm D bằng CmCn p : Suy ra số cặp các
đường đi ngắn nhất từ điểm A đến điểm C và từ điểm B đến điểm D bằng
q n
CmCq  CmCn p: .2/

Từ .1/ và .2/ ta được


n q q n
S D CmCn  CmCq p CmCq  CmCn p:

Bài toán được chứng minh.


Bài toán 5. Cho a; b; m; n; p là các số nguyên. Tìm số quỹ đạo nối điểm A.a; b/ với điểm
M.x; y/ sao cho quỹ đạo đó không cắt đường thẳng y D p ‹
Lời giải. Nếu A và M nằm khác phía so với đường thẳng y D p thì số quỹ đạo bằng 0:
Nếu A và M nằm cùng phía so với đường thẳng y D p: Lấy điểm A0 đối xứng với điểm A qua
đường thẳng y D p suy ra A0 .a; 2p b/: Khi đó theo định lí 1:2:4 số quỹ đạo nối điểm A với
điểm M bằng số quỹ đạo nối điểm A0 với điểm M:
Phương pháp quỹ đạo 161

xCy a b
2
Theo định lý 4 thì số quỹ đạo nối điểm A với điểm M bằng Cx a và số quỹ đạo nối điểm
xCy a 2pCb
0 2
A với điểm M bằng Cx a :
Do đó số quỹ đạo nối điểm A.a; b/ với điểm M.x; y/ sao cho quỹ đạo đó không cắt đường
thẳng y D p bằng
xCy a b xCy a 2pCb
2 2
Cx a Cx a :
Nếu A nằm trên đường thẳng y D p thì điểm A sẽ được nối với điểm A1 .a C 1; p C 1/ hoặc
A2 .a C 1; p 1/: Khi đó số quỹ đạo nối điểm A.a; b/ với điểm M.x; y/ mà có điểm chung
với đường thẳng y D p; không tính điểm chung A gồm các quỹ đạo nối A1 .a C 1; p C 1/ với
điểm M.x; y/ và A2 .a C 1; p 1/ với điểm M.x; y/: Do đó số quỹ đạo loại này là
xCy a p xCy a p xCy a p
2 2 2
Cx a 1 C Cx a 1 D 2Cx a 1 :
Do đó số quỹ đạo nối điểm A.a; b/ với điểm M.x; y/ mà không có điểm chung với đường thẳng
y D p ngoài điểm A bằng
xCy a p xCy a p
2 2
Cx a 2Cx a 1 :
Lời giải hoàn tất.
Bài toán 6. Cho hai số nguyên dương m; n: Tìm số quỹ đạo nối điểm O.0; 0/ với điểm
M.m; n/ sao cho quỹ đạo đó không có điểm chung với trục hoành.
Lời giải. Điểm O.0; 0/ chỉ nối với điểm A1 .1; 1/ hoặc điểm A2 .1; 1/: Khi đó số quỹ đạo nối
điểm O.0; 0/ với điểm M.m; n/ cắt trục hoành bằng hai lần số quỹ đạo nối A2 .1; 1/ với điểm
xCyC1 1 xCy
2 2
M.m; n/ và bằng 2Cx 1 D 2Cx 1 :
Do đó số quỹ đạo nối điểm O.0; 0/ với điểm M.m; n/ sao cho quỹ đạo đó không có điểm chung
xCy xCy
2 2
với trục hoành bằng Cx 2Cx 1 : Mặt khác
xCy
2
xCy
2
xCy
2
.x 1/Š
Cx 2Cx 1 D Cx 2
. xCy
2
/Š  .x 1 xCy
2

xCy
xCy
2
xCy
2
2.x 2
/
D Cx Cx 
x
y xCy
D  Cx 2 :
x
Vậy số quỹ đạo nối điểm O.0; 0/ với điểm M.m; n/ và không có điểm chung với trục hoành
xCy
y 2
bằng x
 Cx : Lời giải hoàn tất.
Bài toán 7. Cho m; n là các số nguyên dương và m  n: Có m C n người sắp hàng mua vé,
trong đó có n người mang loại tiền 5000 đồng và m người mang tiền loại 10000 đồng. Mỗi vé
giá 5000 đồng. Trước lúc bán người bán vé có p đồng tiền 5000 đồng .p là một số tự nhiên).
Hỏi có bao nhiêu cách sắp xếp m C n người để không có người nào phải chờ trả tiền thừa?
Lời giải. Giả sử m C n người mua vé đã được sắp xếp hàng theo một cách nào đó. Đặt "i D 1
nếu người thứ i có 5000 đồng và "i D 1 nếu người thứ i có 10000 đồng. Khi đó nếu có k sắp
Pk
hàng đã mua vé thì số tiền của người bán vé bằng Sk D p C "i suy ra sau khi bán vé cho
i D1
người thứ k, để không phải trả tiền thì Sk  0:
162 Các phương pháp giải toán qua các kỳ thi Olympic

Trên mạng lưới ô vuông, vẽ các điểm Ak .k; Sk /; 1  k  m C n; xét quỹ đạo nối điểm
O.0; p/ với điểm AmCn .m C n; n m C p/: Khi đó mỗi cách sắp xếp m C n người để không
có người nào phải chờ trả tiền thừa tương ứng với một quỹ đạo nối điểm O.0; p/ với điểm
AmCn .m C n; n m C p/ và quỹ đạo này không có điểm chung với đường thẳng y D 1: Do
đó theo bài 4 ta được số quỹ đạo này là
2nCp p 2nCp 2. 1/Cp
n nCpC1
CmCn2 CmCn 2
D CmCn CmCn :

n nCpC1
Cách sắp xếp m C n người để không có người nào phải chờ trả tiền thừa là CmCn CmCn :

Bài toán 8 (Bài toán bầu c). Cho m; n là các số nguyên dương và m > n: Trong một cuộc
bầu cử, ứng cử viên A được m phiếu bầu, ứng cử viên B được n phiếu bầu. Cử tri bỏ phiếu liên
tiếp. Hỏi có bao nhiêu cách bỏ phiếu để ứng cử viên A luôn dẫn đầu về số phiếu bầu cho mình.

Lời giải. Giả sử a C b người tham gia bầu cử và đã được sắp xếp hàng theo một cách nào đó.
Đặt "i D 1 nếu người thứ i bầu phiếu cho ứng viên A và "i D 1 nếu người thứ i bầu phiếu cho
ứng viên B: Khi đó nếu có k đã tham gia bầu cử thì hiệu số phiếu của ứng viên A với số phiếu
của ứng viên B bằng Sk D kiD1 "i suy ra sau khi k người tham gia bầu cử, để ứng viên A luôn
P
dẫn đầu về số phiếu thì Sk > 0:
Trên mạng lưới ô vuông, vẽ các điểm Ak .k; Sk /; 1  k  a C b; xét quỹ đạo nối điểm A1 .1; 1/
với điểm AaCb .a C b; a b/: Khi đó mỗi cách bầu cử của a C b người để ứng viên A luôn dẫn
đầu về số phiếu tương ứng với một quỹ đạo nối điểm A1 .1; 1/ với điểm AaCb .a C b; a b/ và
quỹ đạo này không có điểm chung với đường thẳng y D 0: Do đó theo bài 4 số quỹ đạo này là
2a 2
2
2a 1 2.0/C1
2 a 1 a a b a
CaCb CaCb D CaCb 1 CaCb 1 D C :
1 1
a C b aCb
a b a
Số cách bỏ phiếu để ứng cử viên A luôn dẫn đầu về số phiếu bầu cho mình là C :
aCb aCb

Bài toán 9. Cho số nguyên dương n > 1: Tìm số dãy .x1 ; x2 ; : : : ; x2n 2 / thỏa mãn đồng thời
các điều kiện sau đây

a) xi 2 f 1; 1g ; 8i D 1; 2; : : : ; 2n 2:

b) x1 C x2 C    C xk  0; 8k D 1; 2; : : : ; 2n 2:

c) x1 C x2 C    C x2n 2 D 0:

Lời giải. Ta sẽ giải bài toán tổng quát hơn như sau:
Bổ đề 1. Cho hai số nguyên dương m; n và m  n > 1: Tìm số dãy .x1 ; x2 ; : : : ; xmCn / thỏa
mãn đồng thời các điều kiện sau đây:

a) xi 2 f 1; 1g ; 8i D 1; 2; : : : ; m C n:

b) x1 C x2 C    C xk  0; 8k D 1; 2; : : : ; m C n:

c) Dãy .x1 ; x2 ; : : : ; xmCn / có m số hạng bằng 1 và n số hạng bằng 1:


Phương pháp quỹ đạo 163

Trên mạng lưới ô vuông, vẽ các điểm Ak .k; Sk /; 1  k  m C n, xét quỹ đạo nối điểm O.0; 0/
với điểm AmCn .m C n; m n/: Khi đó mỗi dãy .x1 ; x2 ; : : : ; xmCn / thỏa mãn yêu cầu bài toán
tương ứng với một quỹ đạo nối điểm O.0; 0/ với điểm AmCn .m C n; m n/ và quỹ đạo này
không có điểm chung với đường thẳng y D 1: Do đó theo bài 4 ta được số quỹ đạo này là
2nC0 0 2n 2. 1/C0
n nC1
CmCn2 CmCn 2 D CmCn CmCn :
n nC1
Như vậy số dãy .x1 ; x2 ; : : : ; xmCn / thỏa mãn đồng thời yêu cầu bài toán bằng CmCn CmCn :
Bổ đề được chứng minh.
Trở lại bài toán, áp dụng bổ đề khi thay m; n bởi n 1 ta được số dãy bằng

n 1 n .2n 2/Š .2n 2/Š


C2n 2 C2n 2 D
.n 1/Š.n 1/Š nŠ.n 2/Š
.2n 2/Š .2n 2/Š.n 1/
D
.n 1/Š.n 1/Š n.n 1/Š.n 1/Š
 
.2n 2/Š n 1
D 1
.n 1/Š.n 1/Š n
1 n 1
D C2n 2:
n
Lời giải hoàn tất.

Bài toán 10. Cho số nguyên dương n > 1: Tìm số dãy số .a1 ; a2 ; : : : ; a2nC1 / thỏa mãn đồng
thời các điều kiện sau đây:

a) ai  0; 8i D 1; 2; : : : ; 2n C 1:

b) a1 D a2nC1 D 0:

c) jai ai C1 j D 1; 8i D 1; 2; : : : ; 2n:

Lời giải. Đặt xi D ai C1 ai ; kết hợp với các điều kiện trên ta được dãy số .x1 ; x2 ; : : : ; x2n /
thỏa mãn các điều kiện sau:

i) xi 2 f 1; 1g ; 8i D 1; 2; : : : ; 2n:

ii) x1 C x2 C : : : C xk  0; 8k D 1; 2; : : : ; 2n:

iii) x1 C x2 C : : : C x2n D 0:

Dễ thấy mỗi dãy .a1 ; a2 ; : : : ; a2nC1 / tương ứng với một dãy .x1 ; x2 ; : : : ; x2n / và ngược lại.
1 n
Do đó theo kết quả bài 8 ta được số dãy bằng nC1 C2n :
1
Bài toán 11. Kí hiệu Bn D Cn :
nC1 2n
n
Chứng minh rằng trong số C2n quỹ đạo nối điểm O.0; 0/
với điểm A.2n; 0/ có

a) Bn 1 quỹ đạo nằm trên trục hoành và không có điểm chung với trục hoành, trừ các điểm
O và A:

b) Bn quỹ đạo không có đỉnh nằm dưới trục hoành.


164 Các phương pháp giải toán qua các kỳ thi Olympic

Lời giải. a) Mỗi quỹ đạo nằm trên trục hoành và không có điểm chung với trục hoành, trừ các
điểm O và A là quỹ đạo nối điểm A1 .1; 1/ với điểm A2n 1 .2n 1; 1/ và không có điểm chung
với đường thẳng y D 0: Do đó theo bài 4; số quỹ đạo cần tìm bằng
2n 2 2n 1 2.0/C1
n 1 n 1 n 1
C2n2 2 C2n 2 2 D C2n 2 C2n 2 D C D Bn 1 :
n 2n 2
b) Số quỹ đạo nối điểm O.0; 0/ với điểm A.2n; 0/ và không có đỉnh nằm dưới trục hoành
bằng số quỹ đạo nối điểm O.0; 0/ với điểm A.2n; 0/ và không có điểm chung với đường thẳng
y D 1: Do đó theo bài 4 số quỹ đạo bằng
2n 2n 2. 1/C0
n nC1 1 n
C2n2 C2n 2 D C2n C2n D C2n D Bn :
nC1
Bài toán được chứng minh.
Bài toán 12. Cho số nguyên dương n > 1: Tìm số hàm không giảm
f W f1; 2; : : : ; ng ! f1; 2; : : : ; ng
thỏa mãn điều kiện f .x/  x với mọi x 2 f1; 2; : : : ; ng ‹
Lời giải. Với mỗi hàm không giảm
f W f1; 2; : : : ; ng ! f1; 2; : : : ; ng ;
trong mặt phẳng tọa độ vuông góc Oxy; ta xây dựng một đường đi ngắn nhất nối điểm O.0; 0/
với điểm A.n; n/ như sau:
Với mỗi i 2 f1; 2; : : : ; ng ; điểm M.i; f .i// nối với điểm M1 .i C 1; f .i// bởi một đoạn nằm
ngang. Nếu f .i C 1/ > f .i/ thì điểm M1 .i C 1; f .i// nối với điểm M2 .i C 1; f .i C 1// bằng
một đoạn thẳng đứng. Nếu f .i C 1/ D f .i/ thì điểm M1 .i C 1; f .i//  M2 .i C 1; f .i C 1//:
Cho i D 1; 2; : : : ; n 1 ta được một đường đi ngắn nhất nối điểm O.0; 0/ với điểm .n; f .n//:
Điểm O.0; 0/ nối với điểm .1; 0/ bởi một đoạn nằm ngang, và nếu f .n/ < n; ta nối điểm
.n; f .n// với điểm A.n; n/ bởi một dãy các đoạn thẳng đứng liên tiếp. Khi đó ta được một
đường đi ngắn nhất nối điểm O.0; 0/ với điểm A.n; n/ và đường đi ngắn nhất gồm n đoạn thẳng
đứng và n đoạn nằm ngang.
Bây giờ ta xét tập X các hàm không giảm f W f1; 2; : : : ; ng ! f1; 2; : : : ; ng thỏa mãn điều kiện
f .x/  x với mọi x 2 f1; 2; : : : ; ng với tập Y các đường đi ngắn nhất nối điểm O.0; 0/ với
điểm A.n; n/ thỏa mãn tính chất trên. Ta chứng minh ánh xạ g W X ! Y là một song ánh. Dễ
thấy g là một đơn anh, tiếp theo ta chứng minh g là một toàn ánh, thật vậy: giả sử d là một đường
đi ngắn nhất nối điểm O.0; 0/ với điểm A.n; n/ thỏa mãn tính chất đã xây dựng. Ta xây dựng
hàm không giảm như sau:
fd .i/ D max fj j.i; j / 2 d g ; i D 1; 2; : : : ; n:
Ta đếm số đường đi ngắn nhất độ dài 2n nối điểm O.0; 0/ với điểm A.n; n/ và không có đỉnh
nằm phía trên đường thẳng y D x: Theo lí luận ở trên số đường đi ngắn nhất như vậy bằng số
dãy .x1 ; x2 ; : : : ; x2n / thỏa mãn đồng thời các tính chất:
a) xi 2 f 1; 1g ; 8i D 1; 2; : : : ; 2n:
b) x1 C x2 C    C xk  0; 8k D 1; 2; : : : ; 2n:
c) x1 C x2 C    C x2n D 0:
1 1
Theo bài 8 số dãy thỏa mãn là Cn :
nC1 2n
Vậy số hàm thỏa mãn yêu cầu bài toán là Cn :
nC1 2n
Phương pháp quỹ đạo 165

3. Bài tập rèn luyện


Bài tập 37. Cho m; n; k là các số nguyên dương. Chứng minh các đẳng thức sau bằng cách sử
dụng phương pháp xây dựng đường đi ngắn nhất.

a) Cn0  Cmk C Cn1  Cmk 1


C    C Cnk  Cm0 D CmCn
k
; .k  m; k  n/:

b) Cnm  Ck0 C Cnm 11  CkC1


1
C    C Cm0 n
m
 CkCm m
D CnCkC1 ; .n < m/:

Bài tập 38. Kí hiệu B.n; k/ là số quỹ đạo nối điểm O.0; 0/ với điểm M.2n; 0/ có 2k cạnh
nằm trên trục hoành và 2n 2k cạnh còn lại nằm dưới trục hoành. Chứng minh các đẳng thức
sau:
1
a) B.n; k/ D Cn :
nC1 2n

1
b) B0 Bn 1 C B1 Bn 2 C    C Bn 1 B0 D B.n; k/; trong đó Bn D Cn :
nC1 2n

Bài tập 39. Một tập n 2 đường chéo của một đa giác lồi A1 A2    AnC1 được gọi là tập có
tính chất C nếu hai đường chéo bất kì của nó hoặc không có điểm chung hoặc nếu có điểm chung
thì điểm chung đó là đỉnh của đa giác và n 2 đường chéo này chia đa giác thành n 1 tam
giác. Tính số các tập n 2 đường chéo có tính chất C:

Bài tập 40. Cho m; n là các số nguyên thỏa mãn 1  n < m: Chứng minh rằng số các đường
đi ngắn nhất từ điểm .1; 0/ đến điểm .m; n/ sao cho đường đi ngắn nhất này cắt đường thẳng
y D x tại ít nhất một điểm bằng số đường đi ngắn nhất từ điểm .0; 1/ đến điểm .m; n/:

Bài tập 41. Cho m; n là các số nguyên thỏa mãn 1  n < m: Chứng minh rằng số đường đi
ngắn nhất từ điểm .0; 0/ đến điểm .m; n/ cắt đường thẳng y D x chỉ tại duy nhất điểm .0; 0/
m n n
bằng mCn CmCn :

Bài tập 42. Cho n > 1 là số nguyên. Tìm số đường đi ngắn nhất nối điểm .0; 0/ với điểm
.n; n/ và không có đỉnh nào nằm phía trên đường thẳng y D x:

Bài tập 43. Cho n > 1 là số nguyên. Tìm số đường đi ngắn nhất nối điểm .0; 0/ với điểm
.2n; 2n/ và không đi qua .2i 1; 2i 1/ với 1 6 n:

Bài tập 44. Cho n > 1 là số nguyên. Tìm số dãy nguyên .a1 ; a2 ; : : : ; an / thỏa mãn điều kiện
1 6 a1 6 a2 6    6 an với ai 6 i; i D 1; 2; 3; : : : ; n:

Bài tập 45. Cho n > 1 là số nguyên. Tìm số dãy nguyên .a1 ; a2 ; : : : ; an / thỏa mãn điều kiện
ai D 0 và 0 6 ai C1 6 ai ; C1 i D 1; 2; 3; : : : ; n 1:

Bài tập 46. Cho n > 1 là số nguyên. Tìm số dãy nguyên .a1 ; a2 ; : : : ; an / thỏa mãn điều kiện
ai 6 1 và a1 C a2 C    C an > 0; i D 1; 2; 3; : : : ; n 1:

Bài tập 47. Cho n > 1 là số nguyên. Tìm số dãy nguyên .a1 ; a2 ; : : : ; an / thỏa mãn điều kiện
a1 < a2 <    an và 1 6 ai 6 2i; i D 1; 2; 3; : : : ; n:

Bài tập 48. Cho n > 1 là số nguyên. Tìm số dãy nguyên .a1 ; a2 ; : : : ; an / thỏa mãn điều kiện
ai > 1; a1 C a2 C    C ak > 0; và a1 C a2 C    C an D 0; i D 1; 2; 3; : : : ; n 1:
166 Các phương pháp giải toán qua các kỳ thi Olympic

Bài tập 49. Cho n > 1 là số nguyên. Tìm số dãy nguyên .a1 ; a2 ; : : : ; an / thỏa mãn điều kiện
0 6 ai 6 n i với i D 1; 2; : : : ; n 1 và i < j; ai > 0; ai C1 D ai C2 D    D aj 1 D 0 thì

j > i > ai aj :

Bài tập 50. Cho n > 1 là số nguyên. Tìm số dãy nguyên .a1 ; a2 ; : : : ; an / thỏa mãn điều kiện
i 6 ai 6 n và nếu i 6 j 6 aj thì aj 6 ai :

Bài tập 51. Cho n > 1 là số nguyên. Tìm số dãy nguyên .a1 ; a2 ; : : : ; an / thỏa mãn điều kiện
1 6 ai 6 i và nếu ai D j thì ai j 6 j r với 1 6 r 6 j 1:

Bài tập 52. Cho n > 1 là số nguyên. Tìm số hoán vị a1 a2    an của tập f1; 2; : : : ; ng thỏa
mãn đồng thời các điều kiện:

a) 1; 3; : : : ; 2n 1 xuất hiện theo thứ tự tăng dần.

b) 2; 4; : : : ; 2n xuất hiện theo thứ tự giảm dần.

c) 2i 1 đứng trước 2i với 1 6 i 6 n:

Bài tập 53. Tìm số hoán vị a1 a2    an của tập f1; 2; : : : ; ng thỏa mãn điều kiện không có dãy
con tăng có độ dài lớn hơn 2:

Bài tập 54. Cho n > 1 là số nguyên. Tìm số dãy nguyên .a1 ; a2 ; : : : ; an / thỏa mãn ai > 2 và
dãy a1 a2    an có tính chất với mỗi i D 1; 2; : : : ; n ta có ai là ước của tổng hai số đứng liền
trước và liền sau nó.

Bài tập 55 (Canada, 2009). Cho số nguyên dương n > 1 và bảng ô vuông kích thước 3  n:
Một quân xe xuất phát từ ô góc trên bên trái và đi đến ô góc dưới bên trái sao cho đường đi của
nó tạo thành một đường gấp khúc không tự cắt. Hỏi có bao nhiều cách đi như thế?

Tài liệu tham khảo


[1] Nguyễn Văn Mậu (chủ biên), Chuyên Đề Chọn Lọc Tổ Hợp Và Toán Rời Rạc, Nhà xuất bản
Giáo Dục, 2008:

[2] Nguyễn Văn Mậu (chủ biên), Toán Rời Rạc Và Một Số Vấn Đề Liên Quan, Tài liệu bồi dưỡng
giáo viên hè 2007; Trường ĐHKHTN - ĐHQG Hà Nội.

[3] Trần Nam Dũng (chủ biên), Chuyên Đề Toán Học Số 8; 9, Trường PTNK - ĐHQG TP. Hồ
Chí Minh.

[4] Le Hai Chau - Le Hai Khoi, Selected Problems Of The Vietnamese Maththematical Olympiad
.1962 2009/, World Scientific.

[5] Tạp chí Toán học tuổi trẻ, Crux - Canada, AMM - USA

[6] Titu Andresscu - Zuming Feng, A Path To Combinatorics For Underfrduates, Birkhauser.

[7] Arthur Engel, Problem - Solving Strategies, Springer.


Phương pháp quỹ đạo 167

[8] Titu Andreescu and Zuming Feng, 102 Combinatorial Problems From The Training Of The
USA IMO Team.

[9] Phạm Minh Phương, Một Số Chuyên Đề Toán Học Tổ Hợp Bồi Dưỡng Học Sinh Giỏi Trung
Học Phổ Thông, Nhà xuất bản Giáo Dục Giáo dục.

[10] www.mathscope.org, www.mathlinks.org, www.imo.org.yu


168 Các phương pháp giải toán qua các kỳ thi Olympic
TỔNG QUÁT VÀ ỨNG DỤNG MỘT BÀI TOÁN
CHIA ĐÔI ĐOẠN THẰNG
Trần Quang Hùng
(Trường THPT chuyên KHTN)

Bài báo viết về một bài toán chia đôi đoạn thẳng khá kinh điển với hướng tiếp cận dùng các bổ đề
dễ hiểu cùng với một số hướng tổng quát và khai thác với công cụ là các định lý Ceva, Menelaus
và tính chất tứ giác ngoại tiếp, phần ứng dụng có sử dụng ứng dụng của phép nghịch đảo.

1. Mở đầu
Trên báo toán học tuổi trẻ số 390 tháng 12 năm 2009 bài T12/390 [3] tác giả Tạ Hồng Sơn có đề
nghị một bài toán, cũng trên trên đàn AoPS trong [1,2] có đề xuất bài toán đó, bài toán như sau:
Bài toán 1. Cho tam giác ABC nhọn có trực tâm H . Chứng minh rằng tiếp tuyến chung trong
khác HA của đường tròn nội tiếp các tam giác HAB; HAC thì chia đôi BC .
Bài toán trên đã có lời giải trên THTT số 394 tháng 4 [4]. Cũng trên diễn đàn AoPS trong [2,6]
có đề xuất bài toán như sau:
Bài toán 2. Cho tam giác ABC có tâm nội tiếp I . Đường tròn nội tiếp tam giác IBC tiếp xúc
BC tại D. Chứng minh rằng tiếp tuyến chung trong khác IA của đường tròn nội tiếp các tam
giác IAB; IAC đi qua D.
Nhận xét. Cả hai bài toán trên đẹp và đều có chung một cấu hình là tiếp tuyến chung trong của
hai đường tròn ngoại tiếp tam giác. Tôi nghĩ rằng chúng nhất định phải nằm trong một bài toán
chung nào đó. Chúng ta hãy tìm hiểu điều đó ở phần sau.

2. Mở rộng
Sau quá trình tìm hiểu tôi đã đề xuất bài toán tổng quát cho cả hai bài toán trên như sau:
Bài toán 3. Cho tam giác ABC và P bất kỳ nằm trong tam giác. Gọi Q là điểm đẳng giác
của P trong tam giác ABC . Gọi .K/; .L/ lần lượt là các đường tròn nội tiếp của tam giác
PAB; PAC . Chứng minh rằng tiếp tuyến chung trong khác PA của .K/ và .L/ đi qua tiếp điểm
của đường tròn nội tiếp của tam giác QBC với cạnh BC .
Để giải quyết bài toán này chúng ta sẽ dùng một số bổ đề sau:
Bổ đề 1. Cho tứ giác ABCD. S; T bất kỳ lần lượt thuộc BD; AC . ST cắt AB; CD tại M; N .
Chứng minh rằng
MB NC SB T C
 D  :
MA ND SD TA

169
170 Các phương pháp giải toán qua các kỳ thi Olympic

Bổ đề trên là một ứng dụng của định lý Thales.

B
M
A
S

T
K

D
N
C

Chứng minh. Lấy K; L thuộc M; N sao cho DK k CL k AB. Sử dụng định lý Thales ta có
biến đổi tỷ số:
MB MB DK CL SB ND T C
D   D   :
MA DK CL MA SD NC TA
Suy ra MB
MA ND
 NC D SD
SB T C
 TA . Ta có điều phải chứng minh.

Bổ đề 2. Cho tứ giác ABCD. S; T bất kỳ lần lượt thuộc BD; AC . ST cắt AB; CD tại M; N .
CS giao BN tại P . AS giao DM tại Q. Chứng minh rằng AP; CQ và ST đồng quy.

B
M

Q
A S
P

D
N C

Chứng minh. Áp dụng định lý Menelaus cho tam giác ABS với M; Q; D thẳng hàng, ta có
QA DS MB
  D 1: .1/
QS DB MA

Áp dụng định lý Menelaus cho tam giác CDS với N; P; B thẳng hàng, ta có
PS NC BD
  D 1: .2/
P C ND BS
Tổng quát và ứng dụng một bài toán chia đôi đoạn thẳng 171

MB NC SB TC
Nhân hai đẳng thức (1), (2) chú ý theo bổ đề MA
 ND
D SD
 TA
; ta thu được

QA PS DS BD SB T C QA PS T C
     D 1 hay   D 1:
QS P C DB BS SD TA QS P C TA

Vậy AP; CQ và ST đồng quy. Ta có điều phải chứng minh.


Chú ý bổ đề thực chất đúng cho mọi vị trí tứ điểm ABCD mà không cần ABCD là tứ giác lồi.
Ta có thể quan sát hình vẽ sau:

M
S
Q P

D
N
A T C

Bổ đề 3. Cho tứ giác ABCD ngoại tiếp đường tròn .I /. .I / tiếp xúc AB; BC; CD; DA lần
lượt tại E; F; G; H . Chứng minh rằng EF; GH; AC; BD đồng quy tại P và PPA
C CF
D AH .

F
I K
A

P
C
H
G

D
172 Các phương pháp giải toán qua các kỳ thi Olympic

Chứng minh. Gọi HF giao AC tại P . Lấy K thuộc HF sao cho CK k AH dễ thấy tam giác
KF C cân. Từ đó
PC CK CF
D D :
PA AH AH
Nếu EG cắt AC tại Q; tương tự ta dễ chứng minh
QC CG CF
D D :
QA AE AH

Vậy P  Q; suy ra AC đi qua giao điểm P của EF; GH . Tương tự BD đi qua P . Ta có điều
phải chứng minh.
Trở lại bài toán:
A

M
E K
Q
S
U V
L F
P G
H N
I

B T=R C

Lời giải bài toán. Gọi .K/; .L/ là các đường tròn nội tiếp tam giác P CA; PAB. PA tiếp xúc
.K/; .L/ lần lượt tại E; F . PB; P C lần lượt tiếp xúc .K/; .L/ tại G; H . Gọi tiếp tuyến chung
trong khác PA của .K/; .L/ cắt AB; BC tại S; T . AN giao CS tại V . BS giao MP tại U .
Theo bổ đề 2 thì BV; C U và ST đồng quy. Từ đây suy ra
TC V C US
D  : .1/
TB V S UB
Ta chú ý các tứ giác MSPB và ASNC ngoại tiếp. Theo bổ đề 3 ta có
VC CG US SF
D ; D : .2/
VS SE UB BH
Tổng quát và ứng dụng một bài toán chia đôi đoạn thẳng 173

Từ (1), (2) chú ý LK đi qua S là EK k LF; suy ra

TC C G SF SF C G LF C G LH C G
D  D  D  D  : .3/
TB SE BH SE BH KE BH BH KG

Nếu gọi tiếp điểm của đường tròn .I / nội tiếp tam giác QBC với BC là R. Theo tính chất đẳng
giác ta dễ có 4BLH  4BIR; 4CKG  4CIR. Suy ra

LH IR CG RC
D ; D : .4/
HB RB KG IR

Từ (3),(4) ta suy ra
TC IR RC RC
D  D :
TB RB IR RB
Từ đó T  R, hay tiếp tuyến chung trong khác PA của .K/; .L/ đi qua tiếp điểm của đường
tròn nội tiếp tam giác QBC và BC . Ta có điều phải chứng minh.

Nhận xét. Nếu P trùng trực tâm H thì Q trùng tâm ngoại tiếp O khi đó tam giác OBC cân
nên tiếp điểm đường tròn nội tiếp tam giác OBC là trung điểm BC ta có bài toán 1. Khi P
trùng tâm nội tiếp I thì Q cũng trùng I , ta có bài toán 2. Bài toán này là tổng quát với P bất kỳ
nên có giá trị ứng dụng lớn.

Thực chất có một bài toán tổng quát cũng đã xuất hiện trong lời giải bài toán 1 ở trong THTT số
394 [4] nhưng lần đầu tiên bài toán tổng quát xuất hiện ở [5]. Ở thời điểm sau, trong [2] cũng
xuất hiện bài toán tổng quát tương tự và chứng minh giống như đã đăng trên THTT số 394 và
trong [5], bài toán tổng quát như sau:

Bài toán 4. Cho tứ giác ABCD có thể không lồi. Tiếp tuyến chung trong khác BD của đường
tròn nội tiếp các tam giác ABD; CBD cắt AC tại T . Chứng minh rằng

TA cot A2
D :
TC cot C2

Về bản chất bổ đề này cũng là một cách phát biểu khác của bài toán 3 tuy nhiên ta thấy rằng phát
biểu có liên quan tới yếu tố lượng giác và chứng minh như trong THTT và [2] dùng qua các biến
đổi lượng giác không hề đẹp. Hơn nữa cách tiếp cận bài toán 3 tổng quát qua các bổ đề có tính
đối xứng như đã làm ở trên cho ta dễ hiểu ý tưởng của lời giải hơn. Cách phát biểu đi qua tiếp
điểm nội tiếp đẹp và có nhiều ứng dụng. Mặt khác với các phát biểu đi qua tiếp điểm thì một
cách hoàn toàn tương tự, chúng ta có thể chứng minh các bài toán mở rộng cho các đường tròn
bàng tiếp như sau:

Bài toán 5. Cho tam giác ABC và P bất kỳ. Q là điểm đẳng giác của P trong tam giác ABC .
Gọi .K/; .L/ lần lượt là các đường tròn bàng tiếp góc A của tam giác PAB; PAC . Chứng minh
rằng tiếp tuyến chung trong khác PA của .K/ và .L/ đi qua tiếp điểm của đường tròn bàng tiếp
góc Q của tam giác QBC với cạnh BC .
174 Các phương pháp giải toán qua các kỳ thi Olympic

Q
P

B C

Bài toán 6. Cho tam giác ABC và P bất kỳ. Q là điểm đẳng giác của P trong tam giác ABC .
Gọi .K/; .L/ lần lượt là các đường tròn bàng tiếp góc B; C của tam giác PAB; PAC . Chứng
minh rằng tiếp tuyến chung trong khác PA của .K/ và .L/ đi qua tiếp điểm của đường tròn nội
tiếp của tam giác QBC với cạnh BC .

B C
Tổng quát và ứng dụng một bài toán chia đôi đoạn thẳng 175

Bài toán 7. Cho tam giác ABC và P bất kỳ. Q là điểm đẳng giác của P trong tam giác ABC .
Gọi .K/; .L/ lần lượt là các đường tròn bàng tiếp góc P của tam giác PAB; PAC . Chứng minh
rằng tiếp tuyến chung trong khác PA của .K/ và .L/ đi qua tiếp điểm của đường tròn bàng tiếp
góc Q của tam giác QBC với cạnh BC .

3. Một số ứng dụng


Bài toán 3 phát biểu tổng quát và các trường hợp mở rộng sang đường tròn bàng tiếp có nhiều
ứng dụng. Từ 7 bài toán gốc nếu biết sử dụng thêm các phép biến hình làm công cụ ta sẽ tạo ra
được nhiều bài toán thú vị và đa dạng , từ các bài toán điểm cố định, đường cố định tới các bài
toán tam giác cân và điểm đồng viên. Chúng ta hãy bắt đầu từ ví dụ sau:

Bài toán 8. Cho tam giác ABC có điểm P nằm trong tam giác sao cho ∠PBA D ∠P CA.
Chứng minh rằng tiếp tuyến chung trong khác PA của đường tròn nội tiếp hai tam giác
PAB; PAC luôn đi qua một điểm cố định khi P thay đổi.

Lời giải. Gọi Q là đẳng giác của P trong tam giác ABC . Ta có

∠QBC D ∠PBA D ∠P CA D ∠QCB

nên tam giác QBC cân tại Q. Từ đó đường tròn nội tiếp tam giác QBC tiếp xúc BC tại trung
điểm M của BC . Theo bài toán 3 tiếp tuyến chung trong khác PA của đường tròn nội tiếp hai
tam giác PAB; PAC luôn đi qua một điểm cố định M khi P thay đổi.

P Q

B C
M

Bài toán trên là một ứng dụng tuy có phần đơn giản của bài toán 3 nhưng nếu sử dụng phép
nghịch đảo ta sẽ thu được bài toán thú vị dưới đây:

Bài toán 9. Cho tam giác ABC và điểm P thay đổi sao cho phân giác ∠BP C đi qua A.
Đường tròn .K/ đi qua A và tiếp xúc với các đường tròn A-mixtilinear ngoại của tam giác
APB; AP C sao cho tâm hai đường tròn này không cùng ở trong hoặc ở ngoài .K/. Chứng minh
rằng K luôn di chuyển trên một đường thẳng cố định khi P thay đổi.
176 Các phương pháp giải toán qua các kỳ thi Olympic

K
O

B C

D P

Lời giải. Sử dụng phép nghịch đảo cực A phương tích bất kỳ theo bài thì đường tròn đường tròn
.K/ đi qua A tiếp xúc với các đường tròn A-mixtilinear ngoại của tam giác APB; AP C sao cho
tâm hai đường tròn này không cùng ở trong hoạc ngoài .K/ thì luôn đi qua điểm D trên đường
tròn ngoại tiếp tam giác ABC sao cho AD là đường đối trung của tam giác ABC . Vậy D cố
định nên tâm K nằm trên trung trực AD cố định.
Ta lại tiếp tục một bài toán ứng dụng sau của bài toán 1.
Bài toán 10. Cho tam giác ABC nội tiếp đường tròn .O/ trực tâm H và AD là đường kính
của .O/. Đường tròn .K/ tiếp xúc HA; HB và tiếp xúc trong .O/. Đường tròn .L/ tiếp xúc
HA; H C và tiếp xúc trong .O/. Chứng minh rằng tiếp tuyến chung trong khác HA của .K/ và
.L/ đi qua D.

L O
K
H

B C

D
Tổng quát và ứng dụng một bài toán chia đôi đoạn thẳng 177

Lời giải. Theo định lý Feuerbach thì đường tròn nội tiếp tam giác HBC tiếp xúc trong đường
tròn Euler của tam giác HAB tuy nhiên đường tròn Euler của tam giác HAB cũng chính là
đường tròn Euler của tam giác ABC . Vị tự tâm H tỷ số 2 thì đường tròn Euler của tam giác
ABC biến thành đường tròn .O/. Như vậy đường tròn nội tiếp tam giác HAB biến thành đường
tròn tiếp xúc HA; HB và tiếp xúc trong .O/ chính là .K/. Tương tự đường tròn nội tiếp tam
giác HAC qua phép vị tự tâm H tỷ số 2 biến thành .L/. Theo bài toán 1 tiếp tuyến chung trong
khác HA của đường tròn nội tiếp các tam giác HAB; HAC thì đi qua trung điểm M của BC
nên qua phép vị tự tâm H tỷ số 2 thì tiếp tuyến chung trong khác HA của .K/ và .L/ đi qua D
vì dễ chứng minh D là ảnh của M qua phép vị tự tâm H tỷ số 2.

Bài toán 11. Cho 4ABC trực tâm H . Gọi K; L là tâm nội tiếp tam giác HAB; HAC . KL
cắt HA và trung trực BC tại S; T . Gọi M là trung điểm BC . Chứng minh rằng 4MST cân.

T
L
K O
S
H

B M C

Lời giải. Vì AH là một tiếp tuyến chung trong của .K/ và .L/ nên S là tâm vị tự trong của
.K/ và .L/. Từ đó theo bài toán 1 SM là tiếp tuyến chung trong khác của .K/ và .L/. Từ đó dễ
có HA và SM đối xứng nhau qua KL hay ST là phân giác ∠ASM lại có OM k AH ta dễ thu
được tam giác MST cân.
Ta có bài toán mở rộng tương tự:

Bài toán 12. Cho tam giác ABC có P là điểm nằm trong tam giác sao cho ∠PBA D ∠P CA.
Gọi K; L là tâm nội tiếp tam giác PAB; PAC . Gọi M là trung điểm BC . KL cắt PA và đường
thẳng qua M song song PA lần lượt tại S và T . Chứng minh rằng tam giác MST cân.
178 Các phương pháp giải toán qua các kỳ thi Olympic

L T
K S
P

B C
M

Tổng quát hơn nữa ta thu được bài toán sau:

Bài toán 13. Cho tam giác ABC có P là điểm nằm trong tam giác. Gọi K; L là tâm nội tiếp
tam giác PAB; PAC . Q là đẳng giác của P trong tam giác ABC . Đường tròn nội tiếp tam
giác ABC tiếp xúc BC tại M . KL cắt PA và đường thẳng qua M song song PA lần lượt tại S
và T . Chứng minh rằng tam giác MST cân.

Bài toán 14. Cho tam giác ABC có trực tâm H . Gọi K; L là tâm nội tiếp tam giác
HAB; HAC . KL cắt AH tại S . T đối xứng với H qua S . R là hình chiếu của T lên KL.
M đối xứng với S qua R. Gọi G là hình chiếu của A lên TM . Chứng minh rằng G nằm trên
đường tròn ngoại tiếp tam giác ABC .

O Z

T
M L
Y S
R
K
H

B C

D
Tổng quát và ứng dụng một bài toán chia đôi đoạn thẳng 179

Lời giải. Gọi .O/ là đường tròn ngoại tiếp tam giác ABC . Gọi AD là đường kính của .O/.
Phép vị tự tâm H tỷ số 2 biến đường tròn .K/ nội tiếp tam giác HAB thành đường tròn .Y / tiếp
xúc HA; HB và tiếp xúc trong .O/, đường tròn .L/ nội tiếp tam giác HAC thành đường tròn
.Z/ tiếp xúc HA; H C và tiếp xúc trong .O/ và biến S thành T . Từ đó Y Z đi qua T , như vậy T
chính là tâm vị tự trong của .Y / và .Z/. Theo bài toán 10 thì TD là tiếp tuyến chung trong của
.Y / và .Z/. Vậy TD và TA đối xứng nhau qua Y Z, nhưng do TR vuông góc với Y Z nên TD và
TA cũng đối xứng nhau qua TR. Ta lại có S và M đối xứng nhau qua R nên cũng đối xứng nhau
qua TR. Từ đó TD đi qua M . Vậy hình chiếu G của A lên TM phải nằm trên đường tròn đường
kính AD cũng chính là đường tròn .O/ ngoại tiếp 4ABC . Ta có điều phải chứng minh.
Bài toán 15. Cho tam giác ABC có đường cao AK, trung tuyến AM , phân giác AD, tâm
ngoại tiếp O và trực tâm H . Gọi L đối xứng O qua BC . Giả sử K; D; M cố định và B; C thay
đổi. Chứng minh rằng tiếp tuyến chung trong khác LH của đường tròn ngoại tiếp tam giác
LHB và LH C luôn đi qua một điểm cố định.

I O
H

P M Q
B D C
K

Lời giải. Gọi đường tròn nội tiếp .I / và đường tròn bàng tiếp góc A của tam giác ABC tiếp
xúc BC tại P; Q. Ta dễ thấy hàng .AD; IJ / điều hòa, chiếu vuông góc xuống BC cho ta hàng
.HD; PQ/ điều hòa. Theo kết quả đã biết M là trung điểm PQ nên

MP 2 D MQ2 D MD  MK;
180 Các phương pháp giải toán qua các kỳ thi Olympic

do M; D; K cố định nên P cố định. Do O và L đối xứng nhau qua BC nên

∠LBC D ∠OBC D ∠ABH:

Tương tự ∠LCB D ∠ACH nên A và L đẳng giác trong tam giác HBC . Theo bài toán 3 tiếp
tuyến chung trong khác LH của đường tròn ngoại tiếp tam giác LHB và LH C luôn đi qua tiếp
điểm của đường tròn nội tiếp tam giác ABC với BC đó chính là P cố định.
Tổng quát hơn bài toán trên cho ta bài toán sau, lời giải hoàn toàn tương tự.
Bài toán 16. Cho tam giác ABC có đường cao AK, trung tuyến AM , phân giác AD. Hai
điểm P; Q đẳng giác trong tam giác ABC . Gọi L đối xứng Q qua BC . Giả sử K; D; M cố
định và B; C; P; Q thay đổi. Chứng minh rằng tiếp tuyến chung trong khác LP của đường tròn
ngoại tiếp tam giác LPB và LP C luôn đi qua một điểm cố định.

Q
I
P

B C
H

Lời giải. Tương tự lời giải trên ta dễ chứng minh A và L đẳng giác trong tam giác PBC và
đường tròn .I / nội tiếp 4ABC tiếp xúc với BC tại điểm cố định nên tuyến chung trong khác
LP của đường tròn ngoại tiếp tam giác LPB và LP C luôn đi qua một điểm cố định đó.

4. Một số bài luyện tập


Các ứng dụng của các bài toán trên còn rất nhiều, các bạn hãy làm các bài tập sau để luyện tập.
Bài tập 56. Cho tam giác ABC nội tiếp đường tròn .O/ cố định với B; C cố định và A
di chuyển trên .O/. I là tâm nội tiếp tam giác ABC . IA; IB; IC cắt .O/ tại D; E; F khác
A; B; C . Đường tròn .K/ tiếp xúc đoạn ID; IE và tiếp xúc trong .O/. Đường tròn .L/ tiếp xúc
đoạn ID; IF và tiếp xúc trong .O/. Chứng minh rằng tiếp tuyến chung trong khác ID của .K/
và .L/ luôn đi qua điểm cố định khi A thay đổi.
Bài tập 57. Cho tam giác ABC nhọn có tâm ngoại tiếp O. Tiếp tuyên chung trong khác OA
của đường tròn nội tiếp tam giác OAB; OAC cắt BC tại D. Tương tự có E; F . Chứng minh
rằng AD; BE; CF đồng quy.
Tổng quát và ứng dụng một bài toán chia đôi đoạn thẳng 181

Bài tập 58. Cho tam giác ABC với P; Q đẳng giác trong tam giác ABC . R đối xứng với Q
qua BC . Gọi K; L là tâm nội tiếp tam giác RPB; RP C . KL cắt RP tại S. Đường tròn nội tiếp
tam giác ABC tiếp xúc BC tại D. Đường thẳng qua D song song với RP cắt KL tại T . Chứng
minh rằng tam giác DST cân.
Bài tập 59. Cho tam giác ABC có phân giác AD và tâm bàng tiếp góc A là J . Đường tròn
ngoại tiếp tam giác ACJ; ABJ cắt BC tại M; N khác C; B. Đường tròn .K/ tiếp xúc đoạn
DM; DJ và tiếp xúc trong đường tròn ngoại tiếp tam giác ACJ . Đường tròn .L/ tiếp xúc đoạn
DN; DJ và tiếp xúc trong đường tròn ngoại tiếp tam giác ABJ . Đường tròn .P / đi qua A và
tiếp xúc với .K/; .L/ sao cho K và L không cùng ở trong hoặc ở ngoài .P /. Chứng minh rằng
P đi qua chân phân giác ngoài kẻ từ A của tam giác ABC .

D C
E
M B N
P

K L

Bài tập 60. Cho tam giác ABC nội tiếp đường tròn .O/ trực tâm H . Đường tròn .K/ tiếp xúc
đoạn HA; HB và tiếp xúc trong .O/. Đường tròn .L/ tiếp xúc đoạn HA; H C và tiếp xúc trong
.O/. Chứng minh rằng có một tiếp tuyến chung ngoài của .K/ và .L/ song song với BC .
Bài tập 61. Cho tam giác ABC với trực tâm H . Chứng minh rằng có một tiếp tuyến chung
ngoài của đường tròn nội tiếp tam giác HAB; HAC song song với BC .
Bài tập 62. Cho tam giác ABC nội tiếp đường tròn .O/. Đường tròn A-mixtilinear tiếp xúc
(O) tại D. Gọi .K/; .L/ là các đường tròn I - mixtilinear của tam giác IAB; IAC . Đường tròn
.P / đi qua I và tiếp xúc với .K/; .L/ sao cho K và L không cùng ở trong hoặc ở ngoài .P /. .P /
cắt đường tròn ngoại tiếp tam giác IBC tại Q khác I . Chứng minh rằng Q; I; D thẳng hàng.
Bài tập 63. Cho tam giác ABC nội tiếp đường tròn .O/. Đường tròn A-mixtilinear tiếp xúc
(O) tại D. Gọi .K/ tiếp xúc đoạn IA,IB và tiếp xúc trong .O/ và đường tròn .L/ tiếp xúc đoạn
IA; IC và tiếp xúc trong .O/. Đường tròn .P / đi qua I và tiếp xúc với .K/; .L/ sao cho K và
L không cùng ở trong hoặc ở ngoài .P /. Chứng minh rằng .P / đi qua D.
182 Các phương pháp giải toán qua các kỳ thi Olympic

Bài tập 64. Cho tam giác ABC với trực tâm H .

1) Chứng minh rằng có một và có một tiếp tuyến chung trong của đường tròn bàng tiếp góc
H của các tam giác HAB; HAC chia đôi BC và có tiếp tuyến chung ngoài của hai đường
tròn này song song với BC .

2) Chứng minh rằng có một và có một tiếp tuyến chung trong của đường tròn bàng tiếp góc A
của các tam giác HAB; HAC chia đôi BC và có tiếp tuyến chung ngoài của hai đường
tròn này song song với BC .

3) Chứng minh rằng có một và có một tiếp tuyến chung trong của đường tròn bàng tiếp góc
B của tam giác HAB và đường tròn bàng tiếp góc C của tam giác HAC chia đôi BC và
có tiếp tuyến chung ngoài của hai đường tròn này song song với BC .

Bài tập 65. Cho tam giác ABC với điểm P sao cho ∠PBA D ∠P CA.

1) Chứng minh rằng có một và có một tiếp tuyến chung trong của đường tròn bàng tiếp góc
P của các tam giác PAB; PAC chia đôi BC .

2) Chứng minh rằng có một và có một tiếp tuyến chung trong của đường tròn bàng tiếp góc A
của các tam giác PAB; PAC chia đôi BC .

3) Chứng minh rằng có một và có một tiếp tuyến chung trong của đường tròn bàng tiếp góc
B của tam giác PAB và đường tròn bàng tiếp góc C của tam giác PAC chia đôi BC .

Cuối cùng là một kết quả rất thú vị liên quan tới bài toán ban đầu do tác giả tìm ra.

Bài tập 66. Cho 4ABC; trực tâm H . Gọi da là tiếp tuyến chung trong khác HA của đường
tròn nội tiếp tam giác HAB; HAC . Tương tự có db ; dc . Chứng minh rằng da ; db ; dc đồng quy.

Tài liệu tham khảo


[1] Topic common tangent passes through midpoint.
http://www.artofproblemsolving.com/community/c6h411403

[2] Topic A pretty problem with 3 incircles (by Tiks).


http://www.artofproblemsolving.com/community/c6h354536

[3] Tạp chí toán học và tuổi trẻ số 390 tháng 12 năm 2009.

[4] Tạp chí toán học và tuổi trẻ số 394 tháng 4 năm 2010.

[5] Topic Segment ratio.


http://artofproblemsolving.com/community/c6h246658

[6] Circles [incircles of BCI, CAI, ABI].


http://www.artofproblemsolving.com/community/c6h3680
Tổng quát và ứng dụng một bài toán chia đôi đoạn thẳng 183

[7] Topic Tangent to incircles.


http://www.artofproblemsolving.com/community/c6h487427

[8] Topic common tangent and parallel.


http://www.artofproblemsolving.com/community/c6h359843
184 Các phương pháp giải toán qua các kỳ thi Olympic
HÀNG ĐIỂM ĐIỀU HÒA
Huỳnh Chí Hào
(THPT chuyên Nguyễn Quang Diêu, Đồng Tháp)

1. Tóm tắt lý thuyết


1.1. Tọa độ trên trục
1.1.1. Trục
!
Một đường thẳng được gọi là trục (tọa độ) nếu trên đó đã chọn một điểm O và một vectơ i có
! !
độ dài bằng 1: Điểm O gọi là gốc của trục, i được gọi là vectơ đơn vị của trục, hướng của i
được gọi là hướng của trục.

Ký hiệu: x 0 Ox:

1.1.2. Tọa độ của một vectơ trên trục


! !
Cho vectơ !u nằm trên trục x 0 Ox có vectơ đơn vị là i . Vì !
u cùng phương i nên tồn tại duy
!
nhất số x sao cho !
u D x: i . Số x được gọi là tọa độ của vectơ !
u.
Ký hiệu: !u D .x/ hay !
u .x/ :

1.1.3. Tọa độ của một điểm trên trục


!
Cho điểm M thuộc trục x 0 Ox. Tọa độ của vectơ OM được gọi là tọa độ của điểm M .
Ký hiệu: M D .x/ hay M .x/ :

185
186 Các phương pháp giải toán qua các kỳ thi Olympic

1.1.4. Độ dài đại số của vectơ


! !
Tọa độ của của vectơ AB được gọi là độ dài đại số của vectơ AB và ký hiệu là AB:
Chú ý. Khi ta viết AB có nghĩa là đường thẳng AB đã được xem là một trục với một gốc O nào
!
đó và một vectơ đơn vị i nào đó.

Tính chất.
ˇ ˇ
 ˇAB ˇ D AB:
! !
 AB D AB , AB "" i :
! !
 AB D AB , AB "# i :

 AB D BA:

 AB D CB CA; C bất kỳ trên đường thẳng AB:

 AB D AC C CB; C bất kỳ trên đường thẳng AB:

 AB D xB xA :

1.1.5. Các định lý quan trọng có liên quan đến độ dài đại số
Định lý 1 (Định lý Thales (dạng đại số)). Cho các bộ ba điểm A; B; C và A0 ; B 0 ; C 0 , theo thứ
tự thuộc các đường thẳng  và 0 . Nếu các đường thẳng AA0 ; BB 0 ; C C 0 đôi một song song thì

AB A0 B 0
D :
BC B 0C 0
Hàng điểm điều hòa 187

Định lý 2 (Định lý Ceva (dạng đại số)). Cho tam giác ABC và các điểm M; N; P khác A; B; C ,
theo thứ tự thuộc các đường thẳng BC; CA; AB. Khi đó các đường thẳng AM; BN; CP hoặc
đồng quy hoặc đôi một song song khi và chỉ khi

MB NC PA
: : D 1:
M C NA PB

Định lý 3 (Định lý Menelaus (dạng đại số)). Cho 4ABC và các điểm M; N; P khác A; B; C ,
theo thứ tự thuộc các đường thẳng BC; CA; AB. Khi đó M; N; P thẳng hàng khi và chỉ khi
MB NC PA
  D 1:
MC NA PB

1.2. Hàng điểm điều hòa là gì?


1.2.1. Hàng điểm
Bộ 4 điểm đôi một khác nhau, có kể đến thứ tự, cùng thuộc một đường thẳng được gọi là hàng điểm.
188 Các phương pháp giải toán qua các kỳ thi Olympic

1.2.2. Tỉ số kép của hàng điểm


Định nghĩa 1. Tỉ số kép của hàng điểm A; B ; C ; D là một số, kí hiệu là .AB C D / và được
xác định như sau:
C A DA
. AB C D / D W
C B DB
Dạng tọa độ:

Nếu A . a / ; B .b /; C .c /; D .d / thì
a c a d
. AB C D / D W :
b c b d
Tính chấ
 .A B C D / D . C DAB / D . BAD C / D . D C BA/ :
1 1
 .A B C D / D . BA C D /
D . A BD C /
:

 .A B C D / D 1 .AC BD / D 1 . DB C A / :

 .A B C D / D . AB C D 0 / ) D  D 0 :

 .A B C D / ¤ 1:

1.2.3. Hàng điểm điều hòa


Định nghĩa 2. Nếu .AB C D / D 1 thì hàng điểm A; B ; C ; D được gọi là hàng điểm điều
hòa. Nói cách khác: Nếu CC B
A DA
D DB thì A; B ; C ; D được gọi là hàng điểm điều hòa.

dn CA DA
. AB C D / D 1 , : D
CB DB
Khi đó ta nói: cặp điểm A; B và cặp điểm C ; D là hai cặp điểm liên hợp điều hòa.
Lưu ý. Nếu . AB C D / D 1 thì

. C DAB / D . BAD C / D .D C BA / D .BAC D / D .ABD C / D 1:

Biểu thức tọa độ đối với hàng điểm điều hòa:


 Hệ thức 1. Nếu A.a /; B .b /; C .c /; D .d /thì

. AB C D / D 1 , 2 . a b C c d / D .a C b / .c C d / :
Hàng điểm điều hòa 189

Chứng minh. Chọn một điểm O bất kỳ trên trục là gốc. Ta có:

CA DA a c a d
. AB C D / D 1, D , D
CB DB b c b d

, 2 . a b C c d / D . a C b / .c C d / :

2 1 1
 Hệ thức Descartes. . AB C D / D 1, AB
D AC
C AD
:

Chứng minh. Chọn O  A .a D 0/ trên trục là gốc. Ta có:

. AB C D / D 1 , 2 . a b C c d / D .a C b / .c C d /

2 1 1 2 1 1
, 2 c d D b . c C d / .do a D 0/ , D C , D C :
b c b AB AC AD

2 2
 Hệ thức Newton. . AB C D / D 1 , I A D IB D I C  ID:

Chứng minh. Chọn O  I .I là trung điểm của AB / trên trục là gốc. Ta có:

. AB C D / D 1 , 2 . a b C c d / D .a C b / .c C d /
2
, a 2 D c d .do a D b/ , IA D I C  ID:

 Hệ thức Maclaurin. . AB C D / D 1 , AC  AD D AB  AJ (trong đó J là


trung điêm của C D).
190 Các phương pháp giải toán qua các kỳ thi Olympic

Chứng minh. Chọn O  A trên trục là gốc. Ta có:

2 1 1 AC C AD
.AB C D / D 1, D C , AC  AD D AB 
AB AC AD 2

, AC  AD D AB  AJ :

Những hàng điểm điều hòa cơ bản:

Định lý 4. Nếu AD ; AE theo thứ tự là phân giác trong và phân giác ngoài của tam giác
A B C thì . B C DE / D 1:

Chứng minh. Theo tính chất của phân giác trong và phân giác ngoài của tam giác ta có:
DB AB EB AB
D ; D :
DC AC EC AC
Lại do, D nằm trong đoạn B C và E nằm ngoài đoạn B C nên ta có:

DB AB EB AB
D ; D :
DC AC EC AC
DB EB
Suy ra: DC
D EC
) . B C DE / D 1.

Định lý 5. Cho tam giác AB C và điểm O không thuộc các đường thẳng B C ; C A; AB . Các
đường thẳng AO ; BO ; C O theo thứ tự cắt B C ; C A; AB tại M ; N ; P . Hai đường thẳng
B C ; N P giao nhau tại Q. Khi đó, . B C M Q / D 1.

Chứng minh. Áp dụng định lý Ceva cho 4AB C với sự đồng quy của AM ; B N ; C P , ta có:

MB NC PA
  D 1: (13.1)
MC NA PB
Hàng điểm điều hòa 191

Áp dụng định lý Menelaus cho tam giác AB C với sự thẳng hàng Q ; N ; P , ta có hệ thức:

QB NC PA
  D 1: (13.2)
QC NA PB
MB QB
Từ (13.1) và (13.2) suy ra: MC
D QC
) .B C MQ/ D 1.
Định lý 6. Từ điểm S nằm ngoài đường tròn . O /, ta kẻ tới . O / các tiếp tuyến SA; SB
. A ; B 2 . O //. Một đường thẳng qua S cắt . O / tại M ; N . Gọi I là giao điểm của AB và
M N . Khi đó, . S I M N / D 1.

Để chứng minh định lý nầy ta cần sử dụng 3 bổ đề sau.


Bổ đề 1. Qua điểm S không thuộc đường tròn . O /, kẻ một đường thẳng cắt . O / tại M ; N .
Khi đó:
S M :S N D SO 2 R 2 :

Bổ đề 2. Nếu các đường thẳng AB ; C D cắt nhau tại S khácA; B ; C ; D thì A; B ; C ; D


cùng thuộc một đường tròn khi và chỉ khi SA  SB D S C  SD.
192 Các phương pháp giải toán qua các kỳ thi Olympic

Bổ đề 3. Nếu các đường thẳng AB ; S C cắt nhau tại S khácA; B thì đường tròn ngoại tiếp
2
tam giác A B C tiếp xúc với S C khi và chỉ khi SA  SB D S C .

Chứng minh. Gọi H là hình chiếu của O trên M N và K D SO \ AB .

Do I\KO D I\ H O (cùng bằng 90 ı /. Suy ra tứ giác OH I K nội tiếp. Theo các bổ đề trên và
theo hệ thức lượng trong tam giác vuông ta suy ra:
2
S M  S N D SA D S K  SO D S I  SH :
Do H là trung điểm của M N nên theo hệ thức Maclaurin ta suy ra . S I M N / D 1.

1.3. Tỉ số kép của chùm đường thẳng, phép chiếu xuyên tâm, chùm
điều hòa
1.3.1. Chùm đường thẳng và tỉ số kép của nó
Định nghĩa 3. Tập hợp các đường thẳng cùng đi qua một điểm được gọi là một chùm đầy đủ
đường thẳng.
Hàng điểm điều hòa 193

Định nghĩa 4. Bộ bốn đường thẳng đôi một khác nhau, có kể đến thứ tự, cùng thuộc một chùm
đầy đủ đường thẳng được gọi là một chùm đường thẳng.

Định lý 7. Cho a ; b ; c ; d là chùm đường thẳng tâm O . Đường thẳng  không đi qua O , theo
thứ tự cắt a ; b ; c ; d tại A; B ; C ; D. Đường thẳng  0 không đi qua O , theo thứ tự cắt a ; b ; c
0 A0
tại A 0 ; B 0 ; C 0 . Khi đó:  0 k d , . AB C D / D CC 0 B 0
.

Định lý 8. Cho a ; b ; c ; d là chùm đường thẳng tâm O . Đường thẳng  không đi qua O ,
theo thứ tự cắt a ; b ; c ; d tại A; B ; C ; D. Đường thẳng  0 không đi qua O , theo thứ tự cắt
a ; b ; c ; d tại A 0 ; B 0 ; C 0 ; D 0 . Khi đó: . AB C D / D . A 0 B 0 C 0 D 0 / :
194 Các phương pháp giải toán qua các kỳ thi Olympic

Định nghĩa 5. Số không đổi .AB C D / nói trên được gọi là tỉ số kép của chùm a ; b ; c ; d và
được kí hiệu là . a b c d /.

1.3.2. Phép chiếu xuyên tâm

Định nghĩa 6. Cho hai đường thẳng  ;  0 và điểm S không thuộc  ;  0 . Gọi K là điểm
thuộc  sao cho S K k  0 :Gọi f là ánh xạ đi từ tập hợp các điểm thuộc  n fK g tới tập hợp
các điểm thuộc  0 , xác định như sau: f . M / D M 0 sao cho S ; M ; M 0 thẳng hàng. Ánh xạ
f được gọi là phép chiếu xuyên tâm đi từ n fK gtới  0 . Điểm S được gọi là tâm của f .

Lưu ý. Nếu phép chiếu xuyên tâm f biến hàng điểm A; B ; C ; D thành hàng điểm A 0 ; B 0 ; C 0 ; D 0
thì . A B C D / D . A 0 B 0 C 0 D 0 / (hay Phép chiếu xuyên tâm bảo toàn tỉ số kép).

Định lý 9. Cho hai đường thẳng  ;  0 cắt nhau tại O . Các điểm A; B ; C thuộc ; các điểm
A 0 ; B 0 ; C 0 thuộc  0 . Khi đó: AA 0 ; BB 0 ; C C 0 hoặc đồng quy hoặc đôi một song song khi và
chỉ khi . OAB C / D . OA 0 B 0 C 0 /.

Định lý 10. Cho hai chùm O . AB C O 0 / và O 0 . AB C O /. Khi đó: A; B ; C thẳng hàng khi
và chỉ khi O . AB C O 0 / D O 0 . AB C O /.
Hàng điểm điều hòa 195

1.3.3. Chùm điều hòa


Định nghĩa 7. Chùm a ; b ; c ; d được gọi là điều hòa nếu . a ; b ; c ; d / D 1.
Định lý 11. Với chùm a ; b ; c ; d các điều kiện sau là tương đương:
1) . a ; b ; c ; d / D 1:
2) Tồn tại một đường thẳng song song với một đường thẳng của chùm và định ra trên ba
đường thẳng còn lại hai đoạn thẳng bằng nhau.
3) Mọi đường thẳng song song với một đường thẳng của chùm định ra trên ba đường thẳng
còn lại hai đoạn thẳng bằng nhau.
Định lý 12. Với chùm điều hòa a ; b ; c ; d các điều kiện sau là tương đương
1) c ?d :
2) c là một phân giác của các góc tạo bởi a ; b :
3) d là một phân giác của các góc tạo bởi a ; b =
196 Các phương pháp giải toán qua các kỳ thi Olympic

1.3.4. Một số kết quả thường sử dụng


 Kết quả 1. Cho .AB C D / D 1. Lấy O sao cho O C là phân giác trong của AOB
\
thì OD là phân giác ngoài của AOB
\.

 Kết quả 2. Cho . AB C D / D 1 và điểm O nằm ngoài hàng điểm điều hòa trên. Một
đường thẳng d cắt ba tia O C ; OB ; ODlần lượt tại E ; I ; F . Khi đó I là trung điểm của
EF khi và chỉ khi d song song với OA.

 Kết quả 3. Cho . OA; OB ; O C ; OD / D 1. Một đường thẳng d bất kì cắt các cạnh
OA ; OB ; O C ; OD lần lượt tại E ; F ; G ; H khi đó ta có . EF GK / D 1.
Hàng điểm điều hòa 197

 Kết quả 4. Cho chùm điều hòa .O x ; O y ; O z ; O t / D 1 khi đó nếu z[ O t D 90 0


thì O z là phân giác trong của góc x O y và O t là phân giác ngoài x O y .

 Kết quả 5. Cho chùm điều hòa . O x ; O y ; O z ; O t / D 1 một đường thẳng d bất kì
cắt O z ; O t ; O y lần lượt tại A; B ; I khi đó d song song O x khi và chỉ khi I là trung
điểm của AB .

2. Các ví dụ
Ví dụ 1 (Định lý Pappus). Cho hai đường thẳng  và . Các điểm A; B ; C thuộc . Các
điểm A 0 ; B 0 ; C 0 thuộc . X D B C 0 \ B 0 C I Y D C A 0 \ C 0 A I Z D AB 0 \ A 0 B .
Chứng minh rằng X ; Y ; Z thẳng hàng.
198 Các phương pháp giải toán qua các kỳ thi Olympic

Lời giải. Ta bỏ qua trường hợp đơn giản  k  0 . Xét trường hợp  và  0 cắt nhau. Đặt
 \  0 D O , E D B C 0 \ C A 0 , F D AC 0 \ BA 0 : Ta có: . BE X C 0 / D . OA 0 B 0 C 0 /
(xét phép chiếu xuyên tâm C / D . BA 0 ZF / (xét phép chiếu xuyên tâm A/. Theo định lý về
phép chiếu xuyên tâm ta suy ra: E A 0 ; X Z ; C 0 F đồng quy. Vậy X ; Y ; Z thẳng hàng.
Ví dụ 2 (Định lý Desargues). Cho các tam giác AB C và A 0 B 0 C . Đặt X D B C \ B 0 C 0 ;
Y D C A \ C 0 A 0 ; Z D AB \ A 0 B 0 . Chứng minh rằng X ; Y ; Z thẳng hàng khi và chỉ khi
A A 0 ; BB 0 ; C C 0 hoặc đồng quy hoặc đôi một song song.

Lời giải. Ta bỏ qua trường hợp đơn giản BB 0 k AC _ BB 0 k A 0 C 0 : Gọi E ; E 0 theo thứ tự là
giao điểm của BB 0 với AC ; A 0 C 0 . Ta có: X ; Y ; Z thẳng hàng khi và chỉ khi B . AB 0 C Y / D
B 0 .A 0 B C 0 Y / (định lý về chùm đường thẳng), tức là . AE C Y / D . A 0 E 0 C 0 Y / , AA 0 ;
E E 0 ; C C 0 hoặc đồng quy hoặc đôi một song song , AA 0 ; BB 0 ; C C 0 hoặc đồng quy hoặc
đôi một song song.
Ví dụ 3 (Định lý Blanchet mở rộng). Cho tam giác AB C , lấy D ; E ; F lần lượt thuộc các
đoạn B C ; C A; AB sao cho 3 đường thẳng AD ; BE ; C F đồng quy tại một điểm I (EF
không song song với B C /. Gọi L là giao điểm của AD và EF . Gọi H là hình chiếu của L
xuống B C . Chứng minh rằng H L là phân giác của E\HF .

Lời giải. Đặt K D B C \ EF . Ta có: .K DB C / D 1 ) . K LF E / D 1 (qua phép


chiếu xuyên tâm A/. Lại có: LH
\ K D 90 ı ) H L là phân giác của E
\ HF .
Hàng điểm điều hòa 199

MA
Ví dụ 4. Cho đoạn thẳng AB và số k > 0. Tìm quỹ tích các điểm M sao cho MB
D k ¤ 1.

DA EA
Lời giải. Lấy D ; E trên đường thẳng AB sao cho DB
D EB
D k ) . DE AB / D 1.
MA EA DA
Thuận: Ta có M B
D E B
D DB D k ) M D ; M E là phân giác ngoài và trong của AM
\ B
ı
) DM E D 90 ) M nằm trên đường tròn đường kính DE .
\

Đảo: Lấy M 0 trên đường tròn đường kính DE ) DM


\ 0 E D 90 ı ) M 0 E là phân giác

trong của AM
\ B (vì .DE AB / D 1.
Vậy quỹ tích các điểm M thỏa mãn điều kiện đề bài là đường tròn đường kính DE .
Chú ý. Đường tròn đường kính DE được gọi là đường tròn Apollonius xác định bởi đoạn AB
và số k .
Ví dụ 5. Cho tứ giác AB C D nội tiếp (AB không song song C D /. M ; N theo thứ tự là trung
điểm của AB ; C D. Đường tròn ngoại tiếp tam giác AB N cắt C D tại P . Đường tròn ngoại
tiếp tam giác C DM cắt AB tại Q. Chứng minh rằng AC ; BD ; P Q đồng quy.

Lời giải. Đặt S D AB \ C D. Ta có:

SA  SB D S C  SD D SQ  S M ; S C  SD D SA  SB D SP  S N :
200 Các phương pháp giải toán qua các kỳ thi Olympic

Do M ; N theo thứ tự là trung điểm của AB ; C D, nên theo hệ thức Maclaurin, suy ra:
. SQA B / D 1 và .SP C D / D 1. Vậy AC ; BD ; P Q đồng quy.

Ví dụ 6. Từ điểm S nằm ngoài đường tròn .O /, kẻ tới . O / các tiếp tuyến SA; SB (A; B
thuộc . O //. Một đườg thẳng qua S , cắt . O / tại hai điểm M ; N . Đường thẳng qua M , song
song với SA theo thứ tự cắt AB ; AN tại E ; F . Chứng minh rằng E M D EF .

Lời giải. Đặt I D AB \ M N . Ta có: . S I M N / D 1 (hàng điểm điều hòa cơ bản)


) A .S B M N / D 1: Do M F k SA ) E M D EF (định lý về chùm điều hòa).

Ví dụ 7. Cho tam giác AB C . Đường tròn nội tiếp . I / tiếp xúc với AC ; AB tại E ; F (EF
không song song với B C /. Đặt K D B I \ EF . Chứng minh rằng BK \ C D 90 ı .

Lời giải. Đặt L D EF \ B C . Gọi D là tiếp điểm của .I / và B C . Ta có:


     
DB EC FA DB EC FA F A DB E C
  D   D   D 1:
DC EA FB DC EA FB EA F B D C
Hàng điểm điều hòa 201

Do A D ; BE ; C F không thể đôi một song song nên theo định lý Ceva ta suy ra AD ; BE ; C F
đồng quy. Suy ra: .B C DL / D 1 (hàng điểm điều hòa cơ bản) ) K .B C DL / D 1.
Mặt khác ta lại có: K BD D K BF ) BK
\ D D BK
\ F . Vậy BK
\ C D 90 ı (định lý về
chùm điều hòa).
Ví dụ 8. Cho tam giác AB C . Đường tròn nội tiếp . I / tiếp xúc với B C ; C A; AB tại
D ; E ; F . EF không song song với B C /. H là hình chiếu của D trên EF . Chứng minh rằng

BH
\ D DC
\ HD:

Ý tưởng. Xem H là đỉnh của 1 chùm điều hòa trong đó có 2 đường thẳng vuông góc nhau.
Lời giải. Đặt S D EF \ B C . Ta có: AD ; BE ; C F đồng quy (theo định lý Ceva). Suy ra:
. S DB C / D 1 (hàng điểm điều hòa cơ bản). Do đó:

H . SDB C / D 1:

Vậy BH
\ D DC
\ H D (do H S ?H D, định lý về chùm điều hòa).
Ví dụ 9. Cho tam giác AB C . Đường tròn nội tiếp . I / theo thứ tự tiếp xúc với các cạnh
B C ; C A ; AB tại D ; E ; F . Gọi K là giao điểm của AI và EF . Chứng minh rằng

K
\ DE D ADF
\:
202 Các phương pháp giải toán qua các kỳ thi Olympic

Ý tưởng. Xem D là đỉnh của 1 chùm điều hòa trong đó có 2 đường thẳng vuông góc nhau.
Lời giải. Gọi M ; N là giao của AI với . I /. Ta có: D . AK M N / D 1 (hàng điểm điều
hòa cơ bản). Do DM ?DN nên M \DK D M \ DA (định lý về chùm điều hòa). Mặt khác ta lại
có: K DE D M DA. Suy ra: K DE D ADF .
\ \ \ \

Ví dụ 10. Đường thẳng  đi qua đỉnh A của hình bình hành AB C D, theo thứ tự cắt các
đường thẳng BD ; B C tại M ; N . Chứng minh rằng:
1 1 1
D C :
AM AN AP

Lời giải. Đặt O D AC \ BD. Trên  lấy Q sao cho C Q k BD. Vì AB C D là hình bình
hành nên OB D OD. Do C Q k BD nên theo định lý về chùm điều hòa ta suy ra được
C . AQBD / D 1: Suy ra: . AQN P / D 1. Khi đó, theo hệ thức Descartes ta có:
2 1 1
D C : .1/
AQ AN AP

Mặt khác, cũng vì tứ giác AB C D là hình bình hành nên ta suy ra OA D O C . Từ đó, kết hợp
với C Q k BD ta có:
AQ D 2AM : .2/
1 1 1
Từ (1) và (2) suy ra AM
D AN
C AP
:

Ví dụ 11. Cho tam giác AB C nội tiếp đường tròn . O /. D là điểm đối xứng với A qua O .
Tiếp tuyến với . O / tại D cắt B C tại E . OE theo thứ tự cắt AB ; AC tại M ; N . Chứng minh
rằng O M D O N .
Hàng điểm điều hòa 203

Lời giải. Gọi K ; L theo thứ tự là giao điểm của AB ; AC với DE . Trên DE lấy F sao cho
A F k E O . Ta thấy: AB
\ C D AD\ C DC \LD. Do đó, tứ giác BK LC nội tiếp. Từ đó, nên
theo định lý về hệ thức lượng trong đường tròn ta suy ra được:
2
ED D EB  E C D E K  E L:
Mặt khác, vì O là trung điểm của AD và vì AF k OE nên E là trung điểm của F D. Vậy,
theo hệ thức Newton ta có: . F DK L/ D 1. Suy ra A . F OM N / D 1. Từ đó, với chú ý
rằng A F k M N , theo định lý về chùm điều hòa ta suy ra được: OM D O N :
Ví dụ 12. AD ; BE ; C F là các đường cao của tam giác nhọn AB C . Đặt P D B C \ EF .
Đường thẳng qua D, song song với EF theo thứ tự cắt AB ; AC tại Q ; R. Chứng minh rằng
đường tròn ngoại tiếp của tam giác P QR đi qua trung điểm của B C .
204 Các phương pháp giải toán qua các kỳ thi Olympic

Lời giải. Gọi M là trung điểm của B C . Theo giả thiết BE \ C D BF \ C D 90 ı . Do đó,
bốn điểm B ; C ; E ; F cùng thuộc một đường tròn. Từ đó, với chú ý rằng QR k F E , suy ra
B ; C ; Q ; R cùng thuộc một đường tròn. Vậy, theo định lý về hệ thức lượng trong đường tròn
ta suy ra
DQ  DR D DB  D C : (13.3)
Mặt khác, theo định lý về hàng điểm điều hòa ta suy ra: . DP B C / D 1. Từ đó, với chú ý
rằng M B D M C , theo hệ thức Maclaurin:

DP  D C D DB  D C : (13.4)

Từ (13.3) và (13.4), theo định lý về hệ thức lượng trong đường tròn, suy ra đường tròn ngoại
tiếp của tam giác P QR đi qua trung điểm của B C .

Ví dụ 13. Cho tứ giác AB C D nội tiếp đường tròn . O /. AB ; AC ; AD theo thứ tự cắt
C D ; DB ; B C tại X ; Y ; Z . Chứng minh rằng O là trực tâm tam giác X Y Z .

Lời giải. Qua X , kẻ tới O các tiếp tuyến X M ; X N . Gọi P ; Q là giao điểm của M N với
A B ; C D: Suy ra: .XPAB / D . XQD C / D 1. Do đó: AD ; B C ; P Q đồng quy, suy ra

Z 2 P N: .1/

Mặt khác, ta lại có:


. XPAB / D . XQ C D / D 1:
Suy ra: A C ; BD ; P Q đồng quy ) Y 2 P N : (2)
Từ (1) và (2) suy ra: M N  Y Z . Do OX ?M N ) OX ?Y Z .
Chứng minh tương tự ta cũng được: O Z ? Y X : Vậy O là trực tâm tam giác X Y Z .
Hàng điểm điều hòa 205

Ví dụ 14. Cho tam giác AB C . Các điểm M ; N thuộc B C . Các điểm P ; Q theo thứ tự thuộc
A C ; A B . Đặt O D M P \ N Q ; K D BO \ N P ; L D C O \ M Q. Chứng minh rằng
AO ; B L ; C K đồng quy.

Lời giải. Đặt I D B L \ C K ; U D BO \ M Q ; V D C O \ N P . Ta có:


B . ALO C / D . QLM U /
D . M U LQ / (theo tính chất của tỉ số kép của hàng)
D . P K V N / (xét phép chiếu xuyên tâm O)
D C . AKOB / :

Từ đó suy ra: A; I ; O thẳng hàng. Vậy AO ; B L ; C K đồng quy.


Ví dụ 15. Cho tam giác AB C và điểm O nằm trong tam giác. BO ; C O theo thứ tự cắt
A C ; A B tại E ; F (EF không song song với B C /. I D AO \ EF . H là hình chiếu của I
trên B C . Chứng minh rằng AH
\ E D OH\ F.

Ý tưởng. Xem H là đỉnh của 1 chùm điều hòa trong đó có 2 đường thẳng vuông góc nhau.
Lời giải. Đặt S D EF \ B C ; T D SO \ B C . Ta có: . B C T S / D 1 (hàng điểm điều
hòa cơ bản). Suy ra: .F E I S / D 1 (qua phép chiếu xuyên tâm A/ và . AO T I / D 1 (qua
phép chiếu xuyên tâm F /. Do đó:
H .F E I S / D 1 H .AO T I / D 1:

H E D I\
Suy ra: I\ H A D I\
H F ; I[ H O (do H I ? H S , định lý về chùm điều hòa). Từ đây ta
có điều phải chứng minh.
206 Các phương pháp giải toán qua các kỳ thi Olympic

Ví dụ 16. Cho tam giác AB C . Các đường phân giác BE ; C F cắt nhau tại I (EF không
song song với B C /. Đường thẳng qua I , vuông góc với EF theo thứ tự cắt B C ; EF tại P ; Q.
Giả sử I P D 2I Q. Tính góc BAC
\.

Lời giải. Đặt K D EF \ B C ; R D AK \ P Q ; D D I A \ B C ; L D I A \ EF .


Ta có: . C BK D / D 1 (hàng điểm điều hòa cơ bản), suy ra . AI LD / D 1 (qua phép
chiếu xuyên tâm E /: Từ đây ta có . R I QP / D 1 (qua phép chiếu xuyên tâm K /: Suy ra:
RQ
RP
D II Q
P
D 12 và Q là trung điểm P R : Với kết quả này, ta có 4KP R cân tại K : Suy ra

AK
\ E D BK
\ E ) BAC
\ DK\AC : .1/
Mặt khác ta lại có: . B C DK / D 1 ) A . B C DK / D \ DC
1. Do BAD \AD nên
C
\ AK D x
\ AK : .2/
\ D 60 ı .
Từ (1) và (2) suy ra: BAC
Ví dụ 17. Cho hình thang AB C D . AB k C D / có B C D BD. Đường thẳng đối xứng
với C A qua C D theo thứ tự cắt AD ; BD tại E ; F . Chứng minh rằng E C D EF .
Hàng điểm điều hòa 207

Lời giải. Lấy K sao cho B là trung điểm của đoạn AK . Vì AK k D C và B C D BD nên
A K C D là hình thang cân. Suy ra: K
\ D C D AC
\ D.
Do A
\ CD D E
\ CD ) K
\ DC D E
\ C D. Do đó: DK k C E : (1)
Mặt khác vì D C k AK và BA D BK ) D . C BAK / D 1 (định lý về chùm điều
hòa). Do đó: D . C F E K / D 1: (2)
Từ (1) và (2) suy ra: E C D EF (định lý về chùm điều hòa).

3. Bài tập
Bài tập 67. Cho tam giác AB C cân tại A và b A D 120 ı . D thuộc đoạn B C sao cho
B C D 4 C D. K thuộc đoạn AB sao cho C K ?AC . Đường tròn ngoại tiếp tam giác ABD
cắt A C tại E khác A. Đường tròn ngoại tiếp tam giác K C D cắt AB tại F khác K . Đường
tròn ngoại tiếp tam giác ABD,K C D cắt nhau tại G khác D. Chứng minh rằng BE ; C F ; D G
đồng quy.

Bài tập 68. Cho tam giác AB C nội tiếp đường tròn .O /, các đường cao AA 0 ; BB 0 ; C C 0 .
A A 0 cắt . O / tại điểm thứ hai D. E thuộc A 0 B 0 sao cho BE ?OA. DB 0 cắt . O / tại điểm
thứ hai F . BF cắt AE tại K . Chứng minh rằng K là trung điểm B 0 C 0 .

Bài tập 69. Cho tứ giác AB C D nội tiếp đường tròn . O /. AC cắt BD tại E , AD cắt B C
tại F . Trung điểm của AB và C D lần lượt là G ; H . Chứng minh rằng EF là tiếp tuyến của
đường tròn ngoại tiếp tam giác E GH .

Bài tập 70. Cho tam giác nhọn AB C (AB < AC /. Gọi . I /, . O / lần lượt là các đường
tròn nội tiếp, ngoại tiếp tam giác AB C . Gọi H là chân đường cao kẻ từ A của tam giác AB C
và M là trung điểm của đoạn thẳng AH . Đường tròn .I / tiếp xúc với cạnh B C tại điểm D.
Đường thẳng M D cắt . I / tại điểm thứ hai P và đường thẳng qua I vuông góc M D cắt đường
thẳng B C tại điểm N . Từ điểm N kẻ các tiếp tuyến N R ; N S với đường tròn . O / (R ; S là
tiếp điểm). Chứng minh rằng các điểm R ; P ; D ; S cùng nằm trên một đường tròn.

Bài tập 71. Cho tam giác AB C nội tiếp đường tròn .O /. Đường tròn . I / nội tiếp tam giác,
tiếp xúc với các cạnh B C ; C A; AB lần lượt tại D ; E ; F . Gọi H là hình chiếu vuông góc của
D trên EF ; AH cắt đường tròn . O / tại điểm thứ hai G . Tiếp tuyến với đường tròn .O / tại G
cắt B C tại T . Chứng minh rằng tam giác T D G cân.

Bài tập 72 (KHTN vòng 3 năm 2011-2012, ngày thứ hai). Cho tứ giác lồi AB C D không là
hình thang nội tiếp đường tròn . O /. AD giao B C tại E . I là trung điểm C D. E I cắt đường
tròn ngoại tiếp tam giác E AB tại M khác E . AC giao BD tại F . EF cắt đường tròn ngoại
tiếp tam giác E AB tại N khác E . Chứng minh rằng bốn điểm C ; D ; N ; M cùng thuộc một
đường tròn.

Bài tập 73 (KHTN vòng 2 năm 2012-2013, ngày thứ nhất). Cho tam giác nhọn AB C . D là
một điểm thuộc đoạn AC . Giả sử đường tròn ngoại tiếp tam giác ABD cắt đoạn thẳng B C tại
E khác B . Tiếp tuyến tại B ; D của đường tròn ngại tiếp tam giác ABD cắt nhau tại T . AT
cắt đường tròn ngoại tiếp tam giác ABD tại F khác A. C F giao DE tại G . AG giao B C tại
H . M là trung điểm của AF . AE giao M D tại N . Chứng minh rằng H N k AT .
208 Các phương pháp giải toán qua các kỳ thi Olympic

Bài tập 74 (Vietnam TST, 2013). Cho tam giác AB C nhọn không cân có góc A bằng 45 ı .
Các đường cao AD ; BE ; C F đồng quy tại trực tâm H . Đường thẳng EF cắt đường thẳng
B C tại P . Gọi I là trung điểm của B C , đường thẳng I F cắt P H tại Q.

QH D AI
a) Chứng minh rằng I\ [ E:

b) Gọi K là trực tâm tam giác AEF và . J / là đường tròn ngoại tiếp tam giác KP D.
Đường thẳng C K cắt . J / tại G , đường thẳng I G cắt . J / tại M , đường thẳng J C cắt
đường tròn đường kính B C tại N . Chứng minh rằng các điểm G ; M ; N ; C cùng thuộc
một đường tròn.

Bài tập 75 (IMO Shortlist, 1995). Cho tam giác AB C có D ; E ; F lần lượt là tiếp điểm trên
B C ; C A ; AB của đường tròn nội tiếp tam giác. Gọi X là một điểm bên trong tam giác AB C
sao cho đường tròn nội tiếp tam giác XB C tiếp xúc với B C tại D, tiếp xúc với XB ; X C theo
thứ tự tại Y ; Z . Chứng minh E ; F ; Y ; Z đồng viên.

Bài tập 76 (China TST, 2002). Cho tứ giác lồi AB C D, gọi E ; F ; P lần lượt là giao điểm
của A D và B C ; AB và C D ; AC và BD. Gọi O là chân đường vuông góc hạ từ P xuống
\ D BO
EF . Chứng minh rằng AOD \ C:

Bài tập 77 (Balkan MO, 2007). Cho  AB C vuông tại A. D 2 AC và E đối xứng với A
qua BD, F là giao điểm của đường thẳng qua D vuông góc với B C và đường C E . Chứng
minh rằng AF ; DE ; B C đồng quy.

Tài liệu tham khảo


[1] Nguyễn Minh Hà (CB), Nguyễn Xuân Bình, Toán nâng cao hình học 10, NXB GD 2000.

[2] Đoàn Quỳnh (CB), Văn Như Cương, Trần Nam Dũng, Nguyễn Minh Hà, Đỗ Thanh Sơn,
Lê Bá Khánh Trình, Tài liệu chuyên toán hình học, NXB GD 2010.

[3] https://julielltv.wordpress.com

[4] https://phamquangtoan.wordpress.com

[5] Trần Quang Hùng, Tuyển tập các bài toán hình học chọn đội tuyển KHTN.
CÁC TAM GIÁC CÓ DẠNG AB C AC D kBC
Nguyễn Văn Linh
(ĐH Ngoại Thương, Hà Nội)

1. Mở đầu
Các bài toán về tam giác có điều kiện đặc biệt AB C AC D kBC .k D 2; 3/ đã xuất hiện trong
một số cuộc thi và tạp chí trên thế giới. Tuy nhiên, các tính chất liên quan đến nó còn khá ít và
đơn giản. Trong bài viết này, tác giả xin khai thác và tìm hiểu tính chất của hai dạng tam giác
AB C AC D 2BC , AB C AC D 3BC , đồng thời xem xét một số mở rộng trong trường hợp
tổng quát AB C AC D kBC .
Nếu không có chú thích gì thêm, chúng ta kí hiệu .O/; .I / lần lượt là đường tròn ngoại tiếp,
đường tròn nội tiếp của tam giác ABC:

2. Các tam giác có dạng AB C AC D 2BC


Tính chất 1. AI ? OI:
Lời giải. Cách 1.
A

F
O

B Ma C

Gọi J là điểm chính giữa cung BC không chứa A, Ma là trung điểm BC , F là tiếp điểm của
.I / với AB.
Ta có
AB C AC BC 2BC BC 1
AF D D D BC D BMa :
2 2 2
Đồng thời, ta có
∠IAF D ∠JBMa nên 4AF I D 4BMa J:

209
210 Các phương pháp giải toán qua các kỳ thi Olympic

Do đó, AI D BJ D IJ hay I là trung điểm AJ . Suy ra OI ? AJ:


Cách 2.
Sử dụng hình vẽ của cách 1. Áp dụng định lý Ptolemy cho tứ giác ABJ C; ta có

AB  J C C AC  JB D AJ  BC:

Do JB D J C D JI nên JI.AB C AC / D AJ  BC: Theo giả thiết AB C AC D 2BC nên


AJ D 2JI hay I là trung điểm AJ .
Suy ra OI ? AJ:

Tính chất 2. Gọi G là trọng tâm tam giác ABC . Khi đó IG k BC:

F
O
I G

B D K Ma C

Lời giải. Gọi K là giao của AI với BC . D là tiếp điểm của .I / với BC .
Theo lời giải 1 của tính chất 1 ta thu được Ma J D IF D ID nên theo định lý Thales, ta có
AI AG
KI D KJ . Do đó, IK D 2 D GM a
.
Suy ra IG k BC .

Tính chất 3. Đường tròn .I / tiếp xúc với BC; CA; AB lần lượt tại D; E; F . Gọi Mb ; Mc lần
lượt là trung điểm AC; AB: Các đường thẳng Mb Mc và EF cắt nhau tại L. Khi đó, đường tròn
.L; LI / tiếp xúc với .I / và .AO/.

Lời giải. Gọi Ma là trung điểm BC . Tia AI cắt .O/ tại K khác A.
Do ∠AIO D 90ı nên I 2 .AMb Mc /: Từ đây ta được EF là đường thẳng Simson của I ứng
với tam giác AMb Mc . Suy ra IL ? Mb Mc hay các điểm I; L; D thẳng hàng nên các điểm
A; L; Ma cũng thẳng hàng. Do đó, L là trung điểm Mb Mc :
Từ đó dễ dàng có
.L; LI / tiếp xúc với .AO/:
Do IA D IK; MK D r; nên IL D 21 r: Vậy đường tròn .L; LI / tiếp xúc với .I /.
Các tam giác có dạng AB C AC D kBC 211

E
Mc L
Mb

F O
I

B D Ma C

Tính chất 4. (Hidetosi Fukagawa, tạp chí CRUX năm 1988) Xét trường hợp tam giác ABC
nhọn. Kẻ AK ? BC . Ma là trung điểm BC . Dựng đường tròn .O1 / tiếp xúc với tia KA; KB
và tiếp xúc trong với .O/, đường tròn .O2 / tiếp xúc với tia KA; KC và tiếp xúc trong với .O/.
Khi đó tiếp tuyến chung trong thứ hai của .O1 / và .O2 / đi qua Ma .

T E O2

Mc Mb

F I O

R J
O1
Y
Z
B X K D Ma C

Lời giải. Gọi D; E; F là tiếp điểm của đường tròn .I / với BC; CA; AB, Mb ; Mc là trung điểm
AC; AB, J là tâm đường tròn Euler của tam giác ABC . Kéo dài AK cắt .O/ tại L:
212 Các phương pháp giải toán qua các kỳ thi Olympic

BC
Ta có AE D AF D 2
nên AE C AF D BC D AMb C AMc , suy ra EMb D FMc : Từ đó
4IEMb D 4IFMc suy ra IMb D IMc :
Mà JMb D JMc nên IJ là trung trực của Mb Mc hay trung trực của KM:
Gọi X; Y là tiếp điểm của .O1 / với BC; AH; Z; T là tiếp điểm của .O2 / với BC; AH:
Theo định lý Sawayama-Thebault, XY cắt ZT tại I và I 2 O1 O2 . Do ∠IXZ D ∠IZX D 45ı
nên IX D IZ. Mà IK D IM nên XK D ZMa . Đồng thời, ID là đường trung bình của hình
thang O1 XZO2 nên IO1 D IO2 . Kết hợp với ∠O1 KO2 D 90ı ta suy ra
IO2 D IO2 D IK D IMa hay ∠O1 Ma O2 D 90ı :
Từ Ma kẻ tiếp tuyến Ma R khác BC tới .O1 /. Suy ra, Ma O1 là phân giác ∠XMa R, từ đó Ma O2
là phân giác ∠RMa Z hay Ma R là tiếp tuyến của .O2 /.
Vậy Ma R là tiếp tuyến chung của .O1 / và .O2 /:

Tính chất 5. Gọi H là trực tâm. Khi đó H là tâm vị tự trong của .O1 / và .O2 /.
Lời giải. Trước tiên ta phát biểu một bổ đề.
Bổ đề 1. Cho hai đường tròn .O1 /; .O2 / ngoài nhau và cùng tiếp xúc trong với .O/. Gọi
AE; BF là hai tiếp tuyến chung trong của các đường tròn .O1 / và .O2 / sao cho A; B khác phía
với E; F bờ O1 O2 : Gọi CD là tiếp tuyến chung ngoài của .O1 / và .O2 / sao cho CD cùng phía
với EF bờ O1 O2 . Khi đó CD k EF:

O2

I1
T
I2 O

O1

C D

E L F

Chứng minh.
Gọi L là điểm chính giữa cung CD không chứa A; B. AL; BL cắt O1 O2 lần lượt tại I1 ; I2 . AE
cắt BF tại T:
Áp dụng định lý Thebault suy ra I1 ; I2 lần lượt là tâm nội tiếp các tam giác ACD; BCD. Suy ra
LI1 D LD D LI2 :
Từ đó ∠AI1 T D ∠BI2 T: Mà ∠BT I2 D ∠AT I1 nên ∠I2 BT D ∠I1 AT hay ∠LBF D
∠LAE. Suy ra L là điểm chính giữa cung EF . Vậy CD k EF:
Trở lại bài toán,
Các tam giác có dạng AB C AC D kBC 213

O2

Mc Mb
O
I

J
H
O1

B K D Ma C

L G

Gọi G là đối xứng của A qua O suy ra Ma là trung điểm HG. Từ đó LG k BC . Áp dụng bổ đề
1 suy ra HG là tiếp tuyến chung trong của .O1 / và .O2 /.
Vậy H là tâm vị tự ngoài của .O1 / và .O2 /:
Tiếp theo, chúng ta đến với một tính chất được tìm ra bởi thành viên mr.danh trên diễn đàn Toán
học nổi tiếng AoPS.
Tính chất 6. Gọi T; U; V là giao của tiếp tuyến chung ngoài khác BC của hai đường tròn
.O1 / và .O2 / với AH; AB; AC . Khi đó T U D T V .

T
S O2
U

H O
O1

B Ma C

L G

Lời giải. Gọi S là giao điểm thứ hai của HG với .O/. Áp dụng bổ đề 1 suy ra
AS k U V; GL k BC:
Suy ra G.BCMa L/ D 1, chiếu lên .O/ ta thu được .BCSL/ D 1, từ đó A.BCSL/ D 1
hay A.U V ST / D 1.
Vậy T là trung điểm U V .
214 Các phương pháp giải toán qua các kỳ thi Olympic

Nhận xét. Bằng cách sử dụng tỉ số kép, ta có thể mở rộng tính chất 6 như sau.
Cho tam giác ABC nội tiếp đường tròn .O/, M là điểm bất kì trên BC . Gọi .O1 /; .O2 / là
đường tròn Thebault nội tiếp của tam giác ABC ứng với cát tuyến AM . Gọi d là tiếp tuyến
chung trong khác AM của .O1 / và .O2 /. d cắt .O/ tại D sao cho A và D cùng thuộc một nửa
mặt phẳng bờ BC . DB; DC; d cắt tiếp tuyến chung ngoài thứ hai khác BC của .O1 / và .O2 /
tại U; V; T . Khi đó, ta có hệ thức
TU MC
D
TV MB

Tính chất 7. Đường tròn đường kính AO và đường tròn đường kính HMa tiếp xúc nhau.

Lời giải. Cách 1. (Xem hình vẽ tính chất 5)


Theo tính chất 5 ta suy ra HI là phân giác ∠AHMa hay HI là phân giác ngoài ∠KHMa . Mà
I nằm trên trung trực KM nên HIMa K nội tiếp.
Do I nằm trên trung trực KMa nên I nằm trên đường trung bình của hình thang AKMa O hay
đường thẳng đi qua trung điểm AO và HMa .
Vì thế nên tâm của .AO/; .HMa / và I thẳng hàng.
Suy ra .AO/ tiếp xúc với .HMa / tại I:
Cách 2 (Dựa theo Telv Cohl).

O
I G
H N

B Ma C

Gọi G là trọng tâm tam giác ABC , N là giao của IG với OMa . Ta có NMa D r D Ma J . Áp
dụng định lý Menelaus cho tam giác I NJ với cát tuyến .A; G; Ma / ta có
AI Ma J GN
  D 1:
AJ Ma N GI
GN 2GH
Do đó, GI
D GO
: Ta thu được IO k HN:
Các tam giác có dạng AB C AC D kBC 215

Mà AI ? OI nên AI ? HN . Đồng thời NI ? AH nên I chính là trực tâm của tam giác
AHN . Suy ra HI ? AN:
Lại có IMa là đường trung bình của tam giác AJN nên IMa k AN . Suy ra HI ? IMa hay
I 2 .HMa /. Do I nằm trên đường trung bình của hình thang AOMa H nên I nằm trên đường
nối hai tâm của .AO/ và .HMa /.
Ta có đpcm.

Tính chất 8. Tứ giác AOMa H là hình thang ngoại tiếp.

Lời giải. Tiếp tục sử dụng hình vẽ tính chất 5.


Ta có ∠IMa O D ∠IKH D ∠IMa H nên Ma I là phân giác ∠OMa H . Như vậy I là giao của
phân giác các góc HAO; AHMa ; HMa O nên I là tâm nội tiếp hình thang AHMa O.

Nhận xét. Từ tính chất 8, áp dụng định lý Pythot, bạn đọc có thể chứng minh tam giác AGH có
AG C HG D 3AH:

Tính chất 9. Gọi J là giao của AI với .O/. Khi đó điểm Nagel là đối xứng của J qua BC:

Lời giải. Trước tiên ta phát biểu một bổ đề.


Bổ đề 2. Tâm đường tròn nội tiếp, trọng tâm và điểm Nagel thẳng hàng. Đường thẳng đi qua
ba điểm này là đường thẳng Nagel.
Chứng minh.
A

Mc Mb

I G
N

C
B D Ma L

Gọi D là tiếp điểm của đường tròn nội tiếp .I / với BC; DI cắt .I / lần thứ hai tại T , AT cắt
BC tại L; gọi Ma ; Mb ; Mc lần lượt là trung điểm BC; CA; AB; N là điểm Nagel của tam giác
ABC:
Ta biết rằng Ma là trung điểm DL nên Ma I k AL hay Ma I k AN . Chứng minh tương tự,

Mb I k BN; Mc I k CN:

Do đó I là điểm Nagel của tam giác Ma Mb Mc :


IG
Do G là tâm vị tự của hai tam giác ABC và Ma Mb Mc nên I; G; N thẳng hàng và GN
D 21 :
Trở lại bài toán,
216 Các phương pháp giải toán qua các kỳ thi Olympic

O
G
I
N

B D Ma P C

Gọi N là điểm Nagel, AN cắt .I / tại L, cắt BC tại P . Theo tính chất 2, IG k BC , từ đó áp
dụng bổ đề 2 suy ra I N k BC .
Do I là trung điểm LD do đó N là trung điểm LP . Lại có Ma là trung điểm DP nên NMa k ID
hay NMa ? BC và NMa D r.
Theo phép chứng minh tính chất 1, JMa D r nên J và N đối xứng với nhau qua BC .

Tính chất 10. Gọi Ia là tâm bàng tiếp góc A. Đường tròn .A; AIa / cắt BC tại E; F sao cho
E thuộc tia CB, F thuộc tia BC . Đường tròn .EBIa / cắt AB tại M , .F CIa / cắt AC tại N .
Khi đó tứ giác BCNM là tứ giác lưỡng tâm (là tứ giác vừa nội tiếp, vừa ngoại tiếp).

O
I
E B K C F

Ia

M
Các tam giác có dạng AB C AC D kBC 217

Lời giải. Gọi E 0 là giao của đường tròn qua A; B và tiếp xúc với AIa với BC; K là giao của
AIa với BC; J là giao của AI với .O/:
Ta có AI D IJ D JIa ; AI D 2IK nên AK D KIa . Từ đó
KIa2 D KA2 D KB  KE 0 ;
nghĩa là .E 0 BIa / tiếp xúc với AIa .
Như vậy
∠AE 0 Ia D ∠AE 0 K C ∠KE 0 Ia D ∠BAI C ∠BIa I D ∠BAI C ∠BCI:
Mà ∠E 0 AIa D ∠ABC nên ∠E 0 AIa D 180ı 2∠AE 0 Ia hay tam giác AE 0 Ia cân tại A. Từ đó
suy ra hai điểm E 0 ; E trùng nhau.
Suy ra
∠BIa M D 180ı ∠MBIa ∠BMIa D 180ı .90ı ∠IBC / ∠BIa I
D 90ı C ∠IBC ∠ICB:
Chứng minh tương tự,
∠CIa N D 90ı C ∠ICB ∠IBC:
ı
Suy ra ∠BIa M C ∠CIa N D 180 hay tứ giác BCNM ngoại tiếp.
Mặt khác, AIa là tiếp tuyến chung của .BIa M / và .CIa N / nên AB  AM D AIa2 D AC  AN
hay tứ giác BCNM nội tiếp. Ta có đpcm.
Để kết thúc phần này chúng ta đến với một bài toán nằm trong đề chọn đội tuyển IMO của Thổ
Nhĩ Kỳ năm 2015.
Bài toán. Cho tam giác ABC nội tiếp đường tròn .O/, ngoại tiếp .I /. .I / tiếp xúc với
BC; CA; AB lần lượt tại D; E; F . Gọi E1 ; F1 lần lượt đối xứng với A qua E; F . Đường tròn
!1 đi qua E1 tiếp xúc với .I / tại D cắt AC lần thứ hai tại E2 ; đường tròn !2 đi qua F1 tiếp
xúc với .I / tại D cắt AB tại F2 . Gọi P; Q lần lượt là trung điểm OE; IF . Chứng minh rằng
E2 F2 ? PQ khi và chỉ khi AB C AC D 2BC:

E3

E2

E
Q P
F O E1
I
B
D C
F1

F2
218 Các phương pháp giải toán qua các kỳ thi Olympic

Lời giải. AB C AC D 2BC khi và chỉ khi AI ? OI hay OI k EF hay OI k PQ. Như vậy
ta sẽ chứng minh E2 F2 ? OI:
Gọi E3 là giao của DF với AC . Gọi E20 là trung điểm E3 E. Do .E3 EAC / D 1 nên sau một
số phép tính toán suy ra CE 2 D CE1  CE20 hay .E20 E1 D/ tiếp xúc với .I / hay E20  E2 :
Theo hệ thức Newton, E2 E 2 D E2 A:E2 C hay E2 thuộc trục đẳng phương của .O/ và .I /.
Chứng minh tương tự suy ra E2 F2 ? OI:

Nhận xét. Theo tác giả bài toán này là sự kết hợp cơ học giữa một tính chất của tam giác bất kì
với tính chất 1. Do đó đề thi tuy nhìn đẹp mắt nhưng không phải một đề thi hay.

3. Các tam giác có dạng AB C AC D 3BC


Tính chất 11. (IMO Shortlist 2005). Đường tròn .I / tiếp xúc với AC; AB lần lượt tại E; F .
Gọi L; K là điểm đối xứng với E; F qua I . Khi đó B; C; K; L cùng thuộc một đường tròn.

O E
F
I
K
L

B C

Lời giải. Gọi J là điểm chính giữa cung BC không chứa A. Áp dụng định lý Ptolemy cho tứ
giác ABJ C ta có

AB  J C C AC  JB D AJ  BC hay JI.AB C AC / D AJ  BC:

Do AB C AC D 3BC nên AJ D 3JI . Gọi M là trung điểm AI thì MI D JI . Mà F I D KI


nên
JI D MI D FM D JK:
Suy ra K 2 .BIC /. Tương tự suy ra B; C; K; L cùng thuộc một đường tròn.

Tính chất 12. Gọi G là trọng tâm tam giác ABC . Khi đó IG ? BC:
Các tam giác có dạng AB C AC D kBC 219

AI AG AI
Lời giải. Theo lời giải tính chất 11, IJ
D 2. Gọi Ma là trung điểm BC thì GMa
D2D IJ
.
Suy ra IG k JMa hay IG ? BC .
Tính chất 13. (Iran 2004). Đường tròn .I / nội tiếp tam giác ABC tiếp xúc với AB; AC tại
P; Q: BI; CI giao PQ lần lượt tại L; K. Khi đó .ILK/ tiếp xúc với .I /.
Lời giải. Cách 1.

K
Y P
L
Q

X I

B D C

Gọi D là tiếp điểm của .I / với BC . AI giao .ABC / tại J: Ta có .ILK/ tiếp xúc với .I / khi và
chỉ khi R.ILK/ D 2r :
Dễ thấy BL ? LC; CK ? BK nên L; K nằm trên .BC /. Bằng cộng góc suy ra I là tâm nội
tiếp tam giác DLK. Kẻ IX ? LD; I Y ? PQ.
Do hai tam giác ILK và IBC đồng dạng nên
R.ILK/ IY IX
D D D sin ∠IDX D sin A=2:
R.IBC / ID ID
Suy ra,
R.ILK/ IP r
D D :
IJ AI AI
Do đó, R.ILK/ D 2r khi và chỉ khi IP D 2IJ hay JA D 3JB D 3J C .
Theo lời giải tính chất 11, điều này tương đương AB C AC D 3BC:
Cách 2.
Gọi R; T là giao của CK với .I /. Ta có TPRD là tứ giác điều hoà. Gọi M là giao của IK với
DP thì do TR ? IM nên M T; MR là hai tiếp tuyến của .I /. Suy ra IK  IM D r 2 :
Tương tự gọi N là giao của IL với DQ thì IL  I N D r 2 : Dễ thấy M; N lần lượt là hình chiếu
của A trên IB; IC và MN là đường trung bình ứng với đỉnh A của tam giác ABC .
Xét phép nghịch đảo
2
IIr W .I / 7! .I /; .ILK/ 7! MN
Do đó, .ILK/ tiếp xúc với .I / khi và chỉ khi .I / tiếp xúc với đường trung bình EF của tam
giác ABC .E 2 AC; F 2 AB/ hay .I / là đường tròn bàng tiếp góc A của tam giác AEF .
220 Các phương pháp giải toán qua các kỳ thi Olympic

F T E M
N
K P
L

Q I R

B D C

Khi đó AP D AQ D pAEF D 12 pABC , khi và chỉ khi


AB C AC BC 1
D .BC C CA C AB/
2 4
hay AB C AC D 3BC:
Tính chất 14. Đường tròn .I / tiếp xúc với BC; CA; AB lần lượt tại D; E; F . Gọi L; K lần
lượt đối xứng với E; F qua I . T là giao điểm thứ hai của đường tròn đường kính AI với .O/.
Khi đó .LK T / tiếp xúc với .O/:
Lời giải. Theo tính chất 11, L; K 2 .BIC /:
Kéo dài AT cắt BC tại P suy ra PI ? AI . Suy ra ∠TDP D ∠T IP D ∠TAD D ∠.TJ; BC /,
suy ra T; D; J thẳng hàng.
A

T
X

O
E
I K
F
L

P
B D C

2
Xét phép nghịch đảo IJJI : L 7! L; K 7! K; T 7! D: Do đó .T LK/ 7! .I /. Mà .O/ 7! BC ,
BC tiếp xúc với .I / nên .T LK/ tiếp xúc với .O/:
Các tam giác có dạng AB C AC D kBC 221

Nhận xét. Có một kết quả khá đẹp nằm trong kì thi quốc gia Nga năm 2010 như sau, mời bạn
đọc thử chứng minh.
Cho tam giác ABC nội tiếp đường tròn .O/, ngoại tiếp đường tròn .I /. Đường tròn ngoại tiếp
tam giác BIC cắt .I / tại hai điểm X; Y . Gọi Z là tâm vị tự ngoài của .I / và .BIC /. Chứng
minh rằng .XY Z/ tiếp xúc với .O/.
Tính chất 15. Đường tròn .I / tiếp xúc với BC tại D: Gọi T là điểm đối xứng với D qua I .
BT; C T cắt AC; AB lần lượt tại M; N . X là trung điểm AI . Khi đó X là tâm đường tròn ngoại
tiếp tam giác TMN:
A

X M
T

O
E
I
F
L K

D
B C

Lời giải. Ta có
∠BLI D 180ı ∠ICB; ∠T LE D ∠TDE D ∠ICB
nên B; L; T thẳng hàng. Tương tự C; K; T thẳng hàng. Suy ra M; N là tiếp điểm của đường
tròn bàng tiếp góc B; C với AC; AB.
Ta có
1
∠M T N D ∠BT C D ∠EDF D 90ı ∠BAC:
2
Mặt khác, AM D CE D p c và
AC C BC AB
NF D AB 2BF D AB .AB C BC AC / D AC BC D Dp c:
2
Suy ra AM D NF . Từ đó 4AXM D 4FXN.c:g:c/. Suy ra ∠F NX D ∠AMX hay tứ giác
AM XN nội tiếp.
Mà AX là phân giác ∠NAM nên XM D XN; đồng thời
∠NXM D 180ı ∠BAC D 2∠M T N
nên X là tâm ngoại tiếp tam giác M T N:
222 Các phương pháp giải toán qua các kỳ thi Olympic

Từ lời giải tính chất 15, ta suy ra kết quả sau.


Tính chất 16. Điểm đối xứng với tiếp điểm của .I / với BC qua I là điểm Nagel.
Tính chất 17. Gọi Ia là tâm bàng tiếp góc A. Trên BC lấy hai điểm P; Q sao cho CP D
CA; BQ D BA và theo thứ tự P; B; C; Q. Đường tròn .PBIa / cắt AB lần thứ hai tại M , đường
tròn .QCIa / cắt AC lần thứ hai tại N . Khi đó BCNM là tứ giác lưỡng tâm.

Q
P B D C

Ia

N
M

Lời giải. Gọi D là điểm chính giữa cung BC . Dễ thấy IA D IQ D IP: Từ giả thiết AB C
AC D 3BC suy ra AI D 2ID hay IA D IIa : Vậy APIa Q nội tiếp đường tròn tâm I . Bằng
một số phép cộng góc đơn giản suy ra AICQ; AIBP nội tiếp.
Ta có

∠PMB D ∠PIa B D ∠PIa A ∠BIa A D ∠AQB ∠ICB D ∠Ia IC ∠ICA D ∠IAC:

Suy ra
1
∠BIa M D 180ı∠BPM D ∠PBM C ∠PMB D ∠B C ∠A:
2
1
Tương tự ∠CIa N D ∠C C 2 ∠A. Ta thu được

∠BIa M C ∠CIa N D 180ı :

Suy ra MN tiếp xúc với .Ia / hay tứ giác BCNM ngoại tiếp.
Mặt khác, ∠CNM D 2∠CNIa D 2∠PQIa D 2∠PAI D 2∠IBC D ∠ABC hay tứ giác
BCNM nội tiếp. Ta có đpcm.
Nhận xét. Từ lời giải tính chất 17 ta nhận thấy I là trung điểm AIa . Do đó bán kính đường tròn
nội tiếp bằng một nửa bán kính đường tròn bàng tiếp góc A.
Tiếp theo chúng ta đến với một tính chất được tìm ra độc lập bởi tác giả. Khá thú vị là bài toán
từng nằm trong cuộc thi hình học Sharygin của Nga năm 2014, bài 9.6.
Các tam giác có dạng AB C AC D kBC 223

Tính chất 18. Gọi Y; Z lần lượt là điểm chính giữa các cung ABC; ACB. Khi đó Y Z là tiếp
tuyến của đường tròn .BIC /:

Ib

X
A

Ic

O E
Z
I
F
K
Y L
D
B C

Ia

Lời giải. Gọi Ia ; Ib ; Ic lần lượt là tâm bàng tiếp các góc A; B; C của tam giác ABC I D; E; F
lần lượt là tiếp điểm của .I / với BC; AC; AB; T; L; K lần lượt đối xứng với D; E; F qua I ; X
là điểm chính giữa cung BAC . Khi đó XY Z là tam giác trung tuyến của tam giác Ia Ib Ic :
Do ∠TBI D ∠LBI D 90ı 21 ∠LIK D 90ı ∠BT C nên BI ? T C: Từ đó I là trực tâm
tam giác BT C:
Ta có BI ? Ia Ic nên BI ? XZ. Mà XZ ? ZJ (do JX là đường kính của .O/) nên IB k J Z.
Từ đó J Z ? KC . Vậy ZK D ZC:
Mặt khác, dễ thấy EK k AI k FL. Do đó ∠KEC D ∠JAC D ∠J ZC D 12 ∠KZC . Như vậy
Z là tâm ngoại tiếp của tam giác KEC . Tương tự Y là tâm ngoại tiếp của tam giác FLB.
Suy ra IZ là trung trực của EK, I Y là trung trực của FL. Mà EK k FL nên Y; I; Z thẳng
hàng đồng thời Y Z ? JI . Vậy Y Z là tiếp tuyến của .BIC /:

Tính chất 19. Gọi X; Y; Z lần lượt là điểm chính giữa cung BAC; ABC; ACB; A0 ; B 0 ; C 0
lần lượt là điểm đối xứng với A qua X; B qua Y; C qua Z. Khi đó tiếp tuyến tại B 0 của .Y; YB/
cắt tiếp tuyến tại C 0 của .Z; ZC / tại A0 .

Lời giải. Gọi R là giao của đường thẳng qua Y vuông góc với BB 0 và đường thẳng qua Z vuông
góc với C C 0 .
Ta có YR k J Z; ZR k YJ nên YRZJ là hình bình hành. Suy ra R đối xứng với J qua trung
điểm Y Z. Gọi S là trung điểm AI suy ra S đối xứng với J qua Y Z. Suy ra SRZY là hình
thang cân.
Ta thu được RS ? AI nên RA D RI .
Gọi A00 là điểm đối xứng với I qua R. Suy ra RA D RI D RA00 hay AA00 ? AI hay
AA00  Ib Ic .
224 Các phương pháp giải toán qua các kỳ thi Olympic

Lại có .O/ là đường tròn pedal của I ứng với tam giác Ia Ib Ic , do đó R là điểm liên hợp đẳng
giác của I trong tam giác Ia Ib Ic : Suy ra RX ? Ib Ic hay RX ? AA00 . Như vậy XA D XA00 ,
nghĩa là A00 trùng với A0 :

Ib
A'
X
A

Ic

C'
R
S

O
B'
I Z

Y
B C

Ia

Theo định lý Thales, A0 B 0 k RY k IB, A0 C 0 k RZ k IC . Ta có đpcm.

Nhận xét.

1. Theo tính chất đường tròn pedal, O là trung điểm IR: Như vậy chúng ta thu được kết quả:
đường thẳng OI của tam giác ABC đi qua điểm đối xứng với A qua X.

2. Từ hình vẽ ta phát hiện ra một tính chất thú vị đúng với tam giác bất kì: điểm Nagel là
tâm đẳng phương của các đường tròn .X; XA/; .Y; YB/; .Z; ZC /: Tác giả Trần Quang
Hùng cũng tìm ra tính chất này tại [8].

DB IY
3. Gọi D là tiếp điểm của .I / với BC , ta có thể chứng minh DC D YZ
, từ đó áp dụng định
0 0
lý ERIQ suy ra điểm Nagel của tam giác ABC nằm trên B C :

Tính chất 20. Kẻ AK ? BC . Giả sử .O1 / tiếp xúc với tia KA; KB và tiếp xúc trong với .O/,
.O2 / tiếp xúc với tia KA; KC và tiếp xúc trong với .O/. L là tiếp điểm của đường tròn bàng
tiếp góc A với BC . Khi đó tiếp tuyến chung trong khác AK của .O1 / và .O2 / đi qua L.
Các tam giác có dạng AB C AC D kBC 225

M G T N

T E
F O2
I

O1 Y
X Z
B K D L C

Lời giải. Gọi D; E; F lần lượt là tiếp điểm của .I / với BC; CA; AB: DI cắt .I / lần thứ hai
tại T . Qua T kẻ đường song song với BC cắt AB; AC tại M; N:

Ta biết rằng M; N lần lượt là trung điểm của AB; AC . Mà A; T; L thẳng hàng nên T là trung
điểm AL. Kéo dài LI cắt AK tại G suy ra I là trung điểm GL: Từ đó ta được

IK D IG D IL:

Mặt khác, gọi X; Y lần lượt là tiếp điểm của .O1 / với BC; AK; Z; T lần lượt là tiếp điểm
của .O2 / với BC; AK. Theo định lý Sawayama-Thebault, XY cắt ZT tại I . Mà ∠IXZ D
∠IZX D 45ı nên tam giác XIZ vuông cân tại I , từ đó IX D IZ:

Chứng minh hoàn toàn tương tự tính chất 4 suy ra L nằm trên tiếp tuyến chung trong thứ hai của
.O1 / và .O2 /:

4. Trường hợp tổng quát AB C AC D kBC

Quay lại một số tính chất trong phần 1 và 2. Với tính chất 5, tác giả đã thử kiểm tra trong trường
hợp k D 3 thì tâm vị tự trong L của .O1 / và .O2 / có tính chất gì đặc biệt. Một điều khá thú vị là
khi đó trực tâm H là trung điểm của KL. Từ đó chúng ta có mở rộng trong trường hợp tổng quát
k là số thực bất kì lớn hơn 1 như sau.

Tính chất 21. Cho tam giác ABC có AB C AC D kBC , trực tâm H . Kẻ AK ? BC . Gọi
.O1 / và .O2 / là hai đường tròn Thebault nội tiếp của tam giác ABC ứng với cát tuyến AK; L
là tâm vị tự trong của .O1 / và .O2 /: Khi đó KH
KL
D k11:
226 Các phương pháp giải toán qua các kỳ thi Olympic

O2

N
H I
O
O1
L

B X K M Z C

Q G
J

Lời giải. Gọi Q là giao của AK với .O/. AI cắt .O/ lần thứ hai tại J . Tiếp tuyến chung trong
khác AQ của .O1 / và .O2 / cắt BC tại M , cắt .O/ tại G. Qua M kẻ đường vuông góc với BC
cắt AO tại N . .O1 /; .O2 / tiếp xúc với BC lần lượt tại X; Z:
Ở phần 1 ta đã chứng minh I là trung điểm O1 O2 : Từ đó IX D IZ, suy ra IK D IM:
Gọi R là giao của trung trực KM với AN , suy ra RA D RN .
Dễ thấy 4ARI  4AOJ nên RA D RI:
Suy ra ∠AI N D 90ı :
Mặt khác, ta cũng chứng minh được QG k BC , do đó AG và AQ đẳng giác trong ∠BAC hay
AG là đường kính của .O/. Từ đó ∠AJ G D 90ı . Như vậy I N k J G: Do đó

KH QK GM GN JI
D D D D :
KL KL ML NA IA
Áp dụng định lý Ptolemy cho tứ giác ABJ C ta có

JB.AB C AC / D AJ  BC:
JI 1
Từ đó AJ D kJB hay IA
D k 1
: Ta có đpcm.

Nhận xét. Từ lời giải trên dễ thấy I nằm trên .LM /: Từ đó chúng ta có tổng quát cho tính chất
7 như sau.

Tính chất 22. Cho tam giác ABC có AB C AC D kBC nội tiếp .O/, trực tâm H . Kẻ
AK ? BC , L là điểm nằm trên AK thỏa mãn KH
KL
D k 1 1 . Gọi G là điểm đối xứng với A qua
O, LG cắt BC tại M . Qua M kẻ đường vuông góc với BC cắt AO tại N . Khi đó hai đường
tròn .LM / và .AN / tiếp xúc nhau.

Tiếp theo chúng ta nhận thấy tính chất 9 và tính chất 16 có điểm khá tương đồng nhau, từ đó ta
có bài toán tổng quát.
Các tam giác có dạng AB C AC D kBC 227

Tính chất 23. Cho tam giác ABC có AB C AC D kBC . Đường tròn nội tiếp .I / tiếp xúc
với BC tại D. Gọi T là điểm đối xứng với D qua I; N là điểm Nagel. Khi đó NA
TA
D k21:

N
I

B D Ma C

Lời giải. Gọi J là giao của AI với .O/, Ma là trung điểm BC , AMa cắt I T tại L:
Theo bổ đề 2 trong phần 1, AN k IMa và AN D 2IMa . Suy ra

NA 2Ma I 2Ma L 2JI 2


D D D D :
TA TA LA IA k 1

Tính chất 10 và tính chất 17 cũng có những sự tương đồng. Từ đó dẫn đến mở rộng sau với lời
giải tương tự hai tính chất trên.

Tính chất 24. Cho tam giác ABC có Ia là tâm đường tròn bàng tiếp góc A. Đường tròn qua
B và Ia tiếp xúc với AIa cắt AB tại M . Đường tròn qua C và Ia tiếp xúc với AIa cắt AC tại
N . Khi đó BCNM là tứ giác lưỡng tâm.

Cuối cùng dựa vào tính chất J là trung điểm IIa , ta có tính chất sau trong trường hợp tổng quát.

Tính chất 25. Cho tam giác ABC có AB C AC D kBC . Gọi r và ra là bán kính đường tròn
nội tiếp và đường tròn bàng tiếp góc A. Khi đó rra D kC1
k 1
:

Qua bài viết, chúng ta đã tìm hiểu các tính chất cơ bản và thú vị xung quanh tam giác có dạng
đặc biệt này. Trong khuôn khổ có hạn, bài viết xin được dừng tại đây. Chắc chắn vẫn còn nhiều
tính chất mới lạ đang đợi bạn đọc khám phá. Chúc các bạn thành công!

Tài liệu tham khảo


[1] aloski1687, Turkey TST 2015 P8, AoPS forum.
http://www.artofproblemsolving.com/community/
q1h1072730p4675804
228 Các phương pháp giải toán qua các kỳ thi Olympic

[2] mr.danh, b+c=2a, AoPS forum.


http://www.artofproblemsolving.com/community/
q1h210518p1159893
http://www.artofproblemsolving.com/community/
q1h213165p1176583

[3] livetolove212, (HM) is tangent to (AO), AoPS forum.


http://www.artofproblemsolving.com/community/
q4h1086462p4806122

[4] Omid Hatami, Circumcircle of ILK is tangent to ABC iff AB+AC=3BC, AoPS forum.
http://www.artofproblemsolving.com/community/
q2h250154p1370723

[5] livetolove212, AB+AC=3BC again, AoPS forum.


http://www.artofproblemsolving.com/community/
c6h1086458p4806074

[6] Geometrical Olympiad in honour of I.F.Sharygin, final round.


http://geometry.ru

[7] livetolove212, Bicentric quadrilateral, AoPS forum.


http://www.artofproblemsolving.com/community/
c6h1086456p4806065

[8] buratinogigle, Nagel point is radical center, AoPS forum.


http://www.artofproblemsolving.com/community/c6h521011

[9] Dušan Djukić, Vladimir Janković, Ivan Matić, Nikola Petrović, The IMO Compendium: A
Collection of Problems Suggested for International Mathematical Olympiads 1959-2009,
Springer; 2nd edition (May 16, 2011).
http://www.cs.elte.hu/~nagyzoli/compendium.pdf
VẺ ĐẸP BÀI TOÁN
THI OLYMPIC HUNGARY 2015
Nguyễn Ngọc Giang
(Thành phố Hồ Chí Minh)

Kì thi Olympic Hungary 2015 năm nay có bài toán hình học tuy đơn giản nhưng khá hay. Sở dĩ
bài toán này hay là vì thứ nhất, nó có tính phát triển rất cao. Thứ hai, bản chất đằng sau bài toán
là định lí hình học xạ ảnh nổi tiếng – Định lí Steiner.

1. Bài toán Olympic Hungary 2015


Bài toán 1. Cho đường tròn .O/: QE; QF là các tiếp tuyến. Đường thẳng Qe cắt đường tròn
tại M và N: EC song song với P e và cắt .O/ tại C .khác A/. Chứng minh rằng dây F C chia
đôi dây MN .
Lời giải sau được Nguyễn Văn Lợi [1] đưa lên diễn đàn BÀI TOÁN HAY – LỜI GIẢI ĐẸP –
ĐAM MÊ TOÁN HỌC.
F

N
e
P
M

Q
O

Lời giải. Gọi P là trung điểm của MN: Ta sẽ chứng minh F; P; C thẳng hàng. Ta có OP là
đường trung trực của MN và AC nên ta có
\ D CPN
EPM \: .1/
Ngũ giác QFPOE nội tiếp nên ta có
\ D EOQ
EPQ \ D FOQ
\ D FPQ:
\ .2/
\ D CPN
Từ (1) và (2) ta có FPQ \: Hay F; P; C thẳng hàng.

229
230 Các phương pháp giải toán qua các kỳ thi Olympic

2. Khai thác và phát triển bài toán


Thực hiện phép chiếu xuyên tâm từ điểm O nằm ngoài mặt phẳng tờ giấy xuống mặt phẳng
không song song với mặt phẳng tờ giấy thế thì đường tròn không còn là đường tròn nữa mà trở
thành thiết diện cônic (elip, hypebol, parabol).
Cũng vậy, trung điểm của MN cũng không còn là trung điểm mà nó cùng với điểm vô tận chia
điều hòa hai điểm M; N ; hai đường thẳng song song trở thành hai đường thẳng cắt nhau tại điểm
ở vô tận. Thế thì từ bài toán 1, ta thu được bài toán sau:
Bài toán 2. Cho thiết diện cônic .S/ và hai điểm E; F trên .S/. Qua E và F lần lượt dựng các
tiếp tuyến d; d 0 với .S/ cắt nhau tại Q: Qua Q dựng đường thẳng Qe cắt cônic .S/ tại M; N:
P; P 0 là các điểm trên Qe sao cho .P 0 ; P; M; N / D 1 và P 0 E \ .S/ D C: Chứng minh rằng
ba điểm F; P; C thẳng hàng.
Từ bài toán 2, chúng tôi đề xuất ra bài toán tổng quát như sau:
Bài toán 3. Cho cônic .S/ và hai điểm E; F trên .S/. Qua E và F lần lượt dựng các tiếp
tuyến d; d 0 với .S/ cắt nhau tại Q: Qua Q dựng đường thẳng Qe cắt cônic .S/ tại M; N: P; P 0
là các điểm trên Qe sao cho .P 0 ; P; M; N / D 1 và P 0 E \ .S/ D C: Chứng minh rằng ba
điểm F; P; C thẳng hàng.
Sau đây là lời giải bằng phương pháp cực và đối cực của Quang Dương [2] trên diễn đàn BÀI
TOÁN HAY – LỜI GIẢI ĐẸP – ĐAM MÊ TOÁN HỌC.
F
N e

Q' P
M
Q
P'
O

E
C

Lời giải. Xét chùm .E/ ta có


.E; F; M; N / D E.E; F; M; N / D .Q; Q0 ; M; N /: .1/
Do EF là đường đối cực của điểm Q đối với cônic .S/ nên theo tính chất cực và đối cực ta có
.Q; Q0 ; M; N / D 1: .2/
Mặt khác, ta lại có
.E; F; M; N / D C.E; F; M; N / D .P 0 ; P; M; N /: .3/
Từ (1), (2), (3) ta có .P 0 ; P; M; N / D 1. Hay ta có điều phải chứng minh.
Vẻ đẹp bài toán thi Olympic Hungary 231

Từ cách giải này, ta thấy rằng để chứng minh bài toán 1 ta có thể tách thành hai ý chứng minh:

 Ý thứ nhất. Chứng minh .EQ; EF; EM; EN / là chùm điều hòa.

 Ý thứ hai. Chứng minh tỉ số kép .CE; CF; CM; CN / không đổi khi C thay đổi.

Khi chứng minh được hai ý này thì ta dễ dàng suy ra lời giải bài toán. Thật vậy theo ý thứ hai,
.CE; CF; CM; CN / D .EQ; EF; EM; EN / và theo ý thứ nhất .EQ; EF; EM; EN / D 1.
Vậy ta có .CE; CF; CM; CN / D 1: Do MN k CE nên rõ ràng F C chia đôi dây MN:
Ứng với hai ý này là hai bài toán cần phải chứng minh sau:

Bài toán 4. Cho đường tròn (O/ và điểm Q ở ngoài đường tròn. Vẽ các tiếp tuyến xuất phát
từ Q tiếp xúc với đường tròn ngoại tiếp tại E và F: Vẽ cát tuyến QMN tới đường tròn. Chứng
minh rằng .EQ; EF; EM; EN / là chùm điều hòa.

Bài toán 5. Cho bốn điểm E; F; M; N thuộc đường tròn .S/: Chứng minh rằng với một điểm
C bất kì thuộc .S/ thì tỉ số kép .CE; CF; CM; CN / không đổi.

Bài toán 4 là bài toán hay và bổ ích, có nhiều cách giải. Sau đây là 9 cách giải của nó.

K
M

Q
C H O B

Cách 1. Vẽ cát tuyến QCB qua O: Ta có EC là phân giác của QEH


\ : Do:

N.Q; H; C; B/ D 1

và CN ?NB nên PNC \ D CNM \ ) CP d D CM


d:
Do tính chất đối xứng: MH
\ C D CHP
\ và CH ?EF nên

.HP; HM; H C; HE/ D 1 ) .Q; K; M; N / D 1:

Do đó .EQ; EF; EM; EN / là chùm điều hòa.


232 Các phương pháp giải toán qua các kỳ thi Olympic

N
I
K
M

Q
C O B
H

Cách 2. Hạ OI ?QN: Xét tam giác QOE, ta có QE 2 D QH  QO: Tứ giác HKIO là tứ giác
[ D 90ı : Nên
\ D KIO
nội tiếp vì KHO

QH  QO D QK  QI D PQ=.HKIO/ :

Mặt khác, ta lại có


PQ=.O/ D QM  QN D QE 2 :
Từ QE 2 D QH  QO, QH :QO D QK  QI , PQ=.O/ D QM:QN D QE 2 , ta có:

QM  QN D QK  QI:

Vậy ta có .Q; K; M; N / D 1: Hay .EQ; EF; EM; EN / là chùm điều hòa.

N
I
K
M
Q
C O B
H

F
Vẻ đẹp bài toán thi Olympic Hungary 233

Cách 3. Hạ OI ?QN . Tứ giác HKIO là tứ giác nội tiếp nên ta có

QH  QO D QK  QI:

Vì .Q; H; C; B/ D 1; nên ta có :

QH  QO D QC  QB:

Mặt khác, ta lại có


PQ=.O/ D QC  QB D QM  QN:
Từ QH  QO D QK  QI , QH  QO D QC  QB, QC  QB D QM  QN; ta có

QM  QN D QK  QI:

Vậy ta có .Q; K; M; N / D 1: Hay .EQ; EF; EM; EN / là chùm điều hòa.

I
K
M

Q
O1 C O B
H

Cách 4. Lấy K thuộc MN sao cho .Q; K; M; N / D 1: Vậy ta có KQ  KI D KM  KN :


Dựng OI ?MN: Vẽ đường tròn đường kính QO đi qua I: Ta có

PK=.O/ D KM  KN ; PK=.O1 / D KI  KQ:

Từ KQKI D KM KN , PK=.O/ D KM KN , PK=.O1 / D KI KQ, suy ra PK=.O/ D PK=.O1 / :


Nên K thuộc trục đẳng phương của hai đường tròn. Gọi E; F là các giao điểm của hai đường
tròn thì K 2 EF: Mặt khác EF là dây cung chung của .O1 /; .O/ nên O1 O?EF: Ta có

PH=.O1 / D HO  HQ D HE  HF


PH=.O/ D HE  HF D H C  HB
nên HO HQ D H C HB: Vậy ta có .Q; H; C; B/ D 1: Kẻ các tiếp tuyến QE1 ; QF1 với .O/.
E1 F1 \ QO D H1 thì .Q; H1 ; C; B/ D 1: Từ .Q; H; C; B/ D 1; .Q; H1 ; C; B/ D 1; ta
có H1  H: Hay QE; QF là các tiếp tuyến với (O/.
234 Các phương pháp giải toán qua các kỳ thi Olympic

K
M

Q
C O G B
H

N1

Cách 5. Kéo dài MH cắt .O/ tại N1 . Do .Q; H; C; B/ D 1 và M C ?MB; suy ra MB là


phân giác của góc HMN 1 MB D BMN nên OB?N N1 ; suy ra
\ : Ta có N\ \

N N1 k AB ) GN D GN1 :

Xét chùm .HN1 ; HN; HG; HK/ có N N1 k HK bị 3 tia còn lại chắn thành 2 đoạn bằng
nhau. Vậy đó là chùm điều hòa, cắt bởi đường thẳng QN . Ta có: H.M; N; Q; K/ D 1: Hay
.M; N; Q; K/ D 1; từ đây suy ra .EQ; EF; EM; EN / D 1:

K
M
P
Q
P1 C H O B

Cách 6. Lấy K 2 QN sao cho .Q; K; M; N / D 1: Dựng KH ?QO cắt .O/ ở E và F: Ta


chứng minh rằng QE; QF là các tiếp tuyến. Lấy PQ D PK (hình vẽ). Vậy ta có

PQ2 D PM  PN:
Vẻ đẹp bài toán thi Olympic Hungary 235

Xét Q là đường tròn điểm: PP =.Q/ D PQ2 : Hay P nằm trên trục đẳng phương của hai vòng
tròn. Hạ PP1 ?QO thì PP1 là trục đẳng phương. Do P1 thuộc trục đẳng phương nên
PP1 =.Q/ D PP1 =.O/ ) P1 Q2 D P1 C  P1 B:
Mặt khác, PP1 là đường trung bình của 4QKH nên P1 Q D P1 H: Từ P1 Q2 D P1 C  P1 B và
P1 Q D P1 H , ta có .Q; H; C; B/ D 1:
Vẽ hai tiếp tuyến QE1 ; QF1 với .O/. E1 F1 \ QO D H1 : Ta cũng có .Q; H1 ; C; B/ D 1 nên
H  H1 : Vậy QE; QF là các tiếp tuyến với .O/:
P

N
E

M1
M

Q
C H O B

Cách 7. Giả sử EF \ NB D P; P C \ .O/ D M1 : Ta chứng minh M1  M: Vì:


\DH
CNP b D 90ı

nên CHNP nội tiếp hay ta có CPH \ : Do .QH CB/ D 1 ) CNH


\ D CNH \ D CNQ:
\ Từ
\ D CNH
CPH \ và CNH \ D CNQ \ DM
\ nên ta có CNM \1 PH : Lại có

\ D 1 sđCM
CNM d
2

1 d [  1 d [  1 [
CPH D sđ CF M1 E D sđ CE M1 E D sđCM1 :
\
2 2 2
\ D sđCM 1 1 [1 ; ta có M1  M: Mà .Q; H; C; B/ D
Từ CNM 2
d ; CPH
\ = sđCM
2
1: Nên
.Q; K; M; N / D 1: Hay ta có điều phải chứng minh.
236 Các phương pháp giải toán qua các kỳ thi Olympic

E N

M
I
Q
C O B
H1 H

Cách 8. Giả sử CM \ NB D P; CN \ MB D I; PI \ QB D H1 thì trong tứ giác toàn phần


CMPNBI ta có .Q; H1 ; C; B/ D 1: Do QE; QF là các tiếp tuyến và EF \ QB D H thì

.Q; H; C; B/ D 1:

Từ .Q; H1 ; C; B/ D 1 và .Q; H; C; B/ D 1; ta có H1  H: Mặt khác, trong tam


giác CPB có BM và CN là hai đường cao nên I là trực tâm của tam giác CPB: Hay
PH1 ?QB tại H và EF ?QB tại H: Suy ra P; E; I; F thẳng hàng. Do .Q; H; C; B/ D 1
nên .Q; K; M; N / D 1:
Cách 9. Dựng đường tròn .I / đường kính MN và các tiếp tuyến QR; QS với .I /: Ta có

PQ=.I / D QR2 D QS 2 D QM  QN


PQ=.O/ D QE 2 D QF 2 D QM  QN
nên QR D QS D QE D QF: Suy ra E; F; R; S thuộc đường tròn .Q/: Ta có RS; EF; MN
là các trục đẳng phương của từng cặp hai vòng tròn một. Ba trục đẳng phương đồng quy tại K
là tâm đẳng phương của .I /; .O/; .Q/: Đối với .I /: .Q; K; M; N / D 1 (vì QR; QS là tiếp
tuyến và RS \ QI D K/.
Vẻ đẹp bài toán thi Olympic Hungary 237

E
N
I
K
M

Q
C H B
O

Bài toán 5 đúng cho cả trường hợp .S/ là cônic và nó chính là một định lí quan trọng của hình
học xạ ảnh. Ta chứng minh bài toán 5 cho .S/ là cônic như sau:

M
F N

C C'

Thật vậy, lấy một điểm C 0 khác thuộc .S/: Thế thì, với một điểm A biến thiên trên .S/; định lí
Steiner cho ta một ánh xạ xạ ảnh từ chùm .C / lên chùm .C 0 / sao cho ảnh của CA là C 0 A: Như
238 Các phương pháp giải toán qua các kỳ thi Olympic

vậy CE; CF; CM; CN có ảnh lần lượt là C 0 E; C 0 F; C 0 M; C 0 N; và theo tính chất bảo tồn tỉ số
kép của ánh xạ xạ ảnh, ta có

.C 0 E; C 0 F; C 0 M; C 0 N / D .CE; CF; CM; CN /:

Vậy tỉ số kép .CE; CF; CM; CN / không đổi, không phục thuộc vào điểm C; mà chỉ phụ thuộc
vào bốn điểm E; F; M; N: Tỉ số kép không đổi đó được gọi là tỉ số kép của bốn điểm E; F; M; N
trên đường cônic .S/, kí hiệu là .E; F; M; N /.S / :
Từ bài toán 5, ta rút ra bài toán tổng quát khác cho bài toán thi Olympic Hungary như sau:
Bài toán 6. Cho 6 điểm C 0 ; C; E; M; F; N nằm trên một cônic .S/. Giả sử MN \ CE D Q;
MN \ CF D P; MN \ C 0 E D Q0 ; MN \ C 0 F D P 0 : Chứng minh rằng

.Q; P; M; N / D .Q0 ; P 0 ; M; N /:

P'

P
M
Q

Q'
E

C C'

Nhận xét. Bài toán tổng quát của bài toán 6 vẫn đúng. Tức là khi đường thẳng d bất kì cắt
CE; CF; CM; CN lần lượt tại Q; P; A; B và cắt C 0 E; C 0 F; C 0 M; C 0 N lần lượt tại Q0 ; P 0 ; A0 ;
B 0 thì ta cũng có .Q; P; A; B/ D .Q0 ; P 0 ; A0 ; B 0 /:
Chúng ta vừa đi sâu phân tích và khám phá một bài toán thú vị. Bài toán thi Olympic Hungary
ẩn chứa trong nó những kiến thức toán cao cấp quan trọng và bổ ích. Bài viết này cần trao đổi gì
thêm? Mong được sự chia sẻ của bạn đọc. Sau cùng là một bài toán để bạn đọc luyện tập.
Bài toán 7. Cho tam giác ABC nội tiếp đường tròn .O/: Các tiếp tuyến tại B và C gặp nhau
ở M . Từ M vẽ đường thẳng song song với tiếp tuyến tại A cắt AB; AC kéo dài tại P và R:
Chứng minh rằng M là trung điểm của PR:

Tài liệu tham khảo


[1] Nguyễn Văn Kỷ Cương, Lê Xuân Mai (1960), Bài tập Hình học Cực và Đối cực, Nghịch
đảo – Lớp đệ nhất Khoa học Toán, Nhà xuất bản Khoa học.
Vẻ đẹp bài toán thi Olympic Hungary 239

[2] Quang Dương, Facebook BÀI TOÁN HAY – LỜI GIẢI ĐẸP – ĐAM MÊ TOÁN HỌC.

[3] Nguyễn Văn Lợi, Facebook BÀI TOÁN HAY – LỜI GIẢI ĐẸP – ĐAM MÊ TOÁN HỌC.
240 Các phương pháp giải toán qua các kỳ thi Olympic
ĐỊNH LÝ THẶNG DƯ TRUNG HOA
VÀ MỘT SỐ ỨNG DỤNG
Nguyễn Duy Liên
(Trường THPT Chuyên Vĩnh Phúc)

1. Mở đầu
Định lý thặng dư Trung Hoa là tên người phương Tây đặt thêm, người Trung Quốc gọi nó là bài
toán “Hàn Tín điểm binh”. Hàn Tín là một danh tướng thời Hán Sở, từng được phong tước vương
thời Hán Cao Tổ Lưu Bang đang dựng nghiệp. Sử ký Tư Mã Thiên viết rằng Hàn Tín là tướng
trói gà không nổi, nhưng rất có tài về quân sự, tục kể rằng khi Hàn Tín điểm quân số ông cho
quân lính xếp hàng 3; hàng 5; hàng 7 rồi báo cáo số dư mỗi hàng, từ đó ông tính chính xác quân
số đến từng người. Cách điểm quân số đã được ông thể hiện qua bài thơ sau:

Tam nhân đồng hành thất thập hy.


Ngũ thụ mai hoa trấp nhất chi
Thất tử đoàn viên chính bán nguyệt
Trừ bách linh ngũ tiện đắc chi.

Dịch nghĩa

Ba người cùng đi ít bảy chục


Năm cỗ mai hoa hăm mốt cành
Bảy gã xum vầy vừa nửa tháng
Trừ trăm linh năm biết số thành

(Người dịch: Trình Đại Vỹ đời nhà Minh)

Bản chất của bài toán Hàn Tín điểm binh đấy là việc giải hệ phương trình đồng dư bậc nhất
8
ˆ
ˆ x  a1 .mod m1 /
ˆ
< x  a .mod m /
2 2
ˆ
ˆ :::
ˆ
x  ak .mod mk /
:

Trong đó m1 ; m2 ; : : : ; mk là các số nguyên dương đôi một nguyên tố cùng nhau, với bài toán
của Hàn Tín thì k D 3; m1 D 3; m2 D 5; m3 D 7:

241
242 Các phương pháp giải toán qua các kỳ thi Olympic

1.1. Định lý Thặng dư Trung Hoa


Định lý 1. Cho k số nguyên dương đôi một nguyên tố cùng nhau m1 ; m2 ; : : : ; mk và a1 ; a2 ; : : : ; ak
là k số nguyên tùy ý khi đó hệ phương trình đồng dư tuyến tính
8
ˆ
ˆ x  a1 .mod m1 /
ˆ
< x  a .mod m /
2 2
ˆ
ˆ 
ˆ
x  ak .mod mk /
:

Có nghiệm duy nhất modulo m1 m2    mk :


Chứng minh. Ta chứng minh định lý thông qua 2 bước:
1: Chứng minh sự duy nhất: Giả sử hệ có hai nghiệm x; y dẫn đến x  y .mod mi /; 8i D
1; k:

Vì m1 ; m2 ; : : : ; mk đôi một nguyên tố cùng nhau nên x  y .mod m1 m2    mk /: Tức


là y và x cùng thuộc một lớp thặng dư m1 m2    mk :
2: Chứng minh sự tồn tại: Ta muốn viết các nghiệm như là một tổ hợp tuyến tính của các số
a1 ; a2 ; : : : ; ak : Chẳng hạn x D A1 a1 C A2 a2 C    C Ak ak :

Với các Ai phải tìm thỏa mãn Aj  0 .mod mi /; 8j ¤ i và Ai  1 .mod mi /: Đặt


N1 D m2 m3    mk ; N2 D m1 m3    mk ; : : : ; Ni D m1 m2    mi 1 mi C1    mk ; : : :
Khi đó .Ni ; mi / D 1 vì
.mi ; m1 / D .mi ; m2 / D    D .mi ; mi 1/ D .mi ; mi C1 / D    D .mi ; mk / D 1;
1 1
và mj j Ni ; 8j ¤ i: Vì .Ni ; mi / D 1 nên tồn tại Ni sao cho Ni Ni  1 .mod mi /:

Đến đây ta đặt Ai D Ni Ni 1 ; vì Ni  0 .mod mj / và Ai  0 .mod mj / nên


Ai  1 .mod mi /; Ai  0 .mod mj /; 8j ¤ i
Khi đó
x D A1 a1 C A2 a2 C    C Ak ak D N1 N1 1 a1 C N2 N2 1 a2 C    C Nk Nk 1 ak ;
sẽ thỏa mãn x  Ni Ni 1 ai  ai .mod mi / vì tất cả các thừa số còn lại đều chia hết cho
mi :
Định lý được chứng minh.
Nhận xét. Định lý thặng dư Trung Hoa khẳng định về sự tồn tại duy nhất của một lớp thặng dư
các số nguyên thỏa mãn đồng thời nhiều đồng dư tuyến tính. Do đó có thể sử dụng định lý để
giải quyết những bài toán về sự tồn tại và đếm các số nguyên thỏa mãn một hệ các điều kiện về
quan hệ đồng dư, quan hệ chia hết, ... hay đếm số nghiệm của phương trình đồng dư, chứng minh
cho bài toán số học chia hết. Việc sử dụng hợp lý các bộ m1 ; m2 ; : : : ; mk và bộ a1 ; a2 ; : : : ; ak
trong định lý, cho ta nhiều kết quả khá thú vị và từ đó ta có thể lập được nhiều bài toán hay và
khó. Sau đây tôi đưa ra một số ứng dụng của định lý thặng dư Trung Hoa giải các bài toán số học
mà chúng ta thường gặp.
Định lý thặng dư Trung Hoa và một số ứng dụng 243

2. Ứng dụng để giải hệ phương trình đồng dư bậc nhất


Đầu tiên, ta đến với bài thơ đố dân gian Việt Nam:

Ví dụ 1. Trung Thu:

Trung thu gió mát trăng trong.


Phố phường đông đúc, đèn lồng sao sa
Rủ nhau đi đếm đèn hoa
Quẩn quanh, quanh quẩn biết là ai hay
Kết năm chẵn số đèn này
Bảy đèn kết lại còn hai ngọn thừa
Chín đèn thì bốn ngọn dư
Đèn hoa bao ngọn mà ngẩn ngơ lòng

(Cho biết số đèn trong khoảng 600 đến 700)

Lời giải. Gọi x là số đèn với thì x là số nguyên dương và 600  x  700: Theo đề bài ta có hệ
phương trình dồng dư sau: 8
< x  0 .mod 5/
ˆ
x  2 .mod 7/
ˆ
x  4 .mod 9/
:

Ta có N1 D 7  9 D 63  3 .mod 5/ suy ra N1 1 D 2; tương dự do N2 D 5  9 D 45  3


.mod 7/ nên N2 1 D 5; và N3 D 5  7 D 35  8 .mod 9/; nên N3 1 D 8:
Từ đó ta có

x D 2  63  0 C 5  45  2 C 8  35  4 D 1570  310 .mod 315/;

suy ra tiếp x D 310 C 315k; với k 2 Z:


Do x 2 Z; 600  x  700 nên k D 1 và x D 625: Vậy số đèn là 625:

Hoặc giải theo các cụ thời xưa như sau:: Gọi x là số đèn, khi đó x chia hết cho 5; x chia hết cho
7 dư 2; x chia cho 9 dư 4: Chú ý rằng số dư khi chia cho 7 và cho 9 đều ít hơn số chia 5 đơn vị ,
suy ra x C 5 sẽ chia hết cho cả 5; 7; 9: Bội số chung nhỏ nhất của 5; 7; 9 nằm trong khoảng
600 đến 700 là 315  2 D 630: Vậy số đèn sẽ là 630 5 D 625: Lời giải rất trong sáng và đẹp
đẽ tiếc rằng tôi chưa chuyển thể về thơ được thôi.

Ví dụ 2. Giải hệ phương trình đồng dư


8
< x  2 .mod 3/
ˆ
x  3 .mod 5/
ˆ
: x  5 .mod 7/
244 Các phương pháp giải toán qua các kỳ thi Olympic

Lời giải. Ta có
N1 D 5  7 D 35  2 .mod 3/; suy ra N1 1 D 2;
N2 D 3  7 D 21  1 .mod 5/; suy ra N2 1 D 1;
N3 D 3  5 D 15  1 .mod 7/; suy ra N3 1 D 1:
Từ đó ta có

x D 2  35  2 C 1  21  3 C 1  15  5 D 278  68 .mod 105/;

là nghiệm hệ phương trình.

Ví dụ 3. Giải hệ phương trình đồng dư


8
ˆ
ˆ x 1 .mod 3/
ˆ
<x 4 .mod 5/
ˆ
ˆ x 1 .mod 7/
ˆ
x 1 .mod 8/
:

Lời giải. Ta có

N1 D 5  7  8 D 280  1 .mod 3/; suy ra N1 1 D 1;

N2 D 3  7  8 D 168  3 .mod 5/; suy ra N2 1 D 2;


N3 D 3  5  8 D 120  1 .mod 7/; suy ra N3 1 D 1;
N4 D 3  5  7 D 105  1 .mod 8/; suy ra N4 1 D 1:
Từ đó có

x D 1  280  1 C 2  168  4 C 1  120  1 C 1  105  1 D 1849  169 .mod 840/;

là nghiệm hệ phương trình.

Ví dụ 4. Giải phương trình đồng dư

x2  1 .mod 144/:

Lời giải. Vì 144 D 16  9 và .16; 9/ D 1: Do đó theo định lý thặng dư Trung Hoa thì nghiệm
của bài toán chính là nghiệm của hệ phương trình
(
x 2  1 .mod 16/
x2  1 .mod 9/

Phương trình x 2  1 .mod 16/ có 4 nghiệm x  ˙1; ˙7 .mod 16/:


Phương trình x 2  1 .mod 9/ có 2 nghiệm x  ˙1 .mod 9/ do đó ta có tất cả 8 hệ sau
( (
x  1 .mod 16/ x  1 .mod 16/
.1/; .2/;
x  1 .mod 9/ x  1 .mod 9/
Định lý thặng dư Trung Hoa và một số ứng dụng 245
( (
x  1 .mod 16/ x 1 .mod 16/
.3/; .4/
x  1 .mod 9/ x 1 .mod 9/
( (
x  7 .mod 16/ x  7 .mod 16/
.5/; .6/;
x  1 .mod 9/ x  1 .mod 9/
( (
x  7 .mod 16/ x  7 .mod 16/
.7/; .8/
x  1 .mod 9/ x  1 .mod 9/
Cả 8 hệ đều ứng với k D 2 và
N1 D 9  9 .mod 16/; suy ra N1 1 D 9; suy ra tiếp N1 N1 1 D 81;
N2 D 16  7 .mod 9/; suy ra N2 1 D 4; suy ra tiếp N2 N2 1 D 28:
Do đó phương trình ban đầu có tất cả 8 nghiệm sau:
.1/ W x D 1  81 C 1  64 D 145  1 .mod 144/;
.1/ W x D 1  81 C . 1/  64 D 17  17 .mod 144/;
.1/ W x D . 1/  81 C 1  64 D 17  17 .mod 144/;
.1/ W x D . 1/  81 C . 1/  64 D 145  1 .mod 144/;
.1/ W x D 7  81 C 1  64 D 631  55 .mod 144/;
.1/ W x D 7  81 C . 1/  64 D 503  71 .mod 144/;
.1/ W x D . 7/  81 C 1  64 D 503  71 .mod 144/;
.1/ W x D . 7/  81 C . 1/  64 D 631  55 .mod 144/:
Lời giải hoàn tất.
Nhận xét. Như vậy dựa vào định lý thặng dư Trung Hoa ta có thể đếm được số nghiệm của một
phương trình đồng dư, thông qua các ví dụ 5; ví dụ 6 ta càng thấy rõ vấn đề này.
Ví dụ 5. Cho m là một số nguyên dương, tìm số nghiệm của phương trình
x2  x .mod m/:
Lời giải. Giả sử m D p1˛1 p2˛2    pk˛k ; .pi 2 }; ˛i 2 N; i D 1; k/; xét
.1/ x 2  x .mod pi˛i /;
.2/ x.x 1/  0 .mod pi˛i /:
Ta có x 2  x .mod m/ khi và chỉ khi .1/ và .2/ tương đương với nhau.
Vì .x; x 1/ D 1 co nên phương trình x.x 1/  0 .mod pi˛i / sẽ có hai nghiệm modulo
pi˛i là x  0 .mod pi˛i / và x  1 .mod pi˛i /: Theo định lí thặng dư Trung Hoa, với mỗi bộ
a1 ; a2 ; : : : ; ak : Hệ phương trình
x  ai .mod pi˛i /
(

i D 1; k
luôn có nghiệm duy nhất modulo m: Do mỗi phương trình x.x 1/  0 .mod pi˛i / đều có hai
nghiệm modulo pi˛i nên phương trình đã cho có 2k nghiệm.
246 Các phương pháp giải toán qua các kỳ thi Olympic

Ví dụ 6. Cho m D 20072008 : Hỏi có bao nhiêu số nguyên dương n  m thoả mãn điều kiện
:
n.2n C 1/.5n C 2/ :: m:

(Việt Nam MO 2008)


Lời giải. Ta có
m D 92008 2232008 D 34016 2232008 D n1  n2 :
:
Do .10; m/ D 1 suy ra n.2n C 1.5n C 2/ :: m; tương đương với

m j 10  5  2n  .2n C 1/.5n C 2/ D 10n.10n C 5/.10n C 4/;

hay
m j x.x C 5/.x C 4/;
trong đó x D 10n: Ta có m j x.x C 5/.x C 4/ nên ta có hệ phương trình đồng dư sau
8
< x  0 .mod 10/
ˆ
x.x C 5/.x C 4/  0 .mod n1 /
ˆ
x.x C 5/.x C 4/  0 .mod n2 /
:

Vì 3 không là ước chung của x; x C 4; x C 5 nên x.x C 5/.x C 4/  0 .mod n1 / khi và chỉ
khi x  r1 .mod n1 / ở đó r1 2 f0; 4; 5g : Tương tự x.x C 5/.x C 4/  0 .mod n2 / khi và
chỉ khi x  r2 .mod n2 / ở đó r1 2 f0; 4; 5g :
Vậy
m j n.2n C 5/.5n C 4/;
tương đương với

x0 .mod 10/; x  r1 .mod n1 /; x  r2 .mod n2 /: .1/

Vậy các số n  m thoả mãn điều kiện bằng số các số x  10n1  n2 thoả mãn .1/: Với mỗi cách
chọn
r1 2 f0; 4; 5g ; r2 2 f0; 4; 5g ;
theo định lí Trung Hoa ta có duy nhất một số x  10n1  n2 thoả mãn .1/: Như vậy có 9 số thoả
mãn điều kiện bài ra.

3. Áp dụng để giải các bài toán chứng minh sự tồn tại


trong số học
Ví dụ 7. Cho p; q 2 N nf1g; với .p; q/ D 1: Chứng minh rằng tồn tại số nguyên dương k sao
cho với mọi số nguyên dương n thì .pq 1/n k C 1 là hợp số.
Lời giải. Do .p; q/ D 1 theo định lí thặng dư Trung Hoa tồn tại số nguyên dương k thoả mãn
hệ phương trình đồng dư (
k  1 .mod p/
k  1 .mod q/
Định lý thặng dư Trung Hoa và một số ứng dụng 247

:
Nếu n :: 2 thì .pq 1/n  1 .mod q/; suy ra .pq 1/n k  1 .mod q/; tương đương với

.pq 1/n k C 1  0 .mod q/:


:
Nếu n 6 :: 2 thì .pq 1/n  1 .mod p/; suy ra .pq 1/n k  1 .mod p/; tương đương với

.pq 1/n k C 1  0 .mod p/:


:
Do .p; q/ D 1; suy ra .pq 1/n k C 1 :: pq; suy ra .pq 1/n k C 1 là hợp số với mọi n 2 N :
Bài toán được chứng minh.
Nhận xét. Chứng minh trên thật gọn gàng nhờ vào việc sử dụng định lý thặng dư Trung Hoa.
Mấu chốt của bài toán là chúng ta thấy được để .pq 1/n k C 1 là hợp số ta cần chỉ ra rằng khi
nào .pq 1/n k C 1 chia hết cho p hoặc q (thông qua việc xét tính chẵn lẻ của n/ từ đó ta xây
dựng được một hệ phương trình đồng dư
(
k  1 .mod p/
k  1 .mod q/

Ví dụ 8. Chứng minh rằng với mọi số nguyên dương n sẽ tồn tại n số tự nhiên liên tiếp sao cho
với bất kì số nào trong các số ấy cũng đều không phải là luỹ thừa (với số mũ nguyên dương) của
số nguyên tố nào đó.
(IMO 1989)
Nhận xét. Lời giải 1. Với mỗi số nguyên dương n xét n số nguyên tố phân biệt p1 ; p2 ; : : : ; pn :
Xét hệ phương trình
x  p1 1 .mod p12 /
8
ˆ
ˆ
1 .mod p 2 /
ˆ
<x  p
2 2
ˆ
ˆ 
ˆ
.mod pn2 /
:
x  pn 1
Theo định lý thặng dư Trung Hoa thì hệ phương trình trên có nghiệm khi và chỉ khi tồn tại số
nguyên a sao cho a  .pi 1/ .mod pi2 /; 8i D 1; n: Từ đó suy ra các số aC1; aC2; : : : ; aCn
đều không phải là luỹ thừa với số mũ nguyên dương của một số nguyên tố.
Nhận xét. Lời giải 2. Với mỗi số tự nhiên n xét 2n số nguyên tố phân biệt p1 ; p2 ; : : : ; pn ; q1 ; q2 ; : : : ; qn :
Xét hệ phương trình 8
ˆ
ˆ x  1 .mod p1 q1 /
ˆ
< x  2 .mod p q /
2 2
ˆ
ˆ 
ˆ
x  n .mod pn qn /
:

Theo định lý thặng dư Trung Hoa thì hệ phương trình trên có nghiệm khi và chỉ khi tồn tại số
nguyên a sao cho a  i .mod pi qi /; 8i D 1; n: Từ đó suy ra các số a C1; a C2; : : : ; a Cn;
đều không phải là luỹ thừa với số mũ nguyên dương của một số nguyên tố.
Nhận xét. Qua sự chọn khéo léo bộ m1 ; m2 ; : : : ; mk cho ta một dãy n số hạng thỏa mãn yêu
cầu. Tư tưởng giống như trên cho các ví dụ tiếp theo dưới đây.
248 Các phương pháp giải toán qua các kỳ thi Olympic

Ví dụ 9. Với n là một số nguyên dương


:
.1/ Giả sử tồn tại dãy fx1 ; x2 ; : : : ; xn g D f1; 2; : : : ; ng sao cho x1 C x2 C    C xk :: k; với
mọi k D 1; n: Tìm n ‹
.2/ Tồn tại hay không một dãy vô hạn fx1 ; x2 ; : : :g D f1; 2; : : :g sao cho xi ¤ xj ; 8i ¤ j
:
và thoả mãn x1 C x2 C    C xk :: k với mọi k D 1; n:
(Nordic 1998)
Lời giải. .1/ Dễ thấy n D 1 thỏa mãn yêu cầu của bài toán, nếu n D 3 ta chọn
fx1 ; x2 ; x3 g D f1; 3; 2g;
do đó n D 3 cũng thỏa mãn. Giả sử bài toán thoả mãn với n nào đó, ta có
n n
X X n.n C 1/ ::
xi D iD : n;
i D1 i D1
2
nC1
suy ra n là số lẻ. Giả sử n  5; đặt m D 2
; theo giả thiết ta có
n 1 n 1
:
xn :: .n
X X
xi D i D mn 1/;
i D1 i D1

nên xn  mn  m .mod n 1/; với 1  xn  n; suy ra xn 1 Dn 1: Tương tự ta có


n 2 n 2
X X ::
xi D i D m.n 1/ xn 1 : .n 2/;
i D1 i D1

suy ra xn 1  m.n 1/  m .mod n 2/; với 1  xn 1  n suy ra tiếp xn 1 D m D xn ; vô


lý.
Vậy chỉ có n D 1; n D 3 thoả mãn yêu cầu đề bài.
.2/ Ta sẽ xây dựng một dãy .xn /C1 nD1 thoả mãn điều kiện đề bài. Chọn x1 D 1; x2 D 3; x3 D 2:
:
Giả sử x1 ; x2 ; x3 ; : : : ; xN là một dãy thoả mãn điều kiện x1 C x2 C    C xk :: k với mọi
k D 1; N :
Đặt s D x1 C x2 C x3 C    C xN và gọi n là số nguyên dương bé nhất không nằm trong dãy
x1 ; x2 ; x3 ; : : : ; xN : Dễ thấy .N C 1; N C 2/ D 1 nên theo định lí thặng dư Trung Hoa tồn tại
một số nguyên m > x1 ; x2 ; x3 ; : : : ; xN thoả mãn
(
m  s .mod N C 1/
m  s n .mod N C 2/

Đặt m D xN C1 ; n D xN C2 ; ta có dãy x1 ; x2 ; x3 ; : : : ; xN ; xN C1 ; xN C2 thoả mãn các điều


kiện của bài toán vì
:
x1 C x2 C x3 C    C xN C xN C1 D s C m :: .N C 1/;
:
x1 C x2 C x3 C    C xN C1 C xN C2 D s C m C n :: .N C 2/;
: :
và x1 C x2 C    C xk :: k với mọi k D 1; N : Do đó x1 C x2 C    C xk :: k với mọi k D 1; N C 2
hiển nhiên dãy .xn /C1nD1 xây dựng như trên thoả mãn điều kiện đề bài.
Định lý thặng dư Trung Hoa và một số ứng dụng 249

Nhận xét. Trong bài toán này ta cần chú ý đến dãy fxn g là một hoán vị của tập N; nếu không
có giả thiết này bài toán trở thành tầm thường, trong phần 2 ta cần quy nạp như sau, mỗi bộ
x1 ; x2 ; : : : ; xn thỏa mãn ta luôn tìm được xnC1 sao cho
:
x1 C x2 C    C xnC1 :: n C 1:

Do đó ta cần phải xây dựng dãy fxn g sao cho dãy fxn g phải quét hết tập N; đây là yêu cầu chính
của bài toán.

Ví dụ 10. Chứng minh rằng nếu p1 ; p2 ; : : : ; pn là các số nguyên tố phân biệt thì phương trình

x1p1 C x2p2 C    C xnpn 11 D xnpn ;

có vô số nghiệm nguyên dương .x1 ; x2 ; : : : ; xn /:

Lời giải. Ta có đẳng thức .n 1/k C .n 1/k C    C .n 1/k D .n 1/kC1 : Khi đó ta chọn
„ ƒ‚ …
n 1

k k k kC1
x1 D .n 1/ p1 ; x2 D .n 1/ p2 ; : : : ; xn 1 D .n 1/ pn 1 ; xn D .n 1/ pn :

Thì ta thu được ngay phương trình x1p1 C x2p2 C    C xnpn 11 D xnpn : Vậy nếu ta chỉ ra được số
nguyên dương k sao cho x1 ; x2 ; : : : ; xn đều nguyên thì ta được điều phải chứng minh. Mà điều
này tương đương với hệ sau có nghiệm
8
ˆ
ˆ k  0 .mod p1 /
ˆ
< k  0 .mod p2 /
ˆ
ˆ
ˆ
 ./
ˆ
ˆ
ˆ
ˆ
ˆ k  0 .mod pn 1 /
ˆ
: k  1 .mod p /
n

Luôn đúng theo định lý thặng dư Trung Hoa, vì p1 ; p2 ; : : : ; pn là các số nguyên tố phân
biệt.

Ví dụ 11. Chứng minh rằng với mọi số nguyên dương n luôn tồn tại n số nguyên a1 ; a2 ; : : : ; an
sao cho ai Caj là lũy thừa của một số tự nhiên với số mũ lớn hơn 1 với mọi i; j 2 f1; 2; : : : ; ng :

Lời giải. Với mọi xi 2 N ta chọn các số sau

a1 D 1x1 C1  2x2  3x3    .2n/x2n D 1x1  2x2 C1  3x3    .2n/x2n ;


an D 1x1  2x2  3x3    nxn C1    .2n/x2n :


ai C aj D 1x1  2x2  3x3    i xi C1    .2n/x2n C 1x1  2x2  3x3    j xj C1    .2n/x2n
D 1x1  2x2  3x3    .2n/x2n .i C j / ;
ai C aj D 1x1  2x2  3x3    .i C j /xiCj C1    .2n/x2n :

Xét các số nguyên tố phân biệt p1 ; p2 ; : : : ; p2n : Xét các hệ phương trình đồng dư tuyến tính
250 Các phương pháp giải toán qua các kỳ thi Olympic

1: Với mọi k 2 f2; 3; : : : ; 2ng ; thì


(
x1  1 .mod p1 /
x1  0 .mod pk /

2: Với mọi k 2 f1; 3; 4; : : : ; 2ng ; thì


(
x2  1 .mod p2 /
x2  0 .mod pk /

3: Với mọi k 2 f1; 2; 3; : : : ; i C j 1; i C j C 1; : : : ; 2ng ; thì


(
xi Cj  1 .mod pi Cj /
xi Cj  0 .mod pk /

4: Với mọi k D 1; 2n 1; thì


(
x2n  1 .mod p2n /
x2n  0 .mod pk /

Theo định lý thặng dư Trung Hoa thì các hệ này chắc chắn có nghiệm. Từ đó suy ra
x1 x2 xi Cj C 1 x2n
; ; :::; ; :::; ; :::
pi Cj pi Cj pi Cj pi Cj

các số này đều là số nguyên. Khi đó

ai C aj D 1x1 2x2 3x3    .i C j /xi Cj C1    .2n/x2n


 x x2 xiCj C1 x2n
piCj
1
piCj pi Cj pi Cj piCj
D 1 2    .i C j /    .2n/ ;

là lũy thừa của một số nguyên dương đây là điều phải chứng minh.

Ví dụ 12. Cho A là một tập hợp khác rỗng gồm các số nguyên dương. Chứng minh rằng tồn tại
số nguyên dương m sao cho mọi phần tử của tập mA đều là lũy thừa của một số tự nhiên với số
mũ lớn hơn 1:

(BalKan 2000)

Lời giải. Giả sử A D fa1 ; a2 ; : : : ; ak g và gọi p1 ; p2 ; : : : ; pN là tất cả các ước số nguyên


Qk
tố của số ai : Với mỗi i D 1; 2; : : : ; k tồn tại các số nguyên không âm ˛i; j sao cho
i D1
N
˛
pj i; j : Gọi q1 ; q2 ; : : : ; qk là các số nguyên tố phân biệt. Theo định lý thặng dư Trung
Q
ai D
j D1
Hoa, với j D 1; N ; khi đó tồn tại

ˇj  ˛i; j .mod qi /; với mọi i D 1; 2; : : : ; k:


Định lý thặng dư Trung Hoa và một số ứng dụng 251

N
ˇ
pj j ; với i D 1; 2; : : : ; k ta có
Q
Đặt m D
j D1

2 3qi
N N ˛i;j Cˇj
˛ Cˇj
Y Y qi
mai D pj i;j D4 pj 5
j D1 j D1

là số lũy thừa. Bài toán được chứng minh.

Ví dụ 13. Chứng minh rằng tồn tại vô hạn các số k nguyên dương chẵn, sao cho với mọi số
nguyên tố p thì p 2 C k là hợp số.

Lời giải. Nếu p D 2 thì p 2 C k là hợp số với mọi k chẵn.


Nếu p > 3 thì p 2  1 .mod 3/; với mọi k chẵn và k  2 .mod 3/ thì p 2 C k là hợp số vì nó
là bội của 3:
Nếu p D 3 thì p 2 C k D 9 C k  0 .mod 5/; nếu k  1 .mod 5/: Vậy k thỏa mãn điều kiện
bài toán khi và chỉ khi k là nghiệm nguyên dương của hệ phương trình đồng dư
8
< k  0 .mod 2/
ˆ
k  2 .mod 3/
ˆ
: k  1 .mod 5/

Theo định lý thặng dư Trung Hoa thì hệ phương trình này có nghiệm k  26 .mod 30/; hay
k D 30h C 26; .h 2 N/; p 2 C k D p 2 C 30h C 26  40 cho nên p 2 C k là hợp số. Vậy có vô
số k nguyên dương chẵn sao cho với mọi số nguyên tố p thì p 2 C k là hợp số.

Nhận xét. Chứng minh trên thật ấn tượng nhờ vào việc sử dụng định lý thặng dư Trung Hoa.
Mấu chốt của bài toán là chúng ta thấy được để p 2 C k là hợp số ta cần chỉ ra rằng khi nào
p 2 C k chia hết cho 2; 3 hoặc 5 (qua việc xét các dạng của p) từ đó ta xây dựng được một hệ
phương trình đồng dư 8
< k  0 .mod 2/
ˆ
k  2 .mod 3/
ˆ
k  1 .mod 5/
:

Từ đó tìm được tất cả giá trị của k:

Ví dụ 14. Chứng minh rằng tồn tại đa thức P .x/ 2 ZŒx; không có nghiệm nguyên sao cho với
:
mọi số nguyên dương n; tồn tại số nguyên dương x sao cho P .x/ :: n:

Lời giải. Xét đa thức P .x/ D .2x C 1/.3x C 1/; với mỗi số nguyên dương n ta biểu diễn n
dưới dạng n D 2k .2m C 1/:

 Vì .2k ; 3/ D 1; nên tồn tại a sao cho 3a  1 .mod 2k //: Từ đó nếu 3x  1 .mod 2k /
thì ta cần chọn x  a .mod 2k /:

 Vì .2; 2m C 1/ D 1; nên tồn tại b sao cho 2b  1 .mod 2m C 1/: Từ đó nếu như ta có
2x  1 .mod 2m C 1/ thì ta cần chọn x  b .mod 2m C 1/:
252 Các phương pháp giải toán qua các kỳ thi Olympic

 Nhưng do .2k ; 2m C 1/ D 1; nên theo định lí thặng dư Trung Hoa, tồn tại số nguyên x là
nghiệm của hệ phương trình đồng dư sau
(
x  a .mod 2k /
x  b .mod 2m C 1/

:
theo lập luận trên P .x/ D .2x C 1/.3x C 1/ :: n:
Bài toán được chứng minh.
Ví dụ 15. Cho tập A D f2; 7; 11; 13g và đa thức P D .x/ 2 ZŒx có tính chất với mỗi n 2 Z
tồn tại p 2 A sao cho p jP .n/ Chứng minh rằng tồn tại p 2 A sao cho p j P .n/ với mọi n 2 Z:
Lời giải. Ta có bổ đề (bạn đọc tự chứng minh)
Bổ đề 1. Với x; y là các số nguyên sao cho x  y .mod p/ tương đương với

.x/  P .y/ .mod p/:

Trở lại bài toán, giả sử không tồn tại p 2 A sao cho p jP .n/ với mọi số nguyên n; suy ra tồn tại
các số nguyên a; b; c; d phân biệt sao cho
0 :
.mod 2/ suy ra a 6 :: 2;
0
P .a/  a
0 :
.mod 7/; suy ra b 6 :: 7;
0
P .b/  b
0 :
.mod 11/; suy ra c 6 :: 11;
0
P .c/  c
0 :
.mod 13/; suy ra d 6 :: 13:
0
P .d /  d

Xét hệ phương trình đồng dừ 8


ˆ
ˆ x  a .mod 2/
ˆ
<x  b .mod 7/
ˆx
ˆ  c .mod 1/1
ˆ
x  d .mod 1/3
:

Theo định lý thặng dư Trung Hoa hệ phương trình này có nghiệm x0 : Kết hợp với bổ đề ta có
8 0
ˆ
ˆ P .x0 /  P .a/  a .mod 2/
< P .x /  P .b/  b 0 .mod 7/
ˆ
ˆ
0
0
ˆ
ˆ
ˆ P .x0 /  P .c/  c .mod 11/
0
ˆ
P .x0 /  P .d /  d .mod 13/
:

Điều này mâu thuẫn với điều mà ta đã giả sử ở trên. Từ đó su yra điều phải chứng minh.
Nhận xét. Qua việc giải hai ví dụ trên và việc kết hợp giữa định lý thặng dư Trung Hoa với các
tính chất của đa thức nguyên cho ta một kết quả thú vị.
Ví dụ 16. Cho n; h là haii số nguyên dương. Chứng minh rằng tồn tại n số tự nhiên liên tiếp
thỏa mãn mỗi số đều có ít nhất h ước số nguyên tố phân biệt.
Định lý thặng dư Trung Hoa và một số ứng dụng 253

Lời giải. Do tập hợp các số nguyên tố là vô hạn nên ta có thể chọn nh số nguyên tố phân biệt

p1 < p2 <    < ph <    < pnh

Theo định lý thặng dư Trung Hoa thì tồn tại số nguyên dương k là nghiệm của hệ phương trình
8
ˆ
ˆ k  1 .mod p1 p2    ph /
ˆ
ˆ
ˆ
ˆ
ˆ k  2 .mod phC1 phC2    p2h /
ˆ
< 
ˆ
ˆ k  i .mod p.i 1/hC1 p.i 1/hC2    pih /
ˆ

ˆ
ˆ
ˆ
ˆ
ˆ
k  n .mod p.n 1/hC1 p.n 1/hC 2    pn h /
:

Từ đó ta có n số tự nhiên liên tiếp là k C 1; k C 2; : : : ; k C n thỏa mãn mỗi số đều có ít nhất h


ước số nguyên tố phân biệt.

Ví dụ 17. Chứng minh rằng với mọi m; n nguyên dương thì tồn tại x nguyên dương thoả mãn
(
2x  1999 .mod 3m /
2x  2009 .mod 5n /

Lời giải. Ta có bổ đề
Bổ đề 2. Ta có 2 là căn nguyên thuỷ của mod 5m ; mod 3n :
Từ đó tồn tại x1 ; x2 sao cho (
2x1  1999 .mod 3m /
2x2  2009 .mod 5n /
Do (
2x1  1 .mod 3/
2x2  4 .mod 5/

Vì x1 ; x2 chẵn nên theo định lý thặng dư Trng Hoa hệ phương trình đồng dư sau có nghiệm
x1
8
<t 
ˆ .mod 3m 1 /
2
x
: t  2 .mod 4  5n 1 /
ˆ
2
Chọn x D 2t thì (
2t  x1 .mod '.3m // D 2  3m 1

2t  x2 .mod '.5n // D 4  5n 1

Suy ra (
2x  1999 .mod 3m /
2x  2009 .mod 5n /

Từ đó suy ra điều phải chứng minh.


254 Các phương pháp giải toán qua các kỳ thi Olympic

Nhận xét. Bài toán cần vận dụng kết hợp kiến thức giữa căn nguyên thủy và định lý thặng dư
Trung Hoa cho ta một lời giải thật chặt chẽ và ngắn gọn.
Ví dụ 18. Cho p là số nguyên tố. Chứng minh rằng tồn tại một bội số của p sao cho 10 chữ số
tận cùng của nó đôi một khác nhau.
Lời giải. Nếu p D 2 ta thấy số 1234567899876543210 sẽ thỏa mãn đề bài.
Nếu p D 5 ta thấy 1234567899876432105 cũng thỏa mãn đề bài.
Nếu p 62 f2; 5g: Xét hệ phương trình đồng dư tuyến tính.
(
x  a0 a1 a2    a9 .mod 1010 /
x  0 .mod p/

Trong đó ai 2 f0; 1; 2; 3; 4; 5; 6; 7; 8; 9g với ai ¤ aj và 0  i ¤ j  9:


Vì p 2 }; p 62 f2; 5g; suy ra gcd.p; 1010 / D 1: Do đó theo định lý thặng dư Trung Hoa thì hệ
này chắc chắn có nghiệm, nghiệm của hệ chính là số thỏa mãn yêu cầu bài toán.
Nhận xét. Từ các trường hợp cơ sở cho các số nguyên tố 2 và 5; xây dựng nên hệ phương trình
đồng dư tuyến tính tối ưu cho số nguyên tố bất kỳ khác 2 và 5:

4. Ứng dụng để giải các bài toán về chứng minh chia hết
và tìm số nguyên thỏa mãn điều kiện cho trước
Ví dụ 19. Chứng minh rằng phương trình x 2 34y 2  1 .mod m/ có nghiệm với mọi số
nguyên dương m:
Lời giải. Ta chia bài toán ra 3 trường hợp
Trường hợp 1 W Nếu .m; 3/ D 1 nên x 2 34y 2  1 .mod m/; hay

.x 5y/.x C 5y/  .3y C 1/.3y 1/ .mod m/:

Tập hợp các số f3y C 1; 3y 1g chạy qua các số không chia hết cho 3 suy ra tồn tại số nguyên
:
y0 thỏa mãn .3y0 C 1/.3y0 1/ :: m; chọn x0 D 5y0 ; suy ra .x0 ; y0 / cần tìm.
Trường hợp 2 W Nếu .m; 5/ D 1 thì x 2 34y 2  1 .mod m/; hay

.x 3y/.x C 3y/  .5y C 1/.5y 1/ .mod m/:

Tập hợp các số f5y C 1; 5y 1g chạy qua các số không chia hết cho 5 suy ra tồn tại số nguyên
:
y0 thỏa mãn .5y0 C 1/.5y0 1/ :: m chọn x0 D 3y0 ta suy ra được .x0 ; y0 / cần tìm. Trường hợp
3 W Nếu .m; 3/ D .m; 5/ ¤ 1 ta đặt m D m1  m2 với

m1 D 3˛ .˛ 2 N /; m2 2 N W .m1 ; m2 / D 1; .m1 ; 5/ D 1:
2
 Nếu .3; m2 / D 1 thì tồn tại .x1 ; y1 / 2 .ZC / W x12 34y12  1 .mod m2 /:
2
 Nếu .5; m1 / D 1 thì tồn tại .x2 ; y2 / 2 .ZC / W x22 34y22  1 .mod m1 /:
Định lý thặng dư Trung Hoa và một số ứng dụng 255

Từ đó theo định lí thặng dư Trung Hoa tồn tại .x; y/ 2 N sao cho
( (
x  x1 .mod m1 / x  x2 .mod m2 /
&
y  y1 .mod m1 / y  y2 .mod m2 /

Vậy ta có điều phải chứng minh.


Nhận xét. Cách giải của bài toán chính là đã dùng phương pháp gen trong phương trình đồng
dư, kết hợp với định lí thặng dư Trung Hoa.

Ví dụ 20. Xác định tất cả các số nguyên dương n sao cho với n này tồn tại số nguyên m sao
cho .2n 1/ j .m2 C 9/:

(Shortlisted IMO 1998)

Lời giải. Ta chứng minh .2n 1/ j .m2 C 9/; hay n D 2s ; .s 2 N /:


Điều kiện cần: Đặt n D 2s ; t.s 2 N; 2 N ; .t; 2/ D 1/: Nếu t  3 thì .2t 1/ j .2n 1/; suy ra

.2t 1/ j .m2 C 9/:

Ta có 2t 1  1 .mod 4/ nên tồn tại p 2 } W p  1 .mod 4/; p j .2t 1/; .p ¤ 3/; suy
ra p j .m2 C 9/; hay p j .m2 C 32 /: Theo định lý Fecma
p 1 p 1
1  mp 1
 .m2 / 2
 . 9/ 2  1 .mod p/;

rõ ràng điều này không xảy ra nếu tồn tại t suy ra p D 3 mâu thuẫn, do đó 2t  11 .mod p/
vậy n D 2s .s 2 N /:
Điều kiện đủ: Do
s 2 s 1
2n 1 D 22 1 D .2 1/.2 C 1/.22 C 1/.22 C 1/    .22 C 1/;
t
suy ra .2n 1/ j .m2 C 9/; suy ra tiếp .22 C 1/j.m2 C 9/; 8t D 1; s 1: Mà
˛ ˇ
.22 C 1; 22 C 1/ D 1; .˛ ¤ ˇ/;
t t
nên theo Định lí thặng dư Trung Hoa hệ phương trình x  22 .mod 22 / C 1; 8t D 0; s 2
t t C1
có nghiệm nên tồn tại c 2 Z W c  22 .mod 22 / C 1; suy ra
tC1
c2 C 1  0 .mod 22 / C 1; 8t D 0; s 2;

từ đây suy ra 2n 1 j 9.c 2 C 1/ D m2 C 9; trong đó m D 3c:


Nhận xét. Cái khó của bài toán này là dự đoán dạng của n (thông qua một số ví dụ cơ sở), với
điều kiện đủ ta cần xây dựng được hệ phương trình đồng dư có m1 ; m2 ; : : : ; ms đôi một nguyên
tố cùng nhau.

Ví dụ 21. Cho số nguyên dương a chỉ có ước nguyên tố dạng 4k C 1 .k 2 N /: Chứng minh
:
rằng tồn tại số nguyên dương b sao cho b 2 C 1 :: a4 C a2 :

(Shortlisted IMO 2001)


256 Các phương pháp giải toán qua các kỳ thi Olympic

Ta áp dụng bổ đề sau:
:
Bổ đề 3. Cho p 2 }; p D 4k C 1 khi đó tồn tại số nguyên dương x sao cho x 2 C 1 :: p n với
mọi số nguyên dương n cho trước.

Lời giải. Ta chứng minh bổ đề bằng quy nạp:


Thật vậy nếu n D 1 ta thấy x D .2k/Š thoả mãn bổ đề. Giả sử bổ đề đúng với n D h 2 N khi
:
đó tồn tại xh W x 2 C 1 :: p h ; hay x 2 C 1 D up h ; .u 2 N /: Đặt xhC1 D xh C tp h ; thì
h h

2 :
2
xhC1 C 1 D .xh C tp h / C 1 D p h .u C 2xh t/ C p 2h t 2 :: p hC1 :

: x
Ta cần chọn t 2 N sao cho u C 2xh t :: p: Áp dụng a2 D ph˛h .ph  1 .mod 4//; suy ra
Q
hD1

s
X
2
a C1D ph˛h .ph  1 .mod 4//;
hD1

s
X
2
a .a C 1/ D2
pk˛k ; 8h D 1; s:
kD1

: :
Suy ra tồn tai bh 2 Z sao cho bh2 C 1 :: p h (theo bổ đề), b02 C 1 :: 2.b0 2 Z/: Xét hệ phương trình
đồng dư sau đây: (
x  b0 .mod ph˛h /
x  bs .mod ps˛s /
s
: Q
Theo định lí thặng dư Trung Hoa thì hệ có nghiệm x D b 2 Z W b 2 C 1 :: ph˛h : Chứng minh
hD1
của ta được hoàn tất.

Ví dụ 22. Tìm tất cả các số nguyên dương a sao cho 2n n2 jan na ; với mọi số n nguyên
dương n  5:
ˇ ˇ
Lời giải. Chọn số nguyên tố p sao cho p > 2; a; ˇa2 2a ˇ : Theo định lý thặng dư Trung
˚

Hoa thì tồn tại n 2 N ; n  5 là nghiệm của hệ


(
n  2 .mod p/
n  2 .mod p 1/

Từ đó ta có 2n n2  ˇ 22 22  0 .mod p/; suy ra p jan na : Mà anˇ na  a2 2a


.mod p/; suy ra tiếp p ˇa2 2a : Vậy mọi p nguyên tố thỏa mãn thì p ˇa2 2a ; ta được
a2 2a D 0; suy ra tiếp a D 2; a D 4 do 2a > a2 với mọi a  5:
Thử lại ta thấy a D 2; a D 4 thỏa mãn yêu cầu đề bài.
ˇ ˇ
Nhận xét. Cái hay của bài toán qua việc chọn số nguyên tố p sao cho p > 2; a; ˇa2 2a ˇ :
˚

Ví dụ 23. Với n là một số nguyên dương.


Định lý thặng dư Trung Hoa và một số ứng dụng 257

: :
.1/ Giả sử n không có ước chính phương khác 1: Chứng minh rằng nếu an 1 :: n thì an 1 :: n2 :
:
.2/ Tìm số nguyên dương n > 1 có tính chất với mọi số nguyên dương a thỏa mãn an 1 :: n
:
thì an 1 :: n2 :
: :
Lời giải. .1/: Với n D p 2 }; 8a 2 N ta có ap 1 :: p hay ap aCa 1 :: p hoặc a  1
.mod p/: Từ đó có được
ap 1 D .a 1/.ap 1
C ap 2
C    C a C 1/  0 .mod p 2 /:
: :
Với n D p1 p2    pk ; an 1 D ap1 p2 pk 1 :: p1 p2    pk và .ap1 p2 pk /pi 1 :: pi ; ta suy ra
:
.ap1 p2 pk /pi 1 :: pi2 ;
: :
suy ra tiếp ap1 p2 pk 1 :: .p1 p2    pk /2 ; tương đương với an 1 :: n2 :
.2/: Với n D 2˛ p1 p2    pk ; 0  ˛  2; pi 2 }; .pi ; 2/ D 1; i D 1; k:
Điều kiện đủ: Ta chỉ cần xét n D 4p1 p2    pk ; pi 2 }; .pi ; 2/ D 1; i D 1; k là đủ. Thật vậy
:
đặt A D ap1 p2 pk ; thì an 1 D A4 1 :: 4; do A lẻ nên
:
A4 1 D .A 1/.A C 1/.A2 C 1/ :: 16
: :
suy ra an 1 :: 42 : Đặt tiếp B D a4 ; thì an 1 D B p1 p2 pk 1 :: p1 p2    pk ; suy ra
:
an 1 D B p1 p2 pk 1 :: .p1 p2    pk /2 ; theo .1/:
:
Mà .42 ; p12    pk2 / D 1; nên an 1 :: .4p1 p2    pk /2 D n2 :
Điều kiện cần: Giả sử n D p ˛ q; p 2 }; .p; 2/ D 1; .q; 2/ D 1; q; ˛ 2 N: Lấy a D pq C 1 ta
˛
có an 1 D ap q 1 D p ˛C1 h; .h; p/ D 1 (chứng minh bằng quy nạp). Do đó
< an 1 ::: a 1:::p
8

:
1 :: p ˛
: n
a
: : :
Do an 1 :: n suy ra an 1 :: n2 hay p ˛C1 h :: p 2˛ q 2 ; tương đương với ˛ C 1  2˛; do đó ˛  1:
Vậy số mũ của ˛ là 0 _ 1:
Xét p D 2; giả sử n D 2˛ t; t 2 N; .t; 2/ D 1 thì 52t 1 D 25t 1 D 8m; m 2 N; m lẻ. Theo
định lý thặng dư Trung Hoa tồn tại số nguyên dương a là nghiệm hệ phương trình đồng dư
(
a  5 .mod 16/
a  1 .mod t/

Đặt A D at thì
˛ i ˛ 1
an 1 D A2 1 D .A2 1/.A2 C 1/    .A2 C 1/    .A2 C 1/ D 2˛C2 V; .V; 2/ D 1:
i
Do A2 1 D a2t 1  25t 1  8 .mod 16/; nên A2 C 1  2 .mod 4/; 8i D 1; s 1:
: : :
Do an 1 :: n nên an 1 :: n2 : suy ra 2˛C2 V :: 22˛ t 2 ; hay ˛ C 2  2˛; hoặc ˛  2; nên
˛ 2 f0; 1; 2g: Vậy n D 2˛ p2 p2    pk ; 0 6 ˛  2; pi 2 .pi ; 2/ D 1; i D 1; k:
258 Các phương pháp giải toán qua các kỳ thi Olympic

Nhận xét. Cái khó của bài toán ở phần 2 với điều kiện cần, để giải quyết được vấn đề này ta cần
nắm vững số mũ đúng của một số với một số nguyên tố, và chọn được bộ a1 ; a2 ; : : : ; ak hợp lý
của hệ phương trình đồng dư.

Ví dụ 24. Cho a; b là hai số nguyên dương, sao cho với mỗi số nguyên dương n ta luôn có
an C nj b n C n: Chứng minh rằng a D b:

(Selection tests for the BMO and IMO Romanian teams 2006)

Lời giải. Giả sử a ¤ b; khi đó nếu n D 1 thì a C 1j b C 1 suy ra b > a: Gọi p là số nguyên tố
sao cho p > b; theo định lý phần dư Trung Hoa tồn tại n số nguyên dương là nghiệm của hệ
phương trình đồng dư (
n  1 .mod .p 1//
n  a .mod p/
Suy ra n D .a C 1/.p 1/ C 1: Theo định lý Fermat’s

an D a.aC1/.p 1/C1
D a.a

p 1
   ap …1 /  a
ƒ‚ .mod p/;
aC1 t i me

nên an C n  a C n  0 .mod p/; hay p j an C n; suy ra

p j b n C n: .1/

Mà theo định lý Fermat’s


bn C n  b a .mod p/: .2/
Từ .1/ và .2/ ta suy ra p j b a (vô lý). Vậy ta có điều phải chứng minh.

Ví dụ 25. Chứng minh rằng với mọi số N nguyên dương là tích của 2015 số nguyên tố lẻ phân
biệt đều là ước của vô số số có dạng aaC1 C .a C 1/a ; với a là số nguyên dương.

Lời giải. Với số p là số nguyên tố lẻ sao cho p j N thì p ˇ aaC1 C .a C 1/a : Chọn
ˇ

(
a  2 .mod p/
a C 1  0 .mod p 1/

tương đương với (


a 2 .mod p/
a 1 .mod p 1/
Theo định lý thặng dư Trung Hoa hệ phương trình này có nghiệm a D p 2Ckp.p 1/; k 2 N :
Khi N D p1 p2    p2015
ˇ ; với pj là số nguyên tố lẻ. Theo nhận xét trên với mỗi pj .j D 1; 2015/
ˇ a C1
luôn tồn tại aj mà pj ˇ aj j C .aj C 1/aj : Xét hệ phương trình đồng dư
(
a  aj .mod pj /
j D 1; 2015

Theo định lý thặng dư Trung Hoa hệ luôn tồn tại vô hạn số a như vậy. Ta có điều chứng minh.
Định lý thặng dư Trung Hoa và một số ứng dụng 259

Nhận xét. Qua hai ví dụ trên ta thấy một lời giải đẹp nếu ta biết cách sử dụng thành thạo Định
lý hệ thặng dư Trung Hoa.

Ví dụ 26. Tìm tất cả các số nguyên a; b; c với c  0 sao cho .an C 2n /j .b n C c/ với mỗi số
nguyên dương n đồng thời 2ab không là sô chính phương.

(The 54t h IMO Team Selection tests - 2013/

Lời giải. Ta có .an C 2n / j .b n C c/; tương đương với

bn  c .mod an C 2n /;

suy ra
b 3n C c 3  0 .mod an C 2n /: ./
ˇ
Lại có .a3n C 23n / ˇ .b 3n C c/ tương đương với b 3n C c  0 .mod a3n C 23n /; mà

an C 2n j a3n C 23n ;

suy ra
b 3n C c  0 .mod an C 2n /: ./
Từ ./; ./ ta suy ra c 3 c  0 .mod an C 2n /; hay

an C 2n j c.c C 1/.c 1/: .  /

Khi cho n ! C1 ta được jc.c C 1/.c 1/j < jan C 2n j ; kết hợp .  / ta được

c.c C 1/.c 1/ D 0;

tức c D 0 _ c D 1:

 Nếu c D 0 W
ˇ
ı Khả năng a D 2 theo đề bài 2nC1 ˇ b n suy ra 4 j b thỏa mãn điều kiện.
ı Khả năng jaj ¤ 2 tập các ước số nguyên tố của fan C 2n ; n 2 N g là vô hạn. Thật
vậy nếu a j 2 thì a D 2a1 ; .a1 2 Z; ja1 j > 1/; suy ra an C 2n D 2n .a1n C 1/ với
mọi số tự nhiên k; l và k ¤ l:
k l
Ta có .a12 C 1; a12 C 1/  2; suy ra fan C 2n ; n 2 Ng ; có vô hạn ước số nguyên
k k l l
tố. Nếu a lẻ thì suy ra .a2 C 22 ; a2 C 22 / D 1; suy ra tiếp fan C 2n ; n 2 Ng có
vô hạn ước số nguyên tố, do an C 2n j b n nên b có vô hạn ước số nguyên tố (vô lý).

 Nếu c D 1 W
ˇ
ı Khả năng 2 j a theo đề bài a2 C 22 ˇ b 2 C 1 suy ra b 2 C 1  0 .mod 4/ vô lý.
ı Khả năng a lẻ, do 2ab không là số chính phương cho nên
(
2a D l 2 p1 p2    ps
b D m2 pk pkC1    pt Cs _ b D m2
260 Các phương pháp giải toán qua các kỳ thi Olympic

Với l; m 2 N ; p1 ; p2 ; : : : ; pt Cs 2 }; t; k 2 N ; s 2 N; các số nguyên tố pi .i D 1; t C s/


phân biêt, vậy k D 1; s D 0 vô lý do 2ab là số chính phương.
Theo định lý thặng dư Trung Hoa tồn tại số nguyên tố p sao cho p  1 .mod 4/:
   
 b D m2 suy ra ppi D 1; .i D 2; t/; pp1 D 1:
     
pi pi p1
 k > t suy ra p
D 1; .i D 2; t/; p
D 1; 8i D k; t C s & p
D 1:
   
pi p1
 1 < k  t suy ra p
D 1; .i D 2; t C s/; p
D 1:
   
pi pi
 k D 1; s > 0 suy ra p
D 1; .i D 2; t C s 1/; p
D 1; 8i D k; t C s và
   
p1 pt Cs
D D 1:
p p
    p 1
2a b
Từ đó ta được p
D 1; p
D 1; theo tiêu chuẩn Euler’s .2a/ 2 D 1 .mod p/; suy ra
p 1 p 1 p 1
a 2 C 2 2  0 .mod p/; b 2 D 1 .mod p/:
p 1
ˇ p 1
p 1 ˇ p 1
Nhưng a 2 C 2 2 ˇ b 2 C 1 nên b 2  1 .mod p/ mâu thuẫn (do p ¤ 2/:

Vậy các bộ .a; b; c/ D .2; 4k; 0/; k 2 N ; k không là số chính phương thỏa mãn đề bài.
Nhận xét. Một ví dụ khá khó của bài viết, bài toán là sự hợp giữa đồng dư thức, thặng dư bình
phương và một chút của định lý thặng dư Trung Hoa, cái khéo của bài toán qua viêc chọn các số
2a và b sao cho 2ab không là số chính phương.
2016
Ví dụ 27. Tìm tất cả các số nguyên dương n để số n22 n 76 là số chính phương.
2016
Lời giải. Giả sử tồn tại số nguyên dương n để số n22 n 76 là số chính phương. Ta có
2016 2015 2015
A D 22 1 D .22 C 1/.22 1/
22015 22014 21 0
D .2 C 1/.2 C 1/    .2 C 1/.22 C 1/
2015
22016 l
Y
D2 1D3 .22 C 1/:
lD1

l m l
Gọi d D .22 C 1; 22 C 1/; 1  l ¤ m  2015 ta có d j 2 suy ra d D 1 _ d D 2; do 22 C 1
l
lẻ suy ra d D 1: Do đó các số 22 C 1; .l D 1; 2015/ nguyên tố cùng nhau nên theo định lý
thặng dư Trung Hoa, sẽ tồn tại số nguyên C sao cho
l 1 l
C  22 .mod 22 C 1/; 8l D 1; 2015:

Suy ra
l l
C 2 C 1  22 C 1 .mod 22 C 1/;
hay
l
C 2 C 1  22 C 1; 8l D 1; 2015:
Định lý thặng dư Trung Hoa và một số ứng dụng 261

Suy ra tiếp
2015
: Y  2l 
3.C 2 C 1/ :: 3 2 C 1 D A;
lD1

hoặc
:
81.C 2 C 81/ :: A;
Do đó tồn tại
2016
n 2 N W .9c/2 C 81 D n.22 1/:
2016
Suy ra .9c/2 C 5 D n22 n 76 vô lý do một số chính phương khi chia cho 9 cho các số dư
là 0; 1; 4; 7: Nhưng .9c/2 C 5  5 mod 9: Vậy điều giả sử là sai tương đương với không có số
2016
nguyên dương n để số n22 n 76 là số chính phương.

Ví dụ 28. Một số nguyên n được gọi là số tốt nếu jnj không là số chính phương. Xác định tất
cả các số nguyên m sao cho m có thể biểu diễn bằng vô hạn cách là tổng của 3 số tốt khác nhau
và tích của chúng là một số chính phương lẻ.

Lời giải. Điều kiện cần: Giả sử m D u C v C w; với u; v; w là các số tốt và uvw là số chính
phương lẻ. Khi đó ta có 8
ˆ
ˆ u  1; 3 .mod 4/
ˆ
< v  1; 3 .mod 4/
ˆ
ˆ w  1; 3 .mod 4/
ˆ
uvw  1 .mod 4/
:

Suy ra m D u C v C w  3 .mod 4/:


Điều kiện đủ: Ta chứng minh với mọi số nguyên m mà m  3 .mod 4/ đều thỏa mãn yêu cầu
của bài toán. Trước tiên ta chứng minh:
Với mọi số nguyên dạng m D 4k C 3; .k 2 Z/ đều phân tích được về dạng

m D 4k C 3 D xy C yz C zx; .1/

trong đó x; y; z là các số nguyên lẻ.


Thật vậy nếu chọn x D 1 C 2t và y D 1 2t; .t 2 Z n f0g/ thì x; y là hai số lẻ và khi đó .1/
sẽ trở thành
4k C 3 D 1 4t 2 C .1 C 2t/z C .1 2t/z; .2/
suy ra z D 2t 2 C 2k C 1 nên z cũng là số nguyên lẻ.
Với mỗi số nguyên m có dạng .1/ và với chọn x; y; z như trên ta có các kết quả sau:

 Nếu t 2 Zn f0g thì có vô hạn bộ số xy; yz; zx phân biệt thỏa mãn .1/:

 Tích các số xy; yz; zx là mọt số chính phương lẻ.

 Vậy ta chỉ cần chứng minh có vô hạn t 2 Zn f0g sao cho jxyj ; jyzj ; jzxj là các số tốt.

ı Trước hết ta thấy jxyj D 4t 2 1 là số tốt với t 2 Zn f0g :


262 Các phương pháp giải toán qua các kỳ thi Olympic

ı Chọn hai số nguyên tố phân biệt p; q > m: Ta xét hệ phương trình đồng dư ẩn t
(
1 C 2t  p .mod p 2 /
.3/
1 2t  q .mod q 2 /

Theo định lí thặng dư Trung Hoa hệ phương trình .3/ có vô số nghiêm.


ı Với mỗi t ta có z không chia hết cho p; ngược lại z chia hết cho p thì từ .2/ và .3/ ta
có p là ước của 4k C 3; điều này vô lý vì p > m:
ı Từ đó ta có jzxj chia hết cho p nhưng không chia hết p 2 : Tương tự jyzj chia hết
cho q nhưng không chia hết q 2 : Vậy jzxj ; jyzj là các số tốt, đây là điều phải chứng
minh.

Vậy đáp số của bài toán là m  3 .mod 4/

Nhận xét. Một ví dụ khó của bài viết, bài toán là sự kết hợp giữa định lý thặng dư Trung Hoa,
việc chọn a1 ; a2 và m1 ; m2 và lập luận logic, số chính phương ...

Ví dụ 29. Trong lưới điểm nguyên của mặt phẳng tọa độ Oxy; một điểm có tọa độ là các số
nguyên A.x; y/ 2 Z2 được gọi là nhìn thấy được từ điểm O; nếu trên đoạn OA không có điểm
nào thuộc Z2 ; trừ O và A: Chứng minh rằng với mọi số tự nhiên n tùy ý, luôn tồn tại hình vuông
n  n có các đỉnh nguyên và mọi điểm nguyên bên trong và trên biên của hình vuông đều không
nhìn thấy được từ điểm O:

(Shortlisted IMO 2002)

Lời giải. Ta có nếu .x; y/ D d thì điểm M dx ; dy là điểm nguyên thuộc đoạn OA với A.x; y/:


Do đó A.x; y/ là điểm nhìn thấy được từ điểm O khi và chỉ khi .x; y/ D 1: Gọi pi; j là các số
nguyên tố đôi một khác nhau, với 0  i; j  n (có .n C 1/2 số nguyên tố như vậy). Xét hai hệ
phương trình đồng dư tuyến tính sau đây
8
ˆ
ˆ x  0 .mod p0; 0 p0; 1 p0; 2    p0; n /
ˆ
< x  1 .mod p p p    p /
1; 0 1; 1 1;2 1; n
ˆ
ˆ 
ˆ
x  n .mod pn; 0 pn; 1 pn; 2    pn; n /
:

và 8
ˆ
ˆ y  0 .mod p0; 0 p0; 1 p0; 2    p0; n /
ˆ
< y  1 .mod p p p    p /
1; 0 1; 1 1; 2 1; n
ˆ
ˆ   
ˆ
y  n .mod pn; 0 pn; 1 pn; 2    pn; n /
:

Theo định lý thặng dư Trung Hoa thì hai hệ phương trình trên đều có nghiệm nên tồn tại các số
tự nhiên x; y như vậy. Mà x C i và y C j đều chia hết cho pi; j : Do đó mọi điểm trong hình
vuông n  n với .n C 1/2 điểm nguyên Ai; j .x C i; y C j / trên đều không nhìn thấy được từ
điểm O:
Định lý thặng dư Trung Hoa và một số ứng dụng 263

0 0 0
Ví dụ 30. Tìm số các bộ sắp thứ tự .a; b; c; a ; b ; c / thỏa mãn
8 0 0

< ab C a b  1
ˆ .mod 15/
0 0
bc C b c  1 .mod 15/
ˆ
: 0 0
ca C c a  1 .mod 15/
0 0 0
Với a; b; c; a ; b ; c 2 f0; 1; 2; : : : ; 14g :
(Việt Nam MO 2013)
0 0 0
Lời giải. Với mỗi số nguyên dương k; gọi Nk là số bộ sắp thứ tự .a; b; c; a ; b ; c / thỏa
0 0 0 0 0 0 0 0 0
mãn điều kiện ab C a b  bc C b c  ca C c a  1 .mod k/ và a; b; c; a ; b ; c 2
f0; 1; 2; : : : ; 14g :
Theo định lý thặng dư Trung Hoa thì Nmn D Nm  Nn nếu m; n 2 N và .m; n/ D 1: Do đó
để tính giá trị của N15 ; ta cần tính giá trị của N3 và N5 : Trước tiên ta tính Np với mỗi p là số
0 0 0 0
nguyên tố. Cố định các giá trị .a; b; a ; b / của phương trình ab C a b  1 .mod p/; ta cần
tính số nghiệm của hệ sau
0 0 0 0
bc C b c  ca C c a  1 .mod p/; .1/
Ta xét các trường hợp sau:
0
 Nếu .a; a / 6 t.b; b 0 / .mod p/ với mọi t 2 f0; 1; 2; : : : ; p 1g : Khi đó hệ .1/ có một
0 0 0
nghiệm duy nhất c  aa0 b bb0 a .mod p/; c  aba0 ab 0 b .mod p/:
0 0
 Nếu .a; a /  t.b; b / .mod p/ với mọi t ¤ 1: Khi đó hệ .1/ không có một nghiệm.
0 0
 Nếu .a; a /  .b; b / .mod p/: Khi đó hệ .1/ trở thành một phương trình duy nhất là:
0 0
bc C b c  1 .mod p/:
0 0 0
Do ba C b a  1 .mod p/; ta có thể giả sử rằng b ¤ 0: Do đó với mỗi cách chọn c ;
0 0
ta có duy nhất một cách chọn c  1 bb c .mod p/: Điều này cho thấy hệ .1/ có đúng p
nghiệm.
0 0 0 0 0 0
Đặt Tp là số bộ sắp thứ tự .a; b; a ; b / thỏa mãn ab C a b  1 .mod p/ và a; b; a ; b thuộc
0 0
tập f0; 1; 2; : : : ; p 1g : Với mỗi bộ .a; a / ¤ .0; 0/; có đúng p cặp .b; b / thỏa mãn phương
trình. Suy ra Tp D p.p 2 1/: Đặt Cp là số bộ sắp thứ tự .a; b/ thỏa mãn a2 C b 2  t .mod p/
và a; b 2 f0; 1; 2; : : : ; p 1g : Từ lập luận ở trên ta có
p 1
X
Np D Tp Cp .t/ C pCp .1/ D p.p 2 1/ p 2 C Cp .0/ C pCp .1/: .2/
i D1

Ta dễ dàng tính được C3 .0/ D 1; C3 .1/ D 4; C5 .0/ D 9; C5 .1/ D 4 suy ra N3 D 28; N5 D


0 0 0
124 và N15 D 28  124 D 3472: Vậy số các bộ .a; b; c; a ; b ; c / thỏa mãn điều kiện đề bài
là 3472:
Nhận xét. Một ví dụ áp dụng định lý thặng dư Trung Hoa khá cơ bản, ta cần nhớ về ánh xạ
“phục hồi”  W Zp  Zq D Zpq vốn là ứng dụng quan trọng nhất của định lý thặng dư TrungHoa
này: Một số thuộc Zpq được xác định một cách duy nhất qua cặp số dư của nó khi chia cho p và
q: Từ đó ta có thể chuyển các bài toán trên Zpq về các bài toán trên Zp và trên Zq :
264 Các phương pháp giải toán qua các kỳ thi Olympic

5. Các bài tập rèn luyện


Bài tập 78. Với 0 < x < 120 giải hệ phương trình đồng dư sau
8
ˆ
ˆ x  1 .mod 2/
ˆ
< x  2 .mod 3/
ˆ
ˆ x  3 .mod 4/
ˆ
x  4 .mod 5/
:

Bài tập 79. Giải hệ phương trình đồng dư


8
ˆ
ˆ x  2 .mod 11/
ˆ
< x  3 .mod 12/
ˆ
ˆ
ˆ
x  4 .mod 13/
ˆ
ˆ
ˆ
ˆ
ˆ x  5 .mod 17/
ˆ
: x  6 .mod 19/

Bài tập 80. Cho các số nguyên dương n; h; d: Chứng minh rằng luôn tồn tại một cấp số cộng
n số hạng có công sai d; sao cho mọi số hạng của cấp số cộng đều có ít nhất h ước số nguyên tố
phân biệt.
Bài tập 81. Tìm tất cả các số tự nhiên n sao cho 2n 1 chia hết cho 3 và tồn tại m 2 Z sao
cho 4m2 C 1 chia hết cho 2n3 1 :
(Korea MO 1999)
Bài tập 82. Cho f .x/ là đa thức với hệ số nguyên. Giả sử rằng có một tập hữu hạn các số
nguyên tố A D fp1 ; p2 ; : : : ; pn g sao cho với mọi số nguyên a luôn tồn tại số pi 2 A sao cho
f .a/ chia hết cho pi : Chứng minh rằng tồn tại một số nguyên tố p sao cho f .x/ chia hết cho p
với mọi số nguyên x:
Bài tập 83. Cho các số nguyên dương a; b: Chứng minh rằng luôn tồn tại n số liên tiếp của
dãy số a C b; a C 2b; a C 3b; : : : ; a C nb; là hợp số.
Bài tập 84. Chứng minh rằng với mọi số nguyên dương n; luôn tồn tại mọt tập hợp S gồm n
phần tử, sao cho bất kì một tập con nào của S cũng có tổng các phần tử là lũy thừa của một số tự
nhiên.
Bài tập 85. Chứng minh rằng với mọi số nguyên dương n; luôn tồn tại n số liên tiếp của dãy
số sao cho bất kì số nào trong dãy cũng đều có ước dạng 2k 1; với k là số tự nhiên.
Bài tập 86. Chứng minh rằng không tồn tại đa thức f .x/ với hệ số nguyên có bậc nguyên
dương, sao cho f .k/ là số nguyên tố với mọi số nguyên dương k:
Bài tập 87. Chứng minh rằng tồn tại một dãy só tăng fan gC1
nD1 các số tự nhiên sao cho với mọi
C1
k 2 N; dãy fk C an gnD1 chỉ chứa hữu hạn các số nguyên tố.
(Czech-Slovak 1997)
Bài tập 88. Chứng minh rằng tồn tại vô hạn số nguyên dương a thỏa mãn các điều kiện sau:
Định lý thặng dư Trung Hoa và một số ứng dụng 265

.i / Tồn tại x; y 2 Z; .x; y/ D 1 sao cho a2 D x 3 C y 3 :

.i i / Tồn tại b 2 Z sao cho b 2 C 3 chia hết cho a2 a2 C 3/

Bài tập 89. Cho f1 .x/ ; f2 .x/ ; : : : ; fn .x/ là n đa thức với hệ số nguyên khác 0: Chứng minh
rằng tồn tại đa thức P .x/ với hệ số nguyên sao cho với mọi i D 1; n ta luôn có P .x/ C fi .x/
là đa thức bất khả quy trên Z:
Bài tập 90. Ta gọi một tập hợp các số nguyên dương C là tốt nếu với mọi số nguyên dương k
thì tồn tại a; b khác nhau trong C sao cho .a C k; b C k/ > 1: Giả sử ta có một tập tốt mà tổng
các phần tử trong đó bằng 2003: Chứng minh rằng ta có thể loại đi một phần tử c trong C sao
cho tập còn lại vẫn là tập tốt.
(Bulgaria TST 2003)

Bài tập 91. Chứng minh rằng với mọi số nguyên dương n .n  2/ ; luôn tồn tại hai số nguyên
dương a; b sao cho .a C i; b C j / > 1; 8i; j 2 f1; 2; : : : ; n 1g :

Bài tập 92. Ta gọi một hình vuông là hình vuông tốt, nếu nó có 4 đỉnh là các điểm nguyên,
đồng thời đoạn thẳng nối tâm O với tất cả các điểm nguyên trên biên và trong hình vuông đó
chưa ít nhất một điểm nguyên khác hai đầu mút. Chứng minh rằng với mọi số nguyên dương n
đều tồn tại một hình vuông tốt dạng n  n:
˚
Bài tập 93. Tìm số nguyên dương n sao cho với mọi hệ thặng dư thu gọn n là a1 ; a2 ; : : : ; a'.n/
ta có a1 a2    a'.n/  1 .mod n/:
Bài tập 94. Chứng minh rằng tồn tại một dãy só tăng fan gC1 nD1 các số tự nhiên sao cho với mọi
n thì a1 a2    an 1 là tích của hai số nguyên liên tiếp.
(USA-TST 2009)

Bài tập 95. .a/ Chứng minh rằng tập các số nguyên có thể phân hoạch thành các cấp số cộng
với công sai khác nhau.
.b/ Chứng minh rằng tập các số nguyên không thể viết được dưới dạng hợp của các cấp số cộng
với công sai đoi một nguyên tố cùng nhau.
(Moldova TST 2009)
Bài tập 96. Cho số nguyên dương n D p1˛1 p2˛2    pk˛k ; trong đó p1 ; p2 ; : : : ; pk là các số
nguyên tố đôi một khác nhau. Tìm số nghiệm của phương trình đồng dư x 2 C x  0 .mod n/:
Bài tập 97. Cho tập An D f a 2 N j 1  a  n; .a; n/ D 1g : Tìm jAn j :
0 0 0 
Bài tập 98. Cho p là số nguyên tố, gọi f .p/ là số tất cả bộ sắp thứ tự a; b; c; a ; b ; c
thỏa hệ phương trình đồng dư
8 0 0

< ab C a b  x .mod p/
ˆ
0 0
bc C b c  .mod p/
ˆ
: 0 0
ca C c a  z mod p
0 0 0
Với a; b; c; a ; b ; c 2 f0; 1; 2; : : : ; p 1g và 0  x; y; z  p 1: Tìm f .p/ :
266 Các phương pháp giải toán qua các kỳ thi Olympic

Bài tập 99. Tìm tất cả các số nguyên dương n sao cho nếu .x; n/ D 1 thì x 2  1 .mod n/:
2015
Bài tập 100. Tồn tại hay không số nguyên dương n để n22 n 81 là số chính phương.

Bài tập 101. Cho số nguyên dương n: Chứng minh rằng tồn tại số nguyên dương m thỏa mãn
hệ đồng dư. (
2m  2015 .mod 3n /
2m  32015 .mod 2n /

Tài liệu tham khảo


[1] Hà Huy Khoái, Số học.

[2] Nguyễn Vũ Lương, Các bài giảng về số học.

[3] Tài liệu tập huấn giáo viên chuyên Toán toàn quốc 2011, 2012.

[4] Tạp chí Toán Học và Tuổi Trẻ.

[5] Đề thi học sinh giỏi lớp 12 các tỉnh, thành phố.

[6] Tuyển tập dự tuyển OLYMPIC toán hoc Quốc tế Từ năm 1991 2014:

[7] Dan Brânzei, Ioan Serdean, Vasile Serdean, JunorBalkan Mathematical Olympiads.

[8] Titu Andreescu, Dorin Andrica, DiophantinEquations.

[9] Vasile Berinde, Gazeta Matematică-A bridge.

[10] Mathematical Reflections.

[11] Nguyễn Văn Nho, OLYMPIC toán học Châu Á Thái Bình Dương.

[12] Nguyễn Văn Nho, Số học nâng cao.

[13] Titu Andreescu, Razvan Gelca, Mathematical Olympiad Challenges 2001:

[14] itu Andreescu, Bogdan Enescu, Mathematical Olympiad Treasures 2004 Birkhauser Boston,
USA.

[15] Vô địch các quốc gia và vùng lãnh thổ từ 1991 2014:
S(N) – HÀM TỔNG CÁC CHỮ SỐ
Lê Phúc Lữ
(FPT Software, thành phố Hồ Chí Minh)

1. Kiến thức cần nhớ


Gọi S.n/ là hàm tổng các chữ số của số nguyên dương n viết trong hệ thập phân. Cụ thể là nếu
biểu diễn n D ak ak 1 : : : a2 a1 với a1 ; a2 ; : : : ; ak là các chữ số và ak ¤ 0 thì
n
X
S.n/ D ai :
i D1

Dễ thấy các tính chất cơ bản sau, với mọi số nguyên dương m; n thì:

1) 0 < S.n/  n.
:
2) S.n/ n :: 9.

˚
Pk
Hai tính chất này khá hiển nhiên vì nếu n có k chữ số thì n D i D1 10i 1
ai và do đó 10a  1
với mọi số tự nhiên a nên

k
X k
X
i 1
nD 10 ai  ai D S.n/
iD1 i D1
k k
:
: : : …9 ai ::9
X X
i 1

n S.a/ D 10 1 ai D 999
„ ƒ‚
i D1 i D1 i 1

3) S.m C n/  S.m/ C S.n/

4) S.mn/  S.m/  S.n/:

Đây là hai tính chất quan trọng và có sử dụng khá nhiều trong các bài toán liên quan đến tổng
các chữ số. Điểm mấu chốt của chứng minh, và cũng là điểm khó xử lý là phép cộng có nhớ.
Gọi c là số lần cộng có nhớ trong việc tính tổng m C n thì ta có

S.m C n/ D S.m/ C S.n/ 9c  S.m/ C S.n/:

Tương tự, ta có S.mn/ D S.m/S.n/ 9c  S.m/S.n/ với c là số phép tính có nhớ trong tích.
Nếu chứng minh trực tiếp, ta để ý kết quả sau và từ đó quy nạp lên:

5) S.10k m C n/ D S.m/ C S.n/ với n là số có không quá k chữ số.

267
268 Các phương pháp giải toán qua các kỳ thi Olympic

Điều này đúng do 10k m có tận cùng bởi ít nhất k chữ số 0 nên khi cộng với n, toàn bộ các chữ
số của n cộng tương ứng với các số 0 và không có phép nhớ nào. Khi đó, biểu diễn
k
X h
X
i
m D ak ak 1 : : : a2 a1 a0 D 10 ai ; n D bh bh 1 : : : b2 b 1 b 0 D 10i bi :
i D0 i D0

Ta có
n n n
!
X X   X
k k
S.mn/ D S 10 mbk  S 10 mbk D S .mbk /
kD0 kD0 kD0
Xn Xn
 bk S.m/ D S.m/ bk D S.m/S.n/:
kD0 kD0

Ngoài ra, ta cũng còn có một số tính chất cơ bản sau:

6) S.n/  9 blog10 n C 1c.

Thật vậy, do k D blog10 n C 1c chính là số các chữ số của n, mỗi chữ số có giá trị không vượt
quá 9 nên rõ ràng S.n/  9k D 9 blog10 n C 1c.

7) S.2m C 1/ D S.2m/ C 1.

Ta thấy rằng do số 2m chẵn nên không thể có tận cùng là 9; vì thế phép tính 2m C 1 không có
nhớ. Do đó nó có giá trị đúng bằng tổng các chữ số của số liền trước đó cộng thêm 1:

8) S.999
„ ƒ‚: : : …9 a/ D 9k S.a/ với a là số có không quá k chữ số.
k

Ta thấy rằng a có thể viết dưới dạng a1 a2 : : : ak với 0  ai  9; i D 1; k. Ta có

999 : : : 9 a D .9 a1 /.9 a2 /.9 a3 / : : : .9 ak /:

Từ đây suy ra
k
X k
X
S.999
„ ƒ‚: : : …9 a/ D .9 ai / D 9k ai D 9k S.a/:
k i D1 i D1

PC1 j n
k
9) S.n/ D n 9 kD1 10k
.

Đặt n D am am 1 am 2 : : : a1 a0 thì n D am 10m C am 1 10m 1


C : : : C 10a1 C a0 . Ta có
1  
X n
k
D .am 10m 1
C : : : C a1 / C .am 10m 2
C : : : C a2 / C   
kD1
10

Sắp xếp lại các tổng, ta được

.am C 10am C : : : C 10m 1 am / C .am 1 C : : : C am 10m 2 / C : : : C a1 :


S.n/ – hàm tổng các chữ số 269

Do đó
C1  
X N
9 k
D am .10m 1/ C am 1 .10m 1
1/ C    C a1 .10 1/:
kD1
10
Vậy ta được
C1  
X n
n 9 D am C am 1 C am 2 C    C a1 C a0 D S.n/:
kD1
10k

Từ công thức trên đây, ta thấy bất đẳng thức S.m C n/  S.m/ C S.n/ là đúng theo bất đẳng
thức phần nguyên bm C nc  bmc C bnc: Dưới đây là một số bài toán liên quan đến việc tính
tổng các chữ số cũng như vận dụng khéo léo các tính chất đã nêu ở trên. Trong đó, ta cũng có thể
phát hiện ra thêm nhiều tính chất thú vị mới.

2. Các dạng toán thường gặp


2.1. Dạng 1: Tính giá trị của biểu thức
Bài toán 1 (Hongkong TST, 1994). Gọi m; n là các số nguyên dương và m có d chữ số với
d  n: Tính S ..10n 1/m/.
Lời giải. Đặt m D a1 a2 : : : ad thì ta biểu diễn .10n 1/m D .m 1/10n C999
„ ƒ‚: : : …9 .m 1/.
n
Do n  d nên số .m 1/  10n có ít nhất n chữ số 0 tận cùng. Suy ra
!
S ..10n 1/m/ D S ..m 1/10n / C S 999
„ ƒ‚: : : …9 .m 1/
n
!
D S.m 1/ C S 999
„ ƒ‚: : : …9 .m 1/ :
n

Theo tính chất đã nêu thì


!
S 999
„ ƒ‚: : : …9 .m 1/ D 9n S.m 1/:
n

Từ đây suy ra tổng các chữ số cần tính là 9n:


Nhận xét. Đây là một kết quả quan trọng và ta còn sử dụng nó để xử lý nhiều bài toán ở phía
sau. Trong các hệ cơ số khác, kết quả này vẫn đúng.
Ta có bài toán tương tự áp dụng của bài toán trên như sau:
 n 
1. (USA MO, 1992) Đặt A D 9  99  9999      102 1 ; tính S.A/:

  n 
k
Theo kết quả trên thì S .10 1/m D 9k với .m; 10/ D 1. Ta có A D 102 1 m
và .m; 10/ D 1. Do đó S.A/ D 9  2n .
270 Các phương pháp giải toán qua các kỳ thi Olympic

2. Cho số nguyên dương m không chia hết cho 10: Chứng minh rằng tồn tại số nguyên dương
n thỏa mãn đồng thời:
 Số n không có chứa chữ số 0 nào.
 S.n/ D S.mn/:
Từ bài toán 1, rõ ràng số cần tìm là n D 10k 1 với k là số chữ số của m:
Bài toán 2.
a) Tính giá trị của A D S.1/ S.2/ C S.3/ S.4/ C    C S.2014/ S.2015/:
b) Tính giá trị của B D S.3/ C S.6/ C S.9/ C : : : C S.300/:
Lời giải. a) Với mọi n nguyên dương thì S.2nC1/ D S.2n/C1. Suy ra S.2nC1/ S.2n/ D 1
với mọi n: Do đó,
1007
X
A D S.1/ C ŒS.2n C 1/ S.2n/ D 1008:
nD1

b) Ta có thể coi các số 3; 6; 9;    ; 297 là các số nguyên dương có 3 chữ số và bắt đầu bởi 0; 1
hoặc 2; ngoài ra, nó được xác định bởi 2 chữ số cuối. Mỗi chữ số 0; 1; 2;    ; 9 xuất hiện 10 lần
trong chữ số hàng đơn vị và 10 lần trong chữ số hàng chục; còn mỗi số 0; 1; 2 xuất hiện 33 lần
trong chữ số hàng trăm. Suy ra
S.0/ C S.3/ C : : : C S.297/ D .0 C 1 C : : : C 9/.10 C 10/ C 33.0 C 1 C 2/ D 999:
Do đó, tổng cuối cùng chỉ cần bổ sung thêm S.300/ D 3 và ta được B D 1002.
Bài toán 3.
a) (ZIMO, 2008) Tìm tất cả các số n sao cho n D 2S 3 .n/ C 8.
b) Có bao nhiêu số tự nhiên không vượt quá 5000 và có tổng các chữ số chia hết cho 4?
Lời giải. a) Gọi d là số chữ số của n: Ta có n  10d 1
và S.n/  9d . Suy ra
10d 1
 2.9d /3 C 8:
Ta thấy bất đẳng thức này sai với d  8 nên ta chỉ cần xét d  7: Khi đó, ta có S.n/  63: Chú
ý rằng n  S.n/ theo modulo 9 nên
2S.n/3 C 8 S.n/  0 .mod9/
3
, 2S.n/ S.n/ 1  0 .mod9/
, .S.n/ 1/ 2S 2 .n/ C 2S.n/ C 1  0 .mod9/

"
S.n/  1 .mod9/
,
2S 2 .n/ C 2S.n/ C 1  0 .mod9/

Phương trình đồng dư thứ 2 vô nghiệm do


2S 2 .n/ C 2S.n/ C 1 D S 2 .n/ C .S.n/ C 1/2  0 .mod9/
mà 9 có ước nguyên tố là 3 nên cả hai số S.n/; S.n/ C 1 đều phải chia hết cho 3; vô lý. Suy
ra S.n/  1 .mod9/ và ta có S.n/ D 1; 10; 19; 28; 37; 46; 55. Thử trực tiếp các số này, ta thấy
nghiệm cần tìm là 10; 2008; 13726:
S.n/ – hàm tổng các chữ số 271

b) Ta xét các trường hợp:

 Nếu số tự nhiên cần tìm là 4 chữ số với dạng abcd và 1  a  4 thì mỗi cách chọn chuỗi
bcd , tồn tại duy nhất một cách chọn a (nếu b Cc Cd chia 4 dư k thì chọn a  k . mod 4/).
Suy ra có 103 D 1000 số thỏa mãn.

 Còn lại, ta xét các số từ 0 đến 999; ta thấy có ba chữ số chia hết cho 4, ba chữ số chia 4 dư
1, hai chữ số chia 4 dư 2 và hai chữ số chia 4 dư 3. Hàm sinh biểu diễn số các chữ số khi
chia cho 4 theo các số dư là

f .x/ D 3 C 3x C 2x 2 C 2x 3 :

Ta cần tìm tổng hệ số của tất cả các số hạng có số mũ chia hết cho 4 trong khai triển của
f 3 .x/: Theo định lý RUF (Root of unity filter), đại lượng đó chính là
1 3
f .1/ C f 3 .i/ C f 3 . 1/ C f 3 . i/ với i2 D

1:
4
Tổng cần tính là 14 .1000 2 C 2i C 9 2 2i/ D 249.

Vậy tổng cộng có 1000 C 249 D 1249 số thỏa mãn đề bài.

Bài toán 4.

a) (Nga, 1984) Số n gọi là số may mắn nếu S.n/ chia hết cho 7: Tìm số nguyên dương n nhỏ
nhất sao cho n; n C 1 đều là các số may mắn.

b) Hỏi nếu thay 7 bởi 2014 thì số cần tìm có ít nhất bao nhiêu chữ số?

Lời giải. a) Giả sử số n cần tìm có dạng m „


999ƒ‚ 9, trong đó k  0 và m có tận cùng khác 9:
: : :…
k

Dễ dàng thấy rằng S.n/ D S.m/ C 9k và S.n C 1/ D S.m/ C 1. Do đó, để S.n C 1/; S.n/ đều
chia hết cho 7 thì ta cần có 9k 1 chia hết cho 7: Số k nhỏ nhất thỏa mãn là 4: Ngoài ra, số m
nhỏ nhất mà S.m/ C 36 là m D 6: Vậy số cần tìm là 69999:
b) Tương tự trên, ta cũng cần tìm k nhỏ nhất sao cho 9k 1 chia hết cho 2014: Dễ thấy phương
trình 9k 2014m D 1 có nghiệm nguyên dương và m  5 nên k  20145C1 9
D 1119.
Khi đó S.m/ C 1 chia hết cho 2014 thì S.m/  2013 D 9  663 C 6 nên số cần tìm có ít nhất
1119 C 663 C 1 D 1783 chữ số. Số đó có dạng 999
„ ƒ‚: : : …9 8 „
999ƒ‚
: : :…
9.
663 chữ số 1119 chữ số

Bài toán 5.

a) (Nordic MOl 1993) Biết rằng a D 888


„ ƒ‚: : : …8; b D 555
„ ƒ‚: : : …5. Tính S.9ab/:
1985 1985

b) (PAMO, 2007) Tìm tất cả các số nguyên dương n có đúng 1112 chữ số sao cho S.n/; S.nC
1/ đều chia hết cho 2000 và 1 là một trong các chữ số của n:

c) Hỏi có tồn tại số nguyên dương a; b; c nào sao cho S.aCb/ < 5; S.bCc/ < 5; S.cCa/ <
5 và S.a C b C c/ > 50?
272 Các phương pháp giải toán qua các kỳ thi Olympic

Lời giải. a) Ta viết lại biểu thức như sau

: : : …4 0 D 10 101985

9ab D 9  888
„ ƒ‚: : : …8  555
„ ƒ‚: : : …5 D 999
„ ƒ‚: : : …9  444
„ ƒ‚ 1 444 : : : …4
„ ƒ‚
1985 1985 1985 1985 1985
!
D 10 444
„ ƒ‚: : : …4 000
„ ƒ‚: : : …0 444
„ ƒ‚: : : …4 D 444
„ ƒ‚: : : …4 3 „
555ƒ‚
: : :…
5 60
1985 1985 1985 1984 1984

Vậy S.9ab/ D 4  1984 C 3 C 5  1984 C 6 D 9  1985 D 17865.


b) Ta thấy rằng nếu n có tận cùng là k chữ số 9 thì 9k 1 chia hết cho 2000 và do 0  k  1112
nên k D 889: Gọi tổng của 1112 k D 223 chữ số đầu tiên là t thì t < 223  9 D 2007 và do
S.n/ D t C 9  889 chia hết cho 2000 nên t C 1 chia hết cho 2000 và t D 1999:
Do 1 là một trong 223 chữ số đầu tiên của n nên tổng của 222 số còn lại là 1998 D 222  9 và
điều này chỉ xảy ra khi toan bộ các số kia là 9:
Vậy số cần tìm là 999
„ ƒ‚: : : …9 1 „
999ƒ‚
: : :…
9 (chữ số thứ 223 là 1, còn lại đều là 9).
222 889

c) Câu trả lời là khẳng định. Ta chọn


€ a C b D 100000001110
b C c D 100001110000
c C a D 111100000000

Khi đó S.a C b C c/ D 51, thỏa mãn điều kiện.

Nhận xét. Chú ý rằng giá trị 51 cũng là GTLN của số S.a C b C c/ và đó là một câu trong đề
thi Olympic Toán toàn Nga.

Bài toán 6.

a) Tìm số nguyên dương n nhỏ nhất sao cho n có thể biểu diễn thành tổng của đồng thời:

 2007 số nguyên dương có cùng tổng các chữ số.


 2008 số nguyên dương khác cũng có cùng tổng các chữ số.

b) Câu hỏi tương tự khi thay 2007; 2008 bởi 2008; 2009:

Lời giải. a) Do 2007 chia hết cho 9 nên n cũng chia hết cho 9: Ngoài ra, do 2008 số nguyên
dương có tổng các chữ số chia hết cho 9 vì gcd.9; 2008/ D 1.
Suy ra n  9  2008. Ta thấy giá trị này cũng thỏa mãn vì

9  2008 D 9„ C 9 C ƒ‚
9 C : : : C…
9 D 18 C 9„ C 9 C
ƒ‚: : : C …
9:
2008 2006

b) Lập luận tương tự, ta thấy số thỏa mãn là 2008  2 D 223  10 C 1786  1.

Bài toán 7.

a) Tìm số nguyên dương k nhỏ nhất sao cho S.k/ D S.2k/ D    D S.2013k/ D S.2014k/.
S.n/ – hàm tổng các chữ số 273

b) Tìm GTNN của S.2017n/ với n là số nguyên dương.

Lời giải. a) Ta sẽ chứng minh rằng k D 9999 là GTNN cần tìm. Thật vậy:

 Nếu k có 1 chữ số thì S.11k/ D S.10k C k/ D S.10k/ C S.k/ D 2S.k/, trường hợp này
không thỏa mãn.

 Nếu k có 2 chữ số thì S.101k/ D S.100k C k/ D S.100k/ C S.k/ D 2S.k/; trường hợp
này cũng không thỏa mãn.

 Nếu k có 3 chữ số thì S.1001k/ D S.1000k C k/ D S.1000k/ C S.k/ D 2S.k/, trường


hợp này không thỏa mãn.

Suy ra k có ít nhất 4 chữ số. Trong trường hợp k có 4 chữ số, ta sẽ chứng minh rằng chỉ có số
k D 9999 thỏa mãn.
Đặt k D a3 a2 a1 a0 thì 1001k D a3 a2 a1 .a0 C a3 /a2 a1 a0 . Ta thấy rằng nếu phép cộng a3 C a0
không có nhớ thì S.1001k/ D 2.a3 C a2 C a1 C a0 / > S.k/, mâu thuẫn. Ngoài ra, nếu a1 < 9
thì S.1001k/ D a3 C a2 C .a1 C 1/ C 0 C a2 C a1 C a0 > S.k/, mâu thuẫn.
Suy ra a1 D 9. Lập luận tương tự, ta có a2 D a3 D 9 và k D 999a0 . Ta thấy a0 ¤ 0 vì nếu
không thì 1001k D 9999990 và S.1001k/ D 54 D 2S.k/, cũng mâu thuẫn.
Ngoài ra, do k chia hết cho 9 (vì k  S.k/  S.2k/  2k.mod9/) nên ta phải có a0 D 9. Do
đó, điều kiện cần của k là k D 9999:
Với k D 9999, theo bài toán 1 đã nêu thì ta có S.9999i/ D 36 với mọi 1  i  2014 nên thỏa
mãn điều kiện đề bài. Vậy số k nhỏ nhất cần tìm là 9999:

Nhận xét. Với n D 2012, ta có đề thi Rioplatense San Isidro 2012. Hãy thử tổng quát bài
toán lên khi thay các số 2012; 2014 bởi các số nguyên dương tùy ý.

b) Trước hết, ta thấy 2017n không thể có dạng 10k nên S.2017n/  2: Ta sẽ chứng minh 2 là
GTNN cần tìm. Chú ý rằng 2017 là số nguyên tố nên theo định lý Fermat nhỏ thì 102016 1 chia
hết cho 2017: Mặt khác 102016 1 D .1063 1/.1063 C 1/.10126 C 1/.10504 C 1/.101008 C 1/.
Ta sẽ chứng minh rằng 1063 1 không chia hết cho 2017. Thật vậy,
Ta tính trực tiếp các số dư của phép lấy lũy thừa.

104  85.mod2017/; 108  1174.mod2017/; 1016  665.mod2017/;


:
1032  502.mod2017/; 1064  121.mod2017/:

Suy ra
1064 10  131.mod2017/:
Ngoài ra, do 1064 10 D 10 1063 1 nên 1063 1 không chia hết cho 2017:


Từ đây suy ra một trong các biểu thức còn lại chia hết cho 2017 và rõ ràng tổng các chữ số của
số đó là 2: Vậy GTNN cần tìm là 2:
274 Các phương pháp giải toán qua các kỳ thi Olympic

Nhận xét. Ở bài b, ta thấy vấn đề đặt ra khá thú vị, đó là tìm trong các bội số dương của 2017;
số có tổng các chữ số nhỏ nhất. Ý tưởng chính là thử chọn các trường hợp nhỏ

S.2017n/ D 1; 2; 3; : : :

để xem có tồn tại số n tương ứng thỏa mãn hay không. Rõ ràng câu trả lời cho trường hợp
tổng quát khi thay 2017 là số nguyên tố ở trên bởi một số nguyên dương bất kỳ là không dễ
dàng. Dưới đây có một bài toán tương đối tổng quát và xử lý được khá nhiều trường hợp (với
k D 3; 5; 11; 23; : : :).
(Brazilian MO, 2009) Đặt p D 2q C 1 với p; q nguyên tố. Chứng minh rằng tồn tại một bội của
q có tổng các chữ số không vượt quá 3.
Với q D 2003, ta có đề VMO 2004 với chú ý rằng p D 2003  2 C 1 D 4007 là số nguyên tố.
Ta giải bài toán này như sau:
Do p D 2q C 1 nên theo định lý Fermat nhỏ thì

10p 1
 1.modp/ , 102q  1.modp/ , 100q  1.modp/:

Đặt h D ordp 100 thì rõ ràng h là ước của q và chỉ có thể là 1 hoặc q: Ta xét 2 trường hợp:

 Nếu h D 1 thì 100  1 .modp/ , p j99 nên p 2 f3; 11g. Dễ thấy p D 3 , q D 1


không thỏa. Còn với p D 11 thì q D 5 thỏa mãn và khi đó, bản thân số 11 có tổng các số
là 2, thỏa mãn đề bài.

 Nếu h D q thì 100k chỉ có đúng q số dư khi chia cho p: Do 100k luôn là thặng dư chính
phương theo modp nên tồn tại đúng q thặng dư chính phương modp và với mọi thặng dư
chính phương x của p, luôn tồn tại k sao cho 100k  x .modp/.

Chú ý rằng phương trình x 2 C y 2 C 1  0.modp/ luôn có nghiệm x; y nên tồn tại a; b
sao cho 10a C 10b C 1  0.modp/. Khi đó, bội tương ứng này của q có tổng các chữ số
là 3; thỏa đề bài.

Trong mọi trường hợp, ta luôn có đpcm.

Bài toán 8.

a) Hỏi tập hợp fjS.n C 2/ S.n/j ; n 2 Ng có bao nhiêu giá trị không vượt quá 2014?

b) (British MO, 2013) Biết rằng số n có đúng 32013 số 3 trong biểu diễn nhị phân. Tính
v3 .n/:
S.n/
c) (Nhật Bản, 2012) Hỏi có bao nhiêu số nguyên dương n không quá 999 mà S.nC1/
là một
số tự nhiên?

Lời giải. a) Gọi di là chữ số thứ i của x (nhân với 10i ) và f là số nhận được của các số đó sau
khi cộng thêm 2. Đặt Ti D di f .di /. Xét số hàng đơn vị d0 , ta thấy rằng nếu 0  d0  7 thì
không có phép cộng có nhớ nào ở đây và T .x/ D 2.
Các giá trị khác của tập hợp đã cho sẽ được tạo thành khi d0 D 8 hoặc d0 D 9: Khi đó,
f .d0 / D 0; 1 tương ứng. Khi đó, ta có T0 D 8:
S.n/ – hàm tổng các chữ số 275

Tiếp theo, nếu 0  d1  8 thì f .d1 / D d2 C 1; T2 D 1 và quá trình cộng của số nhớ kết thúc.
Ngược lại, nếu d1 D 9; f .d1 / D 0; T1 D 9 thì số nhớ lại thêm vào d2 và cứ thế.
Giả sử quá trình này đến số thứ n C 1 thì
ˇnC1 ˇ ˇ ˇ
ˇX ˇ ˇ ˇ
T .x/ D ˇ Ti ˇ D ˇ8 C 9„ C 9 C : : : C…
9 1ˇ D 9n C 7:
ˇ ˇ ˇ ˇ
ˇ ˇ ˇ ƒ‚ ˇ
i D0 n

Do T .x/  1999 nên 9n C 7  1999 ) 0  n  221, có tổng cộng 222 số. Chú ý rằng có số 2
đã tính trước đó. Vậy tổng cộng có 223 giá trị của T .x/:
b) Đặt bk D 111
„ ƒ‚: : : …1 với k  1 thì ta có n D 3b2013 . Tiếp theo, ta xét số
3k

mk D 1 „
000ƒ‚ 01„
: : :… 000ƒ‚
: : :…
01
3k 1 3k 1

thì rõ ràng mk bk D bkC1 . Ta thấy S.mk / D 3 nên số mk chia hết cho 3 nhưng không chia hết
cho 9. Do đó v3 .bkC1 / D 1 C v3 .bk /.
Với k D 1, ta có b1 D 111 có v3 .b1 / D 1 nên suy ra với mọi k thì v3 .bk / D k.
Từ đó suy ra v3 .n/ D 1 C v3 .2013/ D 2014.
c) Ta thấy rằng nếu kí hiệu n D abc với a; b; c có thể bằng 0 thì S.n/ D a C b C c và S.n C 1/
có thể có các trường hợp:

1. S.n C 1/ D a C b C c C 1 nếu c < 9:

2. S.n C 1/ D a C b C 1 nếu c D 9; b < 9.

3. S.n C 1/ D a C 1 nếu b D c D 9; a < 9.

4. S.n C 1/ D 1 nếu a D b D c D 9:

Rõ ràng số n thỏa mãn sẽ không rơi vào trường hợp (1). Còn trường hợp (4) thì hiển nhiên thỏa
mãn điều kiện.
S.n/ aC18 17
Xét trường hợp .3/, ta thấy S.nC1/
D aC1
D1C aC1
là số nguyên khi và chỉ khi a D 0:
S.n/ aCbC9 8
Xét trường hợp (2), ta thấy S.nC1/
D aCbC1
D1C aCbC1
nên ta có a C b 2 f0; 1; 3; 7g.
Từ đó tìm được 15 bộ .a; b/ thỏa mãn.
Vậy kết hợp với các trường hợp, ta thấy có tổng cộng 17 số thỏa mãn.

Bài toán 9. Tìm n nguyên dương sao cho có các đẳng thức

a) S.11n / D 2n :

b) S.43n / D 2n 1.
276 Các phương pháp giải toán qua các kỳ thi Olympic

Lời giải. a) Ta có 11n D .10 C 1/n D niD0 Cni 10i : Ta thấy với n 2 f1; 2; 3; 4g thì
P

n n
!
X X
S.11n / D S Cni 10i D Cni D .1 C 1/n D 2n do Cni < 10:
i D0 iD0

Pn  Pn bnc
Với n  5 thì S.11n / D S i
i D0 Cn 10
i
< i D0 Cni D .1 C 1/n D 2n vì Cn 2 > 10 (nên
phép tính tổng là có nhớ và không thể có đẳng thức xảy ra).
Vậy các giá trị cần tìm của n là n D 1; 2; 3; 4:
b) Ta có
2n 1 D S.43n /  . 2/n .mod9/ , 2n C . 2/n  1 .mod9/:
Nếu n lẻ thì dễ thấy đồng dư thức trên không thể xảy ra, do đó n chẵn và ta có 2nC1  1 .mod9/
hay n C 1 chia hết cho 6:
 Với n D 5 thì S.435 / D 25 1.
 Với n  11 thì ta sẽ chứng minh bằng quy nạp rằng đẳng thức trên không thể xảy ra.

Ta có bất đẳng thức S.43n /  9 .blog10 43n c C 1/ nên ta chỉ cần chứng minh

9 .blog10 43n c C 1/ < 2n 1 với mọi n  11:

Do vế phải tăng nhanh hơn vế trái nên ta chỉ cần chứng minh khi n D 11 hay

9 .b11log10 43c C 1/ < 211 1:

Do 9 .b11log10 43c C 1/ < 9  23 D 207 < 211 1 nên ta dễ dàng có nhận xét đã nêu.
Vậy giá trị duy nhất của n thỏa mãn là n D 5:
Bài toán 10.
a) (South African MO, 2010) Gọi U.n/ là chữ số hàng đơn vị của n: Tìm n sao cho

n D S.n/ C U 2 .n/:

b) Hỏi số n không chứa chữ số 0 nào lớn nhất sao cho S.n/ D 15; S.2n/ < 20?
Lời giải. a) Do n  S.n/ .mod9/ nên U.n/ 2 f0; 3; 6; 9g. Ngoài ra, vì

S.n/  9 .blog10 nc C 1/

nên n  9 blog10 nc C 18 nên dễ thấy n có không quá 2 chữ số. Kết hợp các điều kiện trên, ta có
n D 13; 46; 99 thỏa mãn đề bài.
b) Ta thấy n  6 .mod9/ nên 2n  12  3 .mod9/. Do tổng các chữ số của 2n nhỏ hơn 20
nên có thể có hai trường hợp S.2n/ D 3 hoặc S.2n/ D 12.
Gọi k là số phép tính có nhớ trong phép nhân 2n thì

S.2n/ D S.2/S.n/ 9k , 30 9k D S.2n/:

Do đó, chỉ có thể S.2n/ D 12; k D 2 hoặc S.2n/ D 3; k D 3:


S.n/ – hàm tổng các chữ số 277

 Với k D 3, ta thấy số cần tìm là 555:

 Với k D 2, ta thấy có 2 số 5 ở phía trước và tương ứng, có 5 số 1 theo sau. Do cần tìm số
lớn nhất nên đó cũng là thứ tự cần tìm.

Cụ thể, ta có n D 5511111 và 2n D 11022222, thỏa mãn điều kiện đề bài.


Vậy số n cần tìm là 5511111:

2.2. Dạng 2: Chứng minh tính chất của tổng các chữ số
Bài toán 11.

a) Chứng minh rằng S.n/  8S.8n/ với mọi số nguyên dương n:

b) Chứng minh rằng S.n/  5S 55 n với mọi số nguyên dương n:




Lời giải. a) Ta có S.n/ D S.103 n/  S.125/S.8n/ D S.8/S.8n/:


b) Ta có S.n/ D S.105 n/  S.32/S.55 n/ D 5S.55 n/:

Bài toán 12.

a) (Imar MO, 2014) Hỏi dãy số S.2n / có thực sự tăng từ một giá trị đủ lớn của n không?

b) (Polish MO 2012) Chứng minh rằng với tồn tại vô hạn số nguyên dương n sao cho

S.2n C n/ < S.2n /:

Lời giải. a) Ta thấy số dư của 2n khi chia cho 9 tuần hoàn với chu kỳ 6 là: 1; 2; 4; 8; 7; 5: Đặt
sn D S.2n / với n D 0; 1; 2; 3; : : : Giả sử rằng dãy này tăng với n  n0 , trong đó n0 là một giá
trị cố định nào đó.
Ta cố định số m sao cho 6m  n0 thì

s6mC1  s6m C 1; s6mC2  s6mC1 C 2; s6mC3  s6mC2 C 4;


s6mC4  s6mC3 C 8; s6mC5  s6mC4 C 7; s6mC6  s6mC5 C 5

Suy ra s6mC6  s6m C 27. Do đó, với mọi số tự nhiên n thì s6mC6n  s6m C 27n.
Mặt khác, ta cũng có

s6mC6n D S.26mC6n /  9 log10 26mC6n C 1 D 9 Œ.6m C 6n/log10 2 C 1


 
 
1
<9 .6m C 6n/ D 18m C 18n:
3

Suy ra 18m C 18n > 27n, sai với n đủ lớn. Ta có đpcm.


Nhận xét. Liên quan đến bài toán trên, có một bài toán nổi tiếng hơn là: Chứng minh giá trị
của S.g n / với .g; 10/ D 1 tiến tới vô cực khi n tiến tới vô cực.
278 Các phương pháp giải toán qua các kỳ thi Olympic

b) Với mỗi số nguyên dương k, xét số n D 10kC1 2 D 999


„ ƒ‚: : : …9 8 thì S.n/ D 9k C 8:
k
n
Giả sử có c phép nhớ trong việc tính tổng 2 C n thì rõ ràng c  k C 1. Do đó, theo tính chất
của tổng các chữ số thì

S.2n C n/ D S.2n / C S.n/ 9c  S.2n / C 9k C 8 9.k C 1/ < S.2n /:

Ta có đpcm.
Bài toán 13.
a) Giả sử x; y là các số tự nhiên mà trong biểu diễn thập phân có cùng số lần xuất hiện của
các chữ số 0; 1; 2; : : : ; 9. Chứng minh rằng S.5x/ D S.5y/:

b) Cho k là số có dạng an : : : a3 a2 a1 sao cho an ¤ 0; ai > ai C1 với mọi i D 1; n 1.


Chứng minh rằng S.9k/ D 9:
Lời giải. a) Rõ ràng các chữ số của hai˚ số x;
ˇ y thuộc cùng một multiset.
˚ Ta chiaˇchúng ra thành 2
phần là các P
số chẵn và các số lẻ.
PGiả sử 2ai 0  ai  4; i D 1; n và 2bi C 1 ˇ0  ai  4; i D 1; m .
ˇ
n m
Ta có x D i D1 2ai 10 C i D1 .2bi C 1/10yi và các số xi ; yi đôi một khác nhau. Suy ra
xi

n
X m
X n
X m
X m
X
xi yi xi C1 yi C1
5x D 5 2ai 10 C 5 .2bi C 1/10 D ai 10 C bi 10 C 5  10yi :
i D1 i D1 i D1 i D1 i D1

Từ đó, ta tính được S.5x/ D niD1 ai C m


P P
i D1 bi C 5m không phụ thuộc vào thứ tự của các số
ai ; bi nên suy ra S.5x/ D S.5y/: Ta có đpcm.
b) Với N D anC1 an : : : a3 a2 a1 thì N D nC1 i
P
i D0 ai 10 và ta có

nC1
X nC1
X nC1
X
i i
9N D 9ai 10 D .10 1/ai 10 D .10i C1 10i /ai
i D0 i D0 i D0
nC1
X
D an 10nC2 C .ai 1 ai /10i C .a0 a1 1/10 C .10 a0 /:
i D2

Chú ý rằng 10 a0  9 và a0 a1 1  0 nên đây cũng là các chữ số của 9N . Suy ra


n
X
S.9k/ D an C .ai 1 ai / C .c0 c1 1/ C .10 c0 / D 9:
i D2

Bài toán 14.


a) Hỏi số tự nhiên m nhỏ nhất là bao nhiêu để trong m số nguyên dương liên tiếp, luôn tồn
tại ít nhất một số có tổng các chữ số chia hết cho 13?

b) (USA MO, 2005) Đặt A D f1; 2; 3; : : : ; ng. Gọi f .n/ là GTNN củaP  k sao cho
số tự nhiên
tồn tại n số nguyên dương phân biệt x1 ; x2 ; : : : ; xn có tính chất S i 2I i D k với mọi
x
I  A; I ¤ ;. Chứng minh tồn tại 0 < c1 < c2 sao cho

c1 log10 n  f .n/  c2 log10 n; 8n 2 N :


S.n/ – hàm tổng các chữ số 279

Lời giải. a) Ta sẽ chứng minh GTNN cần tìm là m D 79: Trước hết, xét 78 số tự nhiên liên tiếp
sau (bắt đầu là 1010 39 và kết thúc là 1010 C 38):

99 : : : …9 61; „
„ ƒ‚ 99ƒ‚
: : :…
9 62; : : : ; 99
„ :ƒ‚
: : 999
…;
8 8 10
1„
00ƒ‚
: : :…
0; 1 00 : : :…
„ ƒ‚ 1; : : : ; 1 „
00ƒ‚
: : : …0 38
10 10 8

Dễ dàng kiểm tra được trong dãy này không có số n nào mà S.n/ chia hết cho 13 nên m > 78:
Xét một dãy gồm 79 số nguyên dương n; n C 1; n C 2; : : : ; n C 78 tùy ý. Ta xét 2 trường hợp sau:
1. Trong các số, có một số chia hết cho 100; đặt là n0 và S.n0 / D k. Ta có thể giả sử
n0  n C 39 (trường hợp ngược lại chứng minh tương tự). Ta thấy rằng

S.n C i/ D S.n/ C S.i/; 0  i  39:

Ngoài ra, rõ ràng 0  S.i/  12 với 0  i  39: Từ đây suy ra trong các số trên, phải có
1 số chia hết cho 13:

2. Nếu các số trong dãy đều không chia hết cho 100; tức là trong dãy này, các số chỉ khác
nhau bởi hai chữ số tận cùng, còn lại thì đều giống nhau cả. Đặt S.n/ D a thì tương tự
trên, ta cũng chứng minh được các số S.n/; S.n C 1/; : : : ; S.n C 78/ nhận đủ các giá trị
từ a đến a C 12 nên cũng có một số chia hết cho 13:
Vậy số nhỏ nhất cần tìm là m D 79:
b) Gọi S là tập hợp của n số nguyên dương thỏa mãn điều kiện đã nêu.
Ta sẽ chứng minh tồn tại c1 . Đặt m D blog10 nc thì 10m 1 < n. Xét S D fx1 ; x2 ; : : : ; xn g và
ta đặt ai D it D1 xt thì theo nguyên lý Dirichlet, phải tồn tại hai số 1  i < j  n sao cho
P
ai  aj .mod10m 1/. Khi đó, ta có aj ai D xi C1 C xi C2 C    C xj chia hết cho 10m 1.
Theo tính chất của số chia hết cho 10m 1 thì

f .n/ D S.ai C1 C ai C2 C    C aj /  9m D 9 blog10 nc :

Do đó, ta chọn c1 D 9: Tiếp theo, ta sẽ chứng minh sự tồn tại của c2 . Thật vậy, gọi p là số nguyên
dương nhỏ nhất sao cho 10p  n.nC1/2
và đặt S D f10p 1; 2.10p 1/; : : : ; n.10p 1/g. Khi
đó, rõ ràng tổng các phần tử của bất kỳ tập con nào của S đều có dạng k.10p 1/ với
n.n C 1/
1k  10p :
2
Khi đó, ta có S.k.10p 1// D 9p. Do đó
10n.n C 1/
f .n/  9p  9log10 D 9log10 .5n.n C 1// :
2
Để đưa về dạng c2 log10 n, ta chú ý rằng với n  2 thì 5.n C 1/ < n4 nên

f .n/ < 9log10 n5 D 45log10 n:

Bài toán được giải quyết hoàn toàn.


280 Các phương pháp giải toán qua các kỳ thi Olympic

Nhận xét. Ở câu a, ta có một bài toán tương tự là: Chứng minh rằng trong 39 số nguyên dương
liên tiếp, tồn tại ít nhất một số có tổng các chữ số chia hết cho 11:
Tổng quát, ta có kết quả sau: Với mỗi số nguyên dương n, gọi f .n/ là số lớn nhất các số hạng
của một dãy số tự nhiên liên tiếp mà không có số nào có tổng các chữ số chia hết cho n: Chứng
minh rằng f .9k C a/ D .a C 1/10k 2:
Bài toán 15 (Các bài toán về số Niven).
a) (Italy MO, 2005) Chứng minh rằng trong 18 số nguyên dương liên tiếp không vượt quá
2005; có ít nhất một số chia hết cho tổng các chữ số của chính nó.
b) Chứng minh rằng không tồn tại 21 số tự nhiên liên tiếp mà mỗi số chia hết cho tổng các
chữ số của nó.
Lời giải. a) Ta thấy rằng với 1  n  2005 thì S.n/  28: Xét các số chia hết cho 18 thì tổng
các chữ số của nó sẽ chia hết cho 9. Ta có các trường hợp:
 Nếu tổng đó là 9 hoặc 18 thì hiển nhiên số này thỏa mãn.
 Nếu tổng đó là 27 thì chỉ có thể là 1998 và rõ ràng 27j1998 cũng thỏa mãn.
Trong 18 số bất kỳ thì tồn tại ít nhất một số chia hết cho 18 nên ta có đpcm.
Nhận xét. Một bài tương tự trong Tournament of the town 1984: Chứng minh rằng trong 18
số nguyên dương liên tiếp có 3 chữ số, tồn tại ít nhất một số n sao cho n chia hết cho S.n/:
b) Giả sử ngược lại rằng tồn tại một dãy số thỏa mãn đề bài là a1 ; a2 ; : : : ; a21 .
Ta thấy rằng nếu tồn tại hai số lẻ trong dãy cách nhau 10 đơn vị, đặt là ai ; ai C 10 mà chữ số
hàng chục của chúng hơn kém nhau 1 thì ta có S.ai C 10/ D S.ai / C 1 là hai số khác tính chẵn
lẻ. Khi đó, có một trong hai số trong ai ; ai C 10 sẽ không thỏa mãn tính chất đã cho, mâu thuẫn.
Suy ra các chữ số hàng chục của chúng phải là 0 và 9: Do đây là các số liên tiếp nên phải có một
số trong chúng hết cho 100; đặt là c.
Rõ ràng hai số c C 1; c C 11 không thể cùng xuất hiện trong dãy theo nhận xét trên. Tương tự,
hai số c 1; c 11 cũng thế. Do đó, để dãy này có 21 số tự nhiên thì nó phải là
c 10; c 9; : : : ; c 1; c; c C 1; : : : ; c C 9; c C 10:
Với 1  i  9 thì S.c C i/ D S.c/ C i . Suy ra một trong các số S.c/; S.c C 1/; S.c C 2/;
: : : ; S.c C 9/ chia hết cho 10: Rõ ràng các số c C 1; c C 2; : : : ; c C 9 đều không chia hết cho 10
và chỉ có c chia hết cho 10 nên S.c/ chia hết cho 10 (theo tính chất của các số). Do đó,
S.c/  10:
Chú ý rằng S.c C 1/ D S.c C 10/ nên S.c C 1/ j c C 1 ; S.c C 10/ j c C 1 0 và suy ra
S.c C 1/ j ..c C 10/ .c C 1// D 9 ) S.c C 1/  9:
Do đó 9  S.c C 1/ D S.c/ C 1  10 C 1 D 11, vô lý. Điều giả sử trên là sai.
Vậy không tồn tại dãy số thỏa mãn đề bài.
Nhận xét. Người ta cũng chứng minh được rằng: Trong hệ cơ số b, luôn tồn tại 2b số liên tiếp
mà mỗi số chia hết cho tổng các chữ số của nó; nhưng không tồn tại 2b C 1 số như thế.
S.n/ – hàm tổng các chữ số 281

2.3. Dạng 3: Chứng minh sự tồn tại của các số thỏa mãn điều kiện
cho trước
Bài toán 16.
a) Chứng minh rằng với mọi n, tồn tại k sao cho S.k/ D n và S.k 2 / D n2 :

b) Hỏi có tồn tại các số nguyên dương a; b sao cho S.an/ < S.bn/ với mọi a; b‹
i
Lời giải. a) Ta chọn số k D niD1 102 thì S.k/ D n. Khi đó, ta thấy rằng
P

n
!2 n
2i i j
X X
2
k D 10 D 102 C2 :
i D1 1i j n

Do các số 21 ; 22 ; : : : ; 2n đều có tổng của hai số tùy ý trong đó phân biệt nên suy ra S.k 2 / D n2 .
Ta có đpcm.
i
Nhận xét. Ý tưởng xây dựng số theo dạng 102 là rất thú vị và cũng rất quan trọng, nó giúp ta
có thể giải quyết được nhiều bài toán chứng minh tồn tại tương tự.
b) Câu trả lời là phủ định, chẳng hạn, ta chọn n D 10k 1 với k là số các chữ số lớn nhất của
a; b. Khi đó, ta có S.an/ D S.bn/ D 9k. Câu trả lời vẫn là phủ định nếu thay S.an/ < S.bn/
bởi S.an/  S.bn/.
Bài toán 17.
a) Chứng minh rằng tồn tại n nguyên dương sao cho S.n/; S.2n/; : : : ; S.2003n/ đều chia
hết cho 2003:
n.nC1/
b) Chứng minh tồn tại vô số tam giác dạng 2
mà nó chia hết cho tổng các chữ số của nó.

c) Chứng minh rằng với mọi số nguyên dương m > 0, tồn tại n sao cho S.n/ > mS.n2 /:
Lời giải. a) Xét số n D 10a1 C 10a2 C : : : C 10a2003 với a1 D 1 và anC1 D 10an .
Rõ ràng các phép nhân ni; 1  i  2003 đều không có số nhớ và tổng các chữ số của các số thu
được đều bằng 2003: Do đó, số n như thế thỏa mãn đề bài.

b) Ta chọn số tam giác có dạng:

T10i C1 D 5  10i .10i C1 C 1/:

Khi đó, rõ ràng S.T10i C1 / D 5 C 5 D 10 với i  1: Từ đó dễ thấy S.T10i C1 / j T10iC1 . Do có vô


số cách chọn số i nên cũng có vô số số tam giác thỏa mãn đề bài.
c) Ta chọn n D 1 C 10m 102m 1 C 103m C 104m hoặc n D 1 C 10m 102m 1
C 103m C 104m .
Dễ thấy rằng các số dạng này thỏa mãn đề bài.
Bài toán 18. Chứng minh rằng tồn tại n sao cho n; S.n/ đều là các số chính phương với:
a) n có chứa chữ số 0:

b) n không có chữ số 0:
282 Các phương pháp giải toán qua các kỳ thi Olympic
 
10k 1
Lời giải. a) Ta chọn ak D 333
„ ƒ‚: : : …3 2 D 10 3
C 2 thì ak2 D 111
„ ƒ‚: : : …1 0 „
222ƒ‚
: : :…
2 4.
k k k

Suy ra S.ak2 / D 3k C 4. Rõ ràng ta có thể chọn vô số số k sao cho 3k C 4 là số chính phương.


 k 
b) Tương tự, ta chọn ak D 333
„ ƒ‚: : : …3 4 D 10 10 3 1 C 4 thì ak2 D 111
„ ƒ‚: : : …1 555
„ ƒ‚: : : …5 6 và ta
k kC1 k
cũng chọn được vô số số k sao cho 6k C 7 là số chính phương.

Bài toán 19.

a) Chứng minh rằng tồn tại n sao cho S.n/ D 100 và S.n3 / D 1003 .

b) Với mọi số nguyên dương m, chứng minh tồn tại bội km của m mà S.km/ lẻ.

Lời giải. a) Xét số P .x/ D nkD1 x 2 thì rõ ràng ta luôn có P .10/ D n; P 3 .10/ D n3 và ở
P k

bài toán này, ta chỉ cần chọn n D 100 là thỏa mãn.


Chú ý rằng do các số hệ số các lũy thừa tính được không vượt quá 9 nên khi tính, tổng có phép
cộng có nhớ nào và P 3 .10/ vừa đúng n3 .
Ý tưởng xét tổng có dạng lũy thừa tầng để tránh các số nhớ đã được sử dụng ở các bài toán trên.
b) Ta xét hai trường hợp:

.10; m/ D 1 thì theo định lý Euler, ta có 10'.m/ 1 chia hết cho m nên suy ra số
1. Nếu P
nD m i D1 10
i'.m/
chia hết cho m. Do m lẻ nên S.n/ D m là số lẻ, thỏa mãn điều kiện.

2. Nếu .10; m/ > 1. Ta đặt m D 2k  5t  m0 với .m0 ; 10/ D 1. Tương tự trên, ta chọn số n0
thỏa mãn n0 chia hết cho m0 và S.n0 / lẻ. Khi đó, số n D n0  10kCt chia hết cho m và có
S.n/ D S.n0 / là số lẻ.

Ta có đpcm.

Bài toán 20.

a) Chứng minh rằng với mọi cặp số nguyên dương liên tiếp, có ít nhất một số biểu diễn được
dưới dạng n C S.n/ với n là số nguyên dương nào đó.

b) Tìm điều kiện cần và đủ của k sao cho tồn tại n để S.n2 / D k:

Lời giải. a) Đặt f .n/ D n C S.n/; n > 0. Ta sẽ chứng minh rằng f .n C 1/  f .n/ C 2 với mọi
n: Thật vậy, ta thấy rằng nếu n tận cùng là 9 thì f .nC1/ < f .n/ vì S.n/ S.nC1/ D 9k 1 > 1
với k > 0 là số phép cộng có nhớ. Còn nếu n không tận cùng là 9 thì f .n C 1/ D f .n/ C 2: Do
đó, nhận xét được chứng minh.
Trở lại bài toán, ta thấy với n D 1; 2 thì khẳng định đúng.
Xét hai số liên tiếp .m; m C 1/ với m > 2. Ta chọn số k lớn nhất sao cho f .k/ < m (rõ ràng số
này tồn tại vì f .1/ D 2). Do f .k/ < m nên f .k/  m 1.
Tuy nhiên, theo nhận xét ở trên thì f .k C 1/ < f .k/ hoặc f .k C 1/ D f .k/ C 2 nên ta luôn có

f .k C 1/  m C 1:
S.n/ – hàm tổng các chữ số 283

Hơn nữa, theo định nghĩa thì f .k C 1/  m nên ta có m  f .k C 1/  m C 1 và suy ra


m D f .k C 1/ hoặc m C 1 D f .k C 1/. Như vậy, ta có đpcm.
b) Ta thấy n2  S.n2 /.mod9/ mà một bình phương đúng thì chia 9 dư 0; 1; 4; 7 nên điều kiện
cần của k để tồn tại n sao cho S.n2 / D k là k chia 9 dư 0; 1; 4; 7.
Ta sẽ chứng minh đó cũng là điều kiện đủ. Xét các trường hợp sau:
10t 1
1. Nếu k D 9t với t 2 N. Đặt xt D 3
D 333
„ ƒ‚: : : …3 thì
t
2
10t 102t 1 2 10t

1

1
xt2 D D D 111 : : : …1 0 „
888ƒ‚
: : :…
8 9:
3 9 3 „ ƒ‚
t 1 t 1

Suy ra S.xt2 / D 9t D k và ta chọn n D xt .


2. Nếu k D 3t C 1 (tương ứng với các trường hợp 9t C 1; 9t C 4; 9t C 7). Đặt:
10t C 5
xt D D 33 : : : …3 5
3 „ ƒ‚
t 1
 2
10t C5 102t 1 10t C1 1
thì xt2 D 3
D 9
C 9
C 3 D 11 : : : …1 22
„ ƒ‚ : : : …2 5. Suy ra S.xt2 / D 3t C 1
„ ƒ‚
t 1 t
và ta chọn n D xt .
Nhận xét. Bài toán ở câu b là một dạng phát biểu tổng quát về miền giá trị của S.n2 / khi
n 2 ZC . Một bài cùng dạng trong đề thi Olympic Toán toàn Nga 1964: Chứng minh rằng S.n2 /
đều khác 5 với mọi số nguyên dương n: Hỏi có tồn tại hay không số nguyên dương n sao cho
S.n2 / D 100?
Bài toán 21 (British MO, 1999). Xét các số m D 3n2 C n C 1 với n là số nguyên dương.
a) Tìm GTNN của S.m/:

b) Hỏi có tồn tại n hay không sao cho S.m/ D 1999‹


Lời giải. a) Ta thấy với n D 8 thì m D 201và có S.m/ D 3: Ta sẽ chứng minh rằng không thể
có S.m/ D 1 hay S.m/ D 2: Ta xét các trường hợp:
1. Nếu S.m/ D 1 thì số m có dạng 10k là số chẵn. Trong khi đó:
3n2 C n C 1 D n.n C 1/ C 2n2 C 1
lại là số lẻ nên không thỏa.
2. Nếu S.m/ D 2 và do 3n2 C n C 1 phải lẻ nên m chỉ có thể có dạng 10k C 1: Suy ra
3n2 C n C 1 D 10k C 1 , n.3n C 1/ D 10k :
Ta thấy .n; 3n C 1/ D 1 nên n D 1; 3n C 1 D 10k hoặc n D 2k ; 3n C 1 D 5k .
Dễ thấy trường hợp thứ nhất không thể xảy ra; ở trường hợp thứ hai, ta suy ra 32k C1 D 5k .
Nếu k  2 thì 5k > 2k  2k  4  2k > 3  2k C 1 và dễ kiểm tra với k D 0; 1 thì không
thỏa, suy ra phương trình trên cũng vô nghiệm. Do đó, không tồn tại n thỏa mãn.
284 Các phương pháp giải toán qua các kỳ thi Olympic

Vậy GTNN cần tìm là S.m/ D 3:


b) Câu trả lời là khẳng định. Ta chọn số
2
10a 1 10a 1 102a 2 .10a

1 1/
nD ) n2 D D D 111 : : : …1 0 „
888ƒ‚
: : :…
89
3 3 9 3 „ ƒ‚
a 1 a 1

Suy ra 3n2 D 333


„ ƒ‚: : : …3 2 „
666ƒ‚ 6 7 nên 3n2 C n D 333
: : :… „ ƒ‚: : : …3 3 „
000ƒ‚
: : :…
0 0.
a 1 a 1 a 1 a 1

Do đó S.3n2 C n C 1/ D 3a C 1 với a là số nguyên dương có thể chọn tùy ý.


1999 1
Để có S.3n2 C n C 1/ D 1999 thì ta chỉ cần chọn a D 3
D 666.

Bài toán 22 (Chọn đội tuyển PTNK, 2008).

a) Chứng minh rằng các số tự nhiên n D 999 và n D 2999 không thể biểu diễn thành tổng
a C b mà S.a/ D S.b/:

b) Chứng minh rằng với mọi n mà 999 < n < 2999 thì điều kiện trên được thỏa mãn.

Lời giải. Ta sẽ giải bài toán tổng quát sau: Tồn tại các số a; b thỏa mãn điều kiện khi và chỉ khi
số n > 1 không có dạng n D m999 : : : 9 với 0  m  8 và S.n/ lẻ.
Thật vậy, xét n D d1 d2 d3 : : : dk với k là số các chữ số của n: Ta có 2 trường hợp:

1. Nếu S.n/ chẵn (tương đương với có chẵn chữ số lẻ trong trong n), ta thực hiện như sau:
di di
 Nếu di chẵn thì tách thành di D 2
C 2
và chữ số ở hàng tương ứng của a; b sẽ là 2
số này.
 Nếu di lẻ thì tách thành di D di2 1 C di 2C1 thì di 2C1 di2 1 D 1 và như thế, ta luân
phiên thay đổi các số lớn nhỏ để ghép vào a; b để đảm bảo có S.a/ D S.b/: Do có
chẵn số di lẻ như thế nên quá trình trên thực hiện được và trong trường hợp này, tồn
tại số a; b thỏa mãn.

2. Nếu S.n/ lẻ (tương đương với có lẻ chữ số lẻ trong trong n). Ta lại xét 2 trường hợp:

 Nếu n có dạng m999 : : : 9 với 0  m  8 thì rõ ràng khi tách ra thành 2 phần, các
phép tính tổng phía sau để thu được các số 9 là không có nhớ và chúng có dạng
di C d 0 i D 9.
Khi đó, ta có S.a/ C S.b/ D S.n/ mà
(
S.a/ D S.b/
S.n/  1 .mod 2/

Điều mâu thuẫn trên cho thấy trường hợp này không tồn tại cách tách n thành a; b
thỏa mãn.
 Nếu n không có dạng trên thì dễ thấy tồn tại di ¤ 0; di C1 ¤ 9. Ta viết lại n như sau:

n D d1 d2 : : : di di C1 : : : dk D d1 d2 : : : .di 1/9 : : : dk C .di C1 C 1/10k di


:
S.n/ – hàm tổng các chữ số 285

Đặt a D d1 d2 d3 : : : .di 1/9 : : : dk ; b D .di C1 C 1/10k di


thì S.a/; S.b/ cùng tính
chẵn lẻ và a C b D n.
Ta có thể giả sử S.a/ > S.b/, trường hợp còn lại chứng minh tương tự. Nếu chọn
một vị trí t mà dt ¤ 0 thì có thể đổi các số a; b thành

a0 D d1 d2 : : : .di 1/9 : : : .dt 1/ : : : dk ; y D .di C1 C 1/10k i


C 10k t

với k ¤ i . Khi đó S.a0 / S.b 0 / D S.a/ 1 .S.b/ C 1/ D 2. Cứ như vậy, ta


thực hiện liên tiếp đến khi nào chênh lệch giữa hai tổng các chữ số bằng 0 thì dừng
lại (tồn tại thời điểm như vậy vì ban đầu chúng cùng tính chẵn lẻ nên hiệu của chúng
là số chẵn và mỗi lần thực hiện quá trình trên thì hiệu giảm đi 2 đơn vị). Các số a; b
lúc đó sẽ thỏa mãn đề bài và cũng tồn tại cách tách.

Bài toán được giải quyết hoàn toàn và với các số cụ thể, bài toán trong đề gốc là đúng.

Bài toán 23.

a) (Romania JBTST, 2008) Chứng minh rằng với mọi n, tồn tại một bội của n có tổng các
chữ số đúng bằng n:

b) (USA MO, 2013) Chứng minh rằng với mọi m; n thì tồn tại c nguyên dương sao cho
cm; cn có cùng số lượng xuất hiện của các chữ số từ 0 đến 9:

Lời giải. a) Đặt n D 2a  5b  c với a; b  0 và c  1; .c; 10/ D 1. Gọi k là số nguyên


 dương
k aCb k k2 k3 kn
thỏa mãn 10 1 chia hết cho L: Đặt N D 10 10 C 10 C 10 C : : : C 10 . Ta thấy
rằng
i
10k  1 .modL/ ) 10k  1 .modL/:
:
Suy ra 10k C 10k C 10k C : : : C 10k  n  0 .modn/ và 10aCb ::.2a  5b / nên N chia hết
2 3 n

cho n: Ngoài ra, trong biểu diễn thập phân của N có n chữ số 1 nên số này có S.n/ D n. Từ đây
suy ra số N đã cho thỏa mãn điều kiện đề bài.
b) Với mỗi số nguyên dương k, ta viết 10k m n D 2r 5s t với .t; 10/ D 1.
Rõ ràng r  v2 .n/; s  v5 .n/ nên các giá trị r; s là hữu hạn và ta có thể chọn số mũ k sao cho
giá trị t lớn tùy ý. Khi đó, ta chọn k sao cho t > max fm; ng : Gọi b là số nhỏ nhất sao cho
10b 1 chia hết cho t , suy ra b chính là chu kỳ của số thập phân vô hạn tuần hoàn 1t , bao gồm
các bộ b số lặp lại vô hạn lần.
Mặt khác, nếu viết 10b 1 D tc thì các bộ được lặp lại là b chữ số của c (nếu c không đủ b chữ
số thì có thể thêm số 0 vào trước đó). Chẳng hạn nếu t D 41, ta chọn b D 5 và c D 2439; trong
1
khi đó, chu kỳ của 41 là .02439/. Do t > m; t > n nên trong biểu diễn số thập phân của mt ; nt
thì có các bộ gồm b chữ số biểu diễn cho cm; cn. Ta viết lại 10k mt D 2r 5s C nt thì rõ ràng phần
thập phân của nt nhận được từ mt bằng cách chuyển dấu phẩy sang phải k lần.
Do đó, biểu diễn thập phân của cm; cn cũng tương ứng chuyển đổi cho nhau bằng cách lấy một
đoạn các số ở đầu ghép vào phía cuối (tương ứng với việc chuyển dấu phẩy).
Vậy cm; cn có cùng số lượng các chữ số trong biểu diễn thập phân.
286 Các phương pháp giải toán qua các kỳ thi Olympic

Bài toán 24.


a) Cho số nguyên dương k > 1: Chứng minh rằng tồn tại k số nguyên dương phân biệt
a1 ; a2 ; : : : ; ak sao cho ai C S.ai / không đổi với mọi i D 1; k.
b) (Argentina IMO TST 2007 - Ukraine journal) Chứng minh rằng với mọi số nguyên dương
S.n/
r thì tồn tại n sao cho S.n 2 /  r:

Lời giải. a) Ta sẽ chứng minh bằng quy nạp theo k: Với k D 2, ta xây dựng số theo cách như
p p
sau sử dụng vào bước quy nạp: Xét 2 số Ap D 1010 CpC1 10p và Bp D 1010 CpC1 C 8  10p
thì hiển nhiên Ap ¤ Bp và ta sẽ chứng minh rằng 2 số này thỏa mãn. Ta thấy rằng:
 Ap có 10p chữ số đầu tiên gồm toàn số 9 và còn lại gồm toàn bộ là 0 nên S.Ap / D
9.10p C 1/,
 Bp có chữ số đầu tiên và chữ số thứ 10p C 1 từ phải sang là 8; còn lại đều là 0 nên
S.Bp / D 9:
Do đó, ta có
p p
Ap C S.Ap / D 1010 CpC1
C 8  10p C 9 và Bp C S.Bp / D 1010 CpC1
C 8  10p C 9:

Suy ra Ap C S.Ap / D Bp C S.Bp /:


Giả sử khẳng định đúng đến k, ta sẽ chứng minh nó cũng đúng với 2k (khi đó hiển nhiên
cũng đúng với các số k C 1; k C 2; : : : ; 2k 1). Ta đã có các số a1 ; a2 ; : : : ; ak thỏa mãn
a1 C S.a1 / D a2 C S.a2 / D : : : D ak C S.ak /. Ta chọn p đủ lớn sao cho 10p > max fai g.
Khi đó, với mọi i D 1; k thì
Ap C S.Ap / C ai C S.ai / D Bp C S.Bp / C ai C S.ai /
, .Ap C ai / C S.Ap C ai / D .Bp C ai / C S.Bp C ai /
Do đó, 2 tập hợp ˚ ˚
Ap C ai j1  i  k [ Bp C ai j1  i  k
gồm 2k số đôi một phân biệt thỏa mãn điều kiện. Vậy ta có đpcm.
b) Trước hết, ta có nhận xét: Nếu a; b; c; d là các số tự nhiên mà 2a C 2b D 2c C 2d thì

max.a; b/ D max.c; d /; min.a; b/ D min.c; d /:


n n n n 2n 1
Ta xét số an D 5  102 1
bn và bn D 102 2
C 102 4
C : : : C 102 C 1: Ta sẽ xác định
S.an /. Ta có
n 1
n 2i
X
an D 4 „
999ƒ‚
: : :…
9 102
2n 1 i D1

nên trong dãy 2n 1 chữ số 9 trong số hạng thứ nhất, có n 1 số sẽ chuyển thành 8: Khi đó

S.an / D 4 C 8.n 1/ C .2n 1 .n 1// 9 D 9  2n n 4:

Tiếp theo, ta tính


 n 2 n nC1 n
an2 D 25  102 1
2  5  102 1
bn C bn2 D 25  102 2
102 bn C bn2 :
S.n/ – hàm tổng các chữ số 287

nC1 n n
Chú ý rằng 102 2
D 102  102 2
và với 1  i  n thì
 2
2n 2n 2i 2n 2i 1
10  10 D 10 :

Do đó, ta viết lại


 n 
2nC1 2 2j 2n 2k
X
an2 D 24  10 C1C2 102  10 :
1j <kn

Theo nhận xét ban đầu thì ta có


n 2a n 2b n 2c n 2d
102  102 ¤ 102  102 , 2a C 2b D 2c C 2d

nên các số hạng trong tích đều đôi một khác nhau. Do đó, an2 có dạng 24 : : : 1 và giữa các số 4; 1
thì một số số là 2 và còn lại là 0: Chú ý rằng số các số 2 chính là số cách chọn ra 2 số .j; k/ trong
n số và là Cn2 . Suy ra S.an2 / D 2 C 4 C 1 C 2Cn2 D n2 n C 7. Từ đây ta được

S.an / 9  2n n 4
D
S.an2 / n2 n C 7

và biểu thức này tiến đến vô cực khi n tiến tới vô cực. Từ đó suy ra với mọi r lớn tùy ý thì tồn tại
S.n/
n sao cho S.n 2 /  r: Ta có đpcm.

Bài toán 25 (Polish 2nd Round, 2008). Cho n là số nguyên dương không chia hết cho 3:
Chứng minh rằng tồn tại số nguyên dương m sao cho với mọi k  m thì k có thể biễu diễn thành
tổng các chữ số của bội số dương nào đó của n:

Lời giải. Ta có nhận xét: Nếu a; b là các số nguyên dương nguyên tố cùng nhau thì với mọi
n > ab a bthì phương trình ax C by D n có nghiệm thỏa mãn x; y  0:
Trước hết, ta sẽ chứng minh rằng b có hai bội số thỏa mãn tổng các chữ số của chúng nguyên tố
cùng nhau. Đến đây, chỉ cần áp dụng nhận xét trên là bài toán được giải quyết. Đặt n D 2s 5p q
với .q; 10/ D 1. Theo định lý phần dư Trung Hoa thì tồn tại số nguyên dương m sao cho
(
mq  19 .mod100/
m  1 .mod3/

Giả sử mq có d chữ số và tổng các chữ số của nó là t . Ta xét số mq.10d 1 C 1/ có tổng các chữ
số là 2t 9. Ta thấy t; 2t 9 là hai số nguyên tố cùng nhau vì t không chia hết cho 3:
Khi đó, ta nhân hai số mq và mq.10d 1 C 1/ cho lũy thừa của 10 thích hợp để chúng là bội của
n và tổng các chữ số của chúng không đổi và nguyên tố cùng nhau. Gọi các bội này là a; b:
Đến đây, theo bổ đề thì với k > S.a/S.b/ S.a/ S.b/ thì tồn tại các số không âm x; y sao
cho xS.a/ C yS.b/ D k. Từ đây ta có đpcm.
288 Các phương pháp giải toán qua các kỳ thi Olympic

2.4. Dạng 4: Các hàm số học tương tự


Bài toán 26.

a) Hỏi có bao nhiêu số nguyên dương n không vượt quá 2016 mà trong biểu diễn nhị phân
của nó, có ít nhất 3 chữ số 1?
2 2
b) Chứng minh rằng trong hệ cơ số n2 C 1 thì hai số n2 .n2 C 2/ và n4 .n2 C 2/ có cùng
tổng các chữ số.

Lời giải. a) Do 2048 là lũy thừa của 2 nhỏ nhất mà lớn hơn 2016 nên ta sẽ tìm số các số n
không vượt quá 2048 và có 0; 1; 2 chữ số 1 trong biểu diễn nhị phân. Đặt số cần tìm trong hệ nhị
phân có dạng a1 a2 : : : a11 với ai 2 f0; 1g ; i D 1; 11. Ta xét các trường hợp sau:

1. Nếu số này có 0 chữ số 1; rõ ràng chỉ có 1 số như vậy là n D 0, không thỏa mãn.
1
2. Nếu số này có 1 chữ số 1; có C11 D 11 cách chọn vị trí cho số 1:
2
3. Nếu số này có 2 chữ số 1; có C11 D 55 cách chọn vị trí cho các số 1:

Suy ra, số các số không vượt quá 2048 và có ít nhất 3 chữ số 1 trong biểu diễn nhị phân là
2048 .11 C 55/ D 1982: Chú ý rằng các số từ 2017 đến 2048 đều có hơn 3 chữ số 1 trong
biểu diễn nhị phân nên ta cần trừ đi số lượng thỏa mãn này và số các số thỏa mãn là

1982 .2048 2016/ D 1950:

b) Ta đặt b D n2 C 1 thì có ngay 10.b 2/.b 1/.b/ , .b 1/.b 2/01.b/ : Rõ ràng hai số này
viết trong hệ cơ số b thì có cùng multiset của các chữ số nên chúng có cùng tổng các chữ số.

Bài toán 27. Gọi f .n/ là lũy thừa lớn nhất của 2 trong phân tích của n thành thừa số nguyên
tố và g.n/ là tổng các chữ số của n trong hệ nhị phân. Chứng minh rằng:

a) f .nŠ/ D n g.n/:
n
b) C2n chia hết cho 4 khi và chỉ khi n không là lũy thừa của 2:

Lời giải. a) Giả sử g.n/ D k > 0 thì trong biểu diễn nhị phân của n, có đúng k chữ số 1, suy
ra nó là tổng của k lũy thừa của 2 với số mũ phân biệt, đặt là a1 < a2 <    < ak . Biểu diễn

n D 2ak C 2ak 1
C    C 2a3 C 2a2 C 2a1 :

Theo công thức Legendre, để tính f .nŠ/, ta sẽ tính tổng


C1 j
X nk
i
: ./
i D1
2

Ta có
n D 2a1 .2ak a1
C 2 ak 1 a1
C    C 2 a2 a1
C 1/ D 2a1 m1
với m1 là biểu thức trong dấu ngoặc. Với i D 1; 2; 3; : : : ; a1 , tổng trên chính là

m1 2a1 1 C 2a1 2 C    C 21 C 1 D m1 .2a1 1/ D n m1 :



S.n/ – hàm tổng các chữ số 289

 m1 ˘
Đến đây, sau khi chia cho 2a1 C1 trong công thức ./; tương ứng với xét 2
thì tổng giảm đi 1
đơn vị do số m1 lẻ. Tương tự, đặt
2ak a1
C 2 ak 1 a1
C    C 2 a2 a1
m2 D
2a2 a1

thì số này cũng là tổng của các lũy thừa của 2 và số hạng cuối bằng 1. Sau khi tính tổng trong
./ tiếp đến 2a1 Ca2 thì kết quả thu được là n m1 C . 1/ C m1 m2 D n m2 1.
Cứ như thế, sau k lần thực hiện, tổng cuối cùng sẽ giảm k đơn vị so với giá trị n ban đầu và ta
được f .nŠ/ D n k D n g.n/:
n
b) Ta thấy C2n chia hết cho 4 khi
n

f C2n  2 hay f ..2n/Š/ 2f .nŠ/  2:

Suy ra
2n g.2n/ 2 .n g.n//  2 , 2g.n/  g.2n/ C 2:
Nếu n là lũy thừa của 2 thì g.n/ D g.2n/ D 1 và dẫn đến g.n/  2, vô lý. Do đó, n không phải
là lũy thừa của 2. Ngược lại, nếu n không là lũy thừa của 2 thì g.2n/ D g.n/ > 1 và g.n/  2,
đúng. Vậy ta có đpcm.
Bài toán 28 (Thổ Nhĩ Kỳ TST, 2011). Gọi S2 .n/ là tổng các chữ số của n trong hệ nhị phân
và xét số nguyên dương k  2:
a) Chứng minh rằng tồn tại một dãy số .an / sao cho am  3 là một số lẻ và S2 .a1 a2 a3 : : : am / D
k với mọi m  1:

b) Chứng minh rằng tồn tại số nguyên dương N sao cho

S2 .3  5  7  : : :  .2m C 1// > k

với mọi số nguyên dương m  N:

Lời giải. a) Ta sẽ chứng minh bằng quy nạp. Với m D 1, ta chỉ cần chọn a1 D kiD0 2i thì có
P
ngay S.a1 / D k. Giả sử ta đã xây dựng được cho m số hạng đầu tiên của dãy. Đặt tích của chúng
là Tm . Ta xét số sau
.Tm 1/ 2'.Tm / 1

amC1 D C1
Tm
thì rõ ràng do Tm là số lẻ và theo định lý Euler thì 2'.Tm / 1 chia hết cho Tm nên số amC1 ở trên
là số lẻ. Ta có amC1 Tm D .Tm 1/2'.Tm / C 1.
Rõ ràng, trong hệ nhị phân, Tm có tận cùng là 1 nên khi trừ nó cho 1 sẽ làm giảm tổng số lượng
các số 1 là 1 đơn vị; tuy nhiên, sau khi nhân cho 2'.Tm / thì nó tận cùng bởi các số 0 và khi cộng
thêm 1 thì số lượng các số 1 sẽ vẫn là k: Suy ra S.amC1 Tm / D k và thỏa mãn điều kiện. Theo
nguyên lý quy nạp thì bài toán được chứng minh.
b) Theo bài toán trên thì chỉ cần chọn N là số chia hết cho 2kC1 1 thì

S2 .3  5  7  : : :  .2m C 1//  k C 1 > k:

Bài toán được giải quyết nhanh chóng.


290 Các phương pháp giải toán qua các kỳ thi Olympic

Bài toán 29 (APMO, 2014). Với mỗi số nguyên dương m, ký hiệu S.m/; P .m/ là tổng, tích
các chữ số của số m: Chứng minh với mọi n; tồn tại dãy số gồm n số a1 ; a2 ; : : : ; an sao cho
(
S.a1 / < S.a2 / <    < S.an /
S.ai / D P .ai C1 /; 8i D 1; 2; 3; : : : ; n

Lời giải. Đặt N.a; b/ D 22 : : : …2 111


„ ƒ‚ „ ƒ‚: : : …1 là số tự nhiên gồm a chữ số 2 đứng trước và b chữ số
a b
1 đứng sau. Suy ra S.N.a; b// D 2a C b và P .N.a; b// D 2a .
Với n D 1, ta xét a1 D 1. Với n > 1, ta chọn m là số nguyên dương sao cho 2m  m C n thì
a1 D N.2m ; 0/; ak D N.m C k 1; 2mCk 2.m C k 1// với 2  k  n 1:
m
Đồng thời, chọn an D N.m C n 1; 22 2.m C n 1//. Khi đó, ta thấy rằng S.a1 / D 2mC1
2m mCk
và P .a1 / D 2 ; ngoài ra, S.ak / D 2 và P .ak / D 2mCk 1 nên với mọi 2  k  n 1.
m
Cuối cùng S.an / D 22 và P .an / D 2mCn 1
thỏa mãn các điều kiện đã nêu. Vậy tồn tại dãy số
thỏa mãn điều kiện, ta có đpcm.
Bài toán 30 (Mở rộng IMO Shortlist, 1993). Chứng minh rằng nếu a chia hết cho b n 1 thì
trong biểu diễn hệ cơ số b thì S.a/  n.b 1/:
Lời giải. Đặt S.n/ là tổng các chữ số của n trong hệ cơ số b và f .n/ là số k lớn nhất mà
.b k 1/ j n. Đặt g.n/ D kiD1 f .i/ và g.0/ D 0: Ta có
P

C1  
X n x S.x/
g.x/ D D :
i D1
bi b 1

Hơn nữa, g.x/ C g.y/  g.x C y/. Đặt a D k.b n 1/ với k là số nguyên dương nào đó. Ta có
g.a C k 1/ g.a/  g.k 1/ , g.a C k 1/ C f .a C k/ g.a/  g.k 1/ C f .a C k/:
Ta cũng có f .a C k/ D f .kb n / D f .k/ C n nên ta được
g.a C k/ g.a/  g.k 1/ C f .k/ C n
a C k S.a C k/ a S.a/ k S.k/
,  Cn
b 1 b 1 b 1
k S.a C k/ S.a/ k S.k/
, C  Cn
b 1 b 1 b 1
Hơn nữa, S.a C k/ D S.kb n / D S.k/ nên
S.a/
 n , S.a/  n.b 1/:
b 1
Ta có đpcm.
Nhận xét. Trong hệ cơ số 10, ta có thể phát biểu và chứng minh bài toán đơn giản hơn khá
nhiều: Với mỗi số nguyên dương M là bội của 10q 1 thì S.M /  9q.
Thật vậy, giả sử ngược lại, tồn tại số M không thỏa mãn. Trong các số như vậy, ta chọn số M
nhỏ nhất mà M chia hết cho 10q 1 và S.M / < 9q.
Khi đó M ¤ 10q 1 vì S.10q 1/ D 9q nên M > 10q . Giả sử M có m C 1 chữ số với m  q
thì N D M 10m q .10q 1/ là số nhỏ hơn 10q 1 nhưng có S.N /  S.M / < 9q, mâu thuẫn.
S.n/ – hàm tổng các chữ số 291

Bài toán 31.

a) Gọi T .n/ là tổng bình phương các chữ số của số nguyên dương n. Tìm n sao cho T .n/ D n:

b) Gọi P .n/ là tích các chữ số của n: Xét dãy số:


(
a1 D a
anC1 D an C P .an /

Chứng minh an hội tụ.

Lời giải. a) Ta thấy nếu n có k  4 chữ số thì T .n/  81k < 10k 1
 n, không thỏa mãn. Do
đó k D 1; 2; 3. Ta xét các trường hợp:

1. Nếu k D 1 thì dễ dàng kiểm tra được chỉ có n D 1 là thỏa mãn.

2. Nếu k D 2 thì đặt n D ab, ta có 10a C b D a2 C b 2 nên a.10 a/ D b.b 1/ và


cũng vô nghiệm. Vế trái chỉ có thể là 9; 16; 21; 24; 25 trong khi vế phải thì chỉ có thể là
0; 2; 6; 12; 20; 30; 42; 56; 72:

3. Nếu k D 3 thì n D abc và 100a C 10b C c D a2 C b 2 C c 2 . Do vế phải không quá 243


nên a 2 f1; 2g.

 Nếu a D 1 thì 99 C 10b C c D b 2 C c 2 . Dễ thấy b; c phải ít nhất có 1 số lớn hơn 7.


Ta có các trường hợp sau:
C n D 18c ) 99 C 80 C c D 64 C c 2 , c 2 c 115 D 0, không có nghiệm
nguyên.
C n D 19c ) 99 C 90 C c D 81 C c 2 , c 2 c 108 D 0, không có nghiệm
nguyên.
C n D 1b8 ) 99 C 10b C 8 D b 2 C 64 , b 2 10b 43 D 0, không có nghiệm
nguyên.
C n D 1b8 ) 99 C 10b C 9 D b 2 C 81 , b 2 10b 27 D 0, không có nghiệm
nguyên.
 Nếu a D 2 thì 196 C 10b C c D b 2 C c 2 . Rõ ràng vế phải không vượt quá
81 C 81 D 162 nên cũng vô nghiệm. Do đó, trường hợp này không có nghiệm.

Vậy chỉ có n D 1 thỏa mãn đề bài.


Nhận xét. Nếu thay điều kiện tổng bình phương các chữ số thành tổng lập phương các chữ số
thì kết quả sẽ là 1; 153; 370; 371; 407.
Còn nếu thay bằng tổng lũy thừa bậc 4 thì có các số là 1; 1634; 8208; 9474.
Tất nhiên, các lũy thừa bậc cao hơn khó có thể xử lý bằng lập luận thông thường được mà cần có
sự hỗ trợ của máy tính.
P .n/ P .n/ 9k
b) Chú ý rằng limn!C1 n
D 0 do 0 < n
< 10k 1 với k là số chữ số của n:
Giả sử dãy an không bị chặn. Ta xét 2 trường hợp:

1. Nếu an bắt đầu bởi 10 với n nào đó thì P .an / D 0 và suy ra anC1 D an nên dãy này là
dãy hằng, mâu thuẫn với điều giả sử.
292 Các phương pháp giải toán qua các kỳ thi Olympic

2. Nếu an không bắt đầu bởi 10 thì do an không bị chặn nên với n đủ lớn và k là số nguyên
dương nhỏ nhất sao cho ak có n chữ số thì ak 1 < 10n 1 và ak > 11  10k 2 (ta xét ak
không bắt đầu bởi 10). Suy ra
P .ak 1 / 10n 2
1
P .ak 1 / D ak ak 1 > 10n 2
nên > n 1
D :
ak 1 10 10
P .n/
Điều này mâu thuẫn do ta có lim n
D 0:
n!C1

Vậy điều giả sử là sai nên dãy đã cho hội tụ.


Bài toán 32 (British MO, 2012). Gọi a là số nguyên có có 2013 chữ số và trong biểu diễn nhị
phân của nó, số lượng số 0 nhiều hơn số 1: Gọi n là số các số a có dạng như thế và s là tổng tất
cả các số a: Chứng minh rằng trong hệ nhị phân, n C s cũng có số lượng số 0 nhiều hơn số 1:
Lời giải. Theo giả thiết thì trong hệ nhị phân, số n có 2013 chữ số và chữ số đầu tiên là 1; có ít
nhất 1007 số 0: Mỗi số khác nhau bởi vị trí của các số 0 đó nên suy ra
2012 2012
X
i 1X i 1 1006 1 1006
nD C2012 D C C2012 D 22011 C :
i D1007
2 i D0 2012 2 2 2012

Tiếp theo, gọi ai là số các số mà trong biểu diễn nhị phân của nó có số 1 ở vị trí thứ i: Ta có
2012
X
sD ai 2 i :
i D0

Ngoài ra, số các số ai cũng chính là số cách chọn ra số 0 trong các vị trí còn lại nên
2011 2011
X
i 1X i 1 1005 1006
D 22010 1005

ai D C2011 D C C2011 C C2011 C2011 :
i D1007
2 i D0 2012 2

Suy ra
2012
X 2012
X
ai 2i D 22012 n C ai 2i D 22012 n C 22010 1005
22012
 
sD C2011 1 :
i D0 i D0

Từ đó, ta có
n C s D 22012 C 1 22011 1005
C 22012 1 22010 1005
   
C2011 C2011
D 24023 C 24022 C 22010 22013 C2011
1005
:

Chú ý rằng n C s < 24024 nên nó có không quá 4024 chữ số và ta viết lại

n C s D 22013 22010 C 22009 C20111005


C 22010 :


Suy ra 2012 trong 2013 chữ số tận cùng của n C s trong hệ nhị phân là 0: Cuối cùng, nếu như
n C s có đúng 4024 chữ số thì tất cả các chữ số còn lại đều là 1 và giá trị này lúc bấy giờ là

22013 .1 C 2 C 22 C    C 22010 / C 22010 :

Số này rõ ràng lớn hơn giá trị của n C s nên suy ra n C s có nhiều số 0 hơn 1: Ta có đpcm.
S.n/ – hàm tổng các chữ số 293

Bài toán 33. Với n là số nguyên dương không có chứa chữ số 0; gọi P .n/ là tích các chữ số
của n: Giả sử n2 C P .n/ là một số chính phương. Chứng minh rằng n có không quá 4 chữ số.

Lời giải. Trước hết, ta thấy rằng P .n/  n. Thật vậy, giả sử n có k chữ số là a1 a2 a3 : : : ak thì
 k 1  k 1  k 1
k 1 10 k 1 10 10
n  10  ak D 9  ak  a1 a2 a3    ak D P .n/;
9 9 9
nên ta có k
 1
n 10
 : ./
P .n/ 9
Ta có n2 C P .n/ > n2 . Tuy nhiên, n2 C 6P .n/ D .n C 1/2 , 6P .n/ D 2n C 1, vô lý. Do đó

n2 C 6P .n/  .n C 2/2 :

Ngoài ra, ta có P .n/  n nên

n2 C 6P .n/  n2 C 6n < n2 C 6n C 9 D .n C 3/2

hay
n2 C 6P .n/ D .n C 2/2 , 3P .n/ D 2n C 2:
Nếu n có ít nhất 5 chữ số thì theo ./; ta có
 4
10 3
n P .n/ > P .n/
9 2

nên 2n C 2 > 3P .n/ C 2, mâu thuẫn. Do đó, n có không quá 4 chữ số. Ta có đpcm.

Nhận xét. Chú ý rằng chỉ có duy nhất số n D 2 thỏa mãn điều kiện trên nhưng để chứng minh,
đòi hỏi nhiều đánh giá rắc rối.

3. Bài tập tự giải.


Bài tập 1.

a) Tìm số tự nhiên n sao cho S.n/ D n2 2003n C 5:

b) Tìm số tự nhiên n sao cho n C S.n/ D 2003:

c) Hỏi có tồn tại số n sao cho S.2n / D S.2nC1 / không?

d) Chứng minh rằng tồn tại n > 1000 sao cho S.2n / > S.2nC1 /.

Bài tập 2.
1999
a) Đặt A D 44444444 và B D 29 . Tính S.S.S.A/// và S.S.S.B///:

b) Viết các số từ 1 đến 106 1 thành một dãy liên tiếp được số N: Tính S.N /:

Bài tập 3. Gọi R.n/ là tổng bình phương các chữ số của n.
294 Các phương pháp giải toán qua các kỳ thi Olympic

a) Tính R.R.: : : .21990 / : : :// với hàm R lặp lại 2004 lần.

b) Chứng minh rằng nếu thực hiện hàm trên với số lần đủ lớn thì ta được 1 hoặc 4:

Bài tập 4.

a) (Argentina, 2012) Tìm tất cả các số nguyên dương n sao cho 9S.n/ D 16S.2n/:

b) Chứng minh rằng 39 là số nguyên dương n nhỏ nhất sao cho S.10n n/ chia hết cho 170.

c) Tìm tất cả các số nguyên dương n sao cho S.2n / D 5:

d) Tìm n sao cho n C S.n/ S.S.n// D 2014:

Bài tập 5.

a) (Nga, 1961) Biết rằng với mỗi số nguyên dương k, số S.k/; S.k C l/ đều chia hết cho 11
và không có số nào giữa hai số k; k C l nào thỏa mãn điều này. Tính GTLN của l:

b) Hỏi có 2 số nguyên dương liên tiếp nào mà tổng các chữ số của chúng đều chia hết cho
125 hay không? Vì sao?

c) Chứng minh rằng tồn tại vô số số nguyên dương n sao cho n  2014S.3n/:

Bài tập 6.

a) Một số có 6 chữ số thỏa mãn: tổng các chữ số ở vị trí chẵn bằng tổng các số ở vị trí lẻ.
Tính độ chênh lệch lớn nhất giữa hai số liên tiếp thỏa mãn điều kiện trên.

b) Cho số nguyên dương n D abcdef thỏa mãn def abc D 6n. Tính S.n/:

c) Hỏi tập hợp các số n sao cho S.n/  10100 là hữu hạn hay vô hạn? Vì sao?

d) (AMC 10A) Chứng minh rằng có đúng 4 số nguyên dương thỏa

n C S.n/ C S.S.n// D 2007:

Bài tập 7.

a) (Czech and Slovak MO vòng 1, 2012) Gọi A là tập hợp tất P cả các số nguyên dương có
đủ 10 chữ số và mỗi số xuất hiện đúng 1 lần. Tính số dư của a2A S.a/ khi chia cho 77:

b) (Czech and Slovak MO vòng 2, 2012) Gọi Sn là tổng của tất cả các số có n chữ số và chỉ
chứa các chữ số 1; 2; 3: Tìm tất cả các giá trị của n sao cho S.n/ chia hết cho 7:

Bài tập 8. a) (Oliforum contest lần 3) Cho đa thức P .n/ có hệ số nguyên và đặt sn D P .n/:
Chứng minh rằng tồn tại một số nguyên dương nào đó xuất hiện vô hạn lần trong dãy .sn /:

b) Gọi S.n/; P .n/ lần lượt là tổng và tích các chữ số của n: Chứng minh rằng với mọi m thì
tồn tại vô hạn n sao cho S.P .n// C S.S.n// C P .S.n// C P .P .n// D m:

c) (Brazil 2011) Chứng minh rằng với mọi n, tồn tại số nguyên dương a có n chữ số sao cho
tổng bình phương các chữ số của a là một số chính phương.
S.n/ – hàm tổng các chữ số 295

Bài tập 9. Tìm số nguyên dương x sao cho:

a) x C S.x/ C S.S.x// D 1993:

b) x C S.x/ C S.S.x// C S.S.S.x/// D 1993:

Bài tập 10. Chứng minh rằng tồn tại vô số số N sao cho N không có chứa chữ số 0 và:

a) N chia hết cho S.N /:

b) N chia hết cho S.N / C 1:

Bài tập 11. Chứng minh rằng với mỗi số nguyên dương n, tồn tại số tự nhiên a có n chữ số
khác 0 và chia hết cho S.a/.

Bài tập 12.

a) (World of mathematic 232) Xét dãy số .an / định nghĩa bởi


(
a1 D 1
anC1 D S.an C S.an //

với n  1: Tính tổng a1 C S.a2 / C S.S.a3 // C    C S.: : : S .a2005 / : : :/.


„ ƒ‚ …
2004

b) (World of mathematic 194) Chứng minh rằng không tồn tại số nguyên dương nào xuất
hiện vô hạn lần trong các dãy fS.2n/; n  1g ; fS.nŠ/; n  1g.
nC1
c) (World of Mathematic 223) Tìm tất cả các số nguyên dương n sao cho S.n/
bằng số các
chữ số của n:

Bài tập 13.

a) Có bao nhiêu số tự nhiên có n  2 chữ số sao cho S.n/ chia hết cho 5?

b) Tìm n sao cho S.n/ D n2 10n 35.

c) Chứng minh rằng không tồn tại n sao cho n D 75S.n/? Câu hỏi tương tự với 63?

d) Hỏi có bao nhiêu số tự nhiên n có 2 chữ số mà S.n/ D S.6n/?

Bài tập 14.

a) Xét k  1 . mod 9/. Chứng minh rằng tồn tại n sao cho S.3n2 C 3n C 1/ D k. Tính GTNN
của S.3n2 C 3n C 1/:

b) Gọi p ¤ 2; 3; 5 là các số nguyên tố. Với mỗi k ¤ 1, chứng minh rằng tồn tại một bội của
p k có tổng các chữ số đúng bằng p:

Bài tập 15.

a) Cho số chính phương n có 4 chữ số. Chứng minh rằng S.n/  7.


296 Các phương pháp giải toán qua các kỳ thi Olympic

b) Gọi M là giá trị lớn nhất của S.a/ C S.b/ C S.c/ với a C b C c D 2005: Hỏi có bao
nhiêu bộ có thứ tự .a; b; c/ sao cho S.a/ C S.b/ C S.c/ D M ?

Bài tập 16 (IMO Shortlist, 1992). Với mỗi n , ký hiệu S.n/ là tổng các chữ số của n viết trong
hệ nhị phân. Chứng minh rằng

a) S.n2 /  12 S.n/ .S.n/ C 1/ với mọi n:

b) Tồn tại vô số số n sao cho bất đẳng thức trên là đẳng thức.
S.u2n /
c) Tồn tại dãy số .un / sao cho S.un /
khi n ! C1.

Bài tập 17.

a) Chứng minh rằng nếu n D 700S.n/ thì n 2 f2100; 4200; 6300; 8400g.

b) (Italy MO, 2012) Chứng minh rằng n D 2700 là số duy nhất thỏa mãn n D 300S.n/:

c) (XII Cono Sur Mathematical Olympiad, 2001) Tìm tất cả các số nguyên dương n sao cho

2001  S.m/ D m:

d) Tìm N sao cho S.kN / là số chẵn với mọi 1  k  1992 nhưng S.1993N / là số lẻ. Từ đó
chứng minh rằng không tồn tại N sao cho S.kN / là số chẵn với mọi số nguyên dương k:

Bài tập 18.

a) (Đức, 1998) Chứng minh rằng tồn tại vô số số nguyên dương n sao cho f .3k /  f .3kC1 /:

b) Tìm các giá trị nguyên dương của m sao cho tồn tại n nguyên dương để S.n/ D mS.3n/.

c) Hỏi có tồn tại hay không số k 2 N sao cho mọi n D k; 2k; 3k; : : : ; 2008k thì S.n/ j n?

Bài tập 19.

a) (OSN 2011 Mathematics West Java Province Selection Test) Tìm số nguyên dương
n  123456 sao cho tồn tại số nguyên dương z để tổng các chữ số của z 2 là n:

b) (Mathcounts, 2004-2005) Tính tổng tất cả các số có 2 chữ số và chia hết cho tổng và tích
các chữ số của nó.

c) (Iran TST, 2010) Chứng minh rằng với mọi m, tồn tại số N sao cho với mỗi 2  b  1389
thì S.N / trong hệ cơ số b là lớn hơn m:

d) Tìm số nguyên dương n nhỏ nhất sao cho S.n/ D 100 và S.2n/ D 110:

Bài tập 20.

a) (Nhật Bản MO, 2012 - vòng 1) Gọi A là số nguyên dương là bội của 3 nhưng không là
bội của 9: Biết rằng khi cộng vào A tích các chữ số của A thì ta được một bội của 9: Tìm
giá trị lớn nhất của A:
S.n/ – hàm tổng các chữ số 297

b) (Romani MO, 2014) Đặt an D 18 „


777ƒ‚
: : :…
7 889 với mỗi số không âm n: Chứng minh với
n
an
mọi n thì an chia hết cho 13 và đặt thương cn D 13
. Tìm tất cả các số n sao cho

S.an / D 2S.cn /:

Bài tập 21. Với mỗi số tự nhiên n D at at 1 : : : a3 a2 a1 , ta đặt:


X X
T .n/ D 10 ai C ai :
i 0 . mod 2/ i 1 . mod 2/

Tìm số nguyên dương A nhỏ nhất sao cho tồn tại các số tự nhiên n1 ; n2 ; : : : ; n148 và m1 ; m2 ; : : : ;
m149 thỏa mãn các điều kiện:
€A D n C n2 C    C n148 D m1 C m2 C    C m149
1
T .n1 / D T .n2 / D    D T .n148 /
T .m1 / D T .m2 / D    D T .m149 /

Tài liệu tham khảo


[1] Diễn đàn toán học:
 artofproblemsolving.com.
 mathscope.org.
 diendantoanhoc.net.

[2] Tuyển tập diễn đàn Toán học, 2006.

[3] Trần Nam Dũng, đề thi chọn đội tuyển PTNK 2008.

[4] IMO Booklet của Anh, 2012.

[5] Phan Huy Khải, Các chuyên đề Số học - quyển 4, 2004.


298 Các phương pháp giải toán qua các kỳ thi Olympic
LỜI GIẢI VÀ BÌNH LUẬN
ĐỀ THI VMO 2015
Ban biên tập

1. Đề thi
Ngày thi thứ nhất. (08/01/2015)

Bài 1. (5,0 điểm) Cho a là số thực không âm và .un / là dãy số xác định bởi
1 n2 q 2
u1 D 3; unC1 D un C 2 un C 3 với mọi n  1:
2 4n C a
a) Với a D 0; chứng minh rằng dãy số có giới hạn hữu hạn và tìm giới hạn đó.
b) Với mọi a 2 Œ0I 1; chứng minh rằng dãy số có giới hạn hữu hạn.
Bài 2. (5,0 điểm) Cho a; b; c là các số thực không âm. Chứng minh rằng
p p p 
3.a2 C b 2 C c 2 /  .a C b C c/ ab C bc C ca C
C.a b/2 C .b c/2 C .c a/2  .a C b C c/2 :

Bài 3. (5,0 điểm) Cho số nguyên dương k. Tìm số các số tự nhiên n không vượt quá 10k thỏa
mãn đồng thời các điều kiện sau:
i. n chia hết cho 3;
ii. các chữ số trong biểu diễn thập phân của n thuộc tập hợp f2; 0; 1; 5g :
Bài 4. (5,0 điểm) Cho đường tròn .O/ và hai điểm B; C cố định trên .O/, BC không là đường
kính. Điểm A thay đổi trên .O/ sao cho tam giác ABC nhọn. Gọi E; F lần lượt là chân đường
cao kẻ từ B; C của tam giác ABC . Cho .I / là đường tròn thay đổi đi qua E; F và có tâm là I:
q
DB cot B
a) Giả sử .I / tiếp xúc với BC tại điểm D: Chứng minh rằng DC D cot C
:

b) Giả sử .I / cắt cạnh BC tại hai điểm M; N . Gọi H là trực tâm tam giác ABC và P; Q
là các giao điểm của .I / với đường tròn ngoại tiếp tam giác HBC . Đường tròn .K/ đi
qua P; Q và tiếp xúc với .O/ tại điểm T (T cùng phía A đối với PQ). Chứng minh rằng
đường phân giác trong của góc ∠M T N luôn đi qua một điểm cố định.
Ngày thi thứ hai. (09/01/2015)

Bài 5. (7,0 điểm) Cho fn .x/ là dãy đa thức xác định bởi

f0 .x/ D 2; f1 .x/ D 3x; fn .x/ D 3xfn 1 .x/ C .1 x 2x 2 /fn 2 .x/ với mọi n  2:

299
300 Các phương pháp giải toán qua các kỳ thi Olympic

Tìm tất cả các số nguyên dương n để fn .x/ chia hết cho x 3 x 2 C x:


Bài 6. (7,0 điểm) Với a; n là các số nguyên dương, xét phương trình a2 x C 6ay C 36z D n,
trong đó x; y; z là các số tự nhiên.
a) Tìm tất cả các giá trị của a để với mọi n  250, phương trình đã cho luôn có nghiệm tự
nhiên .x; y; z/.
b) Biết rằng a > 1 và nguyên tố cùng nhau với 6. Tìm giá trị lớn nhất của n theo a để phương
trình đã cho không có nghiệm .x; y; z/:
Bài 7. (6,0 điểm) Có m học sinh nữ và n học sinh nam .m; n  2/ tham gia một liên hoan song
ca. Tại liên hoan song ca, mỗi buổi biểu diễn một chương trình văn nghệ. Mỗi chương trình văn
nghệ bao gồm một số bài hát song ca nam – nữ mà trong đó, mỗi đôi nam – nữ chỉ hát với nhau
không quá một bài và mỗi học sinh đều được hát ít nhất một bài. Hai chương trình được coi là
khác nhau nếu có một cặp nam – nữ hát với nhau ở chương trình này nhưng không hát với nhau ở
chương trình kia. Liên hoan song ca chỉ kết thúc khi tất cả các chương trình khác nhau có thể có
đều được biểu diễn, mỗi chương trình được biểu diễn đúng một lần.
a) Một chương trình được gọi là lệ thuộc vào học sinh X nếu như hủy tất cả các bài song ca
mà X tham gia thì có ít nhất một học sinh khác không được hát bài nào trong chương trình
đó. Chứng minh rằng trong tất cả các chương trình lệ thuộc vào X thì số chương trình có
số lẻ bài hát bằng số chương trình có số chẵn bài hát.
b) Chứng minh rằng ban tổ chức liên hoan có thể sắp xếp các buổi biểu diễn sao cho số các
bài hát tại hai buổi biểu diễn liên tiếp bất kỳ không cùng tính chẵn lẻ.

2. Nhận xét chung


Kỳ thi chọn học sinh giỏi quốc gia môn Toán năm học 2014-2015 (VMO 2015) diễn ra trong 2
ngày 08 và 09/01/2015. Về tổng quan, ta có thể thấy rằng đề thi năm nay hay, phù hợp với việc
tuyển chọn học sinh giỏi.
Các vấn đề đặt ra trong đề khá căn bản, quen thuộc nhưng cũng có những khó khăn nhất định ở
từng bài. Ví dụ bài tổ hợp khai thác chủ đề quen thuộc về các số chia hết cho 3 có các chữ số
thuộc 1 tập hợp (dùng căn bậc 3 của đơn vị hoặc truy hồi), nhưng đưa chữ số 0 vào gây chút rắc
rối. Bài 6 số học thì khai thác định lý Sylvester về biểu diễn dạng tổ hợp tuyến tính ax C by: Bài
bất đẳng thức rất nhẹ nhàng (so với bất đẳng thức khủng năm ngoái), có thể giải bằng kiến thức
THCS nhưng cũng gây khó cho không ít thí sinh.
Năm nay có một điểm đặc biệt là chỉ có một bài hình học, lại là bài khá khó nên có thể sẽ khiến
một số bạn giỏi hình chưa có cơ hội bộc lộ hết sở trường của mình. Thay vào một bài hình là một
bài tổ hợp dài, không quá khó về bản chất nhưng đòi hỏi khả năng đọc hiểu của thí sinh. Nhiều
thí sinh đã chia sẻ rằng: "Em không hiểu bài tổ hợp họ hỏi gì?". Quả thật, khả năng đọc hiểu, xây
dựng và chuyển đổi mô hình là điểm yếu cố hữu của học sinh Việt Nam.
Đi vào chi tiết, ta có thể điểm qua từng bài như sau:
Ngày 1 khá cơ bản, gồm các vấn đề ít nhiều đều đã có giới thiệu trong chương trình Toán chuyên
một cách đại trà.
 Bài 1 (dãy số) có câu a quá dễ và quen thuộc, câu b lại khó khăn kỹ thuật nhất định, cần
sử dụng định lý kẹp, đơn điệu hoặc bổ đề ánh xạ co. Nhiều thí sinh bị lấn sâu vào câu b
của bài toán dẫn đến thiếu thời gian để giải quyết các câu còn lại.
Lời giải và bình luận đề thi VMO 2015 301

 Bài 2 (bất đẳng thức) thì không quá dễ cũng không quá khó, nó là một bất đẳng thức đối
xứng, đồng bậc, dạng tương đối phổ biến với các học sinh. Đổi biến để khử căn xong là có
thể nhìn ngay ra bất đẳng thức Schur bậc 4 và AM-GM. Bài này có rất nhiều cách giải nên
sẽ rất tiếc cho học sinh nào bỏ vế sau, vì đây là vế chính của bài toán. Đây là một bài toán
khá hợp lý tương xứng với vị trí của nó trong đề thi.

 Bài 3 (tổ hợp) khai thác chủ đề quen thuộc (đã xuất hiện trong các đề thi Romania 2003,
Phổ thông năng khiếu 2009, Lâm Đồng 2014). Có hơi rắc rối ở chỗ số 0 nhưng lại được
"giải" bằng điều kiện n < 10k (chứ không phải có m chữ số). Phương pháp căn đơn vị giải
quyết gọn nhưng cũng cần trình bày chặt chẽ. Phương pháp truy hồi sẽ gây khó một chút vì
có đến 3 dãy. Ở bài này, có lẽ thí sinh làm trọn vẹn không nhiều, nhưng giám khảo chấm
sẽ khá mệt.

 Bài 4 (hình học phẳng) là bài hình duy nhất, có hai câu đều ở mức độ trung bình khó,
nhưng số học sinh làm trọn vẹn bài này sẽ không nhiều. Chủ đề về phương tích, trục đẳng
phương vẫn đóng vai trò chủ đạo trong suốt các năm gần đây.

Ở ngày thi thứ hai, đề thi so với ngày đầu tiên đã "gây sốc", khó cả về kỹ thuật lẫn tư duy. Nguyên
nhân là không có bài hình và bài tổ hợp phát biểu quá dài. Hai bài 5, 6 tuy quen thuộc nhưng lại
là phần mà các thí sinh ít để ý. Về ý tưởng thì đề ngày 2 hay hơn.

 Bài 5 (dãy số, đa thức) tương đối cơ bản. Dùng phương trình đặc trưng hoặc quy nạp
dễ dàng tìm được fn D .2x 1/n C .x C 1/n . Kiểu bài toán chia hết này khá giống với
những bài chia hết trong số nguyên. Cách làm truyền thống là khai thác tính tuần hoàn của
số dư. Tuy nhiên, thực tế nhiều thí sinh không có phương hướng gì.

 Bài 6 (số học) sẽ không khó khăn để làm nếu đã quen với định lý Sylvester. Có hai bài
toán mẫu trước đó là IMO 1983 và Vietnam TST 2000. Cũng như các năm, bài số học ít
khi xuất hiện, nhưng nếu có thì nó sẽ là một bài khó. Tuy nhiên, nếu chưa biết định hướng
sử dụng định lý này thì đây quả là một thử thách thực sự.

 Bài 7 (tổ hợp) đề khá dài, quan hệ giữa các khái niệm khá rối và dễ dẫn đến hiểu nhầm.
Học sinh cần chuyển về một mô hình toán học nào đó (bảng, graph hoặc hàm số) để thấy
rõ hơn vấn đề. Thực ra về bản chất thì nó chỉ là một bài toán đếm có thể giải bằng song
ánh và quy nạp.

Như vậy, nhìn chung thì đề năm nay hay. Ngày 1 ra thật cơ bản và quen để cho đại trà. Ngày 2
gây khó và phân loại. Qua khảo sát một số đội tuyển, thí sinh đánh giá đề này vừa sức và ít nhiều
cũng giải quyết được một số nội dung trong đề bài. Dự đoán năm nay điểm đạt giải khuyến khích
sẽ ở vào khoảng 13-15, còn điểm lọt vào vòng 2 để thi chọn đội tuyển IMO là 24. Năm nay chắc
sẽ ít giải nhất.

Tài liệu này chúng tôi đã tổng hợp, biên tập lại từ nhiều nguồn, đặc biệt là mathscope.org
để giới thiệu cho học sinh, thầy cô Chuyên Toán tham khảo. Do hoàn thành trong thời gian
gấp rút và khối lượng tính toán lớn nên có thể còn nhiều sai sót. Mọi góp ý, thắc mắc xin
gửi về các địa chỉ mail: trannamdung@yahoo.com, nguyentatthudn@gmail.com,
babylearnmath@yahoo.com, lephuclu@gmail.com. Xin chân thành cảm ơn!
302 Các phương pháp giải toán qua các kỳ thi Olympic

3. Lời giải chi tiết và bình luận


Bài toán 1. Cho a là số thực không âm và dãy số .un / được xác định bởi

1 n2 q 2
u1 D 3; unC1 D un C 2 un C 3 với mọi n  1:
2 4n C a

a) Với a D 0, chứng minh rằng dãy số .un / có giới hạn hữu hạn và tìm giới hạn đó.

b) Với mọi a 2 Œ0I 1, chứng minh rằng dãy số .un / có giới hạn hữu hạn.

Lời giải.
a) Với a D 0, ta có dãy số unC1 D 21 un C 14 u2n C 3.
p
p
Xét hàm số f .x/ D 12 x C 41 x 2 C 3 với x > 0. Ta có

1 1 x
f 0 .x/ D C p > 0, 8x > 0:
2 4 x C3
2

Suy ra hàm số f .x/ đồng biến trên .0I C1/.


Từ công thức truy hồi ta suy ra với mọi n thì
p
3 3
un > 0 , unC1 D f .un / và u2 D C < u1
2 2
nên bằng quy nạp ta suy ra được .un / là dãy giảm và bị chặn dưới bởi 0 nên nó có giới hạn hữu
hạn.
Đặt lim un D x thì x  0 và là nghiệm của phương trình
(
1 1p 2 p x0
xD xC x C 3 , x 2 C 3 D 2x , , x D 1:
2 4 x 2 C 3 D 4x 2

Vậy giới hạn cần tìm là lim un D 1.


n2 n2 1
b) Với a 2 Œ0I 1 ta có 4n2 C1
 4n2 Ca
 4
. Do đó

1 n2 q 2 1 1q 2
un C 2 un C 3  unC1  un C un C 3:
2 4n C 1 2 4
Xét hai dãy .xn / và .yn / được xác định bởi
8 8
< x1 D 3 < y1 D 3
và 2
: xnC1 D 1 xn C 1 xn2 C 3 : ynC1 D 1 yn C n
q q
yn2 C 3
2 4 2 4n2 C 1
Theo kết quả câu a) ta có lim xn D 1.
Ta sẽ chứng minh lim yn D 1 và từ đó ta suy ra được lim un D 1 và đó là điều cần chứng minh.
Dưới đây ta sẽ nêu hai cách để xử lý giới hạn này.
Lời giải và bình luận đề thi VMO 2015 303

Cách 1. Ta có

1 n2 .yn 1/ .yn C 1/ 2n2 1


ynC1 1D .yn 1/ C C
4n2 C 1 4n2 C 1
p
2 yn2 C 3 C 2 2
1 n2 .yn 1/ .yn C 1/ 1
D .yn 1/ C
4n2 C 1 2.4n2
p
2 yn2 C 3 C 2 C 1/
1 n2 .yn 1/ .yn C 1/
< .yn 1/ C
4n2 C 1
p
2 yn2 C 3 C 2
!
1 n2 yn C 1
D .yn 1/ C
2 4n2 C 1 yn2 C 3 C 2
p

Dễ thấy
1 n2 yn C 1 1 n2 1 1 3
0< C 2 p < C 2 < C D
2 4n C 1 yn2 C 3 C 2 2 4n C 1 2 4 4
với mọi n D 1; 2; :::
3 3 n 1

 Nếu yn > 1 với mọi n thì ta có jynC1 1j < 4
jyn 1j <    < 4
jy1 1j.
Từ đó, suy ra lim yn D 1.
 Nếu tồn tại n0 2 N để yn0  1 thì ta có yn < 1; 8n > n0 .
2
Nếu tồn tại N > n0 sao cho yN C1 > yN , khi đó do f .x/ D 4xx2 C1 là một hàm đồng biến trên
.0I C1/ nên ta có
.N C 1/2 N2
> :
4.N C 1/2 C 1 4N 2 C 1
Suy ra
1 .N C 1/2 q 2 1 N2
q
2
yN C1 C y N C1 C 3 > y N C yN C3
2 4.N C 1/2 C 1 2 4N 2 C 1
hay yN C2 > yN C1 .
Do đó, bằng quy nạp ta chứng minh được dãy .yn / tăng từ số hạng thứ N trở đi. Do đó, dãy .yn /
hội tụ. Đặt lim yn D y ta tìm được y D 1:
Nếu .yn / là dãy giảm kể từ số hạng thứ n0 trở đi, kết hợp với dãy .yn / bị chặn dưới bởi 0 ta suy
ra dãy .yn / hội tụ. Đặt lim yn D y ta tìm được y D 1 nên trường hợp này loại.
Vậy lim yn D 1 và bài toán được chứng minh.
2
Cách 2. Ta sẽ chứng minh bằng quy nạp rằng yn  1 n
với mọi n  2. ./
Thật vậy, với n D 2, dễ dàng thấy rằng ./ đúng.
2
Giả sử ./ đúng với n  2 thì ta có yn  1 n
 0. Suy ra
s
2
n2 q 2 n2
  
1 1 2 2
yn C 2 yn C 3  1 C 2 1 C 3:
2 4n C 1 2 n 4n C 1 n
304 Các phương pháp giải toán qua các kỳ thi Olympic

Ta cần chứng minh


s
2
2 2
  
1 2 n 2
1 C 2 1 C31
2 n 4n C 1 n nC1
s
n2 2 2
     
2 1 2
, 2 1 C3 1 1
4n C 1 n nC1 2 n
2n p 2 n2 n C 2
, 2 n nC1
4n C 1 2n.n C 1/
p
2
, 4n .n C 1/ n2 n C 1  .4n2 C 1/.n2 n C 2/
2 2
, 16n4 .n C 1/2 .n2 n C 1/  .4n2 C 1/ .n2 n C 2/

Ta thấy bất đẳng thức trên đúng với n D 2; 3, ta xét n  4. Chú ý rằng
2
16n2 .n C 1/2  .4n2 C 1/ , 4n.n C 1/  4n2 C 1, đúng và
2
n2 .n2 n C 1/  .n2 n C 2/ , n.n 2/2  4 cũng đúng với mọi n  4:
2
đúng với mọi n, mà lim xn D lim 1 n2 D 1 nên ta có

Từ đó ta được xn  un  1 n
lim un D 1.

Nhận xét. Ý thứ nhất của bài toán rất cơ bản, thuộc dạng mẫu mực và là nội dung quen thuộc
của phần lý thuyết giới hạn dãy số mà hầu hết các học sinh đã được giới thiệu. Ta chỉ cần chứng
minh dãy số giảm bằng quy nạp là coi như bài toán được giải quyết xong.

Tuy nhiên, ý thứ hai khá mới lạ và có thể coi đây là câu khó nhất trong ngày thi đầu tiên. Việc
đánh giá dãy .un / kẹp giữa hai dãy .xn / và .yn / là suy nghĩ rất tự nhiên. Từ kết quả câu a, ta thấy
rằng lim xn D 1 nên ta suy nghĩ đến việc chứng minh lim yn D 1 để có được giới hạn. Để làm
được điều này, ta đi chứng minh dãy .yn / là dãy giảm và bị chặn dưới.
2 p 4
Do 2 .ynC1 yn / D 4n2n2 C1 yn2 C 3 yn nên để có .yn / giảm thì ta cần có yn2  12n412n C8n2 C1
.
Tuy nhiên, không suôn sẻ như vậy, việc chứng minh tính chất này quá khó do phải thực hiện quy
nạp với biểu thức phức tạp. Còn nếu chứng minh dãy này bị chặn dưới bởi 1 thì cũng quy nạp
không thành công bởi hệ số biến thiên theo n gây ra khá nhiều rắc rồi.

Điều này gợi ý cho ta một đánh giá nhẹ nhàng hơn là yn  1 kn với k dương nào đó. Để thực
hiện được quy nạp, ta cần có đánh giá
s
k
1 2 n 2
k 2 k
C 2 3 C .1 / 1
2 4n C 1 n nC1
s
n2 k 2 n2 2k n C n C k
, 2 3 C .1 /  :
4n C 1 n 2n.n C 1/

Xét với k D 1, ta thấy bất đẳng thức trên chính là

56n7 52n6 C 48n5 37n4 C 18n3 11n2 C 2n 1  0; đúng do n  1:


Lời giải và bình luận đề thi VMO 2015 305

Trong lời giải theo cách 2, ta đã sử dụng đánh giá dạng trên với k D 2:

Dưới đây là một số bài toán tương tự:


Bài 1.(Đề chọn đội tuyển ĐHSP 2010) Cho dãy số .xn / thỏa mãn:
8
< x1 D x2 D 1;
2
: xnC2 D xnC1 1
xn ; n  1
2
Chứng minh rằng dãy số này có giới hạn hữu hạn và tìm giới hạn đó.
Gợi ý. Ta đánh giá jxn j < n1 ; n  5.
Bài 2. (VMO 2012) Cho dãy số .xn / thỏa mãn:
8
< x1 D 3;
: xn D n C 2 .xn 1 C 2/ ; n  2
3n
Chứng minh rằng dãy số này có giới hạn hữu hạn và tìm giới hạn đó.
3
Gợi ý. Chứng minh dãy giảm trực tiếp hoặc đánh giá xn > 1 C n
với n  2.
Bài 3. (Hưng Yên, 2011) Cho dãy số .xn / xác định bởi
8
< x1 D a > 0;
2
: xnC1 D xn C xn ; n  1
n2
Tìm tất cả các giá trị a sao cho dãy số có giới hạn hữu hạn.
1 p 1
Gợi ý. Ta chứng minh rằng xn Cn2
 an.nC1/
.
Nói chung, cách đánh giá các dãy số thông qua một hàm biến thiên theo n không phải quá mới
mẻ và khá phổ biến trong các bài dãy số có hệ số thay đổi theo n hoặc là tổng của nhiều số hạng
của dãy.
Ngoài ra, ta có thể tiếp cận bài này theo kiểu dùng để chứng minh các dãy nghiệm của phương
trình thông qua điểm bất động. Dưới đây là một cách như thế:
2 p
Đặt fn .x/ D x2 C 4nn2 Ca x 2 C 3 với n  1 và x > 0.
Rõ ràng hàm số có điểm bất động duy nhất là
s
3
an D a 2
.2 C 2n2
/ 1

Nếu x > an thì fn .x/ > an và ngược lại, nếu 0 < x < an thì fn .x/ < an . Ngoài ra, ta thấy rằng
an xác định như trên tăng ngặt và có giới hạn 1.
Xét dãy số .un /, nếu tất cả các số hạng đều lớn hơn 1 thì u2n C 3 < 4u2n nên suy ra

1 4n2 q 2 1 1
unC1 < un C 2 4un < un C  2un D un :
2 4n C 1 2 4
306 Các phương pháp giải toán qua các kỳ thi Olympic

Do đó, dãy un giảm và bị chặn dưới bởi 1 nên có giới hạn.


Ngược lại, nếu tồn tại m sao cho um < 1 thì giả sử m đó là số nhỏ nhất thì từ đó trở đi, tất cả các
số hạng của .un / đều nhỏ hơn 1 vì

1 n2 q 2 1 1p
umC1 D um C 2 um C 3 < C 1 C 3 D 1:
2 4n C a 2 4

Ta sẽ chứng minh rằng tồn tại một chỉ số N nào đó mà từ đó trở đi, dãy số này tăng.
Vì um < 1 nên tồn tại N sao cho um < aN < 1. Do dãy an tăng nên ta có thể chọn N sao cho
2
N > m. Chú ý rằng hàm 4xx2 C1 đồng biến trên RC nên

um m2 q 2 aN N2
q
2
umC1 D C u m C 3 < C aN C 3 D aN
2 4m2 C 1 2 4N 2 C 1
tức là umC1 < aN . Từ đó quy nạp được các số hạng umC1 ; umC2 ; : : : ; uN 1 ; uN đều bé hơn aN
và cũng có uN C1 < aN :
Do đó, ta được uN C1 < aN < aN C1 : Tương tự, ta cũng có uN Ck < aN Ck với mọi k  0. Ngoài
ra, vì uN Ck < aN Ck nên fN Ck .uN Ck / > uN Ck , tức là uN CkC1 > uN Ck : Suy ra dãy .un / tăng
ngặt kể từ 1 và nó bị chặn trên bởi 1 nên có giới hạn. Chứng minh kết thúc.

Ta thấy rằng cách lập luận chia trường hợp trên cũng khá thú vị, nó cũng tương tự cách 1 của lời
giải đã được giới thiệu. Tuy nhiên, điểm mạnh của lời giải này là nó không phụ thuộc vào giá trị
a 2 Œ0I 1 nên có thể thấy rằng bài toán vẫn đúng với mọi a không âm. Ngoài ra, nó cho ta một
cách đánh giá nhẹ nhàng hơn khi khảo sát tính hội tụ của dãy nhờ ý tưởng: dù chưa biết tính biến
thiên của dãy cụ thể như thế nào, ta cứ thử xét mọi tình huống: nếu rơi vào trường hợp thuận lợi
thì tốt; nhưng nếu không thuận lợi thì bằng các lập luận thích hợp, ta vẫn chứng minh được; suy
ra nó luôn đúng trong mọi trường hợp.
Lập luận này đã từng được dùng để giải quyết bài toán khá thú vị sau:
Bài 4. (VMO bảng A, 2005) Xét dãy số thực .xn / xác định bởi công thức
(
x1 D a;
; n D 1; 2; 3; : : :
xnC1 D 3xn3 7xn2 C 5xn

Xác định tất cả các giá trị a sao cho dãy số trên có giới hạn hữu hạn và trong từng trường hợp,
hãy xác định giới hạn đó.

Bài toán 2. Cho các số thực a; b; c  0: Chứng minh rằng


p p p 
3.a2 Cb 2 Cc 2 /  .aCbCc/ abC bcC ca C.a b/2 C.b c/2 C.c a/2  .aCbCc/2 :

Lời giải. Vế trái của bất đẳng thức khá đơn giản. Dạng phát biểu của nó với tổng các bình
phương gợi cho ta nghĩ đến đồng nhất thức Lagrange – một hằng đẳng thức quen thuộc được
dùng để chứng minh bất đẳng thức Cauchy-Schwarz:
n
X n
X  n
X 2 X
ai2 bi2 ai bi D .ai bj aj bi /2  0:
i D1 i D1 i D1 1i <j n
Lời giải và bình luận đề thi VMO 2015 307

Cụ thể hơn, ta có đẳng thức sau:

3.a2 C b 2 C c 2 / .a C b C c/2 D .a b/2 C .b c/2 C .c a/2 :

Do đó, bất đẳng thức vế trái có thể được viết dưới dạng:
p p p 
.a C b C c/2  .a C b C c/ ab C bc C ca :
2 2 2
Đến đây thì có lẽ bạn nào cũng sẽ nghĩ
p đến việc pđẳng thức quen thuộc x Cy Cz 
psử dụng bất
xy C yz C zx (áp dụng cho x D a; y D b và z D c) để hoàn tất phép chứng minh.

Ở đây, ta sẽ dành sự quan tâm nhiều hơn cho bất đẳng thức vế phải. Nhận xét ban đầu cho thấy
đây là một bất đẳng thức tương đối chặt vì dấu bằng xảy ra tại hai trường hợp a D b D c và
a D b; c D 0 (cùng các hoán vị tương ứng). Do đó, ta cần phải rất cẩn trọng trong các đánh giá
của mình.

Ngoài ra, ta cũng thấy rằng chỗ khó của bài toán chính là ở các căn thức. Nếu ta có thể phá được
dấu căn đưa bất đẳng thức về dạng đơn giản hơn thì chắc chắn bài toán cũng sẽ trở nên sáng sủa
hơn. Đến đây, có hai ý tưởng chính như sau:

1. Đặt ẩn phụ để khử căn: Đây là một hướng đi khá tự nhiên vì các căn thức ở đây cũng đơn
giản,p
các biểu thức
p dưới dấup căn chỉ có dạng bậc một. Do đó, chỉ cần một lần đặt ẩn phụ
x D a; y D b; z D c là ta có thể khử được hết các căn thức và đưa về xét một bất
đẳng thức thuần nhất bậc 4 đối với x; y; z: Bậc của bất đẳng thức mới cũng không quá
cao nên đây là hướng đi hoàn toàn khả thi.

2. Sử dụng đánh giá để khử căn: Đây là ý tưởng thường thấy khi xử lý các bài toán có căn.
Vấn đề được đặt ra ở đây là ta phải lựa chọn đánh giá đủ chặt sao cho các điều kiện dấu
bằng phải được đảm bảo.

Các hướng tiếp cận được trình bày dưới đây hầu hết đều sử dụng hai ý tưởng trên làm tư tưởng
chủ đạo.
Cách 1. Khai triển trực tiếp.
p p p
Đây có lẽ là hướng đi tự nhiên nhất cho bài toán này. Ta chỉ việc đặt x D a; y D b; z D c
rồi nhân tung hết ra. Khi đó, bất đẳng thức cần chứng minh có thể được viết lại dưới dạng:
X X X X
x 4 C xyz xC xy.x 2 C y 2 /  4 x2y 2: (1)

Đến đây, nếu bạn nào có tìm hiểu sẽ nghĩ ngay đến bất đẳng thức Schur bậc 4:

x 2 .x y/.x z/ C y 2 .y z/.y x/ C z 2 .z x/.z y/  0:

Dạng khai triển của nó chính là:


X X X
x 4 C xyz x xy.x 2 C y 2 /: (2)

Sự tương đồng giữa hai bất đẳng thức (1) và (2) gợi cho ta nghĩ đến việc dùng (2) để đánh giá
cho (1). Ngoài ra, (2) cũng có dấu bằng tại x D y D z và x D y; z D 0 (cùng các hoán vị)
308 Các phương pháp giải toán qua các kỳ thi Olympic

tương ứng với trường hợp đẳng thức của (1). Do đó, đây sẽ là một đánh giá khá ổn và ta có thể
yên tâm về độ an toàn của nó. Thật vậy, sau khi đánh giá, ta chỉ cần xét bất đẳng thức:
X X X X
2 xy.x 2 C y 2 /  4 x2y 2 , xy.x 2 C y 2 /  2 x2y 2

và nó chỉ là một hệ quả trực tiếp của bất đẳng thức AM-GM:
X X X
xy.x 2 C y 2 /  .xy  2xy/ D 2 x2y 2:

Lời bình. Đặt ẩn phụ là một trong những kỹ năng cơ bản cần có trong bất đẳng thức. Nhiều bài
toán có hình thức cồng kềnh phức tạp, tuy nhiên sau những bước đặt ẩn phụ đơn giản, ta có thể
đưa bài toán trở về dạng mới mà ở đó nhiều ý tưởng (mà trong đó cũng có thể là gốc của bài
toán) sẽ được phơi bày ra.

Có nhiều kiểu đặt ẩn phụ, trong đó có ba kiểu sau rất thông dụng: Đặt ẩn phụ để làm đơn giản
hình thức bài toán, đặt ẩn phụ để thuần nhất hóa hoặc đối xứng hóa, và đặt ẩn phụ lượng giác
dựa vào dấu hiệu từ điều kiện giả thiết.

Cách 2. Phương pháp SOS.


Đây là hướng đi tự nhiên thứ hai sau phương pháp khai triển. Trước hết, ta cũng sẽ đặt ẩn phụ
x; y; z như hướng 1 ở trên để khử căn tiện cho việc quan sát. Ta đưa bài toán về chứng minh:
X
.x 2 C y 2 C z 2 /.xy C yz C zx/ C .x 2 y 2 /2  .x 2 C y 2 C z 2 /2 :

Trong bất đẳng thức trên, có hai số hạng cùng chứa nhân tử x 2 C y 2 C z 2 : Một cách tự nhiên, ta
nghĩ đến việc ghép hai số hạng đó với nhau. Lúc này, bất đẳng thức được viết lại thành:
X
.x 2 y 2 /2  .x 2 C y 2 C z 2 /.x 2 C y 2 C z 2 xy yz zx/:

.x 2 y 2 /2 bên vế trái và phân tích đã quá quen thuộc:


P
Sự xuất hiện của tổng bình phương
X X 1X
x2 xy D .x y/2
2
gợi cho ta nghĩ ngay đến việc dùng phương pháp phân tích bình phương SOS để xử lý bài toán.
Cụ thể, ta viết được bất đẳng thức cần chứng minh dưới dạng:

Sx .y z/2 C Sy .z x/2 C Sz .x y/2  0;

trong đó Sx D f .x; y; z/ D 2.y C z/2 .x 2 C y 2 C z 2 / D y 2 C z 2 C 4yz x 2 ; còn


Sy D f .y; z; x/; Sz D f .z; x; y/ được định nghĩa tương tự.

Đến đây, ta chỉ việc sử dụng các tiêu chuẩn của phương pháp là được. Giả sử x  y  z; khi đó
ta có
Sy D z 2 C x 2 C 4zx y 2  0; Sz D x 2 C y 2 C 4xy z 2  0

Sx C Sy D 2z 2 C 4zx C 4yz  0:
Lời giải và bình luận đề thi VMO 2015 309

Do x zy z  0 nên .z x/2  .y z/2 : Từ đó suy ra

Sx .y z/2 C Sy .z x/2 C Sz .x y/2  .Sx C Sy /.y z/2  0:

Lời bình. Một điều cần chú ý là khi sử dụng phương pháp SOS, các bạn cần phải chứng minh
lại các tiêu chuẩn của nó. Nhiều bạn cẩu thả chỉ ghi gọn là Sx C Sy  0; Sy C Sz  0 rồi suy ra
điều phải chứng minh. Như thế là chưa được.

Ngoài cách sử dụng các tiêu chuẩn SOS như trên, ta cũng có cách biến đổi mà không phải sử
dụng tiêu chuẩn nào dựa trên đồng nhất thức đơn giản:
X
.x y/2 .x z/.y z/ D 0: (3)

Có thể thấy điểm mấu chốt gây khó khăn trong việc xử lý tổng:
X
.x y/2 .x 2 C y 2 C 4xy z 2 /  0

chính là phần số âm ở mỗi số


Phạng, chẳng hạn như z 2 trong số hạng .x y/2 .x 2 Cy 2 C4xy z 2 /:
Nếu ta đem cộng với tổng .x y/2 .x z/.y z/ với một số lượng thích hợp vào sẽ làm tăng
số lượng z 2 lên ở số hạng này và rất có thể sẽ thu được một đại lượng không âm. Cụ thể, ta hy
vọng sẽ có số k sao cho:

x 2 C y 2 C 4xy z 2 C k.x z/.y z/  0


, .x C y/2 C .2 C k/xy kz.x C y/ C .k 1/z 2  0:

Quan sát một chút, cho thể thấy ngay nếu chọn k D 2 thì ta sẽ viết được biểu thức .x C y/2
kz.x C y/ C .k 1/z 2 dưới dạng bình phương. Từ đó, ta thu được một lời giải ngắn gọn thú vị
sau: Bất đẳng thức cần chứng minh tương đương với
X X
.x y/2 .x 2 C y 2 C 4xy z 2 / C 2 .x y/2 .x z/.y z/  0;

hay X
y/2 .x C y z/2 C 4xy  0:
 
.x
Đồng nhất thức (3) đã giúp chúng ta xử lý được bài toán theo một lối SOS rất thú vị để đưa đến
một bất đẳng thức hiển nhiên. Đây cũng là một kinh nghiệm của chúng tôi tích lũy được khi tìm
hiểu về phương pháp SOS. Tất nhiên, đồng nhất (3) chỉ hiệu quả ở các bất đẳng thức đối xứng
bậc 4: Với các bất đẳng thức bậc cao, chúng ta cần một đồng nhất thức tổng quát hơn để tăng
cường tính hiệu quả. Chúng ta có một kết quả thú vị sau (bạn đọc có thể tự chứng minh): Cho
f .x; y; z/ là một đa thức đối xứng với hai biến x; y: Khi đó, ta có thể phân tích:
X
.x y/2 .x z/.y z/  f .x; y; z/ D .x y/2 .y z/2 .z x/2  g.x; y; z/


trong đó g.x; y; z/ là một đa thức đối xứng với ba biến x; y; z:

Nhờ vào đồng nhất thức trên mà chúng tôi đã xử lý thành công rất nhiều bất đẳng thức bằng
phương pháp SOS rất đơn giản chứ không cần phải dùng tiêu chuẩn phức tạp nào.

Cách 3. Đánh giá khử căn.


310 Các phương pháp giải toán qua các kỳ thi Olympic

Một hướng đi khác thaypcho đặt p ẩn phụ


p là tìm cách đánh giá phá căn thức. Cụ thể, ta sẽ tìm các
đánh giá thích hợp cho ab; bc; ca với chiều  để phá dấu căn. Thường thì với các dạng
căn tích như thế này, cách phá căn thông dụng là sử dụng bất đẳng thức AM-GM. Tuy nhiên, ở
bài toán này, nó lại cho đánh giá với chiều ngược lại:
p aCb p bCc p cCa
ab  ; bc  ; ca 
2 2 2
không phải chiều ta cần. Có cách nào để điều chỉnh không nhỉ?

Một ý tưởng thú vị ở đây là sử dụng nghịch đảo. Như đã biết,p


với bất đẳng thức dương thì nghịch
đảo của nó sẽ đảo chiều. Do đó, ta có thể nghĩ đến việc viết ab thành pabab rồi đánh giá:
p 2ab
ab  : (4)
aCb
Như vậy là sẽ phá được căn thức với chiều ta muốn. Tuy nhiên, một điều cần lưu ý ở đây là số
0 không có nghịch đảo. Thế nên nếu một trong các căn thức có một số bằng 0 thì ta không thể
dùng cách này được. Do đó, cần phải xét trường hợp để loại trừ tình huống ngoài ý muốn này.

Nếu trong a; b; c có một số bằng 0; chẳng hạn c D 0; thì bất đẳng thức cần chứng minh sẽ trở
thành: p
.a C b/ ab C .a b/2 C a2 C b 2  .a C b/2 :
p
Công việc ở đây là khá đơn giản vì ta đã có .a C b/ ab  2ab:

Tiếp theo, ta xét trường hợp a; b; c > 0: Lúc này, ta đã có thể sử dụng được (4). Bài toán được
đưa về chứng minh
X X ab  X X 2
2
2 a C .a b/  a :
aCb
Sau khi thu gọn, nó có dạng:
X X ab  X X
2 a 4 ab a2 :
aCb
Vì a C b C c có
P thể tách ra các đại lượng a C b; b C c; c C a liên quan đến mẫu của các số
ab
hạng của tổng aCb
nên ta có thể xử lý rút gọn vế trái theo cách sau:
X ab.a C b C c/ X  c 
VT D2 D2 ab 1 C
aCb aCb
X X 1
D2 ab C 2abc :
aCb
Khi đó, bất đẳng thức có thể viết lại thành:
 1 1 1 
2abc C C  2.ab C bc C ca/ a2 b2 c2:
aCb bCc cCa
Đến đây thì ý tưởng tự nhiên là sử dụng bất đẳng thức Cauchy-Schwarz dạng cộng mẫu để làm
giảm số lượng các phân thức:
1 1 1 9
C C 
aCb bCc cCa 2.a C b C c/
Lời giải và bình luận đề thi VMO 2015 311

và đưa bài toán về xét một bất đẳng thức mới:

9abc
 2.ab C bc C ca/ a2 b2 c2:
aCbCc

Tuy nhiên, đây chính là bất đẳng thức Schur bậc ba.

Lời bình. Ở đây, chúng tôi muốn chú ý với các bạn về cách tách các tích ab; bc; ca; abc được
sử dụng trong lời giải trên.

Như ta đã biết, những bất đẳng thức mà trong các trường hợp dấu bằng của chúng có trường hợp
không tại tâm thì thường khó đánh giá hơn các bất đẳng thức bình thường. Nguyên nhân là ở các
bộ hoán vị. Một bất đẳng thức đối xứng (hoặc hoán vị) nếu có dấu bằng tại bộ .A; B; C / thì
cũng sẽ đạt được dấu bằng tại các hoán vị của nó là .B; C; A/ và .C; A; B/: Do đó, để đánh
giá thành công thì ta phải tìm được một đánh giá sao cho nó đảm bảo được cả ba trường hợp. Rõ
ràng rất khó!

Đối với các bài toán có dấu bằng tại biên thì cách tách trên cho ta một kỹ thuật xử lý đặc biệt
hiệu quả. Thật vậy, giả sử ta cần chứng minh bất đẳng thức f .a; b; c/  0 với dấu bằng là
a D kb .k ¤ 0/; c D 0 (và các hoán vị) chẳng hạn (ở đây chỉ xin lấy ví dụ một trường hợp cụ
thể để phân tích, còn nhiều trường hợp khác cũng có thể xử lý tương tự). Khi đó, nếu viết được
bất đẳng thức trên dưới dạng:

ab  g.a; b; c/ C bc  g.b; c; a/ C ca  g.c; a; b/  0

thì ta chỉ cần quan tâm đánh giá biểu thức đại diện g.a; b; c/ theo dấu bằng a D kb; c D 0 là
đủ mà không cần chú ý nhiều đến các hoán vị của bộ này. Nếu đánh giá thành công thì sau khi
nhân thêm ab vào hai vế, ta sẽ thu được một đánh giá cho số hạng ab  g.a; b; c/ với dấu bằng
xảy ra tại ab D 0 và a D kb; c D 0: Hiển nhiên đánh giá này sẽ đảm bảo được cả ba trường hợp
hoán vị của a D kb; c D 0:

Từ đây, ta thấy rằng các điều kiện sẽ càng thuận lợi hơn nếu ta tách ra được số hạng có dạng
abc  h.a; b; c/: Lúc này, h.a; b; c/ có thể được đánh giá khá vô tư, bởi lẽ tại trường hợp biên
thì tích abc đã bằng 0 mất rồi, thế nên khi nhân vào thì kiểu gì cũng đảm bảo được dấu bằng
biên.
Cách 4. Sử dụng hàm lồi.
Chắc hẳn bạn đọc yêu Toán đều biết đến tính chất thú vị sau của hàm lồi: Nếu hàm số f .x/ liên
tục và lồi trên đoạn Œa; b thì giá trị lớn nhất của nó sẽ đạt được một trong hai điểm x D a hoặc
x D b; còn đối với hàm lõm thì sẽ là giá trị nhỏ nhất.

Thế nhưng, lại không có nhiều bạn nghĩ đến việc sử dụng tính chất này vào giải toán. Một trong
những nguyên nhân có lẽ là ở tính chất của hàm lồi. Như ở trên đã đề cập, hàm lồi sẽ đạt cực đại
tại biên và hàm lõm sẽ đạt cực tiểu tại biên. Nhưng ở bài toán này thì lại không có biên rõ ràng,
các biến có biên dưới nhưng lại không có biên trên.

Một điều nữa cũng cần phải nói đến là hầu hết các bạn học sinh đều chỉ có tầm nhìn vĩ mô mà
chưa có đến cái nhìn vi mô. Có nghĩa là đề bài cho bất đẳng thức bao nhiêu biến thì các bạn chỉ
nhìn bằng đúng bấy nhiêu biến chứ không nghĩ đến tầm nhìn khác đi. Đó là một tầm nhìn sai
312 Các phương pháp giải toán qua các kỳ thi Olympic

lầm. Trên thực tế, để có được lời giải thành công thì ta nên bắt đầu bằng những thứ nhỏ nhặt
nhất, chú ý đến mọi khía cạnh.

Bất đẳng thức đã cho có dạng đối xứng với ba biến a; b; c; thế thì nó cũng là bất đẳng thức đối
xứng với hai biến bất kỳ nào đó trong ba biến trên. Chắn hẳn các bạn vẫn con nhớ chúng ta đã
học từ cấp 2 rất nhiều rằng các bài toán đối xứng hai ẩn có thể được xử lý hiệu quả bằng phép đặt
ẩn phụ tổng-tích S D x C y; P D xy nhờ vào quan hệ của chúng: S 2  4P:

Cụ thể hơn, nếu ta cố định c thì bất đẳng thức sẽ có dạng đối xứng với a và b: Khi đó, nhờ phép
đặt S D a C b; P D ab; ta có thể chuyển bất đẳng thức về dạng:

g.S; P /  0:

Khi đó, nếu ta cố định S nữa thì đây sẽ chỉ còn là một bất đẳng thức với một biến là P và lúc này
2
P đã được chặn miền với đủ biên trên lẫn biên dưới là 0; S4 :


Đến đây, nếu ta có thể suy xét được tính đơn điệu hoặc tính lồi lõm của gP .S; P / thì cũng có
thể đưa ra được kết luận về tính chất các cực trị của nó để rồi từ đó đi đến lời giải.

Lời giải chi tiết theo hướng này như sau: Đặt f .a; b; c/ D V T VP: Khi đó, ta phải chứng
2
minh f .a; b; c/  0: Cố định c và S D a C b: Đặt P D ab thì ta có 0  P  S4 : Ta có biến
đổi:
p p p p
q q
a C b D a C b C 2 ab D S C 2 P
và .a b/2 C .b c/2 C .c a/2 .a C b C c/2 D h.P /; trong đó h.P / là một biểu thức bậc
nhất của P: Do đó:
p p
 q 
f .a; b; c/ D .S C c/ PC S C2 P C h.P / D g.P /:

Nếu cả hai số a; b đều bằng 0 thì bất đẳng thức hiển nhiên đúng nên ta chỉ cần xét trường hợp
2
S > 0 là đủ (lý luận này là để đảm bảo khoảng 0; S4 tồn tại, đảm bảo cho việc xét đạo hàm
2 2
của g.P /). Dễ thấy hàm số g.P / liên tục trên 0; S4 : Ngoài ra, với mỗi P 2 0; S4 ; ta tính


được:
p1
!
1 P
g 0 .P / D .S C c/ p C p p Ck
2 P 2 S C2 P
2
trong đó k là hệ số cao nhất của h.P /: Rõ ràng g 0 .P / là hàm giảm ngặt trên 0; S4 nên g.P /
2
là hàm lõm trên 0; S4 : Từ đó suy ra


(  2 )
S2
 
S
g.P /  min g.0/; g ; 8P 2 0; :
4 4

S2

Như thế, để chứng minh bất đẳng thức đã cho, ta chỉ cần chứng minh g.0/  0 và g 4
 0:
Mặt khác, ta lại dễ thấy:
 2  
S S S
g.0/ D f .S; 0; c/; g Df ; ;c :
2 2 2
Lời giải và bình luận đề thi VMO 2015 313

Do đó, từ những lý luận ở trên, có thể thấy rằng ta chỉ cần xét bất đẳng thức tại hai trường hợp: có
một số bằng 0 hoặc có hai số bằng nhau, là đủ. Ở hướng 3, ta đã chứng minh được bất đẳng thức
đúng tại trường hợp thứ nhất. Như vậy, ta chỉ còn phải kiểm tra trường hợp thứ hai nữa là được.

Giả sử a D b: Khi đó, bất đẳng thức trở thành


p 
.2a C c/ a C 2 ac C 2.a c/2  .2a C c/2 :
Sau khi khai triển và rút gọn, ta phải chứng minh
p p
4a ac C 2c ac C c 2  7ac:
Vế lớn có dạng tổng, còn vế bé có dạng tích gợi cho ta nghĩ ngay đến bất đẳng thức AM-GM để
đánh giá:
p p p 4 p 2
q
2 7
4a ac C 2c ac C c  7 a ac c ac c 2 D 7ac:
Lời bình. Bằng cách sử dụng hàm lồi và bất đẳng thức Karamata1 , ta sẽ có thêm cách nhìn nhận
tổng quan hơn cho nhiều vấn đề, nắm bắt được bản chất tốt hơn.
Nhận xét. Qua tìm hiểu và nghiên cứu tài liệu, chúng tôi suy đoán rằng bài toán này có xuất xứ
từ bài toán sau đây: Cho a; b; c là các số thực không âm thỏa mãn điều kiện a C b C c D 1:
Chứng minh rằng
p p p p
a C .b c/2 C b C .c a/2 C c C .a b/2  3: (8)
Tất nhiên, đây chỉ là những suy đoán có tính chủ quan, nhưng nếu suy xét kỹ, các bạn sẽ thấy bài
VMO 2015 chính là một mấu chốt quan trọng trong chứng minh bất đẳng thức (8). Thật vậy,
bình phương hai vế của (8), ta thấy nó tương đương với
X q  X
.a b/2  2:

2 a C .b c/2 b C .a c/2 C

Đến đây, sử dụng bất đẳng thức Cauchy-Schwarz, ta có


X q   Xp 
a C .b c/2 b C .a c/2  ab C .a c/.b c/
Xp X
D ab C .a c/.b c/
Xp 1X
D ab C .a b/2 :
2
1
Bất đẳng thức Karamata được xây dựng dựa trên khái niệm bộ trội và tính chất tiếp tuyến của hàm lồi:
Cho hai bộ số không tăng A D .a1 ; a2 ; : : : ; an / và B D .b1 ; b2 ; : : : ; bn /; trong đó ai ; bi đều cùng thuộc vào
một miền I: Ta nói rằng A trội hơn B; ký hiệu A  B; nếu các điều kiện sau được thỏa mãn đồng thời:
8
ˆ
ˆ a1  b1
ˆ
< a1 C a2  b1 C b2
ˆ
ˆ
ˆ
:::::::::
ˆ
a 1 C a2 C    C an  b1 C b2 C    C bn
ˆ
1 1
ˆ
ˆ
ˆ
ˆ
a1 C a2 C    C an C an D b1 C b2 C    C bn C bn
:
1 1

Lúc này, nếu f .x/ là một hàm khả vi bậc hai và lồi trên I thì:
f .a1 / C f .a2 / C    C f .an /  f .b1 / C f .b2 / C    C f .bn /:
314 Các phương pháp giải toán qua các kỳ thi Olympic

Do đó, để chứng minh (8), ta chỉ cần chứng minh được


Xp X
ab C .a b/2  1;

hay
X X p  X X 2
2
a ab C .a b/  a :

Đây chính là bất đẳng thức vế phải trong bài số 2 của đề VMO năm nay. Còn vế trái có lẽ tác giả
đã đặt thêm ra với mục đích gỡ điểm cho các thí sinh tham dự kỳ thi.

Nói riêng về bất đẳng thức (8), nó cũng có một xuất xứ rất thú vị... từ hình học. Chính xác hơn là
từ sự tương tự hóa một bất đẳng thức về đường trung tuyến của tam giác. Ta biết rằng, trong một
tam giác với độ dài ba cạnh là a; b; c thì:

ma C mb C mc  2p:

Xét các tam giác có nửa chu vi p D 1: Đặt a D y C z; b D z C x và c D x C y với x; y; z > 0:


Khi đó, ta có x C y C z D 1 và:

2b 2 C 2c 2 a2
2.z C x/2 C 2.x C y/2 .y C z/2
m2a D D
4 4
2
.y z/ .y z/2
D x.x C y C z/ C DxC :
4 4
Từ phép biến đổi này, ta thu được bất đẳng thức
r r r
.y z/2 .z x/2 .x y/2
xC C yC C zC  2:
4 4 4
Có thể thấy bất đẳng thức (8) chính là một sự tương tự hóa bằng cách thay đổi hệ số của các bình
phương dưới dấu căn.
Như đã đề cập, ở phần cuối này, chúng tôi xin được đề xuất một tổng quát cho bất đẳng thức vế
phải trong đề VMO năm nay như sau (trường hợp bài VMO chính là ứng với n D 3):
Cho số tự nhiên n  2: Xét n số thực không âm x1 ; x2 ; : : : ; xn : Chứng minh rằng
n
X  X  n
2 p X X
xi xi xj C .n 2/ xi2  4 xi xj :
n 1 i D1 1i <j n i D1 1i <j n

Bài toán 3. Cho số nguyên dương k. Tìm số các số tự nhiên n không vượt quá 10k thỏa mãn
đồng thời các điều kiện sau:

i) n chia hết cho 3:

ii) các chữ số trong biểu diễn thập phân của n thuộc tập hợp f2; 0; 1; 5g :

Lời giải. Vì 10k không chia hết cho 3 nên ta chỉ cần xét các số từ 0 cho đến 999
„ ƒ‚: : : …9, tức là các
k chữ số
số có không quá k chữ số.
Lời giải và bình luận đề thi VMO 2015 315

Đặt S D f2; 0; 1; 5g, bổ sung các chữ số 0 vào trước nếu cần thiết, ta đưa về xét các số có dạng
a1 a2 : : : ak với ai 2 S. Ta cần đếm các số như vậy và chia hết cho 3: Chú ý là a1 a2 : : : ak chia
hết cho 3 khi và chỉ khi a1 C a2 C a3 C : : : C ak chia hết cho 3; ta đưa bài toán về việc đếm số
các bộ .a1 ; a2 ; a3 ; : : : ; ak / 2 S k sao cho tổng chia hết cho 3:
Đến đây ta có các hướng giải quyết như sau.
Cách 1. Dùng công thức truy hồi.
Với i D 0; 1; 2, ta đặt

A.n; i/ D f.a1 ; a2 ; a3 ; : : : ; an / 2 S n ja1 C a2 C a3 C : : : C an  i.mod3/g

và đặt
an D A.n; 0/; bn D A.n; 1/; cn D A.n; 2/:
Dễ dàng thấy rằng
a1 D 1; b1 D 1; c1 D 2:
Xét phần tử .a1 ; a2 ; : : : ; an ; anC1 / của A.n C 1; 0/:
 Nếu anC1 D 0 thì .a1 ; a2 ; : : : ; an / 2 A.n; 0/:

 Nếu anC1 D 2 hoặc 5 thì .a1 ; a2 ; : : : ; an / 2 A.n; 1/:

 Nếu anC1 D 1 thì .a1 ; a2 ; : : : ; an / 2 A.n; 2/:


Từ đây ta suy ra anC1 D an C 2bn C cn .1/. Hoàn toàn tương tự, ta cũng có

bnC1 D an C bn C 2cn .2/


cnC1 D 2an C bn C cn .3/
Từ đây ta tính được a2 D 5; b2 D 6; c2 D 5; a3 D 22; b3 D 21; c3 D 21. Cộng các đẳng thức
.1/; .2/; .3/ lại, vế theo vế, ta có anC1 C bnC1 C cnC1 D 4.an C bn C cn /. Từ đây suy ra

an C bn C cn D 4n

(Chú ý rằng điều này cũng có thể suy ra dễ dàng bằng quy tắc đếm).
Trừ .1/ cho .2/, trừ .2/ cho .3/, trừ .3/ cho .1/, ta được

anC1 bnC1 D bn cn ; bnC1 cnC1 D cn an ; cnC1 anC1 D an bn :

Do đó, anC3 bnC3 D bnC2 cnC2 D cnC1 anC1 D an bn . Tương tự thì

bnC3 cnC3 D bn cn ; cnC3 anC3 D cn an với mọi n:

Sử dụng các giá trị ban đầu ai ; bi ; ci với i D 1; 2; 3 và tính chất ở trên, ta suy ra rằng
 Nếu k chia hết cho 3 thì bk D ck D ak 1.

 Nếu k chia 3 dư 1 thì ak D bk D ck 1.

 Nếu k chia 3 dư 2 thì ak D ck D bk 1.


316 Các phương pháp giải toán qua các kỳ thi Olympic

Từ đây, kết hợp với đẳng thức ak C bk C ck D 4k , ta suy ra kết quả

4k 1
 aK D 3
nếu k không chia hết cho 3;
4k C2
 aK D 3
nếu k chia hết cho 3:

Cách 2. Dùng đa thức và số phức


Xét đa thức
k
P .x/ D .x 2 C 1 C x C x 5 /
Ta có
k
X
P .x/ D .x 2 C 1 C x C x 5 / D x a1 Ca2 C:::Cak
.a1 ;a2 ;:::;aK /2S k

Ta thấy tổng các hệ số của P .x/ bằng số các bộ .a1 ; a2 ; : : : ; ak / 2 S k và bằng 4k . Hơn nữa số
các bộ .a1 ; a2 ; : : : ; ak / 2 S k sao cho a1 C a2 C : : : C ak bằng tổng các hệ số của các số mũ
chia hết cho 3 trong khai triển của P .x/.
Đặt P .x/ D a0 C a1 x C a2 x 2 C a3 x 3 C : : : C a5k x 5k . Ta cần tính T D a3m là tổng các hệ số
P
chia hết cho 3 trong khai triển. Gọi " là nghiệm của phương trình x 2 Cx C1 D 0 thì ta có "3 D 1.
Từ đó dễ dàng suy ra 1 C "k C "2k D 0 với mọi k không chia hết cho 3 và 1 C "k C "2k D 0 với
k chia hết cho 3: ./
Ta có 8
< P .1/ D a0 C a1 C a2 C a3 C : : : C a5k ;
ˆ
P ."/ D a0 C a1 " C a2 "2 C a3 "3 C : : : C a5k "5k ;
ˆ
P ."2 / D a0 C a1 "2 C a2 "4 C a3 "6 C : : : C a5k "10k
:

Áp dụng tính chất ./; ta suy ra P .1/ C P ."/ C P ."2 / D 3T . Suy ra

P .1/ C P ."/ C P ."2 / 4k C "2k C "4k


T D D
3 3
4k 1 4k C2
Cuối cùng, lại áp dụng tính chất (*) ta suy ra T D 3
nếu k không chia hết cho 3 và T D 3
nếu k chia hết cho 3:

Nhận xét. Đây là một bài toán khá quen thuộc. Dạng này đã gặp ở một số đề thi của Rumani
(2007), PTNK (2009) hay Lâm Đồng năm vừa rồi. Chẳng hạn đề thi PTNK 2009:
Cho số nguyên dương n. Có bao nhiêu số chia hết cho 3; có n chữ số và các chữ số đều thuộc
f3; 4; 5; 6g?

Cái mới và cũng là cái khó của bài toán này là ở chữ số 0 và xét tất cả các số nhỏ hơn 10k (chứ
không phải là số có k chữ số). Tuy nhiên, như đã thấy trong lời giải trên, kết hợp hai cái khó và
mới đó lại ta được một cái cũ (và dễ!). Theo phương pháp công thức truy hồi, có thể sẽ có một số
bạn gặp khó khi giải hệ phương trình truy hồi (nếu có 2 dãy thì dễ hơn, 3 dãy khá rối).

Nếu không xử lý được cái mới bằng cách lý luận như trên và phải đếm số các số có n chữ số lập
từ f2; 0; 1; 5g và chia hết cho 3 thì ta sẽ đưa đến cách giải sau:
Lời giải và bình luận đề thi VMO 2015 317

Gọi An ; Bn ; Cn là số các số có n chữ số lập từ f2; 0; 1; 5g và chia 3 dư 0; 1; 2 tương ứng. Khi đó


ta có A1 D 1, (số 0 ta coi là số có 1 chữ số), B1 D 1; C1 D 2 và

AnC1 D 2.Bn C Bn 1 C : : : C B1 / C .Cn C Cn 1 C : : : C C1 / .1/;


BnC1 D .An C An 1 C : : : C A1 / C 2.Cn C Cn 1 C : : : C C1 / .2/;
CnC1 D 2.An C An 1 C : : : C A1 / C .Bn C Bn 1 C : : : C B1 / .3/

(học sinh rất dễ bị nhầm chỗ này)


Suy ra A2 D 4; B2 D 5; C2 D 3.
Trong .1/ thay n bằng n C 1, ta được

AnC2 D 2.BnC1 C Bn C : : : C B1 / C .CnC1 C Cn C : : : C C1 / .4/:

Lấy .4/ trừ .1/ ta được


AnC2 AnC1 D 2BnC1 C CnC1 ;
tức là
AnC2 D AnC1 C 2BnC1 C CnC1 :
Tương tự,

BnC2 D AnC1 C BnC1 C 2CnC1 ; CnC2 D 2AnC1 C BnC1 C CnC1 :

Do vậy, ta có A2 D 4; B2 D 5; C2 D 3 và

AnC1 D An C 2Bn C Cn ; BnC1 D An C Bn C 2Cn ; CnC1 D 2An C Bn C Cn

với mọi n  2.
Tiếp theo ta giải gần giống như lời giải 1. Chú ý đáp số của bài toán là A1 C A2 C : : : C Ak và
theo .1/; .2/; .3/ thì tổng này bằng
4CKC1 C BKC1 2AKC1
:
9
Trong tình huống đề yêu cầu chia hết cho số khác, không phải 3 thì lời giải sẽ thú vị hơn nhiều
và trong tình huống đó, vai trò của hàm sinh thể hiện rõ hơn nhiều. Ta xem xét thử một số bài
toán tương tự sau:

Bài 1. (Đề tập huấn đội tuyển IMO 2014) Có bao nhiêu số tự nhiên có 9 chữ số, trong đó
không chứa chữ số 0 và chia hết cho 11?
Bài 2. (VMO 2008) Có bao nhiêu số tự nhiên có 2008 chữ số, chia hết cho 9 và trong biểu diễn
của nó có không quá 2 chữ số 9?
Bài 3. Có bao nhiêu số tự nhiên chia hết cho 7; có n chữ số và mỗi chữ số được lấy từ tập hợp
f1; 3; 4; 6; 7; 9g?
Bài 4. Có bao nhiêu số tự nhiên bé hơn 5000 có tổng các chữ số chia hết cho 4?
Bài toán 4. Cho đường tròn .O/ và hai điểm B; C cố định trên .O/, BC không là đường kính.
Một điểm A thay đổi trên .O/ sao cho tam giác ABC nhọn. Gọi E; F lần lượt là chân đường
cao kẻ từ B; C của tam giác ABC . Cho .I / là đường tròn thay đổi đi qua E; F và có tâm là I:
318 Các phương pháp giải toán qua các kỳ thi Olympic
q
DB cot B
a) Giả sử .I / tiếp xúc với BC tại điểm D: Chứng minh rằng DC
D cot C
.

b) Giả sử .I / cắt cạnh BC tại hai điểm M; N . Gọi H là trực tâm tam giác ABC và P; Q
là các giao điểm của .I / với đường tròn ngoại tiếp tam giác HBC . Đường tròn .K/ đi
qua P; Q và tiếp xúc với .O/ tại điểm T (T cùng phía A đối với PQ). Chứng minh rằng
đường phân giác trong của góc ∠M T N luôn đi qua một điểm cố định.
Lời giải.
a) Giả sử điểm D nằm trong cạnh BC , trường hợp điểm D nằm ngoài chứng minh tương tự. Ta
có hai cách xử lý như sau:
Cách 1. Gọi R; S lần lượt là giao điểm của .I / với BC (các giao điểm này có thể tương ứng
trùng E; F trong trường hợp tam giác ABC cân). Ta có
AR AE AB
AR  AF D AS  AE ) D D ) RS k BC:
AS AF AC

Do .I / tiếp xúc với BC tại D nên


BD 2 BF  BR BF BR BF AB BF BE cot B
2
D D  D  D  D :
CD CE  CS CE CS CE AC CE CF cot C
q
DB cot B
Vậy ta có DC D cot C
.
Cách 2. Gọi X; Y lần lượt là giao điểm của .I / với BE; CF . Ta có BD 2 D BX  BE; C D 2 D
C Y  CF (phương tích với đường tròn .I /).
Hai tam giác BXF; C YE có ∠XBF D ∠Y CE; ∠BXF D ∠C YE nên đồng dạng. Suy ra
BX BF cos B
D D :
CY CE cos C
Do đó,
BD 2 BX BE cos B sin C cot B
2
D  D  D
CD C Y CF cos C sin B cot C
Lời giải và bình luận đề thi VMO 2015 319

hay r
BD cot B
D :
CD cot C
b) Trường hợp tam giác ABC cân tại A, bài toán hiển nhiên đúng.
Xét trường hợp tam giác không cân ở A, không mất tính tổng quát, giả sử AB < AC . Gọi G là
giao điểm của EF và đường thẳng BC .
Xét các đường tròn .BH C /; .I / và đường tròn đường kính BC . Ta thấy:

 Trục đẳng phương của .BH C /; .I / là PQ.

 Trục đẳng phương của .I / và đường tròn đường kính BC là EF .

 Trục đẳng phương của .BH C / và đường tròn đường kính BC là BC .

Do đó, PQ; EF; BC đồng quy tại tâm đẳng phương của 3 đường tròn này. Ta có GT 2 D
GP  GQ D GM  GN nên đường tròn .TMN / cũng tiếp xúc với .O/ tại T .
Do đó, ta có ∠GTM D ∠GN T (cùng chắn cung TM của đường tròn .K/).
Theo tính chất góc ngoài của tam giác thì ∠GN T D ∠N T C C∠NC T . Hơn nữa, do GT tiếp xúc
với .O/ nên ∠GTB D ∠GC T . Trừ tương ứng từng vế 2 đẳng thức, ta được ∠BTM D ∠C T N .

Từ đây dễ thấy phân giác của góc ∠M T N và ∠BT C là trùng nhau hay phân giác của ∠M T N
đi qua trung điểm J của cung BC không chứa A, đây là điểm cố định.
Ta có đpcm.

Nhận xét. Về câu a của bài toán, có thể nói đây là một phần tương đối hay, nó không quá đơn
giản nhưng cũng đủ dễ để học sinh có thể tự tin và thoải mái để bắt tay vào thử sức. Việc sử dụng
tính chất cơ bản của phương tích của đường tròn sẽ dẫn đến sự xuất hiện một cách tự nhiên của
các điềm R; S; X; Y như trên.
320 Các phương pháp giải toán qua các kỳ thi Olympic

Ngoài ra, với học sinh nào không quen với việc kẻ đường phụ, ta còn có thể lập luận bằng tỉ số
lượng giác trực tiếp, không cần vẽ thêm bất cứ gì như sau:
Ký hiệu E là góc ∠DEF D ∠BDF , bằng biến đổi góc, ta có

∠B D ∠AEF D 180ı .∠EC∠CED/ D 180ı .∠EC180ı .∠C CF // D ∠C C∠F ∠E:

Suy ra ∠B C ∠E D ∠C C ∠F: Do đó ∠BFD D ∠CED; ∠CFD D ∠BED:


Xét tỉ lệ diện tích các tam giác

BD SFBD FB  FD  sin BFD sin BFD


D D D cot B 
CD SF CD F C  FD  sin CFD sin CFD

BD SEBD EB  ED  sin BED 1 sin BED
D D D  :
CD SE CD EC  ED  sin CED cot C sin CED
Suy ra
 2 r
BD cot B BD cot B
D ) D :
CD cot C CD cot C
Lời giải này đã phát hiện ra một số cặp góc bằng nhau nữa và biến đổi cũng khá nhẹ nhàng.
Khai thác thêm mô hình này, ta cũng phát hiện ra thêm khá nhiều điều thú vị:

1. Bôn điểm R; S; X; Y là các đỉnh của một hình chữ nhật.

2. Đường thẳng nối giao điểm của DE; CF và DF; CE song song với BC .

Ta cũng thấy rằng, ngay từ đầu, học sinh cũng sẽ thắc mắc về cách dựng của đường tròn .I /, tức
là đường tròn đi qua 2 điểm cho trước và tiếp xúc với 1 đường thẳng. Nhiều bạn vẽ hình bằng
cảm tính, dựng đường tròn .I / trước và chính điều này đã khiến họ không đi sâu vào bản chất
của bài toán và trên cơ sở đó, giải quyết được câu b.
Lời giải và bình luận đề thi VMO 2015 321

Đó chính là giao điểm G của EF; BC trong lời giải của bài toán đã nêu. Ta thấy rằng GE  GF D
GD 2 hay GD chính là trung bình nhân của hai đoạn GE; GF , hoàn toàn dựng được. Chú ý rằng,
có 2 điểm D như vậy, một điểm nằm phía trong đoạn BC và một nằm phía ngoài đoạn BC , hai
điểm này cùng với B; C lập thành một hàng điểm điều hòa.
Đến đây, ta phát hiện ra rằng cần phải có câu nhận xét như đầu tiên để hạn chế việc phụ thuộc
hình vẽ, do đề không nêu rõ là tiếp xúc với cạnh hay với đường thẳng BC .
cot B LB
Ta cũng để ý rằng nếu gọi L là chân đường cao góc A của tam giác ABC thì cot C
D LC
nên bài
LB DB 2
a tương đương với LC D DC 2.

Một cách tự nhiên, ta đặt ra câu hỏi tổng quát: Điều kiện nào đối với bộ điểm A0 ; B 0 ; C 0 nằm
trên các cạnh BC; CA; AB của tam giác ABC theo thứ tự, AA0 ; BB 0 ; C C 0 đồng quy và đường
tròn đi qua B 0 ; C 0 tiếp xúc với BC tại A1 thì
A0 B A1 B 2
D ‹
A0 C A1 C 2
Ngoài ra, nếu xét các đường tròn tương tự theo góc độ B; C thì cả ba đường tròn vẽ được sẽ có
đặc điểm gì? Đây là các ý khai thác rất thú vị cho bài toán này.

Về câu b của bài toán, cũng tương tự như những bài toán về yếu tố cố định trong các kỳ thi VMO,
TST gần đây, bài 4 lần này đòi hỏi học sinh khả năng phán đoán, suy luận tốt và nhất là khả năng
"đơn giản hoá" bài toán, tìm ra yếu tố quyết định trong một loạt giả thiết có vẻ phức tạp và "ít tác
dụng".

Bài toán này có lẽ được phát triển từ bổ đề sau: Cho tam giác ABC và M; N là các điểm thuộc
BC , một điểm T nằm trên đường tròn .O/. Khi đó, nếu đường tròn .TMN / tiếp xúc với .O/
tại T thì ∠BT C; ∠M T N có chung đường phân giác trong.
Bổ đề này có thể chứng minh bằng cộng góc như lời giải trên, hoặc sử dụng phép vị tự tâm T
biến M; N thành X; Y thuộc .O/ thì XY k BC và ta có kết quả trên.
Do vậy, với những học sinh đã nhìn ra tính chất GT tiếp xúc với cả .TMN / và .O/ thì chỉ cần
mạnh dạn bỏ qua những "yếu tố thừa" trong đề là sẽ có ngay lời giải.
Ta có thể dự đoán điểm cố định thông qua việc xét hai điểm A đối xứng qua trung trực BC; từ
đó suy ra điểm cố định cách đều B và C; sẽ dễ đưa đến kết luận hơn.
Có khá nhiều đường tròn góp mặt trong bài toán, nhưng với những học sinh có kinh nghiệm thì
việc sử dụng tính chất trục đẳng phương để có các đường đồng quy là điều dễ hiểu. Dù đề bài
được phát biểu tương đối dài và phức tạp, nhưng không phải là khó để xử lý những chuyện này.
Trên thực tế, ta có thể giải mà không cần sự có mặt của E; F . Vẫn với xuất phát từ trục đẳng
phương, xét 3 đường tròn .BH C /; .O/; .TPQ/ thì sẽ có tiếp tuyến tại T đi qua giao điểm của
BC; PQ.
Sự kiện EF cũng đi qua G có thể sẽ dẫn đến nhiều tính chất mới hoặc nếu kết hợp với các tính
chất cũ cũng sẽ đưa ra nhiều bài toán khác để khai thác. Dưới đây là một bài toán mở rộng do
bạn Nguyễn Văn Linh đề xuất:
Cho tam giác ABC nội tiếp đường tròn .O/. Một đường tròn bất kỳ đi qua B; C và một đường
tròn .J / khác thay đổi cắt BC tại M; N và cắt .I / tại P; Q. Đường tròn đi qua P; Q và tiếp xúc
với .O/ tại T nằm cùng phía với A so với BC . Khi đó, phân giác góc M T N luôn đi qua trung
điểm cung BC không chứa A.
322 Các phương pháp giải toán qua các kỳ thi Olympic

Bài toán 5. Cho dãy đa thức fn .x/ được xác định bởi
f0 .x/ D 2; f1 .x/ D 3x; fn .x/ D 3xfn 1 .x/ C .1 x 2x 2 /fn 2 .x/; 8n  2:
Tìm tất cả các giá trị n để đa thức fn .x/ chia hết cho đa thức x 3 x 2 C x.
Lời giải. Từ công thức truy hồi đã cho, ta biến đổi như sau
fn .x/ D 3xfn 1 .x/ C 1 x x 2 fn 1 .x/


, fn .x/ .x C 1/ fn 1 .x/ D .2x 1/ fn 1 .x/ .2x 1/ .x C 1/ fn 2 .x/


, fn .x/ .x C 1/ fn 1 .x/ D .2x 1/ Œfn 1 .x/ .x C 1/ fn 2 .x/
Suy ra
fn .x/ .x C 1/ fn 1 .x/ D .2x 1/n 1 Œf1 .x/ .x C 1/ f0 .x/ D .2x 1/n 1
.x 2/
1/n D .x C 1/ fn 1 .x/ .2x 1/n 1
 
, fn .x/ .2x
Từ đây ta có
fn .x/ .2x 1/n D .x C 1/n Œf0 .x/ .2x 1/0  D .x C 1/n
hay
fn .x/ D .2x 1/n C .x C 1/n :
Đặt Q.x/ D x 3 x 2 C x D x.x 2 x C 1/:
Vì fn .x/ chia hết cho đa thức g.x/ D x 3 x 2 C x nên fn .0/ D 0 hay 1 C . 1/n D 0 nên n lẻ.
Và fn . 2/ D .5n C 1/ (do n lẻ) chia hết cho . 2/2 . 2/ C 1 D 7. Do 125  1.mod7/
nên ta xét các trường hợp sau
 n D 3k, k lẻ, ta có 5n C 1 D 53k C 1  . 1/k C 1 D 0 .mod7/
 n D 3k C 1, k chẵn, ta có 5n C 1 D 5  53k C 1  6 .mod7/
 n D 3k C 2, k lẻ, ta có 5n C 1 D 25  53k C 1  24  3 .mod7/.
Từ đó, ta suy ra điều kiện cần của n là n D 3k với k lẻ.
:
Khi đó fn .x/ D .x C 1/3k C .2x 1/3k ::.x C 1/3 C .2x 1/3 với mọi k.
Nhận thấy .x C 1/3 C .2x 1/3 D 9x 3 9x 2 C 9x chia hết cho đa thức g.x/ nên ta có n D 3k
với k là số tự nhiên lẻ thỏa mãn. Đặt k D 2m C 1 với m nguyên thì n D 6m C 3.
Vậy tất cả các số n cần tìm có dạng 6m C 3 với m là số nguyên dương.
Nhận xét. Ta có thể tìm ra được công thức tổng quát của dãy đa thức đã cho bằng cách coi x là
hằng số và xây dựng số hạng của dãy số tương ứng, tức là xét dãy số
(
u0 D 2; u1 D 3x;
un D 3xun 1 C 1 x 2x 2 un 2


với x là tham số thực nào đó.


Xét phương trình đặc trưng t 2 3xt C 2x 2 C x 1 D 0 , t D x C 1; t D 2x 1.
n n
Suy ra un D ˛.x C 1/ C ˇ.2x 1/ . Dựa vào u0 ; u1 ta tìm được ˛ D ˇ D 1. Do đó,
fn .x/ D .x C 1/n C .2x 1/n .
Một cách khác để làm là dự đoán công thức thông qua việc tính toán trước vài giá trị:
Lời giải và bình luận đề thi VMO 2015 323

 f0 D 2I
 f1 D 3xI
 f2 D 5x 2 2x C 2I
 f3 D 9x 3 9x 2 C 9xI
 f4 D 17x 4 37x 3 C 30x 2 4x C 2:
Ta thấy hệ số bậc cao nhất của các đa thức có dạng 2n C 1 và hệ số cuối là 0 và 2 luân phiên nên
dự đoán đa thức cần tìm có dạng .2x ˙ 1/n C .x ˙ 1/n .
Kiểm tra trực tiếp, ta thấy đa thức cần tìm là .2x 1/n C .x C 1/n và đến đây chỉ cần quy nạp
là xong. Ngoài ra, ta có thể giải quyết ý còn lại là tìm n để có sự chia hết bằng cách dùng số phức
như sau:
Do fn .x/ chia hết cho x 3 x 2 C x nên fn .0/ D fn ."/ D fn "2 D 0 với " là số phức thỏa


"2 " C 1 D 0.
Ta cũng có "3 D 1 và " … R. Ta có fn .0/ D 0 nên n là số lẻ. Đặt n D 2k C 1 thì

fn ."/ D .2" 1/2kC1 C ." C 1/2kC1


k k
D .2" 1/ 4"2 4" C 1 C ." C 1/ "2 C 2" C 1
 
D .2" 1/ . 3/k C ." C 1/ .3"/k D . 3/k .2" 1/ C ." C 1/. "/k

Do fn ."/ D 0 nên ta cần có ." C 1/. "/k C .2" 1/ D 0. Ta xét các trường hợp sau:
 Với k D 3t thì ."C1/. "/k C2" 1 D ."C1/. "/3t C2" 1 D ."C1/C2" 1 D 3" ¤ 0,
không thỏa.
 Với k D 3t C1 thì ."C1/. "/k C2" 1 D ."C1/. "/3t C1 C2" 1 D . "2 "/C2" 1 D
0, thỏa mãn.
 Với k D 3t C2 thì ."C1/. "/k C2" 1 D ."C1/. "/3tC2 C2" 1 D ."2 1/C2" 1 ¤ 0,
không thỏa.
Từ đó suy ra n D 2.3t C 1/ C 1 D 6t C 3 thỏa mãn. Bước thử lại thực hiện tương tự.
Ta thấy bài toán này thuộc dạng tương đối chuẩn về tính chia hết của đa thức kết hợp với đa thức
xác định bởi hệ thức truy hồi. Bài toán đặt ở vị trí đầu tiên của ngày 2 cùng với hai bài khác khó
hơn hẳn khiến cho bài toán này dường như là lựa chọn duy nhất cho các thí sinh.
Dạng đa thức xác định bởi truy hồi này cũng không phải quá xa lạ, ít nhất là với đa thức
Chebyshev nổi tiếng; tuy nhiên, nhiều thí sinh tỏ ra lúng túng và không giải quyết được trọn vẹn
bài toán này là điều rất đáng tiếc.
Dưới đây là một bài tương tự:
Cho dãy đa thức fn .x/ xác định bởi công thức
(
f0 .x/ D 2; f1 .x/ D 2x C 2;
fnC2 .x/ D .2x C 2/fnC1 .x/ .x 2 C 2x 3/fn .x/; n  1

Tìm tất cả các giá trị n sao cho fn .x/ chia hết cho x 2 C 2x C 5:
324 Các phương pháp giải toán qua các kỳ thi Olympic

Bài toán 6. Với a; n nguyên dương, xét phương trình a2 x C 6ay C 36z D n trong đó ; x; y; z
là các số tự nhiên.
a) Tìm tất cả các giá trị của a để với mọi n  250, phương trình đã cho luôn có nghiệm
.x; y; z/.
b) Biết rằng a > 1 và nguyên tố cùng nhau với 6: Tìm giá trị lớn nhất của n theo a để phương
trình đã cho không có nghiệm .x; y; z/.
Lời giải. Ta sẽ sử dụng bổ đề quen thuộc sau:
Cho các số nguyên dương a; b nguyên tố cùng nhau. Khi đó, số nguyên lớn nhất không biểu diễn
được dưới dạng ax Cby với x; y tự nhiên là N0 D ab a b. Nói cách khác, N0 D ab a bC1
là số nguyên dương nhỏ nhất sao cho phương trình ax C by D m có nghiệm tự nhiên với mọi
m  N . (Định lý Sylvester).
a) Giả sử a là một giá trị thỏa mãn điều kiện đề bài. Ta sẽ nói ngắn gọn số n là “biểu diễn được”
nếu tồn tại x; y; z tự nhiên sao cho
a2 x C 6ay C 36z D n:

Rõ ràng .a; 6/ D 1 vì nếu ngược lại, giả sử .a; 6/ D d > 1 thì a2 x C 6ay C 36z chia hết cho d .
Ta chọn n  250 và n không chia hết cho d thì dẫn đến mâu thuẫn.
Dễ thấy rằng điều kiện cần để mọi n  250 biểu diễn được là .a; 6/ D 1. Để ý rằng
a2 x C 6ay C 36z D a2 x C 6.ay C 6z/ D a.ax C 6y/ C 36z
Áp dụng bổ đề trên, ta thấy các số
6a2 a2 6 D 5a2 6 và 36a C 36 a D 35a C 36
tương ứng là các số lớn nhất không biểu diễn được trong từng cách biểu diễn. Suy ra
5a2 5 < 250; 35a C 36 < 250
Suy ra a < 7. Kết hợp với điều kiện .a; 6/ D 1 suy ra chỉ có a D 1; a D 5 thỏa mãn.
Ta sẽ chứng minh a D 1; a D 5 cũng chính là các số cần tìm. Thật vậy,
Với a D 1, ta có x C 6y C 36z D n, phương trình luôn có nghiệm là .nI 0I 0/.
Với a D 5, ta chứng minh mọi số n  250 đều biểu diễn được dưới dạng 25x C 30y C 36z với
x; y; z tự nhiên. Ta viết
25x C 30y C 36z D 25x C 6.5y C 6z 20/ C 120:

Áp dụng bổ đề, mọi số tự nhiên không nhỏ hơn 20 đều có thể biểu diễn được dưới dạng 5y C 6z
nên suy ra mọi số tự nhiên đều có thể biểu diễn dưới dạng 5y C 6z 20 với y; z tự nhiên.
Tương tự, mọi số tự nhiên không nhỏ hơn 120 đều biểu diễn được dưới dạng 25x C 6u: Với
m  250 thì m 120  120 > 120 nên tồn tại x và u tự nhiên sao cho m 120 D 25x C 6u:
Lại chọn y; z tự nhiên sao cho u D 5y C 6z 20 thì được m 120 D 25x C 6.5y C 6z 20/:
Suy ra m D 25x C 30y C 36z:
Vậy tất cả các giá trị a cần tìm là a D 1; a D 5:
Lời giải và bình luận đề thi VMO 2015 325

b) Ta chứng minh bài toán tổng quát sau: Cho các số nguyên dương a và b nguyên tố cùng nhau.
Khi đó
N D a2 b C ab 2 a2 b 2 ab C 1
là số nguyên dương nhỏ nhất để phương trình a2 x C aby C b 2 z D m có nghiệm tự nhiên với
mọi m  N .

Thật vậy, nếu m  N , ta viết phương trình a2 x C aby C b 2 z D m dưới dạng

a .ax C by .ab a b C 1/ C b 2 z D m a2 b C a2 C ab a

Do m a2 b C a2 C ab a  b2a b2 a C 1 nên theo bổ đề ở câu a, tồn tại u; z tự nhiên


sao cho
m a2 b C a2 C ab a D au C b 2 z:
Lại áp dụng bổ đề, tồn tại x; y tự nhiên sao cho u C ab a b C 1 D ax C by: Khi đó

m a2 b C a2 C ab a D a .ax C by .ab a b C 1// C b 2 z , m D a2 x C aby C b 2 z

Cuối cùng, ta chứng minh số a2 b C ab 2 a2 b2 ab không biểu diễn được dưới dạng
a2 x C aby C b 2 z với x; y; z tự nhiên.
Thật vậy, giả sử ngược lại, nếu tồn tại x; y; z tự nhiên sao cho

a2 b C ab 2 a2 b2 ab D a2 x C aby C b 2 z

thì a2 .b 1 x/ C b 2 .a 1 z/ D ab.y C 1/: Suy ra hoặc b 1 x > 0 hoặc a 1 z > 0.


Không mất tính tổng quát, giả sử b 1 x > 0. Ta viết đẳng thức trên dưới dạng a2 .b 1 x/ D
b.ay C bz ab C b C a/.
Do .a2 ; b/ D 1 nên từ đây suy ra b 1 x chia hết cho b: Điều này mâu thuẫn vì 0 < b 1 x < b:
Do đó, a2 b C ab 2 a2 b 2 ab không biểu diễn được dưới dạng a2 x C aby C b 2 z với x; y; z
tự nhiên.
Bài toán được chứng minh.
Câu b của bài toán này chính là trường hợp đặc biệt của bài toán vừa phát biểu với b D 6
nên số n lớn nhất không biểu diễn được dưới dạng a2 x C 6ay C 36z với x; y; z tự nhiên là số
5a2 C 30a 36.

Nhận xét. Chủ đề định lý Bezout, thuật toán Euclid là một chủ đề quan trọng trong chương trình
số học. Nếu nắm vững chủ đề này thì cũng sẽ nắm được định lý Sylvester và làm quen với bài
toán Frobenius về các đồng xu. Đây là nội dung đã gặp trong khá nhiều đề thi, ví dụ như đề thi
IMO 1983, VN TST 2000. Cụ thể là:

Bài 1.(IMO 1983) Cho a; b; c là các số nguyên dương đôi một nguyên tố cùng nhau. Chứng minh
rằng 2abc ab bc ca là số nguyên lớn nhất không biểu diễn được dưới dạng abx Cbcy Ccaz
với x; y; z là các số tự nhiên.
Bài 2. (VN TST 2000) Cho a; b; c đôi một nguyên tố cùng nhau. Số nguyên dương n được gọi
là số bướng bỉnh nếu không biểu diễn được dưới dạng abx C bcy C caz trong đó x; y; z là các
số nguyên dương. Hỏi có tất cả bao nhiêu số bướng bỉnh?
326 Các phương pháp giải toán qua các kỳ thi Olympic

Bài 3. (Titu Andreescu, 104 Number Theory Problems) Cho a và n là các số nguyên dương.
Chứng minh rằng số nghiệm nguyên không âm .x; y; z/ của phương trình ax C by C z D ab là
1
Œ.a C 1/.b C 1/ C .a; b/ C 1 :
2
Định lý Sylvester thực ra được phát biểu đầy đủ như sau (xem thêm tài liệu Số học qua các định
lý và bài toán):
Cho a; b là các số nguyên dương nguyên tố cùng nhau. Chứng minh rằng N0 D ab a b là số
nguyên lớn nhất không biểu diễn được dưới dạng ax C by với x; y là các số nguyên không âm.
Hơn nữa, với mọi p; q nguyên với p C q D N0 , có đúng một trong hai số p; q biểu diễn được
dưới dạng ax C by với x; y là các số nguyên không âm (mà ta gọi tắt là biểu diễn được).

Định lý này được chứng minh thông qua bổ đề sau:


Bổ đề. Cho a; b là các số nguyên dương nguyên tố cùng nhau và b > 1: Chứng minh rằng với
mọi số nguyên N; tồn tại duy nhất cặp số nguyên x; y thỏa mãn điều kiện N D ax C by và
0  x < b.
Chứng minh bổ đề.
Do .a; b/ D 1 nên theo định lý Bezout, tồn tại x; y sao cho N D ax C by: Bây giờ chia cho
b; ta được x D bq C x0 với 0  x0 < b và ta có N D a.bq C x0 / C by D ax0 C b.y C qa/.
Tính tồn tại được chứng minh. Tính duy nhất là hiển nhiên vì nếu có 2 biểu diễn như thế thì
ta có ax C by D ax 0 C by 0 , suy ra a.x x 0 / D b.y 0 y/. Suy ra x x 0 chia hết cho b: Mà
0  x; x 0 < b nên điều này vô lý.
Quay trở lại định lý, nếu n > N0 . Áp dụng bổ đề, tồn tại x; y nguyên sao cho n D ax C by với
0  x < b. Suy ra by > ab a b ax D ab a.b 1/ D b: Suy ra y  0. Vậy ta đã tìm
được x; y tự nhiên sao cho n D ax C by:
Nếu N0 biểu diễn được thì ta có ab a b D ax C by với x; y  0. Ta có hai biểu diễn
ab a b D a.b 1/ C b. 1/ và ab a b D ax C by: Từ tính duy nhất ở trên, ta suy ra
y D 1; mâu thuẫn.
Vậy N0 không biểu diễn được. Do N0 không biểu diễn được nên nếu p C q D N0 thì chỉ có
nhiều nhất 1 trong 2 số p; q biểu diễn được. Cuối cùng, nếu p không biểu diễn được thì chọn
x; y sao cho p D ax C by với 0  x < b thì ta phải có y < 0: Lúc đó
q D N0 p D ab a b ax by D a.b 1 x/ C b. y 1/
biểu diễn được do b 1 x  0 và y 1  0:
Nói thêm về bài toán Frobenius về những đồng xu. Đây là bài toán xác định số tiền lớn nhất
không thể trả được khi chỉ sử dụng các đồng xu có mệnh giá cố định nào đó. Ví dụ với các đồng
xu mệnh giá 3 và 5 đơn vị thì số tiền lớn nhất không trả được là 7 đơn vị. Số lớn nhất với mỗi bộ
số như thế ta gọi là số Frobenius.
Một cách toán học, bài toán được phát biểu như sau: Cho các số nguyên dương a1 ; a2 ; a3 ; :::; an
có ước chung lớn nhất là 1: Tìm số nguyên lớn nhất không biểu diễn được dưới dạng k1 a1 C
k2 a2 C ::: C kn an với k1 ; k2 ; :::; kn là các số nguyên không âm. Số nguyên lớn nhất này được
gọi là số Frobenius và thường được ký hiệu là g.a1 ; a2 ; :::; an /.
Với n D 2, bài toán này có lời giải trọn vẹn và đó chính là định lý Sylvester đã nêu. Xa hơn nữa,
người ta đã chứng minh được một số kết quả liên quan như:
Lời giải và bình luận đề thi VMO 2015 327

1. Định lý Schur khẳng định rằng với điều kiện các số nguyên dương a1 ; a2 ; a3 ; :::; an có ước
chung lớn nhất là 1 thì số Frobenius tồn tại.
p
2. g.a; b; c/  3abc a b c

3. Với n bất kỳ, tìm ra được công thức hàm g cho các cấp số cộng, cấp số nhân, ngoài ra là
một số đánh giá chặn trên, chặn dưới.

Như vậy với n > 2 và các bất kỳ thì tính tới nay chưa tìm được công thức tường minh cho hàm
g.a1 ; a2 ; :::; an / và có vẻ hướng đi này không khả thi. Trong khi đó, với một số lượng đồng xu
cố định, tồn tại thuật toán để tính số Frobenius trong thời gian đa thức (tính theo logarith của giá
trị tiền xu có trong dữ liệu vào). Hiện chưa có thuật toán thời gian đa thức theo số đồng xu, và
bài toán tổng quát khi giá trị đồng xu là lớn tùy ý là bài toán NP-khó.

Bài toán 7. Có m học sinh nữ và n học sinh nam .m; n  2/ tham gia một liên hoan song ca.
Tại liên hoan song ca, mỗi buổi biểu diễn một chương trình văn nghệ. Mỗi chương trình văn
nghệ bao gồm một số bài hát song ca nam – nữ mà trong đó, mỗi đôi nam – nữ chỉ hát với nhau
không quá một bài và mỗi học sinh đều được hát ít nhất một bài. Hai chương trình được coi là
khác nhau nếu có một cặp nam – nữ hát với nhau ở chương trình này nhưng không hát với nhau
ở chương trình kia. Liên hoan song ca chỉ kết thúc khi tất cả các chương trình khác nhau có thể
có đều được biểu diễn, mỗi chương trình được biểu diễn đúng một lần.

a) Một chương trình được gọi là lệ thuộc vào học sinh X nếu như hủy tất cả các bài song
ca mà X tham gia thì có ít nhất một học sinh khác không được hát bài nào trong chương
trình đó. Chứng minh rằng trong tất cả các chương trình lệ thuộc vào X thì số chương
trình có số lẻ bài hát bằng số chương trình có số chẵn bài hát.

b) Chứng minh rằng ban tổ chức liên hoan có thể sắp xếp các buổi biểu diễn sao cho số các
bài hát tại hai buổi biểu diễn liên tiếp bất kỳ không cùng tính chẵn lẻ.

Lời giải.
a) Ta đánh số các học sinh nữ theo thứ tự từ 1 đến m và các học sinh nam từ 1 đến n: Ứng với
mỗi chương trình văn nghệ, ta biểu diễn việc ghép cặp của các cặp nam nữ song ca thành một
bảng m  n gồm m hàng n cột như sau: Bảng sẽ được đánh số 1 hoặc 0; trong đó ô nằm ở hàng i
cột j được điền số:

 Số 1 nếu học sinh nữ thứ i và học sinh nam thứ j có hát với nhau.

 Số 0 nếu học sinh nữ thứ i và học sinh nam thứ j không hát với nhau.

0 0 1 0 1
1 1 0 0 0
0 0 1 0 0
1 0 1 1 1
1 0 1 1 1

Một bảng gọi là tốt nếu trên mỗi hàng và mỗi cột đều phải có ít nhất một số 1: Rõ ràng theo đề
bài thì tất cả các bảng biểu diễn cho chương trình đều là tốt vì học sinh nào cũng có biểu diễn.
328 Các phương pháp giải toán qua các kỳ thi Olympic

Xét một học sinh X nào đó, giả sử đó là nữ; trường hợp học sinh nam chứng minh tương tự.
Chương trình nào đó lệ thuộc học sinh X nếu như trên bảng tương ứng của nó, tồn tại ít nhất 1
cột có đúng một số 1 nằm trên hàng của X , ta gọi bảng này là lệ thuộc X và cột như thế là cột lệ
thuộc X .
Ta cần chứng minh rằng, trong các bảng lệ thuộc X , số bảng có số các số 1 chẵn bằng số bảng
có số các số 1 lẻ.
Thật vậy,
Xét trường hợp trong bảng có k cột lệ thuộc X thì rõ ràng k < n vì nếu không, ngược lại, k D n
thì toàn bộ các ô trên hàng X đều là 1; còn tất cả các ô còn lại của bảng đều là 0: Do m  2 nên
tồn tại một dòng toàn là số 0; mâu thuẫn. Với k < n, ta bỏ k cột đó ra khỏi bảng thì trên bảng sẽ
mất đi đúng k số 1: Mỗi ô trong n k ô còn lại của hàng X sẽ được điền số 0 hoặc 1 tùy ý vì
các cột còn lại đều còn ít nhất một số 1 nữa không thuộc hàng X . Do đó, nếu ta bỏ luôn hàng X
đi thì bảng còn lại vẫn là tốt.
Suy ra số bảng lệ thuộc X trong trường hợp này sẽ là 2n k nhân với số lượng bảng tốt có kích
thước .m 1/  .n k/ còn lại. Trong mỗi bảng đó, ta chọn một ô bất kỳ của hàng X và thay
đổi số từ 0 ! 1; 1 ! 0 thì sẽ dẫn đến thay đổi tính chẵn lẻ của số các số 1 trên bảng.
Do đó, rõ ràng tồn tại một song ánh đi từ tập hợp các bảng lệ thuộc X có số các số 1 chẵn đến
tập hợp các bảng lệ thuộc X có số các số 1 lẻ.
Do đó, số lượng hai loại bảng này là bằng nhau. Ứng với mỗi k D 1; n 1 và các cách chọn k
cột phụ thuộc X thì số lượng bảng có số 1 lẻ và chẵn đều bằng nhau, vì thế nên tổng số bảng có
số các số 1 lẻ bằng với bảng có số các số 1 chẵn. Ta có đpcm.

b) Tiếp theo, ta đặt f .m; n/ và g.m; n/ lần lượt là số các bảng tốt m  n có chẵn và lẻ các số 1.
Xét một học sinh nữ tùy ý, đặt là X . Ta xét các trường hợp sau:

 Nếu tồn tại một cột nào đó lệ thuộc X thì theo câu a, số bảng có số các 1 chẵn bằng số
bảng có số các số 1 lẻ, đặt giá trị này là h.m; n/.

 Nếu không tồn tại cột nào lệ thuộc X thì bỏ hàng tương ứng của X đi, ta còn lại một bảng
tốt có m 1 hàng và n cột.

Mặt khác, số trường hợp mà hàng X có số lẻ và có số chẵn ô điền số 1 lần lượt là


X
LD Cna và
a1. mod 2/
X
C D Cna (do hàng X không thể toàn là số 0)
a0. mod 2/;a>0

Ta biết rằng .1 C x/n D Cn0 CCn1 x CCn2 x 2 C:::CCnn x n nên với x D 1, ta có ngay L D C C1.

Chú ý là tính chẵn lẻ của số các số 1 thuộc dòng X sẽ quyết định đến tính chẵn lẻ của bảng còn
lại nên ta có công thức truy hồi sau
(
f .m; n/ D h.m; n/ C L  g.m 1; n/ C C  f .m 1; n/
g.m; n/ D h.m; n/ C L  f .m 1; n/ C C  g.m 1; n/
Lời giải và bình luận đề thi VMO 2015 329

Do đó
f .m; n/ g.m; n/ D .L C / .g.m 1; n/ f .m 1; n//
D g.m 1; n/ f .m 1; n/
Lặp lại quá trình này đến khi số hàng và số cột nhỏ nhất có thể, tức là m D n D 2, ta có

f .m; n/ g.m; n/ D . 1/mCn 4


.f .2; 2/ g.2; 2// :

Đếm trực tiếp, ta thấy có f .2; 2/ D 3; g.2; 2/ D 4 nên suy ra f .m; n/ g.m; n/ D . 1/mCn 3 .
Từ đó ta thấy rằng số lượng của hai loại bảng không vượt quá 1 và có thể sắp xếp các bảng theo
thứ tự chẵn, lẻ đan xen để được điều kiện đề bài.
Ta cũng có đpcm.

Nhận xét. Có thể nói đây là một bài tổ hợp dài nhất trong lịch sử các kỳ thi VMO. Cách đây
đúng 10 năm, trong đề VMO 2005 bảng A có một bài liên quan đến định lý Turan mà trên tạp
chí THTT đã từng nhận xét: "Đây là một bài toán rời rạc, không phải khó song hầu hết học sinh
đều bó tay! Tại sao vậy? Có lẽ vấn đề là tâm lý. Trước hết khi đọc đề ra học sinh thấy choáng
ngợp bởi các khái niệm đưa ra trong bài toán và các mối quan hệ logic giữa chúng, nếu thoáng
qua thì chưa thể hiểu được ngay." Trong bài toán này, nhận xét trên dường như vẫn phù hợp.
Và nó cùng với bài 5 trong đề VMO 2009 là các bài toán dạng đếm bằng hai cách, mô hình hóa
thành bảng có thể nói là khó nhất trong nhiều năm trở lại đây.

Bài toán này đặt cuối đề thi, có độ dài và số điểm như thế khiến nhiều thí sinh đã bỏ cuộc ngay từ
bước đọc đề. Kỳ thực, bài toán cũng không đến nỗi quá khó hiểu và có lẽ tác giả đã rất cố gắng
trong việc diễn đạt đề bài một cách rõ ràng nhất có thể. Tuy nhiên, trong đề bài có giả thiết mỗi
đôi nam – nữ chỉ hát với nhau không quá một bài và mỗi học sinh đều được hát ít nhất một bài
đã khiến nhiều thí sinh hoang mang và không hiểu là thông tin này mô tả cho một chương trình
hay là cả kỳ liên hoan này. Nếu mô tả cho chương trình thì có vẻ hơi rối rắm và lung tung, mỗi
học sinh có thể lên diễn nhiều lần trong một chương trình và các tiết mục có thể được lặp đi lặp
lại trong nhiều chương trình khác nhau. Nhưng kỳ thực, nó lại chính là ý muốn của đề bài.
Để tiến hành xử lý bài toán, ta cần phải tìm cách mô hình hóa nó thành dạng thích hợp, trong đó,
mọi ràng buộc đều có thể biểu diễn được. Ở đây, việc lập thành bảng nhị phân cho một chương
trình cụ thể như trên có lẽ là lựa chọn sáng sủa, đơn giản nhất. Cách này cũng quen thuộc với hầu
hết các học sinh vì bài toán về tô màu bảng, điền số trên bảng cũng tương đối quen thuộc (ít nhất
là trong đề kiểm tra của Viện Toán trong tháng 12 vừa qua cũng có một bài như thế).

Bên cạnh lựa chọn đó, ta cũng có thể dùng các cách tiếp cận khác như:

 Dùng bipartite graph hay còn gọi là đồ thị lưỡng phân, đồ thị 2 phe rồi thao tác trên các
đỉnh và cạnh.

 Dùng cách chia thành các bộ (nam, nữ, chương trình) rồi đếm bằng 2 cách và tính tổng,
hoặc cách nói khác là xét hàm sốf .a; b; c/ W .A  B  C / ! f0; 1g với A; B; C lần lượt
là tập hợp nữ, nam và các chương trình.

Suy cho cùng, các cách này cũng là song ánh, chuyển đổi cách tiếp cận nhưng bản chất vẫn thế:
bảng tốt chính là một ma trận cạnh kề của đồ thị và nó cũng chính là tổng hợp các giá trị mà hàm
ở trên nhận được với cùng một giá trị c 2 C .
330 Các phương pháp giải toán qua các kỳ thi Olympic

Một nhận xét cơ bản nhưng mang tính quyết định trong cả 2 ý a và b của bài toán là: Trong các
tập hợp con của một tập hợp có n  1 phần tử, số tập hợp con có lẻ phần tử bằng số tập hợp con
có chẵn phần tử và cùng bằng 2n 1 . Ý này có thể giải quyết dễ dàng bằng song ánh hoặc đếm
trực tiếp và dùng nhị thức Newton.

So sánh với các bài toán trong những năm gần đây, có thể coi đây là bài toán mà phần a hỗ trợ rất
tốt cho phần b với việc chia một "trường hợp lớn" thành hai trường hợp nhỏ. Phần a vừa có tác
dụng gợi ý, vừa đóng góp một phần vào lập luận của phần b. Cái khó của bài toán này có lẽ ở chỗ
sử dụng ý tưởng truy hồi quy nạp, và đi đến cùng.
Chúng ta sẽ đặt ra câu hỏi sau đây một cách tự nhiên: “Có tất cả bao nhiêu chương trình trong
liên hoan văn nghệ?”. Đây là một bài toán không dễ và không cho ra kết quả dạng tường minh
nhưng có thể đã có thí sinh nào đó mà trong bài thi của mình, đã cố gắng đếm số lượng này.

Ta có thể giải quyết bằng nguyên lý bù trừ như sau (ta vẫn xét bảng có m hàng và n cột): Ta gọi
một bảng mà mỗi hàng đều có ít nhất một số 1 là bảng “tốt theo hàng”, gọi tập hợp các bảng này
là P . Gọi A là tập hợp các bảng tốt theo hàng nhưng lại có một cột nào đó không có số 1 nào, Ai
là tập hợp các bảng tốt theo hàng nhưng cột thứ i lại không có số 1 nào. Ta thấy

A D A1 [ A2 [ A3 [ ::: [ An :

Rõ ràng số bảng tốt cần tìm chính là jP j jAj. Trước hết, ta tính số bảng tốt theo hàng.
Chú ý rằng mỗi ô có hai cách điền là 0 hoặc 1 nên hàng có n ô sẽ có 2n cách, tuy nhiên, loại trừ
trường hợp tất cả các ô đều là 0 ra thì có 2n 1 cách. Các hàng khác cũng tương tự thế nên có
tổng cộng .2n 1/m bảng tốt theo hàng hay jP j D .2n 1/m . Do các cột bình đẳng với nhau
nên ta có thể đếm đại diện một trường hợp nào đó để suy ra các trường hợp còn lại. Giả sử có k
cột nào đó trong các cột 1; 2; 3; :::; n là không chứa số 1 nào với 1  k < n. Trên mỗi hàng sẽ
còn lại n k ô để điền vào các số 0 hoặc 1 sao cho có ít nhất một số 1 (do k ô thuộc các cột kia
m
đều được điền số 0), số cách điền cho mỗi hàng là 2n k 1 và cho cả bảng là .2n k 1/ .
Từ đó, theo nguyên lý bù trừ, ta có
n
X  m
k k n k
jAj D jA1 [ A2 [ A3 [ ::: [ An j D Cn . 1/ 2 1 :
kD1

Do đó, ta được số bảng tốt cần tìm là


n
X  m n
X  m
m k
.2 n
1/ Cn . 1/ 2n
k k
1 D Cnk . 1/k 2n k
1
kD1 kD0

Với tính bình đẳng của m; n, ta cũng suy ra được đẳng thức sau
n
X  m m
X  n
k
k
Cn . 1/ 2 n k
1 D Cmk . 1/k 2m k
1 :
kD0 kD0

Trên bảng nhị phân m  n, ta còn có thể đặt ra nhiều câu hỏi tương tự như:

Bài 1.(Columbia Olympiad) Có bao nhiêu bảng nhị phân m  n sao cho trong mỗi hình vuông
con 2  2 của bảng thì số các số 0 bằng số các số 1? Đáp số là: 2m C 2n 2.
Lời giải và bình luận đề thi VMO 2015 331

Bài 2.(Đề kiểm tra của Viện Toán 2014) Có bao nhiêu bảng nhị phân m  n mà mỗi hàng và
mỗi cột đều có số chẵn số 1? Đáp số là: 2.m 1/.n 1/ .
Bài 3.(AIME 2007) Có bao nhiêu bảng nhị phân 6  4 sao cho có đúng 12 số 1 và mỗi hàng có
đúng 2 số 1 và mỗi cột có đúng 4 số 1? Đáp số là: 1860:

Cuối cùng, nhận xét chung về bài toán này, ta thấy phát biểu của nó còn hơi khiên cưỡng dẫn đến
bài toán cồng kềnh, nặng nề. Có lẽ xuất phát từ một kết quả nghiên cứu cũ nào đó, tác giả đã
chuyển đổi từ ngôn ngữ Toán học sang một tình huống thực tế trong đời sống. Nhưng do phải
đảm bảo nhiều quan hệ nội tại trong vấn đề gốc nên cuối cùng, tình huống đó lại phản tác dụng
trở nên rắc rối, thiếu tự nhiên. Có lẽ nếu phát biểu thành dạng bảng hoặc dạng đồ thị như đã phân
tích ở trên thì có nhiều học sinh sẽ nắm bắt tốt ý tưởng hơn và sẽ mạnh dạn tiếp cận hơn, nhiều
khi kiểm tra năng lực, khả năng tư duy của học sinh thì cũng chỉ cần ở mức độ như thế mà thôi.
332 Các phương pháp giải toán qua các kỳ thi Olympic
LỜI GIẢI ĐỀ CHỌN ĐỘI TUYỂN VIỆT NAM
DỰ THI IMO 2015
Ban biên tập

Bài toán 1. Gọi ˛ là nghiệm dương của phương trình x 2 C x D 5: Giả sử n là số nguyên
dương và các số nguyên không âm c0 ; c1 ; c2 ; : : : ; cn thỏa mãn đẳng thức

c0 C c1 ˛ C c2 ˛ 2 C    C cn ˛ n D 2015: ./

a) Chứng minh rằng c0 C c1 C c2 C    C cn  2 .mod 3/:


b) Tìm giá trị nhỏ nhất của tổng c0 C c1 C c2 C    C cn :
Lời giải. a) Cách 1. Ta chứng minh bằng quy nạp theo n: Với n D 0; ta có c0 D 2015; như vậy
mệnh đề đúng. Với n D 1; do ˛ là số vô tỷ nên đẳng thức c0 C c1 ˛ D 2015 chỉ có thể xảy ra khi
c1 D 0; c0 D 2015; mệnh đề vẫn đúng.

Giả sử mệnh đề đã đúng đến n  2: Xét các số nguyên không âm c0 ; c1 ; c2 ; : : : ; cn ; cnC1 thỏa
mãn đẳng thức
c0 C c1 ˛ C c2 ˛ 2 C    C cn ˛ n C cnC1 ˛ nC1 D 2015:
Sử dụng đẳng thức ˛ 2 D 5 ˛; ta có cnC1 ˛ nC1 D cnC1 ˛ n 1 .5 ˛/: Suy ra

c0 C c1 ˛ C c2 ˛ 2 C    C .cn 1 C 5cnC1 /˛ n 1
C .cn cnC1 /˛ n D 2015:

Áp dụng giả thiết quy nạp, ta có

c0 C c1 C c2 C    C .cn 1 C 5cnC1 / C .cn cnC1 /  2 .mod 3/:

Nhưng đây cũng có nghĩa là

c0 C c1 C    C cn 1 C cn C cnC1  2 .mod 3/:

Vậy mệnh đế đúng với n C 1: Theo nguyên lý quy nạp toán học, ta có điều phải chứng minh.

Cách 2. Xét đa thức P .x/ D c0 C c1 x C c2 x 2 C    C cn x n 2015 thì P .˛/ D 0:


Ta chứng minh rằng P .x/ chia hết cho Q.x/ D x 2 C x 5; tức là P .x/ D Q.x/  S.x/ với
S.x/ là đa thức với hệ số nguyên.
Thật vậy, giả sử P .x/ D Q.x/  S.x/ C Ax C B; với A; B nguyên. Thay x D ˛ vào ta được
A˛ C B D 0: Do ˛ là số vô tỷ nên điều này chỉ có thể xảy ra khi A D B D 0: Do đó,
P .x/ D Q.x/  S.x/: Thay x D 1 vào ta được

c0 C c1 C c2 C    C cn 2015 D 3  S.1/;

suy ra
c0 C c1 C c2 C    C cn  2 .mod 3/:

333
334 Các phương pháp giải toán qua các kỳ thi Olympic

b) Với mỗi bộ số nguyên không âm .c0 ; c1 ; c2 ; : : : ; cn / thỏa mãn ./; ta gọi

c0 C c1 C c2 C    C cn

là giá của bộ số đó. Do tính sắp thứ tự tốt của tập các số tự nhiên, tồn tại bộ số .c0 ; c1 ; c2 ; : : : ; cn /
thỏa mãn ./ với giá nhỏ nhất.

Ta chứng minh nhận xét quan trọng sau:

Nhận xét. Nếu .c0 ; c1 ; c2 ; : : : ; cn / là bộ có giá nhỏ nhất thì ci < 5 với mọi i D 0; 1; 2; : : : ; n:

Chứng minh. Ta chứng minh bằng phản chứng. Giả sử tồn tại i sao cho ci  5: Khi đó dựa vào
đẳng thức 5 D ˛ 2 C ˛; ta có

ci ˛ i D .ci 5/˛ i C .˛ 2 C ˛/˛ i D .ci 5/˛ i C ˛ i C1 C ˛ i C2 :

Như vậy, bộ số .c0 ; c1 ; c2 ; : : : ; ci 5; ci C1 C 1; ci C2 C 1; : : : ; cn / cũng thỏa mãn ./ và có


giá

c0 C c1 C    C .ci 5/ C .ci C1 C 1/ C .ci C2 C 1/ C    C cn D c0 C c1 C    C cn 3

nhỏ hơn giá của bộ .c0 ; c1 ; : : : ; cn /: Điều này mâu thuẫn với cách chọn .c0 ; c1 ; : : : ; cn / ở trên.
Bây giờ, giống như phần a) ta đặt

P .x/ D c0 C c1 x C c2 x 2 C    C cn x n 2015

và Q.x/ D x 2 C x 5 thì theo a), P .x/ D Q.x/  S.x/: Đặt:

S.x/ D b0 C b1 x C b2 x 2 C    C bn 2 x n 2 :

So sánh hệ số hai vế, ta được

c0 2015 D 5b0 ;
c1 D b0 5b1 ;
c2 D b0 C b1 5b2 ;
c3 D b1 C b2 5b3 ;
:::;
cn D b n 2 :

Từ điều kiện 0  c0  4; ta suy ra được ngay c0 D 0 và b0 D 403: Tiếp tục sang dòng thứ hai,
ta tìm được c1 D 3 và b1 D 80: Nói chung dãy .ci ; bi / được xác định một cách duy nhất theo
công thức ci D bi 2 C bi 1 mod 5 và bi D bi 2 Cb5i 1 ci :

Sử dụng công thức này, ta lần lượt tính được

i 0 1 2 3 4 5 6 7 8 10 11 12
c 0 3 3 1 1 1 3 4 0 0 3 1
b 403 80 96 35 26 12 7 3 2 1 0 0
Lời giải đề chọn đội tuyển Việt Nam dự thi IMO 2015 335

Từ đó tìm được bộ có giá nhỏ nhất là .0; 3; 3; 1; 1; 1; 3; 4; 0; 0; 3; 1/; và giá nhỏ nhất là 20: Vậy
giá trị nhỏ nhất của c0 Cc1 C  Ccn là 20; đạt được ở bộ .0; 3; 3; 1; 1; 1; 3; 4; 0; 0; 3; 1/:

Bình luận. Ý tưởng quy nạp trong cách giải 1 của phần a) là khá tự nhiên. Và sự kiện P .x/ chia
hết cho Q.x/ mà ta dùng trong cách giải 2 không phải là một điều gì đặc biệt. Ta có hai tính chất
đơn giản nhưng quan trọng sau:

Tính chất 1. Nếu P .x/ và Q.x/ là các đa thức với hệ số nguyên, ngoài ra Q.x/ đơn khởi, tức là
có hệ số cao nhất bằng 1 thì tồn tại duy nhất các đa thức với hệ số nguyên S.x/ và R.x/ sao cho

i) P .x/ D Q.x/  S.x/ C R.x/I


 
ii) deg R.x/ < deg Q.x/ :

Tính chất 2. Cho P .x/; Q.x/ là các đa thức với hệ số nguyên, trong đó Q.x/ bất khả quy, cùng
nhận số thực ˛ làm nghiệm. Khi đó, P .x/ chia hết cho Q.x/:

Trong phần b), ta đã dùng phương pháp tìm tính chất của bộ số tối ưu, sau đó dùng tính chất này
để xây dựng bộ số tối ưu đó. Lời giải phần b) liên hệ chặt chẽ đến cả hai cách giải ở phần a). Đây
là một bài toán khá thú vị vì nó liên hệ được nhiều vấn đề trong cùng một bài toán: đa thức, số
nguyên, hệ đếm cơ số, chia hết, thuật toán.

Một số bài toán liên quan

1. (Nga 2014) Kho bạc nhà nước của nước Cộng hòa toán học chọn một số ˛ > 2 và sản xuất
các đồng xu có mệnh giá 1 rúp và ˛ k rúp với mọi k nguyên dương. Người ta nhận thấy
rằng mọi mệnh giá (trừ mệnh giá 1) đều vô tỷ. Có thể xảy ra tình huống là mọi số nguyên
dương n; ta đều có thể chọn ra một số đồng xu có tổng bằng n và mỗi một mệnh giá được
chọn không quá 6 lần?

2. (IMO 1976) Tổng của một số số nguyên dương bằng 1976: Hỏi tích của chúng lớn nhất
bằng bao nhiêu?

Bài toán 2. Cho đường tròn .O/ có dây BC cố định và không phải là đường kính. Xét điểm A
di chuyển trên cung lớn BC sao cho tam giác ABC nhọn và AB < AC: Gọi I; H lần lượt là
trung điểm cạnh BC và trực tâm tam giác ABC . Tia IH cắt lại đường tròn .O/ tại K, đường
thẳng AH cắt đường thẳng BC tại D và đường thẳng KD cắt lại đường tròn .O/ tại M: Từ
điểm M , kẻ đường thẳng vuông góc với đường thẳng BC cắt AI tại điểm N:

a) Chứng minh rằng điểm N luôn thuộc một đường tròn cố định khi A thay đổi.

b) Đường tròn tiếp xúc với AK ở A và đi qua N cắt AB; AC lần lượt tại P; Q: Gọi J là
trung điểm của PQ. Chứng minh rằng đường thẳng AJ luôn đi qua một điểm cố định.

Lời giải. a) Ta sẽ chứng minh bài toán tổng quát hơn:


Cho tam giác ABC nhọn nội tiếp đường tròn .O/ có AD là đường cao và H là trực tâm. M là
trung điểm BC . Đường tròn đường kính AH cắt .O/ tại G khác A. GD cắt .O/ tại K khác A.
Đường thẳng qua K vuông góc BC cắt AM tại L. Chứng minh rằng bốn điểm B; C; L; H cùng
thuộc một đường tròn.
336 Các phương pháp giải toán qua các kỳ thi Olympic

A F

G O
L
H

B D C
M

E
K

Thật vậy,
Do G nằm trên đường tròn đường kính AH nên gọi GH cắt .O/ tại E khác G thì AE là đường
kính của .O/. Từ đó, dễ thấy tứ giác HBEC là hình bình hành nên HE đi qua M . Gọi KM cắt
.O/ tại F khác K. Chú ý tứ giác AGDM nội tiếp nên

∠AMB D 180ı ∠AGD D ∠AFM

Ta có, BC tiếp xúc đường tròn ngoại tiếp tam giác AFM . Do đó, OM ? BC ? SM với S là
tâm ngoại tiếp tam giác SFM do đó S thuộc OM .
Mặt khác, dễ dàng có được SO ? AF nên suy ra AF k BC .
Từ đó, ta cũng có tam giác MAF cân nên

∠LMB D ∠MAF D ∠MFA D ∠BMK:

Tam giác KLM cân nên K; L đối xứng qua BC . Chú ý đối xứng của H qua BC thuộc .O/ nên
H; L; B; C thuộc đường tròn đối xứng với .O/ qua BC .
Trở lại bài toán ban đầu, dễ thấy điểm N luôn di động trên đường tròn ngoại tiếp tam giác BH C
cố định.
b) Tương tự trên, ta lại phát biểu thành bài toán sau:
Cho tam giác ABC nội tiếp đường tròn .O/ và trực tâm H . Đường tròn đường kính AH cắt .O/
tại G khác A. Một đường tròn tiếp xúc AG tại A cắt CA; AB lần lượt tại E; F khác A. Chứng
minh rằng AO chia đôi EF .
Ta chứng minh như sau:
Gọi GH cắt .O/ tại D khác A thì AD là đường kính của .O/. Từ đó tứ giác HBDC là hình
bình hành nên HD đi qua trung điểm M của BC .
Lời giải đề chọn đội tuyển Việt Nam dự thi IMO 2015 337

F
G

N
O E
H
P
B C
M

Gọi AD cắt đường tròn ngoại tiếp tam giác AEF tại P khác A và AP cắt EF tại N . Ta dễ thấy

∠FPN D ∠FAE D ∠GAB D ∠BDM


∠PFE D ∠PAE D ∠DBM
Từ đó tam giác DBM và PFM đồng dạng. Tương tự tam giác DCM và PEN đồng dạng.
Mà M là trung điểm BC dễ suy ra N là trung điểm EF .
Trở lại bài toán ban đầu, dễ thấy điểm cố định cần tìm chính là O.

Nhận xét. Đây bài toán hay dựa trên một cấu hình khá quen thuộc về điểm Miquel. Tuy vậy có
thể thấy rằng hai ý của bài toán hầu như không liên quan tới nhau vì thực chất ý b) chỉ cần đường
tròn bất kỳ qua A tiếp xúc AK là bài toán đúng, mặt khác điểm cố định lại chính là O xuất hiện
ngay trong đề bài nên giảm nhiều thú vị của bài toán.
Chú ý rằng có một số tính chất hệ quả như điểm L cũng thuộc đường tròn đường kính AH và
AK là đường đối trung của tam giác ABC . Thực chất các tính chất đó đều là các mô hình khá
quen thuộc. Sau đây, chúng tôi xin đưa ra hai mở rộng khác nhau cho ý đầu tiên của bài toán này:

Bài 1. Cho tam giác ABC nội tiếp đường tròn .O/. Điểm P thuộc cung BC không chứa A.
Q đối xứng P qua BC . QB; QC cắt CA; AB tại E; F . Đường tròn ngoại tiếp tam giác AEF
cắt .O/ tại G khác A. AP cắt BC tại D. GD cắt .O/ tại K khác G. M là trung điểm của BC .
Đường thẳng qua K vuông góc BC cắt AM tại L.

a) Chứng minh rằng B; C; L; Q cùng thuộc một đường tròn.

b) Chứng minh rằng L nằm trên đường tròn ngoại tiếp tam giác AEF .

Bài 2. Cho tam giác ABC nội tiếp đường tròn .O/. Điểm P thuộc cung BC không chứa A. Q
đối xứng P qua BC . QB; QC cắt CA; AB tại E; F .
338 Các phương pháp giải toán qua các kỳ thi Olympic

a) Chứng minh rằng đường tròn ngoại tiếp tam giác AEF luôn đi qua một điểm cố định khác
A khi P thay đổi.

b) Chứng minh rằng tâm đường tròn Euler của tam giác AEF luôn thuộc một đường thẳng
cố định khi P thay đổi.

Việc chỉ ra bốn điểm thẳng hàng G; H; M; D đóng vai trò quan trọng trong lời giải đã nêu. Cách
dùng kỹ thuật góc và đồng dạng đó có thể được dùng để giải quyết các bài toán tương tự sau:

Bài 3. Cho tam giác ABC nội tiếp đường tròn .O/. Điểm P thuộc BC không chứa A. Q đối
xứng P qua BC . QB; QC cắt CA; AB tại E; F . Đường tròn ngoại tiếp tam giác AEF cắt .O/
tại G khác A. Một đường tròn tiếp xúc AG tại A cắt CA; AB lần lượt tại M; N . R thuộc .O/
sao cho PR k BC . Chứng minh rằng AR chia đôi MN .

Bài 4. Cho tam giác ABC với G thuộc .O/. Một đường tròn tiếp xúc AG tại A cắt CA; AB lần
lượt tại E; F khác A. D thuộc BC . GD cắt .O/ tại L khác G. AL cắt EF tại K. Chứng minh
rằng trung điểm của BF; CE và DK thẳng hàng.

Bài 5. (Đề châu Á Thái Bình Dương 2012) Cho tam giác ABC nhọn. Gọi D là chân đường cao
kẻ từ A đến BC; M là trung điểm của BC và H là trực tâm tam giác ABC: Giả sử E là giao
điểm của đường tròn € ngoại tiếp tam giác ABC với tia MH: Gọi F là giao điểm của đường
BF AB
thẳng ED và đường tròn €: Chứng minh rằng CF D AC :

Bài toán 3. Một số nguyên dương k có tính chất T .m/ nếu như với mọi số nguyên dương a;
tồn tại số nguyên dương n sao cho

1 k C 2 k C 3 k C    C nk  a .mod m/:

a) Tìm tất cả các số nguyên dương k có tính chất T .20/:

b) Tìm số nguyên dương k nhỏ nhất có tính chất T .2015 /:

Lời giải. a) Đặt Sk .n/ D 1k C 2k C    C nk thì theo các công thức quen thuộc, ta có

n.n C 1/ n.n C 1/.2n C 1/ n2 .n C 1/2


S1 .n/ D ; S2 .n/ D ; S3 .n/ D
2 6 4

n.n C 1/.2n C 1/.3n2 C 3n 1/ 6n5 C 15n4 C 10n3 n
S4 .n/ D D :
30 30
Sử dụng tính chất nkC4 nk  0 .mod 20/ với mọi k > 1 ta suy ra k > 1 thỏa mãn tính chất
T .20/ khi và chỉ khi k C 4 thỏa mãn tính chất T .20/: ./

Để có tính chất T .20/; trước hết ta phải có tính chất T .5/: Lập bảng mô-đun 5 như sau

N 1 2 3 4 5 6 7 8 9
S1 .n/ 1 3 1 0 0 1 3 1 0
S2 .n/ 1 0 4 0 0 1 0 4 0
S3 .n/ 1 4 1 0 0 1 4 1 0
Lời giải đề chọn đội tuyển Việt Nam dự thi IMO 2015 339

N 1 2 3 4 5 6 7 8 9 10 11
n4 1 16 1 16 5 16 1 16 1 0 1

N 1 2 3 4 5 6 7 8 9 10 11
S4 .n/ 1 17 18 14 19 15 16 12 13 13 14
N 12 13 14 15 16 17 18 19 20 21 22
S4 .n/ 10 11 7 12 8 9 5 6 6 7 3
N 23 24 25 26 27
S4 .n/ 4 0 5 1 2

Ta thấy k D 1; 2; 3 không thỏa mãn. Vì n5  n .mod 5/ nên k D 5 cũng không thỏa mãn. Vậy
tất cả các số không là bội của 4 đều không thỏa mãn tính chất T .20/:

Ta chứng minh k D 4 thỏa mãn tính chất T .20/; từ đó, sử dụng ./ suy ra mọi bội số của 4 đều
thỏa mãn tính chất T .20/: Ta thấy bảng đồng dư mô-đun 20 của n4 là
Suy ra bảng đồng dư mô-đun 20 của S4 .n/ là:
Như vậy tất cả các số dư đều xuất hiện. Vậy tất cả các số thỏa mãn điều kiện T .20/ là các bội số
của 4:

b) Theo kết quả câu a) thì k D 1; 2; 3 không thỏa mãn điều kiện T .2015 /: Ta chứng minh rằng
k D 4 thỏa mãn điều kiện T .20m / với mọi m: Từ đó suy ra k D 4 là giá trị nhỏ nhất cần tìm.

Để chứng minh điều này, ta chứng minh với mọi a nguyên, tồn tại n sao cho

6n5 C 15n4 C 10n3 n  30a .mod 30  20m /:

Ta lần lượt chứng minh rằng với mọi a nguyên:


1) Tồn tại n sao cho 6n5 C 15n4 C 10n3 n  30a .mod 3/; giả sử là n1 :
2) Tồn tại n sao cho 6n5 C 15n4 C 10n3 n  30a .mod 22mC1 /; giả sử là n2 :
3) Tồn tại n sao cho 6n5 C 15n4 C 10n3 n  30a .mod 5mC1 /; giả sử là n3 :
Khi đó, theo định lý Trung hoa về số dư, tồn tại n sao cho

n  n1 .mod 3/; n  n2 .mod 22nC1 /; n  n3 .mod 5nC1 /

và khi đó với n này thì 6n5 C 15n4 C 10n3 n  30a .mod 30  20n / và ta có điều phải chứng
minh.

Mệnh đề 1) ở trên là hiển nhiên, ta có thể chọn n D 0:

Để chứng minh 2), ta chứng minh quy nạp theo n rằng với mọi a nguyên và với mọi m nguyên
dương, tồn tại n sao cho

6n5 C 15n4 C 10n3 n  30a .mod 2m /:

Với m D 1 điều này đúng, ta có thể chọn n D 0: Giả sử tồn tại n sao cho 6n5 C15n4 C10n3 n 
30a .mod 2m /: Ta chứng minh tồn tại n sao cho 6n5 C 15n4 C 10n3 n  30a .mod 2mC1 /:
Đặt:
6n5 C 15n4 C 10n3 n D 30a C u  2m
340 Các phương pháp giải toán qua các kỳ thi Olympic

và chọn N D n C v  2m : Khi đó, ta có

6N 5 C 15N 4 C 10N 3 N  6n5 C 15n4 C 10n3 n v  2m


D 30a C .u v/  2m .mod 2mC1 /:

Như vậy chỉ cần chọn v D u là ta có N cần tìm.

Mệnh đề 3) được chứng minh bằng cách hoàn toàn tương tự.

Ta đã hoàn tất việc chứng minh k D 4 thỏa mãn điều kiện T .20m / với mọi m 2 N : Vậy k D 4
là giá trị nhỏ nhất cần tìm.
Lời giải đề chọn đội tuyển Việt Nam dự thi IMO 2015 341

Bình luận.

1. Các hệ số của đa thức 6n5 C 15n4 C 10n3 n đóng vai trò khá quan trọng trong bước
chuyển từ m lên m C 1; chẳng hạn khi từ mô-đun 2m lên mô-đun 2mC1 ; các hệ số 6 và 10
là chẵn nên mới có các đồng dư:

6N 5  6n5 .mod 2mC1 /; 10N 3  10n3 .mod 2mC1 /:

Với đồng dư thức 15N 4  15n4 .mod 2mC1 / ta dùng đến nhị thức Newton. Tương tự cho
trường hợp từ mô-đun 5m lên 5mC1 ; các hệ số 15 và 10 chia hết cho 5; còn với hệ thức

6N 5  6n5 .mod 5mC1 /;

ta dùng đến khai triển nhị thức Newton.

2. Việc sử dụng nâng lũy thừa và định lý Trung hoa về số dư là những kỹ thuật rất cơ bản
trong số học mô-đu-la. Vì thế bài toán này chỉ khó đối với các bạn chưa quen với các kỹ
thuật này, ngược lại đó là một bài tập khá đơn giản. Đây cũng là điểm yếu của bài toán.
Trong thực tế chấm thi, số học sinh làm đúng phần b) rất ít.

Một số bài toán liên quan

1. (VMO 1997) Chứng minh rằng với mọi số nguyên dương n; tồn tại số nguyên dương k sao
cho 19k C 97 chia hết cho 2n :

2. (Saudi Arabia TST 2015) Cho n và k là các số nguyên dương. Chứng minh rằng nếu n
nguyên tố cùng nhau với 30 thì tồn tại các số nguyên a và b; mỗi số đều nguyên tố cùng
nhau với n; sao cho a2 b 2 C k chia hết cho n:

Bài toán 4. Có 100 sinh viên tham dự một cuộc thi vấn đáp. Ban giám khảo gồm 25 thành
viên. Mỗi sinh viên được hỏi thi bởi một giám khảo. Biết rằng mỗi sinh viên thích ít nhất 10 giám
khảo trong số các thành viên trên.

a) Chứng minh rằng có thể chọn ra 7 giám khảo mà mỗi thí sinh đều thích ít nhất 1 trong 7
người đó.

b) Chứng minh rằng có thể sắp xếp lịch thi sao cho mỗi thí sinh được đúng 1 giám khảo mình
thích hỏi và mỗi giám khảo hỏi không quá 10 thí sinh.

Lời giải. a) Cách 1. Gọi A1 là giám khảo được nhiều sinh viên thích nhất và a1 là số sinh viên
thích A1 : Khi đó, do mỗi sinh viên thích ít nhất 10 giám khảo, ta có

100  10
a1  D 40:
25
Ta chọn giám khảo A1 và loại đi tất cả các sinh viên thích A1 ; còn lại 100 a1 sinh viên. Ta lại
xét A2 là giám khảo được nhiều trong các sinh viên còn lại thích nhất và gọi a2 là số sinh viên
trong số sinh viên còn lại thích A2 : Ta cũng có

.100 a1 /  10
a2  :
24
342 Các phương pháp giải toán qua các kỳ thi Olympic

Ta có lúc này
.100 a1 /  10 14a1 C 1000 14  40 C 1000
a1 C a2  a1 C D  D 65:
24 24 24
Ta chọn giám khảo A2 và loại đi tất cả các sinh viên thích A2; còn lại 100 a1 a2 sinh viên.
Tiếp theo, ta xét A3 là giám khảo được nhiều trong các sinh viên còn lại thích nhất và gọi a3 là
số sinh viên trong số sinh viên còn lại thích A3 : Ta có
.100 a1 a2 /  10
a3  :
23
Ta có lúc này:
a1 a2 /  10
.100
a1 C a2 C a3  a1 C a2 C
23
13.a1 C a2 / C 1000 13  65 C 1000
D  D 80; 22:
23 23
Suy ra a1 C a2 C a3  81: Hoàn toàn tương tự như vậy ta có
.100 a2 a3 /  10
a1
a1 C a2 C a3 C a4  a1 C a2 C a3 C
22
12.a1 C a2 C a3 / C 1000 12  81 C 1000
D  D 89; 63:
23 23
Suy ra a1 C a2 C a3 C a4  90: Tiếp tục như vậy, ta có a1 C    C a5  95; a1 C    C a6  98
và a1 C a2 C    C a7  100:
Cách 2. Cũng như cách 1, ta gọi A1 là giám khảo được nhiều sinh viên thích nhất và a1 là số
sinh viên thích A1 : Khi đó, do mỗi sinh viên thích ít nhất 10 giám khảo, ta có
100  10
a1  D 40:
25
Ta chọn giám khảo A1 và loại đi tất cả các sinh viên thích A1 ; còn lại tối đa 60 sinh viên. Nếu số
sinh viên không phải là 60; ta sẽ bổ sung thêm một số sinh viên “ảo” cho đủ 60: Các sinh viên
ảo này thích tất cả các giám khảo. Rõ ràng nếu ta chọn được 7 giám khảo cho các sinh viên kể cả
các sinh viên ảo thì cũng là 7 giám khảo phù hợp cho 100 sinh viên ban đầu. Bổ sung như thế, ta
có 60 sinh viên, mỗi sinh viên thích ít nhất 10 giám khảo.
Ta gọi A2 là giám khảo được nhiều sinh viên trong số 60 sinh viên này (cả thật lẫn ảo, nếu có)
thích và a2 là số sinh viên thích giám khảo A2 ; ta có
60  10
a2  D 25:
24
Ta lại chọn giám khảo A2 và loại đi tất cả các sinh viên thích A2 : Số sinh viên còn lại tối đa là
60 25 D 35: Ta lại bổ sung một số sinh viên ảo nếu số sinh viên nhỏ hơn 35; và tiếp tục gọi
A3 ; a3 có định nghĩa tương tự như trên. Ta có
35  10
a3  D 15; 21:
23
Suy ra a3  16: Ta lại chọn A3 và loại đi tất cả các sinh viên thích A3 : Số sinh viên còn lại
không quá 35 16 D 19: Cứ tiếp tục như vậy ta được
Lời giải đề chọn đội tuyển Việt Nam dự thi IMO 2015 343

 a4  9; số sinh viên còn lại sau khi chọn A4 và loại đi tất cả các sinh viên thích A4 không
quá 10:

 a5  5; số sinh viên còn lại sau khi chọn A5 và loại đi tất cả các sinh viên thích A5 không
quá 4:

 a6  2; số sinh viên còn lại sau khi chọn A6 và loại đi tất cả các sinh viên thích A6 không
quá 2:

 a7  2; số sinh viên còn lại sau khi chọn A7 và loại đi tất cả các sinh viên thích A7 không
quá 0:

Vậy sau khi chọn 7 giám khảo A1 ; A2 ; : : : ; A7 thì không còn sinh viên nào, tức là mỗi sinh viên
đều thích 1 trong 7 giám khảo đã chọn (đpcm).

b) Ta cứ sắp xếp lịch thi cho các thí sinh sao cho mỗi giám khảo hỏi thi không quá 10 thí sinh,
đến khi không thể thực hiện được nữa thì dừng lại. Giả sử còn một thí sinh A chưa được thi. Vì A
thích ít nhất 10 giám khảo nên ta xét 10 trong số giám khảo mà A thích. Theo giả sử thì ta không
thể cho A thi nữa do đó các giám khảo đều đã đủ “quota”, tức là đã hỏi đủ 10 thí sinh. Vì mỗi
thí sinh chỉ được hỏi bởi một giám khảo nên số thí sinh đã được hỏi thi là 10  10 D 100; mâu
thuẫn. Vậy không còn thí sinh nào không được hỏi thi và ta có điều phải chứng minh.

Bình luận.

1. Câu a) bài toán này có thể mô hình hóa dưới dạng bài toán tập hợp: Có 100 tập con
A1 ; A2 ; : : : ; A100 của tập hợp X D f1; 2; : : : ; 25g; jAi j  10: Khi đó tồn tại Y  X;
jY j D 7 sao cho Ai \ Y ¤ với mọi i D 1; 2; : : : ; 100: Với mô hình này, ta có thể chuyển
sang mô hình ma trận thuộc. Chẳng hạn, ta có thể đi theo hướng chứng minh phản chứng
và đếm số bộ 7 số 0 trong ma trận thuộc: Giả sử ngược lại, không tồn tại Y thỏa mãn yêu
cầu đề bài. Khi đó với mọi 7 cột ta luôn tìm được ít nhất một dòng mà giao của dòng đó
7
với 7 cột là các số 0: Suy ra số bộ 7 số 0 trên dòng ít nhất là C25 : Mặt khác, trên mỗi dòng
7
có nhiều nhất 15 số 0 nên số bộ 7 số 0 trên dòng nhiều nhất là 100C15 : Nếu ta có được
7 7 7 7
C25 > 100C15 thì sẽ suy ra mâu thuẫn. Đáng tiếc (!) là ở đây thì ta lại có C25 < 100C15
nên cách này không khả thi. Thú vị là chỉ cần giảm số giám khảo từ 25 xuống 24 thì quả
7 7
thật C24 > 100C14 và cách giải này trót lọt. Ngược lại, nếu thay 25 bằng 26 thì quy trình
giải trên đây sẽ cho số giám khảo cần thiết là 8: Như vậy số 25 được chọn khá có chủ ý.

2. Với các chú ý ở trên, có hai câu hỏi được đặt ra là:

1) Nếu cố định các thông số 100 (số sinh viên), 25 (số giám khảo), 10 (số giám khảo tối
thiểu mà một sinh viên thích) thì có thể thay 7 bằng 6 được không?
2) Nếu cố định các thông số 100 (số sinh viên), 10 (số giám khảo tối thiểu mà một sinh
viên thích) và 7 (số giảm khảo cần chọn) thì có thể thay 25 bằng 26 được không? Và
có thể thay bằng số lớn nhất là bao nhiêu?

3. Cách giải câu b) trình bày ở trên cho thấy kết quả câu b) khá tầm thường và như vậy bài 4
thực sự là bài dễ nhất của kỳ thi. Xét tổng thể trong bối cảnh ngày thi thứ hai thì điều này
là hợp lý. Tuy nhiên, kết quả bài làm của thí sinh ở bài 4 một lần nữa cho thấy tổ hợp vẫn
là yếu điểm then chốt của học sinh Việt Nam.
344 Các phương pháp giải toán qua các kỳ thi Olympic

Một số bài toán liên quan

1. (IMC 2002) 200 sinh viên tham dự một cuộc thi toán. Họ phải giải 6 bài toán. Biết rằng
mỗi bài toán được giải đúng bởi ít nhất 120 sinh viên. Chứng minh rằng tồn tại hai sinh
viên mà hợp lại giải đúng cả 6 bài toán.

2. (Putnam) Cho tập hữu hạn X cójXj > 10: A1 ; A2 ; : : : ; A1066 là các tập con của X sao
cho jAi j > jX2 j với mọi i D 1; 2; : : : ; 1066: Chứng minh rằng tồn tại tập con A gồm 10
phần tử của X sao cho jA \ Ai j  1 với mọi i D 1; 2; : : : ; 1066:

3. (Nga 1993) Mỗi một cư dân thành phố N quen với ít nhất 30% cư dân của thành phố. Một
người dân sẽ đi bầu cử nếu như người quen của người đó tranh cử. Chứng minh rằng có thể
tổ chức bầu cử thị trưởng thành phố gồm hai ứng cử viên, sao cho có ít nhất một nửa cư
dân đi bầu.

Bài toán 5. Cho tam giác ABC nhọn không cân và có điểm P nằm trong tam giác sao cho
∠APB D ∠AP C D ˛ > 180ı ∠BAC: Đường tròn ngoại tiếp tam giác APB cắt AC ở E
khác A. Đường tròn ngoại tiếp tam giác AP C cắt AB ở F khác A. Gọi Q là điểm nằm trong
tam giác AEF sao cho ∠AQE D ∠AQF D ˛: Gọi D là điểm đối xứng với Q qua EF , phân
giác góc ∠EDF cắt AP tại T:

a) Chứng minh rằng ∠DET D ∠ABC; ∠DF T D ∠ACB.

b) Đường thẳng PA cắt các đường thẳng DE; DF lần lượt tại M; N . Gọi I; J lần lượt là
tâm đường tròn nội tiếp các tam giác PEM; PF N và K là tâm đường tròn ngoại tiếp tam
giác DIJ . Đường thẳng DT cắt .K/ tại H . Chứng minh rằng đường thẳng HK đi qua
tâm đường tròn nội tiếp của tam giác DMN:

Lời giải. Trước hết, ta sẽ chứng minh các bổ đề này và từ đó đi đến giải quyết bài toán đã nêu:
Bổ đề 3. Cho tứ giác ABCD.

a) Tứ giác ABCD ngoại tiếp khi và chỉ khi AB C CD D AD C BC . Tứ giác ABCD gọi là
ngoại tiếp tức là tồn tại đường tròn .I / tiếp xúc các cạnh AB; BC; CD; DA.

b) Tứ giác ABCD bàng tiếp góc A; C khi và chỉ khi AB C AD D CB C CD. Tứ giác
ABCD gọi là bàng tiếp tức là tồn tại một đường tròn .I / chứa trong góc A hoặc C tiếp
xúc với các cạnh AB; BC; CD; DA kéo dài.

A
D C B

D I B
I

C
Lời giải đề chọn đội tuyển Việt Nam dự thi IMO 2015 345

Bổ đề này là kết quả cơ bản có trong nhiều tài liệu. Để chứng minh, ta chỉ cần dựng các tam giác
cân và chỉ ra các phân giác của các góc đồng quy.

Bổ đề 4. Cho tứ giác ABCD nội tiếp đường tròn .O/. Hai đường chéo AC và BD cắt nhau
tại S . Đường tròn ngoại tiếp tam giác SAD; SBC cắt tại tại T khác S . Dựng ra ngoài tứ giác
tam giác ABR đồng dạng với DC T . Chứng minh rằng tứ giác ATBR là tứ giác ngoại tiếp.

R B

A
K

O
T

D C

Gọi K đối xứng R qua BC . Ta thấy

∠KAS D ∠BAS ∠BAK D ∠BDC ∠RAB


D ∠BDC ∠TDC D ∠SDT D ∠SAT:

Từ đó, AS là phân giác ∠KAT . Tương tự, BS là phân giác ∠KBT . Dễ thấy T S là phân giác
∠ATB.

Từ đó, đường tròn .S/ tiếp xúc với KA; KB; TA; TB suy ra

AK AT D BK BT:

Theo tính đối xứng, suy ra AR AT D BR BT hay AR C BT D BR C AT suy ra tứ giác


ARBT ngoại tiếp.

Bổ đề này được chứng minh.

Bổ đề 5. Cho tứ giác ABCD ngoại tiếp đường tròn .I /. Một đường thẳng đi qua A cắt đoạn
thẳng BC và cắt tia đối tia CD tại N . Gọi J; K; L là tâm nội tiếp tam giác CNM; MAB và
NAD. Chứng minh rằng trực tâm tam giác JKL nằm trên MN .
346 Các phương pháp giải toán qua các kỳ thi Olympic

K B
I
P
L
M

D J
C N

Gọi tiếp tuyến của đường tròn .L/ nội tiếp tam giác NAD cắt AM; AB tại P; Q. Như vậy tứ
giác AP CD nội tiếp.
Kết hợp ABCD nội tiếp suy ra

PA P C D DA DC D BA BC:

Suy ra tứ giác AP CB bàng tiếp hay tứ giác BQPM ngoại tiếp vậy CP tiếp xúc .K/. Từ đó

2∠LCK D ∠BCD D ∠CMN C ∠CNM D 2∠JMN C 2∠JNM D 2∠MJL:

Do đó, tứ giác CJKL nội tiếp. Theo định lý đường thẳng Steiner thì đường thẳng nối đối xứng
của C qua JK; JL đi qua trực tâm tam giác JKL.
Chú ý theo tính chất phân giác thì đường thẳng đó chính là MN . Vậy MN đi qua trực tâm tam
giác JKL.
Bổ đề này được chứng minh.
Kết hợp các bổ đề trên lại, ta có thể giải quyết cả hai câu của bài toán đã nêu.
Nhận xét. Đây là bài toán hay. Câu a) dùng gợi ý hướng giải cho câu b). Nếu để ý kỹ thực chất
bài toán này là sự ghép nối của hai bài toán khác.
Chú ý rằng ta hoàn toàn có thể thay thế AP thành một đường thẳng bất kỳ đi qua P . Và khi đó,
ta có thể phát biểu lại bài toán này đẹp hơn như sau:
Bài 6. Cho tam giác ABC và có điểm P nằm trong tam giác sao cho ∠APB D ∠AP C D ˛:
Đường tròn ngoại tiếp tam giác PAB; PAC lần lượt cắt CA; AB tại E; E khác A. Điểm Q là
điểm nằm trong tam giác AEF sao cho ∠AQE D ∠AQF D ˛. Gọi D là điểm đối xứng với
Q qua EF . Một đường thẳng đi qua P cắt các đường thẳng DE; DF lần lượt tại M; N sao cho
M; N cùng phía với P . Gọi I; J lần lượt là tâm đường tròn nội tiếp các tam giác PEM; PF N
và K là tâm đường tròn ngoại tiếp tam giác DIJ . Phân giác ∠EDF cắt .K/ tại H . Chứng minh
rằng đường thẳng HK đi qua tâm đường tròn nội tiếp của tam giác DMN:
Ngoài ra, dưới đây là một số bài toán liên quan:
Bài 7. Cho tứ giác ABCD ngoại tiếp và điểm P nằm trong tứ giác sao cho các đường thẳng
AP; DP tương ứng cắt đoạn BC tại S; R theo thứ tự. Chứng minh rằng tâm đường tròn nội tiếp
của các tam giác ABS; DCR; PAD; PSR cùng thuộc một đường tròn.
Lời giải đề chọn đội tuyển Việt Nam dự thi IMO 2015 347

Bài 8. Cho tứ giác ABCD nội tiếp đường tròn .O/. Các điểm E; F thuộc cạnh CB; AD sao
cho EF k AB. DE cắt CF tại S . Đường tròn ngoại tiếp tam giác ASD và BSC cắt nhau
tại T khác S. Dựng điểm R ở ngoài tứ giác ABCD sao cho ∠RAB D ∠ASF C ∠TDC và
∠RBA D ∠BSE C ∠T CD. Chứng minh rằng tứ giác ATBR ngoại tiếp.

Cuối cùng, để thú vị hơn, chúng ta có thể đưa ra một ý tưởng khai thác mới cho đề bài này.
Những điểm P nằm trong tam giác ABC sao cho ∠APB D ∠AP C thì các điểm P đó có gì
đặc biệt hay nói cách khác, quỹ tích P là gì. Bài toán sau sẽ giải đáp thắc mắc đó, chúng ta sẽ
không cố gắng tìm quỹ tích P mà sẽ tìm quỹ tích điểm đẳng giác của P . Đó là một quỹ tích đẹp
và chứng minh đơn giản, các bạn hãy làm như một bài luyện tập:

Bài 9. Cho tam giác ABC có đường tròn Apollonius ứng với A là .K/. P là một điểm thuộc
.K/. Q đẳng giác với P trong tam giác ABC . Chứng minh rằng ∠AQB D ∠AQC .

Bài toán 6. Tìm số nguyên dương n nhỏ nhất sao cho tồn tại n số thực a1 ; a2 ; : : : ; an thỏa
mãn điều kiện:

a1 C a2 C    C an > 0; a13 C a23 C    C an3 < 0; a15 C a25 C    C an5 > 0:

Lời giải. Ta chứng minh n D 5 là giá trị n nhỏ nhất thỏa mãn điều kiện đề bài. Ta nhận xét rằng
nếu tồn tại n số thực thỏa mãn điều kiện đề bài thì với mọi m > n cũng tồn tại m số thực thỏa
mãn điều kiện đề bài (chỉ cần bổ sung thêm m n số 0). Vì vậy, để chứng minh n D 5 là giá trị
nhỏ nhất thỏa mãn điều kiện đề bài, ta chỉ cần chứng minh:

1) Không tồn tại 4 số thực a1 ; a2 ; a3 ; a4 sao cho

a1 C a2 C a3 C a4 > 0; a13 C a23 C a33 C a43 < 0; a15 C a25 C a35 C a45 > 0:

2) Tồn tại 5 số thực a1 ; a2 ; a3 ; a4 ; a5 sao cho

a1 C a2 C    C a5 > 0; a13 C a23 C    C a53 < 0; a15 C a25 C    C a55 > 0:

Ta lần lượt chứng minh 1) và 2). Để chứng minh 1), ta giả sử ngược lại rằng tồn tại 4 số thực
a1 ; a2 ; a3 ; a4 thỏa mãn

a1 C a2 C a3 C a4 > 0; a13 C a23 C a33 C a43 < 0; a15 C a25 C a35 C a45 > 0:

Khi đó trong các số a1 ; a2 ; a3 ; a4 có ít nhất một số dương và ít nhất một số âm.

Trường hợp 1: Có 1 số dương, 3 số không dương. Không mất tính tổng quát, giả sử a1 > 0 
a2 ; a3 ; a4 : Đặt bi D ai thì ta có a1 > b2 C b3 C b4 suy ra

a13 > .b2 C b3 C b4 /3  b23 C b33 C b43 D .a23 C a33 C a43 /:

Mâu thuẫn.

Trường hợp 2: Có 3 số không âm, 1 số âm. a1 ; a2 ; a3 > 0 > a4 : Đặt b4 D a4 : Ta có

a1 C a2 C a3 > b4 ; a13 C a23 C a33 < b43 ; a15 C a25 C a35 > b45 :
348 Các phương pháp giải toán qua các kỳ thi Olympic

Từ bất đẳng thức thứ hai suy ra a1 ; a2 ; a3 < b4 và

a15 C a25 C a35 < a13 b42 C a23 b42 C a33 b42 D .a13 C a23 C a33 /b32 < b35 :

Mâu thuẫn.

Trường hợp 3: Có 2 số dương, 2 số âm. Gọi hai số dương là a; b; hai số âm là c; d thì ta có


a; b; c; d > 0 và

a C b > c C d; a5 C b 5 > c 5 C d 5 ; a3 C b 3 < c 3 C d 3 :

Ta chứng minh điều này sẽ dẫn đến mâu thuẫn. Không mất tính tổng quát, giả sử a  b và c  d:
Thật vậy, do

a3 C b 3 D .a C b/.a2 ab C b 2 / < .c C d /.c 2 cd C d 2 /

và a C b > c C d nên c 2 cd C d 2 > a2 ab C b 2 ; suy ra

.c C d /2 C 3.c d /2 > .a C b/2 C 3.a b/2 ;

suy ra c d > a b: Nếu c  a thì từ đây sẽ suy ra b d > a c  0; suy ra b > d: Từ đó


a3 C b 3 > c 3 C d 3 ; mâu thuẫn. Vậy c > a: Và như vậy ta có c > a  b > d:

Đến đây ta có một số hướng giải khác nhau như sau:

Cách 1. Đặt C D dc ; A D da ; B D b
d
thì C > A; B: Xét hàm số

f .x/ D C x C 1 Ax Bx

thì theo điều kiện ta có f .1/ < 0; f .3/ > 0; f .5/ < 0: Ngoài ra, do C > A; B nên f .C1/ D
C1: Theo định lý về tính chất hàm liên tục, phương trình f .x/ D 0 có ít nhất 3 nghiệm. Áp
dụng định lý Rolle, suy ra phương trình f 0 .x/ D 0 có ít nhất hai nghiệm. ./

Nhưng ta lại có
f 0 .x/ D C x ln C Ax ln A B x ln A
  x  x 
x A B
DC 1 ln A ln B :
C C
Hàm trong ngoặc là hàm liên tục và đồng biến trên RC đồng biến nên chỉ có nhiều nhất 1 nghiệm,
suy ra f 0 .x/ có nhiều nhất 1 nghiệm, mâu thuẫn với ./.
  13  13
c 3 Cd 3
Cách 2. Ta có b  2
và a < c 3 C d 3 b3 : Do đó:

 35
c3 C d 3 b3 C b5 > c5 C d 5: ./

 53
  1 
3 3 3 5 c 3 Cd 3 3
Xét hàm số f .x/ D c C d x C x trên d; 2
: Ta có

5 3  32 h  23 i
f 0 .x/ D c C d3 x3 . 3x 2 / C 5x 4 D 5x 2 x 2 c3 C d 3 x3 0
3
Lời giải đề chọn đội tuyển Việt Nam dự thi IMO 2015 349

  13
c 3 Cd 3
do x  2
: Vì vậy f là hàm nghịch biến và ta có

f .x/ < f .d / D c 3 C d 3 :

Do đó ./ không thể xảy ra. Mâu thuẫn.

Cuối cùng, để hoàn tất phép chứng minh, ta sẽ chứng minh tồn tại 5 số thực a1 ; a2 ; : : : ; a5 sao
cho a1 C a2 C    C a5 > 0; a13 C a23 C    C a53 < 0 và a15 C a25 C    C a55 > 0:

Ta sẽ chỉ ra 5 số dương a; b; c; d; e sao cho a C b C c > d C e; a3 C b 3 C c 3 < d 3 C e 3 ;


a5 C b 5 C c 5 > d 5 C e 5 : Đầu tiên ta chọn a D 2x; b D c D 1; d D e D x C 1 thì
a C b C c D d C e:

Ta sẽ chọn x sao cho .2x/3 C 2 < 2.x C 1/3 và .2x/5 C 2 > 2.x C 1/5 : Giải các bất
phương trình này, ta có thể chọn x D 1; 5: Sau đó điều chỉnh d một chút, ta có thể chọn
a D 3; b D c D 1; d D e D 2; 45: Bài toán được giải quyết hoàn toàn.
Bình luận.
1. Bài toán này khá thú vị về cách đặt vấn đề, khá mới mẻ cho dù hiện nay bất đẳng thức
được nghiên cứu một cách khá rầm rộ và có nhiều công cụ mạnh được trang bị cho học
sinh. Để giải được bài này, học sinh cần áp dụng một chút tư duy tổ hợp (trong việc phân
trường hợp), phép chứng minh phản chứng và những biến đổi đại số (hoặc giải tích) khá
tinh tế. Việc xây dựng ví dụ cũng đòi hỏi khả năng dự đoán và khoanh vùng.

2. Tuy nhiên đây là một bài toán hơi nặng về kỹ thuật, không thật phù hợp cho một bài toán
olympic, khi thí sinh phải làm bài trong điều kiện thời gian hạn chế và không được sử dụng
các công cụ trợ giúp.

3. Trong quá trình đi tìm ví dụ cho trường hợp n D 5; một cách tự nhiên ta phát hiện
ra bài toán sau: Nếu a; b; c; d; e là các số thực dương sao cho a C b C c D d C e;
a5 C b 5 C c 5 D d 5 C e 5 thì a3 C b 3 C c 3 < d 3 C e 3 :

4. Một cách tự nhiên, ta có thể đặt câu hỏi tổng quát: Tìm số n nhỏ nhất sao cho tồn tại n số
thực a1 ; a2 ; : : : ; an có tổng các luỹ thừa bậc 4k C 1 dương .k D 0; 1; : : : ; m/ và tổng
các lũy thừa bậc 4k 1 âm .k D 1; 2; : : : ; m/:

Lời giải cho bài toán tổng quát có lẽ là rất khó. Ngay cả trường hợp m D 2 đã là khó.

PnCho ax 1 ; : : : ; am I b1 ; : : : ; bn là
5. Một câu hỏi khác cũng xuất phát từ lời giảiPbài toán trên:
m C n số thực dương. Hỏi phương trình m a
i D1 i
x
D i D1 bi có nhiều nhất bao nhiêu
nghiệm thực?
Một số bài toán liên quan
1. (Ucraina 2014, lớp 9) Tìm tất cả các số nguyên dương n sao cho tồn tại n số thực thỏa mãn
điều kiện a1 C    C an > 0 và a13 C    C an3 < 0:

(Bài này khá đơn giản: Với n D 1; 2 rõ ràng không thỏa mãn, còn với n D 3 thì chọn
a1 D a2 D 23 ; a3 D 1:)
350 Các phương pháp giải toán qua các kỳ thi Olympic

2. (IMC 2006) Cho a; b; c; d; e là các số thực dương sao cho a2 C b 2 C c 2 D d 2 C e 2 ;


a4 C b 4 C c 4 D d 4 C e 4 : Hãy so sánh các số a3 C b 3 C c 3 và d 3 C e 3 :

3. (Tournament of the Towns, 2006) Các số thực dương x1 ; : : : ; xk thỏa mãn điều kiện

x1 C    C xk x13 C    C xk3
x12 C    C xk2 < ; x1 C    C xk < :
2 2
a) Chứng minh rằng k > 50:
b) Tìm ví dụ một bộ như vậy với một giá trị k nào đó.
c) Tìm giá trị nhỏ nhất của k để tồn tại 1 ví dụ như vậy.
ĐỀ THI OLYMPIC TOÁN QUỐC TẾ 2015
Ban biên tập

1. Nhận xét về đề thi


Đánh giá một cách tổng quan thì đề thi năm nay là một đề thi khó. Ngay cả bài 1 và bài 4, các
bài truyền thống là “cho điểm” thì năm nay cũng không đơn giản chút nào, đặc biệt là ý 1b, tức
là chứng minh không tồn tại hệ điểm cân bằng không tâm có số điểm chẵn. Các bài 2 và 4 không
khó về ý tưởng nhưng đòi hỏi có kỹ thuật xử lý điêu luyện. Bài hình số 3 không quá khó nhưng
cũng là một cửa ải không đơn giản. Bài 6 được đánh giá là khó nhất kỳ thi, là một bài nằm giữa
đại số và tổ hợp, phải có thời gian để ngấm tình huống mới xử lý nổi.
Đi chi tiết vào từng bài. Bài 1 là một bài toán hình tổ hợp. Đây là một bài toán hay, dù ý a) có thể
là đã được biết trước. Ý b) là ý mới và lời giải thông qua phép đếm rất thú vị. Phải nói đây là một
bài toán đẹp, đơn giản nhưng không hiển nhiên. Rất IMO! Chỉ tiếc là như một số bạn đã nhận
xét, ý a) đã từng xuất hiện trong các cuốn sách Olympic.
Bài 2 là một bài toán số học. Bài này được chọn có lẽ do cách phát biểu đẹp và đối xứng của nó.
Đi sâu vào lời giải của nó thì không còn đẹp lắm, chủ yếu là vẫn dùng các xử lý kỹ thuật phức
tạp nhưng không mới, không có ý nào hay. Những đề toán thế này sẽ thuận lợi cho các đội được
ôn luyện nhiều, quen tay. Tuy nhiên, bài toán cũng có một ưu điểm là bài thuần túy số học, vì
nhiều năm qua thì bài số học thuần túy bị “xâm lấn” bởi các bài toán “số học tổ hợp”. Ví dụ bài
các đồng xu năm ngoái ở Nam Phi được tính là bài số học nhưng bản chất là tổ hợp.
Bài số 3 là một bài toán hình học. Bài này không quá khó như những bài số 3 trước đó. Mô hình
ở đây không phải quá lạ và chỉ cần chứng minh được một ý mấu chốt là có thể giải quyết được
nhanh chóng bài toán. Nếu dùng các công cụ cao cấp như tứ giác điều hòa, phép nghịch đảo hay
cực đối cực thì công việc đó sẽ rất nhanh chóng. Thậm chí vẫn có thể dùng cách biến đổi góc hay
chứng minh tam giác bằng nhau rất thuần túy hình học. Bên cạnh đó, nếu thử dùng số phức hay
đưa về tọa độ thì dù tính toán phức tạp hơn nhưng hướng đi lại khá rõ ràng. Đây có lẽ là xu thế ra
đề của BGK IMO những năm gần đây, họ chọn các bài đều hơn thay vì rất chênh lệch như trước,
bài 1 nâng độ khó lên và bài 3 giảm độ khó xuống, không còn “killing” như trước. Đồng thời,
đây cũng là năm thứ 3 liên tiếp cho bài hình ở vị trí số 3.
Bài số 4 là một bài toán hình học dễ. Dù phát biểu khá dài dòng với nhiều yếu tố, ràng buộc
nhưng kỳ thực, chính điều đó đã làm hạn chế các trường hợp phát sinh. Thêm nữa, chỉ cần chú ý
một chút là tìm được ý tưởng giải quyết. Lời giải chỉ dùng biến đổi góc rất nhẹ nhàng thông qua
các tứ giác nội tiếp hoặc tam giác đồng dạng. Đây thực sự là bài toán dễ nhất của kỳ thi năm nay.
Bài số 5 là một bài phương trình hàm, thuộc phân môn đại số. Bài này cũng thuần túy kỹ thuật,
giải bằng các phép thế liên tiếp. Bài này cũng không có ý gì mới. Bài này có thể đánh giá ngang
với bài 2 về mọi mặt.
Bài số 6 là bài khó nhất của kỳ thi, bất đẳng thức rời rạc. Ý tưởng cơ bản cần khai thác
là nếu đặt ci D ai C i 1 thì i  ci  i C 2014 và các ci đôi một khác nhau và ci

351
352 Các phương pháp giải toán qua các kỳ thi Olympic

chứa mọi số nguyên dương ngoại trừ b số. Để hình dung được lời giải, ta cứ thay 2015 bằng
1 số lẻ bất kỳ và xét các số nhỏ trước, sẽ thấy được hướng đi rõ ràng hơn. Bài toán này
có lẽ được xuất phát từ khái niệm tung hứng lượng tử (Quantum juggling, hay Siteswap,
https://en.wikipedia.org/wiki/Siteswap) trong kỹ thuật, liên quan đến nhóm
Weyl a-phin trong toán học hiện đại: hãy tưởng tượng là chúng ta tung các quả bóng vào thời
gian i lên độ cao ai 1 và rơi xuống vào thời gian ai –1 C i:

2. Đề thi chính thức


2.1. Ngày thi thứ nhất (ngày 10 tháng 7 năm 2015)
Bài 1. Ta nói tập S gồm hữu hạn điểm trên mặt phẳng là tập cân đối nếu với hai điểm phân biệt
A và B tùy ý thuộc S; tồn tại điểm C thuộc S sao cho AC D BC . Ta nói S là tập vô tâm nếu
với ba điểm phân biệt A; B; C tùy ý thuộc S; không tồn tại điểm P thuộc S sao cho

PA D PB D P C:

a) Chứng minh rằng với mọi số nguyên n  3; tồn tại tập cân đối gồm n điểm.

b) Hãy tìm tất cả các số nguyên n  3 sao cho tồn tại tập cân đối và vô tâm gồm n điểm.

Bài 2. Hãy tìm tất cả các bộ số nguyên dương .a; b; c/ sao cho mỗi số trong các số:

ab c; bc a; ca b

là lũy thừa của 2: .Lũy thừa của 2 là một số nguyên có dạng 2n với n là số nguyên không âm./

Bài 3. Cho tam giác nhọn ABC với AB > AC: Ký hiệu € là đường tròn ngoại tiếp, H là trực
tâm và F là chân đường cao hạ từ A của tam giác đó. Gọi M là trung điểm của BC: Gọi Q là
điểm nằm trên € sao cho ∠HQA D 90ı , và gọi K là điểm nằm trên € sao cho ∠HKQ D 90ı :
Giả sử rằng các điểm A; B; C; K và Q đôi một phân biệt, và nằm trên € theo thứ tự đó. Chứng
minh rằng các đường tròn ngoại tiếp của các tam giác KQH và FKM tiếp xúc với nhau.

2.2. Ngày thi thứ hai (ngày 11 tháng 7 năm 2015)


Bài 4. Cho tam giác ABC nội tiếp đường tròn  tâm O: Đường tròn € tâm A cắt đoạn thẳng
BC tại các điểm D và E sao cho B; D; E và C đôi một phân biệt và nằm trên đường thẳng BC
theo thứ tự đó. Gọi F và G là các giao điểm của € và ; sao cho A; F; B; C và G nằm trên 
theo thứ tự đó. Gọi K là giao điểm thứ hai của đường tròn ngoại tiếp tam giác BDF và đoạn
thẳng AB: Gọi L là giao điểm thứ hai của đường tròn ngoại tiếp tam giác C GE và đoạn thẳng
CA: Giả sử các đường thẳng FK và GL phân biệt và cắt nhau tại điểm X: Chứng minh rằng X
nằm trên đường thẳng AO:

Bài 5. Hãy tìm tất cả các hàm số f W R ! R thỏa mãn phương trình:

f x C f .x C y/ C f .xy/ D x C f .x C y/ C y  f .x/

với mọi số thực x và y:


Đề thi Olympic Toán quốc tế 2015 353

Bài 6. Dãy số nguyên a1 ; a2 ; a3 ; : : : thỏa mãn các điều kiện sau:

i) 1  aj  2015 với mọi j  1I

ii) k C ak ¤ ` C a` với mọi 1  k < `:

Chứng minh rằng tồn tại hai số nguyên dương b và N sao cho
ˇ ˇ
ˇ n ˇ
X
.aj b/ˇˇ  10072
ˇ ˇ
ˇ
ˇ
ˇj DmC1 ˇ

với mọi số nguyên m và n thỏa mãn n > m  N:

3. Kết quả của đội tuyển


Kết quả chính thức của kỳ thi vừa được công bố vào tối ngày 14 tháng 7, sau 3 ngày chấm thi.
Năm nay, đội tuyển Việt Nam đã xuất sắc đạt được 2 Huy chương vàng, 3 Huy chương bạc, 1
Huy chương đồng, giành vị trí thứ 5 toàn đoàn với tổng số điểm là 151.
Dưới đây là kết quả chi tiết:

Họ tên 1 2 3 4 5 6 Tổng điểm Huy chương


Vũ Xuân Trung 7 7 7 7 6 0 34 Vàng
Nguyễn Thế Hoàn 7 5 7 7 5 0 31 Vàng
Hoàng Anh Tài 7 3 1 7 7 0 25 Bạc
Nguyễn Huy Hoàng 4 2 7 7 1 2 23 Bạc
Nguyễn Tuấn Hải Đăng 7 2 0 7 7 0 23 Bạc
Nguyễn Thị Việt Hà 3 2 1 7 2 0 15 Đồng

You might also like